You are on page 1of 397

https://yo

utube.co
m/channe
l/UCYa4_J
rOrf8R5Kz
2uOtccXQ
https://unaca
FOLLOW
demy.app.link/
LOOsHfChfkb>
https://unacademy
.com/goal/bank-
SUBSCRIBE
exams/RTPSX/subs
cribe?referral_cod
e=TT10
https://unac
ademy.com/
goal/bank-
exams/RTPS
X/subscribe
?referral_co
de=TT10
1-5

6 - 39

40 - 64

65 - 73

74 - 179

180 - 238

239 - 377

378 - 386
What will come in the place of question mark (?) in the given expression?
ददए गए व्मॊजक भें प्रश्नवाचक चचन्ह (?) के स्थान ऩय क्मा आएगा?

1. 16.66% of 774 + (?) = 14.28% of 483 + 1142


(A) 1128 (B) 928 (C) 1082 (D) 1211 (E) None Of These

2. (36% of 2000 + 76% of 6000) ÷ 48 = (?) × 22 – 44


(A) 8 (B) 9 (C) 7 (D) 12 (E) None Of these

3. 562 – 795 + 54 × 72 – 15 × 68 = (90 + (?)) × 25


(A) 18.2 (B) 13.8 (C) 14 (D) 15.4 (E) None Of These

4. (222 + 43 – 342 + 3446) ÷ (?) = 43


(A) 54 (B) 66 (C) 48 (D) 72 (E) None Of These

5. 4444 ÷ 44 + 3003 ÷ 77 + 525 ÷ 75 = (?)


(A) 109 (B) 147 (C) 134 (D) 107 (E) None Of These

6. 350.1% of 329.97 – 12.5% of 552.01 = (?) ² – 69.9


(A) 34 (B) 36 (C) 44 (D) 26 (E) None Of These

7. 3√4912 + (13.97 × 15.97) – (?) = 21990


(A) 119 (B) 129 (C) 219 (D) 229 (E) None Of These

8. (?) = (30.982.01 + 13.982.01) ÷ (8.992.02 + 5120.33)


(A) 12 (B) 13 (C) 17 (D) 11 (E) None Of These

9. (?) = 19.89% of 34.98% of (8922.01 + 2628.11 + 31.12 + 19.03)


(A) 872 (B) 812 (C) 722 (D) 942 (E) None Of These

10. (36.01) ² + √840= 2304.99 – (2 × 14.02)


(A) 90 (B) 60 (C) 80 (D) 70 (E) None Of These

11. (145.11 – 95) ÷ 5.09 × (?) = 79.99


(A) 5 (B) 8 (C) 9 (D) 2 (E) 4

https://youtu https://instagra
https ://ww
be.com/chan w.fa cebook.
m.com/aashish https://
arorasocial(?)
1
nel/UCYa4_Jr
Orf8R5Kz2uO
com/a ashis utm_medium = t.me/st
ha rorasocial copy_link
tccXQ udified
12. 447.997 × 9.91 ÷ 4.112 + 44.889 = (?)
(A) 920 (B) 1250 (C) 1165 (D) 1029 (E) None Of These

13. (?) + (29.05 × 12.98) ÷ 4.04 = 244.25


(A) 120 (B) 130 (C) 140 (D) 150 (E) None Of These

14. 515.89 ÷ 4.8 – 83.5 = 448.32 – (?)


(A) 554 (B) 586 (C) 424 (D) 482 (E) None Of These

15. {(42 + 1.99) × 510.110} ÷ 26.92 = (?)


(A) 340 (B) 392 (C) 410 (D) 280 (E) None Of These

https://youtu https://instagra
https ://ww
be.com/chan w.fa cebook.
m.com/aashish https://
arorasocial(?)
2
nel/UCYa4_Jr
Orf8R5Kz2uO
com/a ashis utm_medium = t.me/st
ha rorasocial copy_link
tccXQ udified
1. Ans. (C) 6. Ans. (A)
16.66% of 774 + (?) = 14.28% of 483 350.1% of 329.97 – 12.5% of 552.01
+ 1142 = (?) 2 – 69.9
1/6 × 774 + (?) = 1 7 × 483 + 1142 350% of 330 – 12.5% of 552 = (?) 2 –
129 + (?) = 69 + 1142 70
(?) = 1211 – 129 3.5 × 330 – 0.125 × 552 = (?) 2 – 70
(?) = 1082 1155 – 69 = (?) 2 – 70
(?)2 = 1156
2. Ans. (C) (?) = 34
(36% of 2000 + 76% of 6000) ÷ 48 =
(?) × 22 – 44 7. Ans. (C)
(0.36 × 2000 + 0.76 × 6000) ÷ 48 = 3 √4912 + (13.97 × 15.97) – (?) =

(?) × 22 – 44 21.999
(720 + 4560) ÷ 48 = (?) × 22 – 44 3 √4913 + (14 × 16) – (?) = 22

5280 ÷ 48 = (?) × 22 – 44 17 + 224 – (?) = 22


110 = (?) × 22 – 44 (?) = 241 – 22
(?) × 22 = 154 (?) = 219
(?) = 7
8. Ans. (B)
3. Ans. (D) (?) = (30.982.01 + 13.982.01 ) ÷
562 – 795 + 54 × 72 – 15 × 68 = (90 + (8.992.02 + 5120.33 )
(?)) × 25 (?) = (312 + 142 ) ÷ (92 + 5121/3 )
– 233 + 3888 – 1020 = (90 + (?)) × 25 (?) = (961 + 196) ÷ (81 + 8)
2635 = (90 + (?)) × 25 (?) = 1157 ÷ 89
90 + (?) = 105.4 (?) = 13
(?) = 15.4
9. Ans. (B)
4. Ans. (B) (?) = 19.89% of 34.98% of (8922.01
(222 + 43 – 342 + 3446) ÷ (?) = 43 + 2628.11 + 31.12 + 19.03)
(484 + 64 – 1156 + 3446) ÷ (?) = 43 (?) = 20% of 35% of (8922 + 2628 +
2838 ÷ 43 = (?) 31 + 19)
(?) = 66 (?) = 0.2 × 0.35 × 11600
(?) = 812
5. Ans. (C)
4444 ÷ 44 + 3003 ÷ 77 + 525 ÷ 75 =
(?) 101 + 39 + 7 = (?)(?) = 147

https://youtu https://instagra
https ://ww
be.com/chan w.fa cebook.
m.com/aashish https://
arorasocial(?)
3
nel/UCYa4_Jr
Orf8R5Kz2uO
com/a ashis utm_medium = t.me/st
ha rorasocial copy_link
tccXQ udified
10. Ans. (D) 15. Ans. (A)
(36.01) 2 + √840 = 2304.99 – ((?) × {(42 + 1.99) × 510.110} ÷ 26.92 = (?)
14.02) {(42 + 2) × 510} ÷ 27 = (?)
362 + √841 = 2305 – ((?) × 14) {18 × 510} ÷ 27 = (?)
(?) × 14 = 2305 – 1296 – 29 2 × 510 ÷ 3 = (?)
(?) × 14 = 980 (?) = 340
(?) = 70

11. Ans. (B)


(145.11 – 95) ÷ 5.09 × (?) = 79.99
(145 – 95) ÷ 5 × (?) = 80
50 ÷ 5 × (?) = 80
10 × (?) = 80
(?) = 8

12. Ans. (C)


447.997 × 9.91 ÷ 4.112 + 44.889 =
(?)
448 × 10 ÷ 4 + 45 = (?)
4480 ÷ 4 + 45 = (?)
1120 + 45 = (?)
(?) = 1165

13. Ans. (B)


(?) + (29.05 × 12.98) ÷ 4.04 = 244.25
(?) + (29 × 13) ÷ 4 = 244.25
(?) + 377 ÷ 4 = 244.25
(?) = 224.25 – 94.25
(?) = 130

14. Ans. (C)


515.89 ÷ 4.8 – 83.5 = 448.32 – (?)
516 ÷ 4.8 – 83.5 = 448 – (?)
107.5 – 83.5 = 448 – (?)
(?) = 448 – 24
(?) = 424

https://youtu https://instagra
https ://ww
be.com/chan w.fa cebook.
m.com/aashish https://
arorasocial(?)
4
nel/UCYa4_Jr
Orf8R5Kz2uO
com/a ashis utm_medium = t.me/st
ha rorasocial copy_link
tccXQ udified
https://t.mhttps/t.me
e/studified/studified

https://instagram.com/aashisharora
social?utm_medium = copy_link

https://youtube.com/channel
/UCYa4_JrOrf8R5Kz2uOtccXQ

https://www.facebook.
com/aashisharorasocial

https://youtu https://instagra
https ://ww
be.com/chan w.fa cebook.
m.com/aashish https://
arorasocial(?)
5
nel/UCYa4_Jr
Orf8R5Kz2uO
com/a ashis utm_medium = t.me/st
ha rorasocial copy_link
tccXQ udified
Solve the given equations and choose the correct option.
ददए गए सभीकयणों को हर कयें औय सही ववकल्ऩ चुनें।

1. 2x² – 7x + 28 = 14x – 26, y² – 8y + 7 = 6y – 42


(A) x > y (B) x ≥ y (C) x < y
(D) x ≤ y (E) x = y or no relation can be established

2. 12x² + 38x + 5 = 9x – 32 , 12y² – 11y = 15y – 22


(A) x > y (B) x ≥ y (C) x < y
(D) x ≤ y (E) x = y or no relation can be established

3. 132x² – 23x + 1 = 0, 88y² – 19y + 1 = 0


(A) x > y (B) x ≥ y (C) x < y
(D) x ≤ y (E) x = y or no relation can be established

4. 3x² + 3x + 79 = 87 – 7x, 3y² – 5y + 115 = 125 + 8y


(A) x > y (B) x ≥ y (C) x < y
(D) x ≤ y (E) x = y or no relation can be established

5. 3x² + 10x + 6 = - x – 4, 6y² + 8y + 4 = - 11 – 11y


(A) x > y (B) x ≥ y (C) x < y
(D) x ≤ y (E) x = y or no relation can be established

6. 25x² – 30x + 8 = 0, 25y² + 110y + 21 = 0


(A) x > y (B) x ≥ y (C) x < y
(D) x ≤ y (E) x = y or no relation can be established

7. x² – x – 6 = 0, y² + 4y + 3 = 0
(A) x > y (B) x ≥ y (C) x < y
(D) x ≤ y (E) x = y or no relation can be established

8. 48x² – 206x + 221 = 0, 15y² – 7y – 4 = 0


(A) x > y (B) x ≥ y (C) x < y
(D) x ≤ y (E) x = y or no relation can be established

9. x² – 2x – 195 = 0, y² – 4y – 192 = 0
(A) x > y (B) x ≥ y (C) x < y
(D) x ≤ y (E) x = y or no relation can be established
https://youtu https://instagra
https ://ww
be.com/chan w.fa cebook.
m.com/aashish https://
arorasocial(?)
nel/UCYa4_Jr com/a ashis utm_medium = t.me/st
6 Orf8R5Kz2uO
ha rorasocial copy_link
tccXQ udified
10. x² + 2x – 24 = 0, y² – 12y + 36 = 0
(A) x > y (B) x ≥ y (C) x < y
(D) x ≤ y (E) x = y or no relation can be established

11. x² – 5x + 6 = 0, y² + 3y + 2 = 0
(A) x > y (B) x ≥ y (C) x < y
(D) x ≤ y (E) x = y or no relation can be established

12. 10x² – 13x + 4 = 0, 20y² – 7y – 3 = 0


(A) x > y (B) x ≥ y (C) x < y
(D) x ≤ y (E) x = y or no relation can be established

13. x² + 2x – 224 = 0, y² – 30y + 221 = 0


(A) x > y (B) x ≥ y (C) x < y
(D) x ≤ y (E) x = y or no relation can be established

14. 8x² – 14x + 3 = 0, 8y² – 33y + 4 = 0


(A) x > y (B) x ≥ y (C) x < y
(D) x ≤ y (E) x = y or no relation can be established

15. x² – 23x + 132 = 0, y² – 25y + 156 = 0


(A) x > y (B) x ≥ y (C) x < y
(D) x ≤ y (E) x = y or no relation can be established

16. 32x² – 20x + 3 = 0, 4y² – 17y + 18 = 0


(A) x > y (B) x ≥ y (C) x < y
(D) x ≤ y (E) x = y or no relation can be established

17. x² + 7x + 12 = 0, y² – 2y – 24 = 0
(A) x > y (B) x ≥ y (C) x < y
(D) x ≤ y (E) x = y or no relation can be established

18. 4x² – 7x + 3 = 0, 5y² – 11y + 6 = 0


(A) x > y (B) x ≥ y (C) x < y
(D) x ≤ y (E) x = y or no relation can be established

19. x² + 10x + 24 = 0, y² – 7y + 12 = 0
(A) x > y (B) x ≥ y (C) x < y
(D) x ≤ y (E) x = y or no relation can be established

20. x² + 10x + 21 = 0, y² + 3y – 28 = 0
(A) x > y (B) x ≥ y (C) x < y
(D) x ≤ y (E) x = y or no relation can be established
https://instagra
https://youtu https ://ww
be.com/chan w.fa cebook.
m.com/aashish https://
arorasocial(?)
7
nel/UCYa4_Jr
Orf8R5Kz2uO
com/a ashis utm_medium = t.me/st
ha rorasocial copy_link
tccXQ udified
21. 60x2 + 103x + 44 = 0, 30y2 + 83y + 56 = 0
(A) x > y (B) x ≥ y (C) x < y
(D) x ≤ y (E) x = y or no relation can be established

22. x2 – 25x + 156 = 0, y2 – 31y + 234 = 0


(A) x > y (B) x ≥ y (C) x < y
(D) x ≤ y (E) x = y or no relation can be established

23. 35x2 + 38x + 8 = 0, 15y2 – 38y + 24 = 0


(A) x > y (B) x ≥ y (C) x < y
(D) x ≤ y (E) x = y or no relation can be established

24. x2 – 7x + 10 = 0, y2 – 10y + 21 = 0
(A) x > y (B) x ≥ y (C) x < y
(D) x ≤ y (E) x = y or no relation can be established

25. 6x2 – 17x + 12 = 0, 18y2 – 41y + 21 = 0


(A) x > y (B) x ≥ y (C) x < y
(D) x ≤ y (E) x = y or no relation can be established

26. 63x2 – 85x + 28 = 0, 18y² + 17y – 15 = 0


(A) x > y (B) x ≥ y (C) x < y
(D) x ≤ y (E) x = y or no relation can be established

27. x² – 16x + 63 = 0, y² – 7y + 12 = 0
(A) x > y (B) x ≥ y (C) x < y
(D) x ≤ y (E) x = y or no relation can be established

28. 6x² + 49x + 88 = 0, 24y² + 37y + 14 = 0


(A) x > y (B) x ≥ y (C) x < y
(D) x ≤ y (E) x = y or no relation can be established

29. x² + 4x + 3 = 0, y² – y – 2 = 0
(A) x > y (B) x ≥ y (C) x < y
(D) x ≤ y (E) x = y or no relation can be established

30. 12x² – 97x + 195 = 0, 12y² – 83y + 143 = 0


(A) x > y (B) x ≥ y (C) x < y
(D) x ≤ y (E) x = y or no relation can be established

31. 12x2 – 37x + 28 = 0, 56y2 – 97y + 42 = 0


(A) x > y (B) x ≥ y (C) x < y
(D) x ≤ y (E) x = y or no relation can be established
https://instagra
https://youtu https ://ww
be.com/chan w.fa cebook.
m.com/aashish https://
arorasocial(?)
8
nel/UCYa4_Jr
Orf8R5Kz2uO
com/a ashis utm_medium = t.me/st
ha rorasocial copy_link
tccXQ udified
32. 3x2 + 19x + 20 = 0, 3y2 – 2y – 8 = 0
(A) x > y (B) x ≥ y (C) x < y
(D) x ≤ y (E) x = y or no relation can be established

33. 20x2 – 29X – 33 = 0, 7y2 – 22y + 16 = 0


(A) x > y (B) x ≥ y (C) x < y
(D) x ≤ y (E) x = y or no relation can be established

34. x2 – 15X + 56 = 0, y2 – 25y + 156 = 0


(A) x > y (B) x ≥ y (C) x < y
(D) x ≤ y (E) x = y or no relation can be established

35. 8x2 – 23x + 14 = 0, 8y2 + 41y – 21 = 0


(A) x > y (B) x ≥ y (C) x < y
(D) x ≤ y (E) x = y or no relation can be established

36. 52x2 – 87x + 36 = 0, 112y² – 199y + 85 = 0


(A) x > y (B) x ≥ y (C) x < y
(D) x ≤ y (E) x = y or no relation can be established

37. x² + 37x + 342 = 0, y² + 39y + 380 = 0


(A) x > y (B) x ≥ y (C) x < y
(D) x ≤ y (E) x = y or no relation can be established

38. 28x² + 97x + 84 = 0, 40y² + 66y + 27 = 0


(A) x > y (B) x ≥ y (C) x < y
(D) x ≤ y (E) x = y or no relation can be established

39. x² – 3x – 40 = 0, y² – 16y + 63 = 0
(A) x > y (B) x ≥ y (C) x < y
(D) x ≤ y (E) x = y or no relation can be established

40. 180x² – 399x + 221 = 0, 88y² – 157y + 70 = 0


(A) x > y (B) x ≥ y (C) x < y
(D) x ≤ y (E) x = y or no relation can be established

41. 8x² – 46x + 63 = 0, 24y² – 85y + 56 = 0


(A) x > y (B) x ≥ y (C) x < y
(D) x ≤ y (E) x = y or no relation can be established

42. x² – 38x + 217 = 0, y² – 11 y + 30 = 0


(A) x > y (B) x ≥ y (C) x < y
(D) x ≤ y (E) x = y or no relation can be established
https://instagra
https://youtu https ://ww
be.com/chan w.fa cebook.
m.com/aashish https://
arorasocial(?)
9
nel/UCYa4_Jr
Orf8R5Kz2uO
com/a ashis utm_medium = t.me/st
ha rorasocial copy_link
tccXQ udified
43. 88x² + 7x – 30 = 0, 8y² – 109y – 42 = 0
(A) x > y (B) x ≥ y (C) x < y
(D) x ≤ y (E) x = y or no relation can be established

44. 4x² – 88x + 483 = 0, 4y² – 72y + 315 = 0


(A) x > y (B) x ≥ y (C) x < y
(D) x ≤ y (E) x = y or no relation can be established

45. 56x² + 291x + 378 = 0, 323y² + 610y + 288 = 0


(A) x > y (B) x ≥ y (C) x < y
(D) x ≤ y (E) x = y or no relation can be established

46. 88 x² – 149x + 63 = 0, 42y² – 163y + 156 = 0


(A) x > y (B) x ≥ y (C) x < y
(D) x ≤ y (E) x = y or no relation can be established

47. x² + 23x + 132 = 0, y² + 25y + 156 = 0


(A) x > y (B) x ≥ y (C) x < y
(D) x ≤ y (E) x = y or no relation can be established

48. 45 x² – 19x – 42 = 0, 4y² – 27y – 7 = 0


(A) x > y (B) x ≥ y (C) x < y
(D) x ≤ y (E) x = y or no relation can be established

49. x² – 36x + 315 = 0, y² – 48y + 572 = 0


(A) x > y (B) x ≥ y (C) x < y
(D) x ≤ y (E) x = y or no relation can be established

50. 77x² – 230x + 168 = 0, 12y² + 53y + 56 = 0


(A) x > y (B) x ≥ y (C) x < y
(D) x ≤ y (E) x = y or no relation can be established

51. 24x² + 29x + 7 = 0, 12y² + 31y + 20 = 0


(A) x > y (B) x ≥ y (C) x < y
(D) x ≤ y (E) x = y or no relation can be established

52. x² + 18x + 77 = 0, y² + 29y + 198 = 0


(A) x > y (B) x ≥ y (C) x < y
(D) x ≤ y (E) x = y or no relation can be established

53. 8x² – 47x + 35 = 0, 5y² – 17y + 14 = 0


(A) x > y (B) x ≥ y (C) x < y
(D) x ≤ y (E) x = y or no relation can be established
https://instagra
https://youtu https ://ww
be.com/chan w.fa cebook.
m.com/aashish https://
arorasocial(?)
10
nel/UCYa4_Jr
Orf8R5Kz2uO
com/a ashis utm_medium = t.me/st
ha rorasocial copy_link
tccXQ udified
54. x² + 33x + 270 = 0, y² + 44y + 483 = 0
(A) x > y (B) x ≥ y (C) x < y
(D) x ≤ y (E) x = y or no relation can be established

55. 288x² + 610x + 323 = 0, 187y² + 291y + 110 = 0


(A) x > y (B) x ≥ y (C) x < y
(D) x ≤ y (E) x = y or no relation can be established

56. 12x² + 7x + 1 = 0, 10y² + 23y + 12 = 0


(A) x > y (B) x ≥ y (C) x < y
(D) x ≤ y (E) x = y or no relation can be established

57. x² + 23x + 132 = 0, y² + 21y + 110 = 0


(A) x > y (B) x ≥ y (C) x < y
(D) x ≤ y (E) x = y or no relation can be established

58. 56x² – 89x + 35 = 0, 8y² – 22y + 15 = 0


(A) x > y (B) x ≥ y (C) x < y
(D) x ≤ y (E) x = y or no relation can be established

59. x² – 45x + 504 = 0, y² – 45y + 506 = 0


(A) x > y (B) x ≥ y (C) x < y
(D) x ≤ y (E) x = y or no relation can be established

60. x² + 35x + 306 = 0, y² – 9y + 20 = 0


(A) x > y (B) x ≥ y (C) x < y
(D) x ≤ y (E) x = y or no relation can be established

61. 100 x² – 189x + 80 = 0, 36y² – 127y + 112 = 0


(A) x > y (B) x ≥ y (C) x < y
(D) x ≤ y (E) x = y or no relation can be established

62. 4 x² – 73x + 18 = 0, 45y² + 44y – 12 = 0


(A) x > y (B) x ≥ y (C) x < y
(D) x ≤ y (E) x = y or no relation can be established

63. 55 x² – 29x – 12 = 0, 44y² + 79y + 30 = 0


(A) x > y (B) x ≥ y (C) x < y
(D) x ≤ y (E) x = y or no relation can be established

64. x² – 25x + 150 = 0, y² + 25y + 150 = 0


(A) x > y (B) x ≥ y (C) x < y
(D) x ≤ y (E) x = y or no relation can be established
https://instagra
https://youtu https ://ww
be.com/chan w.fa cebook.
m.com/aashish https://
arorasocial(?)
11
nel/UCYa4_Jr
Orf8R5Kz2uO
com/a ashis utm_medium = t.me/st
ha rorasocial copy_link
tccXQ udified
65. 40 x² – 59x + 21 = 0, 36y² – 43y + 12 = 0
(A) x > y (B) x ≥ y (C) x < y
(D) x ≤ y (E) x = y or no relation can be established

66. 24 x² + 19x + 2 = 0, 12y² + 5y – 2 = 0


(A) x > y (B) x ≥ y (C) x < y
(D) x ≤ y (E) x = y or no relation can be established

67. x² – x – 182 = 0, 3y² + 38y + 24 = 0


(A) x > y (B) x ≥ y (C) x < y
(D) x ≤ y (E) x = y or no relation can be established

68. x² + 7x + 6 = 0, y² + 16y + 63 = 0
(A) x > y (B) x ≥ y (C) x < y
(D) x ≤ y (E) x = y or no relation can be established

69. 9 x² – 15x + 4 = 0, 3y² – 31y + 36 = 0


(A) x > y (B) x ≥ y (C) x < y
(D) x ≤ y (E) x = y or no relation can be established

70. 15 x² + 59x – 80 = 0, 16y² + 159y + 135 = 0


(A) x > y (B) x ≥ y (C) x < y
(D) x ≤ y (E) x = y or no relation can be established

71. 2 x² + 5x + 2 = 0, 2y² – 5y + 2 = 0
(A) x > y (B) x ≥ y (C) x < y
(D) x ≤ y (E) x = y or no relation can be established

72. x² + 2x – 224 = 0, y² + 40y + 384 = 0


(A) x > y (B) x ≥ y (C) x < y
(D) x ≤ y (E) x = y or no relation can be established

73. x² + 14x + 45 = 0, y² – 14y + 48 = 0


(A) x > y (B) x ≥ y (C) x < y
(D) x ≤ y (E) x = y or no relation can be established

74. 32 x² – 12x + 1 = 0, 64y² – 20y + 1 = 0


(A) x > y (B) x ≥ y (C) x < y
(D) x ≤ y (E) x = y or no relation can be established

75. x² – 2x – 99 = 0, y² – 24y + 143 = 0


(A) x > y (B) x ≥ y (C) x < y
(D) x ≤ y (E) x = y or no relation can be established
https://instagra
https://youtu https ://ww
be.com/chan w.fa cebook.
m.com/aashish https://
arorasocial(?)
12
nel/UCYa4_Jr
Orf8R5Kz2uO
com/a ashis utm_medium = t.me/st
ha rorasocial copy_link
tccXQ udified
76. x² – 25x + 156 = 0, y² + 23y + 132 = 0
(A) x > y (B) x ≥ y (C) x < y
(D) x ≤ y (E) x = y or no relation can be established

77. 5x² – 26x – 24 = 0, 25y² + 60y + 32 = 0


(A) x > y (B) x ≥ y (C) x < y
(D) x ≤ y (E) x = y or no relation can be established

78. 4x² + 73x + 144 = 0, 3y² – 52y + 64 = 0


(A) x > y (B) x ≥ y (C) x < y
(D) x ≤ y (E) x = y or no relation can be established

79. 4x² – 33x + 8 = 0, 16y2 – 1 = 0


(A) x > y (B) x ≥ y (C) x < y
(D) x ≤ y (E) x = y or no relation can be established

80. 3x² – 47x + 30 = 0, 3y² – 38y + 24 = 0


(A) x > y (B) x ≥ y (C) x < y
(D) x ≤ y (E) x = y or no relation can be established

81. x² + 15x + 54 = 0, y² + 9y + 18 = 0
(A) x > y (B) x ≥ y (C) x < y
(D) x ≤ y (E) x = y or no relation can be established

82. 56x² + 113x + 56 = 0, y² – 13y + 36 = 0


(A) x > y (B) x ≥ y (C) x < y
(D) x ≤ y (E) x = y or no relation can be established

83. 5x² + 31x + 30 = 0, 5y² + 9y + 4 = 0


(A) x > y (B) x ≥ y (C) x < y
(D) x ≤ y (E) x = y or no relation can be established

84. x² + 15x + 56 = 0, 72y² – 43y + 6 = 0


(A) x > y (B) x ≥ y (C) x < y
(D) x ≤ y (E) x = y or no relation can be established

85. 7x² – 20x – 32 = 0, 8y² – 17y – 21 = 0


(A) x > y (B) x ≥ y (C) x < y
(D) x ≤ y (E) x = y or no relation can be established

86. x² + 11x + 30 = 0, 8y² – 14y + 3 = 0


(A) x > y (B) x ≥ y (C) x < y
(D) x ≤ y (E) x = y or no relation can be established
https://instagra
https://youtu https ://ww
be.com/chan w.fa cebook.
m.com/aashish https://
arorasocial(?)
13
nel/UCYa4_Jr
Orf8R5Kz2uO
com/a ashis utm_medium = t.me/st
ha rorasocial copy_link
tccXQ udified
87. 4 x² + 63x – 16 = 0, 4y² – 17y + 4 = 0
(A) x > y (B) x ≥ y (C) x < y
(D) x ≤ y (E) x = y or no relation can be established

88. x² – 30x + 209 = 0, 3y² + 17y – 176 = 0


(A) x > y (B) x ≥ y (C) x < y
(D) x ≤ y (E) x = y or no relation can be established

89. 25x² – 35x + 12 = 0, 25y² – 65y + 36 = 0


(A) x > y (B) x ≥ y (C) x < y
(D) x ≤ y (E) x = y or no relation can be established

90. x² – x – 56 = 0, 8y² + 69y – 27 = 0


(A) x > y (B) x ≥ y (C) x < y
(D) x ≤ y (E) x = y or no relation can be established

91. 3x² + 14x + 15 = 0, 3y² + 7y + 4 = 0


(A) x > y (B) x ≥ y (C) x < y
(D) x ≤ y (E) x = y or no relation can be established

92. x² + 18x + 77 = 0, 5y² – 52y + 20 = 0


(A) x > y (B) x ≥ y (C) x < y
(D) x ≤ y (E) x = y or no relation can be established

93. x² – 24x + 143 = 0, 2y² – 49y + 299 = 0


(A) x > y (B) x ≥ y (C) x < y
(D) x ≤ y (E) x = y or no relation can be established

94. 3x² – 61x + 240 = 0, 4y² + 35y – 99 = 0


(A) x > y (B) x ≥ y (C) x < y
(D) x ≤ y (E) x = y or no relation can be established

95. x² + 12x + 35 = 0, y² + 16y + 63 = 0


(A) x > y (B) x ≥ y (C) x < y
(D) x ≤ y (E) x = y or no relation can be established

96. 10x² + 29x + 21 = 0, 77y² + 136y + 60 = 0


(A) x > y (B) x ≥ y (C) x < y
(D) x ≤ y (E) x = y or no relation can be established

97. x² + 32x + 255 = 0, y² + 28y + 195 = 0


(A) x > y (B) x ≥ y (C) x < y
(D) x ≤ y (E) x = y or no relation can be established
https://instagra
https://youtu https ://ww
be.com/chan w.fa cebook.
m.com/aashish https://
arorasocial(?)
14
nel/UCYa4_Jr
Orf8R5Kz2uO
com/a ashis utm_medium = t.me/st
ha rorasocial copy_link
tccXQ udified
98. 8x² – 71x + 56 = 0, 8y² – 39y + 28 = 0
(A) x > y (B) x ≥ y (C) x < y
(D) x ≤ y (E) x = y or no relation can be established

99. x² + 19x + 88 = 0, y² + 15y + 56 = 0


(A) x > y (B) x ≥ y (C) x < y
(D) x ≤ y (E) x = y or no relation can be established

100. 12x² – 127x + 323 = 0, 20y² – 123y + 180 = 0


(A) x > y (B) x ≥ y (C) x < y
(D) x ≤ y (E) x = y or no relation can be established

101. 9x² – 45x + 56 = 0, 20y² – 53y + 35 = 0


(A) x > y (B) x ≥ y (C) x < y
(D) x ≤ y (E) x = y or no relation can be established

102. 5x² – 49x + 36 = 0, 25y² – 35y + 12 = 0


(A) x > y (B) x ≥ y (C) x < y
(D) x ≤ y (E) x = y or no relation can be established

103. x² + 23x + 132 = 0, y² + 21y + 110 = 0


(A) x > y (B) x ≥ y (C) x < y
(D) x ≤ y (E) x = y or no relation can be established

104. 56x² – 181x + 143 = 0, 5y² + 14y – 24 = 0


(A) x > y (B) x ≥ y (C) x < y
(D) x ≤ y (E) x = y or no relation can be established

105. 15x² – 149x + 342 = 0, 4y² – 96y + 567 = 0


(A) x > y (B) x ≥ y (C) x < y
(D) x ≤ y (E) x = y or no relation can be established

106. x² – 15x + 56 = 0, y² – 16y + 63 = 0


(A) x > y (B) x ≥ y (C) x < y
(D) x ≤ y (E) x = y or no relation can be established

107. 44x² – 69x + 27 = 0, 10y² – 51y + 54 = 0


(A) x > y (B) x ≥ y (C) x < y
(D) x ≤ y (E) x = y or no relation can be established

108. 2x² + 13x – 7 = 0, 4y² + 33y – 27 = 0


(A) x > y (B) x ≥ y (C) x < y
(D) x ≤ y (E) x = y or no relation can be established
https://instagra
https://youtu https ://ww
be.com/chan w.fa cebook.
m.com/aashish https://
arorasocial(?)
15
nel/UCYa4_Jr
Orf8R5Kz2uO
com/a ashis utm_medium = t.me/st
ha rorasocial copy_link
tccXQ udified
109. x² – 12√2x + 70 = 0, y² – 9√2y + 40 = 0
(A) x > y (B) x ≥ y (C) x < y
(D) x ≤ y (E) x = y or no relation can be established

110. x² + 63x + 992 = 0, y² + 55y + 756 = 0


(A) x > y (B) x ≥ y (C) x < y
(D) x ≤ y (E) x = y or no relation can be established

111. x² – 36x + 323 = 0, y² – 33y + 272 = 0


(A) x > y (B) x ≥ y (C) x < y
(D) x ≤ y (E) x = y or no relation can be established

112. 20x² – 63x + 49 = 0, 77y² – 108y + 36 = 0


(A) x > y (B) x ≥ y (C) x < y
(D) x ≤ y (E) x = y or no relation can be established

113. 30x² + 49x + 20 = 0, 20y² + 49y + 30 = 0


(A) x > y (B) x ≥ y (C) x < y
(D) x ≤ y (E) x = y or no relation can be established

114. x² + 44x + 483 = 0, y² + 55y + 744 = 0


(A) x > y (B) x ≥ y (C) x < y
(D) x ≤ y (E) x = y or no relation can be established

115. x² – 22x + 105 = 0, y² – 24y + 128 = 0


(A) x > y (B) x ≥ y (C) x < y
(D) x ≤ y (E) x = y or no relation can be established

116. 72 x² – 161x + 90 = 0, 48y² – 82y + 35 = 0


(A) x > y (B) x ≥ y (C) x < y
(D) x ≤ y (E) x = y or no relation can be established

117. 6 x² – 79x + 255 = 0, 9y² – 180y + 896 = 0


(A) x > y (B) x ≥ y (C) x < y
(D) x ≤ y (E) x = y or no relation can be established

118. x² + 30x + 221 = 0, y² + 25y + 156 = 0


(A) x > y (B) x ≥ y (C) x < y
(D) x ≤ y (E) x = y or no relation can be established

119. 9x² – 52x + 35 = 0, 7y² – 61y + 40 = 0


(A) x > y (B) x ≥ y (C) x < y
(D) x ≤ y (E) x = y or no relation can be established
https://instagra
https://youtu https ://ww
be.com/chan w.fa cebook.
m.com/aashish https://
arorasocial(?)
16
nel/UCYa4_Jr
Orf8R5Kz2uO
com/a ashis utm_medium = t.me/st
ha rorasocial copy_link
tccXQ udified
120. x² – 40x + 399 = 0, y² – 44y + 483 = 0
(A) x > y (B) x ≥ y (C) x < y
(D) x ≤ y (E) x = y or no relation can be established

121. x² – 42x + 437 + 0, y² – 47y + 552 + 0


(A) x > y (B) x ≥ y (C) x < y
(D) x ≤ y (E) x + y or no relation can be established

122. 6x² + 67x + 165 + 0, 15y² + 41y + 28 + 0


(A) x > y (B) x ≥ y (C) x < y
(D) x ≤ y (E) x + y or no relation can be established

123. 10 x² – 61x + 72 + 0, 48y² – 82y + 35 + 0


(A) x > y (B) x ≥ y (C) x < y
(D) x ≤ y (E) x + y or no relation can be established

124. x² + 35x + 306 + 0, y² + 38y + 360 + 0


(A) x > y (B) x ≥ y (C) x < y
(D) x ≤ y (E) x + y or no relation can be established

125. 56 x² – 89x + 35 + 0, 35y² – 29y + 6 + 0


(A) x > y (B) x ≥ y (C) x < y
(D) x ≤ y (E) x + y or no relation can be established

126. x² + 10x + 24 = 0, y² – 15y + 56 = 0


(A) x > y (B) x ≥ y (C) x < y
(D) x ≤ y (E) x = y or no relation can be established

127. 8x² + 26x + 21 = 0, 8y² + 38y + 45 = 0


(A) x > y (B) x ≥ y (C) x < y
(D) x ≤ y (E) x = y or no relation can be established

128. 10x² – 27x + 18 = 0, 32y² – 20y + 3 = 0


(A) x > y (B) x ≥ y (C) x < y
(D) x ≤ y (E) x = y or no relation can be established

129. 4x² – 23x + 28 = 0, 3y² – 16y + 21 = 0


(A) x > y (B) x ≥ y (C) x < y
(D) x ≤ y (E) x = y or no relation can be established

130. x² + 10x + 24 = 0, y² + 5y + 6 = 0
(A) x > y (B) x ≥ y (C) x < y
(D) x ≤ y (E) x = y or no relation can be established
https://instagra
https://youtu https ://ww
be.com/chan w.fa cebook.
m.com/aashish https://
arorasocial(?)
17
nel/UCYa4_Jr
Orf8R5Kz2uO
com/a ashis utm_medium = t.me/st
ha rorasocial copy_link
tccXQ udified
131. 5x² + 13x – 28 = 0, 8y² – 51y + 18 = 0
(A) x > y (B) x ≥ y (C) x < y
(D) x ≤ y (E) x + y or no relation can be established

132. 20x² – 79x + 66 = 0, y² – 2y – 48 = 0


(A) x > y (B) x ≥ y (C) x < y
(D) x ≤ y (E) x + y or no relation can be established

133. 22x² – 23x + 6 = 0, 63y² – 100y + 32 = 0


(A) x > y (B) x ≥ y (C) x < y
(D) x ≤ y (E) x + y or no relation can be established

134. 7x² – 101x + 42 = 0, 49y² – 35y + 6 = 0


(A) x > y (B) x ≥ y (C) x < y
(D) x ≤ y (E) x + y or no relation can be established

135. 5x² – 34x + 24 = 0, y² – 19y + 88 = 0


(A) x > y (B) x ≥ y (C) x < y
(D) x ≤ y (E) x + y or no relation can be established

https://youtu https://instagra
https ://ww
be.com/chan w.fa cebook.
m.com/aashish https://
arorasocial(?)
18
nel/UCYa4_Jr
Orf8R5Kz2uO
com/a ashis utm_medium = t.me/st
ha rorasocial copy_link
tccXQ udified
1. Ans. (C) 4. Ans. (E)
2x2 – 7x + 28 = 14x – 26 3x2 + 3x + 79 = 87 – 7x
2x2 – 21x + 54 = 0 3x2 + 10x – 8 = 0
(2x – 9)(x – 6) = 0 (3x – 2)(x + 4) = 0
x = 4.5, 6 x = 2/3, - 4

y2 – 8y + 7 = 6y – 42 3y2 – 5y + 115 = 125 + 8y


y2 – 14y + 49 = 0 3y2 – 13y – 10 = 0
(y – 7) 2 = 0 (3y – 2)(y + 5) = 0
y=7 y = 2/3, - 5
x<y x = y or No Relation Established

2. Ans. (C) 5. Ans. (D)


12x2 + 38x + 5 = 9x – 32 3x2 + 10x + 6 = - x – 4
12x2 + 29x + 14 = 0 3x2 + 11x + 10 = 0
(3x + 2)(4x + 7) = 0 (3x + 5)(x + 2) = 0
x = - 2/3, - 7/4 x = - 5/3, - 2

12y2 – 11y = 15y = 22 6y2 + 8y + 4 = - 11 – 11y


12y2 – 26y + 4 = 0 6y2 + 19y + 15 = 0
2(6y – 1)(y – 2) = 0 (2y + 3)(3y + 5) = 0
y = 1/6, 2 y = - 5/3, - 3/2
x<y x≤y

3. Ans. (D) 6. Ans. (A)


132x2 – 23x + 1 = 0 25x2 – 30x + 8 = 0
(11x – 1)(12x – 1) = 0 (5x – 2)(5x – 4) = 0
x = 1/11, 1/12 x = 2/5, 4/5

88y2 – 19y + 1 = 0 25y2 + 110y + 21 = 0


(11y – 1)(8y – 1) = 0 (5y + 1)(5y + 21) = 0
y = 1/11, 1/8 y = - 1/5, - 21/5
x≤y x>y

https://youtu https://instagra
https ://ww
be.com/chan w.fa cebook.
m.com/aashish https://
arorasocial(?)
19
nel/UCYa4_Jr
Orf8R5Kz2uO
com/a ashis utm_medium = t.me/st
ha rorasocial copy_link
tccXQ udified
7. Ans. (E) y2 – 12y + 36 = 0
x2 – x – 6 = 0 (y – 6) 2 = 0
(x + 2)(x – 3) = 0 y=6
x = - 2, 3 x<y

y2 + 4y + 3 = 0 11. Ans. (A)


(y + 1)(y + 3) = 0 x2 – 5x + 6 = 0
y = - 1, - 3 (x – 2)(x – 3) = 0
No Relation Established x = 2, 3

8. Ans. (A) y2 + 3y + 2 = 0
48x2 – 206x + 221 = 0 (y + 2)(y + 1) = 0
(8x – 17)(6x – 13) = 0 y = - 2, - 1
x = 17/8, 13/6 x>y

15y2 – 7y – 4 = 0 12. Ans. (E)


(5y – 4)(3y + 1) = 0 10x2 – 13x + 4 = 0
y = 4/5, - 1/3 (2x – 1)(5x – 4) = 0
x>y x = 1/2, 4/5

9. Ans. (E) 20y2 – 7y – 3 = 0


x2 – 2x – 195 = 0 (5y – 3)(4y + 1) = 0
(x + 13)(x – 15) = 0 y = - 1/4, 3/5
x = - 13, 15 No Relation Established

y2 – 4y – 192 = 0 13. Ans. (E)


(y + 12)(y – 16) = 0 x2 + 2x – 224 = 0
y = - 12, 16 (x + 16)(x – 14) = 0
No Relation Established x = - 16, 14

10 Ans. (C) y2 – 30y + 221 = 0


x2 + 2x – 24 = 0 (y – 13)(y – 17) = 0
(x + 6)(x – 4) = 0 y = 13, 17
x = - 6, 4 No Relation Established

https://youtu https://instagra
https ://ww
be.com/chan w.fa cebook.
m.com/aashish https://
arorasocial(?)
20
nel/UCYa4_Jr
Orf8R5Kz2uO
com/a ashis utm_medium = t.me/st
ha rorasocial copy_link
tccXQ udified
14. Ans. (E) y2 – 2y – 24 = 0
8x2 – 14x + 3 = 0 (y + 4)(y – 6) = 0
(4x – 1)(2x – 3) = 0 y = - 4, 6
x = 1/4, 3/2 x = y No Relation Established

8y2 – 33y + 4 = 0 18. Ans. (D)


(8y – 1)(y – 4) = 0 4x2 – 7x + 3 = 0
y = 1/8, 4 (4x – 3)(x – 1) = 0
No Relation Established x = 3/4, 1

15. Ans. (D) 5y2 – 11y + 6 = 0


x2 – 23x + 132 = 0 (5y – 6)(y – 1) = 0
(x – 11)(x – 12) = 0 y = 6/5, 1
x = 11, 12 x≤y

y2 – 25y + 156 = 0 19. Ans. (C)


(y – 12)(y – 13) = 0 x2 + 10x + 24 = 0
y = 12, 13 (x + 4)(x + 6) = 0
x≤y x = - 4, - 6

16. Ans. (C) y2 – 7y + 12 = 0


32x2 – 20x + 3 = 0 (y – 3)(y – 4) = 0
(4x – 1)(8x – 3) = 0 y = 3, 4
x = 1/4, 3/8 x<y

4y2 – 17y + 18 = 0 20. Ans. (E)


(y – 2)(4y – 9) = 0 x2 + 10x + 21 = 0
y = 2, 9/4 (x + 7)(x + 3) = 0
x<y x = - 7, - 3

17. Ans. (E) y2 + 3y – 28 = 0


x2 + 7x + 12 = 0 (y + 7)(y – 4) = 0
(x + 3)(x + 4) = 0 y = - 7, 4
x = - 4, - 3 x = y or No Relation Established

https://youtu https://instagra
https ://ww
be.com/chan w.fa cebook.
m.com/aashish https://
arorasocial(?)
21
nel/UCYa4_Jr
Orf8R5Kz2uO
com/a ashis utm_medium = t.me/st
ha rorasocial copy_link
tccXQ udified
21. Ans. (A) y2 – 10y + 21 = 0
60x2 + 103x + 44 = 0 (y – 3)(y – 7) = 0
(12x + 11)(5x + 4) = 0 y = 3, 7
x = - 11/12, - 4/5 No Relation Established

30y2 + 83y + 56 = 0 25. Ans. (E)


(6y + 7)(5y + 8) = 0 6x2 – 17x + 12 = 0
y = - 7/6, - 8/5 (3x – 4)(2x – 3) = 0
x>y x = 4/3, 3/2

22. Ans. (D) 18y2 – 41y + 21 = 0


x2 – 25x + 156 = 0 (2y – 3)(9y – 7) = 0
(x – 12)(x – 13) = 0 y = 3/2, 7/9
x = 12, 13 No Relation Established

y2 – 31y + 234 = 0 26. Ans. (A)


(y – 13)(y – 18) = 0 63x2 – 85x + 28 = 0
y = 13, 18 (9x – 7)(7x – 4) = 0
x≤y x = 7/9, 4/7

23. Ans. (C) 18y2 + 17y – 15 = 0


35x2 + 38x + 8 = 0 (9y – 5)(2y + 3) = 0
(7x + 2)(5x + 4) = 0 y = - 3/2, 5/9
x = - 2/7, - 4/5 x>y

15y2 – 38y + 24 = 0 27. Ans. (A)


(5y – 6)(3y – 4) = 0 x2 – 16x + 63 = 0
y = 6/5, 4/3 (x – 7)(x – 9) = 0
x<y x = 7, 9

24. Ans. (E) y2 – 7y + 12 = 0


x2 – 7x + 10 = 0 (y – 3)(y – 4) = 0
(x – 2)(x – 5) = 0 y = 3, 4
x = 2, 5 x>y

https://youtu https://instagra
https ://ww
be.com/chan w.fa cebook.
m.com/aashish https://
arorasocial(?)
22
nel/UCYa4_Jr
Orf8R5Kz2uO
com/a ashis utm_medium = t.me/st
ha rorasocial copy_link
tccXQ udified
28. Ans. (C) 56y2 – 97y + 42 = 0
6x2 + 49x + 88 = 0 (8y – 7)(7y – 6) = 0
(3x + 8)(2x + 11) = 0 y = 7/8, 6/7
x = - 8/3, - 11/2 x>y

24y2 + 37y + 14 = 0 32. Ans. (D)


(8y + 7)(3y + 2) = 0 3x2 + 19x + 20 = 0
y = - 7/8, - 2/3 (3x + 4)(x + 5) = 0
x<y x = - 4/3, - 5

29. Ans. (D) 3y2 – 2y – 8 = 0


x2 + 4x + 3 = 0 (3y + 4)(y – 2) = 0
(x + 1)(x + 3) = 0 y = - 4/3, 2
x = - 3, - 1 x≤y

y2 – y – 2 = 0 33. Ans. (E)


(y + 1)(y – 2) = 0 20x2 – 29x – 33 = 0
y = - 1, 2 (4x + 3)(5x – 11) = 0
x≤y x = - 3/4, 11/5

30. Ans. (A) 7y2 – 22y + 16 = 0


12x2 – 97x + 195 = 0 (7y – 8)(y – 2) = 0
(4x – 15)(3x – 13) = 0 y = 8/7, 2
x = 15/4, 13/3 No Relation Established

12y2 – 83y + 143 = 0 34. Ans. (C)


(3y – 11)(4y – 13) = 0 x2 – 15x + 3 = 0
y = 11/3, 13/4 (x – 7)(x – 8) = 0
x>y x = 7, 8

31 . Ans. (A) y2 – 25y + 156 = 0


12x2 – 37x + 28 = 0 (y – 12)(y – 13) = 0
(4x – 7)(3x – 4) = 0 y = 12, 13
x = 7/4, 4/3 x<y

23
35. Ans. (B) 40y2 + 66y + 27 = 0
8x2 – 23x + 14 = 0 (10y + 9)(4y + 3) = 0
(8x – 7)(x – 2) = 0 y = - 9/10, - 3/4
x = 7/8, 2 x<y

8y2 + y – 7 = 0 39. Ans. (E)


(8y – 7)(y + 1) = 0 x2 – 3x – 40 = 0
y = 7/8, - 1 (x – 8)(x + 5) = 0
x≥y x = 8, - 5

36. Ans. (E) y2 – 16y + 63 = 0


52x2 – 87x + 36 = 0 (y – 7)(y – 9) = 0
(4x – 3)(13x – 12) = 0 y = 7, 9
x = 3/4, 12/13 No Relation Established

112y2 – 199y + 85 = 0 40. Ans. (A)


(7y – 5)(16y – 17) = 0 180x2 – 399x + 221 = 0
y = 5/7, 17/16 (12x – 13)(15x – 17) = 0
No Relation Established x = 13/12, 17/15

37. Ans. (B) 88y2 – 157y + 70 = 0


x2 + 37x + 342 = 0 (11y – 10)(8y – 7) = 0
(x + 18)(x + 19) = 0 y = 10/11, 7/8
x = - 18, - 19 x>y

y2 + 39y + 380 = 0 41. Ans. (E)


(y + 19)(y + 20) = 0 8x2 – 46x + 63 = 0
y = - 19, - 20 (4x – 9)(2x – 7) = 0
x≥y x = 9/4, 7/2

38. Ans. (C) 24y2 – 85y + 56 = 0


28x2 + 97x + 84 = 0 (3y – 8)(8y – 7) = 0
(4x + 7)(7x + 12) = 0 y = 8/3, 7/8
x = - 7/4, - 12/7 No Relation Established

https://youtu https://instagra
https ://ww
be.com/chan w.fa cebook.
m.com/aashish https://
arorasocial(?)
24
nel/UCYa4_Jr
Orf8R5Kz2uO
com/a ashis utm_medium = t.me/st
ha rorasocial copy_link
tccXQ udified
42. Ans. (A) 323y2 + 610y + 288 = 0
x2 – 38x + 217 = 0 (19y + 18)(17y + 16) = 0
(x – 7)(x – 31) = 0 y = - 18/19, - 16/17
x = 7, 31 x<y

y2 – 11y + 30 = 0 46. Ans. (C)


(y – 5)(y – 6) = 0 88x2 – 149x + 63 = 0
y = 5, 6 (8x – 7)(11x – 9) = 0
x>y x = 7/8, 9/11

43. Ans. (E) 42y2 – 163y + 156 = 0


88x2 + 7x – 30 = 0 (7y – 12)(6y – 13) = 0
(11x – 6)(8x + 5) = 0 y = 12/7, 13/6
x = 6/11, - 5/8 x<y

8y2 – 109y – 42 = 0 47. Ans. (B)


(8y + 3)(y – 14) = 0 x2 + 23x + 132 = 0
y = - 3/8, 14 (x + 11)(x + 12) = 0
No Relation Established x = - 11, - 12

44. Ans. (B) y2 + 25y + 156 = 0


4x2 – 88x + 483 = 0 (y + 12)(y + 13) = 0
(2x – 21)(2x – 23) = 0 y = - 12, - 13
x = 21/2, 23/2 x≥y

4y2 – 72y + 315 = 0 48. Ans. (E)


(2y – 21)(2y – 15) = 0 45x2 – 19x – 42 = 0
y = 21/2, 15/2 (5x – 6)(9x – 7) = 0
x≥y x = 6/5, 7/9

45. Ans. (C) 4y2 – 27y – 7 = 0


56x2 + 291x + 378 = 0 (4y + 1)(y – 7) = 0
(7x + 18)(8x + 21) = 0 y = - 1/4, 7
x = - 18/7, - 21/8 No Relation Established

https://youtu https://instagra
https ://ww
be.com/chan w.fa cebook.
m.com/aashish https://
arorasocial(?)
25
nel/UCYa4_Jr
Orf8R5Kz2uO
com/a ashis utm_medium = t.me/st
ha rorasocial copy_link
tccXQ udified
49. Ans. (C) y2 + 29y + 198 = 0
x2 – 36x + 315 = 0 (y + 11)(y + 18) = 0
(x – 15)(x – 21) = 0 y = - 11, - 18
x = 15, 21 x≥y

y2 – 48y + 572 = 0 53. Ans. (E)


(y – 22)(y – 26) = 0 8x2 – 47x + 35 = 0
y = 22, 26 (8x – 7)(x – 5) = 0
x<y x = 7/8, 5

50. Ans. (A) 5y2 – 17y + 14 = 0


77x2 – 230x + 168 = 0 (5y – 7)(y – 2) = 0
(7x – 12)(11x – 14) = 0 y = 7/5, 2
x = 12/7, 14/11 No Relation Established

12y2 + 53y + 56 = 0 54. Ans. (A)


(4y + 7)(3y + 8) = 0 x2 + 33x + 270 = 0
y = 7/4, 8/3 (x + 15)(x + 18) = 0
x>y x = - 15, - 18

51. Ans. (A) y2 + 44y + 483 = 0


24x2 + 29x + 7 = 0 (y + 21)(y + 23) = 0
(8x + 7)(3x + 1) = 0 y = - 21, - 23
x = - 7/8, - 1/3 x>y

12y2 + 31y + 20 = 0 55. Ans. (C)


(4y + 5)(3y + 4) = 0 288x2 + 610x + 323 = 0
y = - 5/4, - 4/3 (16x + 17)(18x + 19) = 0
x>y x = - 17/16, - 19/18

52. Ans. (B) 187y2 + 291y + 110 = 0


x2 + 18x + 77 = 0 (11y + 10)(17y + 11) = 0
(x + 7)(x + 11) = 0 y = - 10/11, - 11/17
x = - 7, - 11 x<y

https://youtu https://instagra
https ://ww
be.com/chan w.fa cebook.
m.com/aashish https://
arorasocial(?)
26
nel/UCYa4_Jr
Orf8R5Kz2uO
com/a ashis utm_medium = t.me/st
ha rorasocial copy_link
tccXQ udified
56. Ans. (A) y2 – 45y + 506 = 0
12x2 + 7x + 1 = 0 (y – 22)(y – 23) = 0
(3x + 1)(4x + 1) = 0 y = 22, 23
x = - 1/3, - 1/4 No Relation Established

10y2 + 23y + 12 = 0 60. Ans. (C)


(5y + 4)(2y + 3) = 0 x2 + 35x + 306 = 0
y = - 4/5, - 3/2 (x + 17)(x + 18) = 0
x>y x = - 17, - 18

57. Ans. (D) y2 – 9y + 20 = 0


x2 + 23x + 132 = 0 (y – 4)(y – 5) = 0
(x + 11)(x + 12) = 0 y = 4, 5
x = - 11, - 12 x<y

y2 + 21y + 110 = 0 61. Ans. (C)


(y + 10)(y + 11) = 0 100x2 – 189x + 80 = 0
y = - 10, - 11 (25x – 16)(4x – 5) = 0
x≤y x = 16/25, 5/4

58. Ans. (C) 36y2 – 127y + 112 = 0


56x2 – 89x + 35 = 0 (4y – 7)(9y – 16) = 0
(8x – 7)(7x – 5) = 0 y = 16/9, 7/4
x = 7/8, 5/7 x<y

8y2 – 22y + 15 = 0 62. Ans. (A)


(4y – 5)(2y – 3) = 0 4x2 – 73x + 18 = 0
y = 5/4, 3/2 (4x – 1)(x – 18) = 0
x<y x = 1/4, 18

59. Ans. (E) 45y2 + 44y – 12 = 0


x2 – 45x + 504 = 0 (9y – 2)(5y + 6) = 0
(x – 21)(x – 24) = 0 y = 2/9, - 6/5
x = 21, 24 x>y

https://youtu https://instagra
https ://ww
be.com/chan w.fa cebook.
m.com/aashish https://
arorasocial(?)
27
nel/UCYa4_Jr
Orf8R5Kz2uO
com/a ashis utm_medium = t.me/st
ha rorasocial copy_link
tccXQ udified
63 . Ans. (A) 12y2 + 5y – 2 = 0
55x2 – 29x – 12 = 0 (4y – 1)(3y + 2) = 0
(11x + 3)(5x – 4) = 0 y = 1/4, - 2/3
x = - 3/11, 4/5 x = y or No Relation

44y2 + 79y + 30 = 0 67. Ans. (E)


(11y + 6)(4y + 5) = 0 x2 – x – 182 = 0
y = - 6/11, - 5/4 (x + 13)(x – 14) = 0
x>y x = - 13, 14

64. Ans. (A) 3y2 + 38y + 24 = 0


x2 – 25x + 150 = 0 (y + 12)(3y + 2) = 0
(x – 10)(x – 15) = 0 y = - 12, - 2/3
x = 10, 15 No Relation Established

y2 + 25y + 150 = 0 68 . Ans. (A)


(y + 10)(y + 15) = 0 x2 + 7x + 6 = 0
y = - 10, - 15 (x + 1)(x + 6) = 0
x>y x = - 1, - 6

65. Ans. (E) y2 + 16y + 63 = 0


40x2 – 59x + 21 = 0 (y + 7)(y + 9) = 0
(5x – 3)(8x – 7) = 0 y = - 7, - 9
x = 3/5, 7/8 x>y

36y2 – 43y + 12 = 0 69. Ans. (D)


(4y – 3)(9y – 4) = 0 9x2 – 15x + 4 = 0
y = 3/4, 4/9 (3x – 1)(3x – 4) = 0
No Relation Established x = 1/3, 4/3

66. Ans. (E) 3y2 – 31y + 36 = 0


24x2 + 19x + 2 = 0 (3y – 4)(y – 9) = 0
(8x + 1)(3x + 2) = 0 y = 4/3, 9
x = - 1/8, - 2/3 x≤y

https://youtu https://instagra
https ://ww
be.com/chan w.fa cebook.
m.com/aashish https://
arorasocial(?)
28
nel/UCYa4_Jr
Orf8R5Kz2uO
com/a ashis utm_medium = t.me/st
ha rorasocial copy_link
tccXQ udified
70. Ans. (E) y2 – 14y + 48 = 0
15x2 + 59x – 80 = 0 (y – 6)(y – 8) = 0
(15x – 16)(x + 5) = 0 y = 7, 8
x = 16/15, - 5 x<y

16y2 + 159y + 135 = 0 74. Ans. (E)


(16y + 15)(y + 9) = 0 32x2 – 12x + 1 = 0
y = - 15/16, - 9 (8x – 1)(4x – 1) = 0
No Relation Established x = 1/8, 1/4

71. Ans. (C) 64y2 – 20y + 1 = 0


2x2 + 5x + 2 = 0 (16y – 1)(4y – 1) = 0
(2x + 1)(x + 2) = 0 y = 1/16, 1/4
x = - 1/2, - 2 x = y or No Relation Established

2y2 – 5y + 2 = 0 75. Ans. (D)


(2y – 1)(y – 2) = 0 x2 – 2x – 99 = 0
y = 1/2, 2 (x – 11)(x + 9) = 0
x<y x = 11, - 9

72. Ans. (B) y2 – 24y + 143 = 0


x2 + 2x – 224 = 0 (y – 11)(y – 13) = 0
(x + 16)(x – 14) = 0 y = 11, 13
x = - 16, 14 x≤y

y2 + 40y + 384 = 0 76. Ans. (A)


(y + 16)(y + 24) = 0 x2 – 25x + 156 = 0
y = - 16, - 24 (x – 12)(x – 13) = 0
x≥y x = 12, 13

73. Ans. (C) y2 + 23y + 132 = 0


x2 + 14x + 45 = 0 (y + 11)(y + 12) = 0
(x + 5)(x + 9) = 0 y = - 11, - 12
x = - 5, - 9 x>y

https://youtu https://instagra
https ://ww
be.com/chan w.fa cebook.
m.com/aashish https://
arorasocial(?)
29
nel/UCYa4_Jr
Orf8R5Kz2uO
com/a ashis utm_medium = t.me/st
ha rorasocial copy_link
tccXQ udified
77. Ans. (B) 3y2 – 38y + 24 = 0
5x2 – 26x – 24 = 0 (3y – 2)(y – 12) = 0
(5x + 4)(x – 6) = 0 y = 2/3, 12
x = - 4/5, 6 x = y or No Relation Established

25y2 + 60y + 32 = 0 81. Ans. (D)


(5y + 4)(5y + 8) = 0 x2 + 15x + 54 = 0
y = - 4/5, - 8/5 (x + 9)(x + 6) = 0
x≥y x = - 9, - 6

78. Ans. (C) y2 + 9y + 18 = 0


4x2 + 73x + 144 = 0 (y + 3)(y + 6) = 0
(4x + 9)(x + 16) = 0 y = - 6, - 3
x = - 9/4, - 16 x≤y

3y2 – 52y + 64 = 0 82. Ans. (C)


(3y – 4)(y – 16) = 0 56x2 + 113x + 56 = 0
y = 4/3, 16 (8x + 7)(7x + 8) = 0
x<y x = - 7/8, - 8/7

79. Ans. (B) y2 – 13y + 36 = 0


4x2 – 33x + 8 = 0 (y – 4)(y – 9) = 0
(4x – 1)(x – 8) = 0 y = 4, 9
x = 1/4, 8 x<y

16y2 – 1 = 0 83. Ans. (C)


y2 = 1/16 5x2 + 31x + 30 = 0
y = ± 1/4 (5x + 6)(x + 5) = 0
x≥y x = - 6/5, - 5

80. Ans. (E) 5y2 + 9y + 4 = 0


3x2 – 47x + 30 = 0 (5y + 4)(y + 1) = 0
(3x – 2)(x – 15) = 0 y = - 4/5, - 1
x = 2/3, 15 x<y

https://youtu https://instagra
https ://ww
be.com/chan w.fa cebook.
m.com/aashish https://
arorasocial(?)
30
nel/UCYa4_Jr
Orf8R5Kz2uO
com/a ashis utm_medium = t.me/st
ha rorasocial copy_link
tccXQ udified
84. Ans. (C) 4y2 – 17y + 4 = 0
x2 + 15x + 56 = 0 (4y – 1)(y – 4) = 0
(x + 7)(x + 8) = 0 y = 1/4, 4
X = - 7, - 8 x≤y

72y2 – 43y + 6 = 0 88. Ans. (A)


(9y – 2)(8y – 3) = 0 x2 – 30x + 209 = 0
y = 2/9, 3/8 (x – 11)(x – 19) = 0
x<y x = 11, 19

85. Ans. (E) 3y2 + 17y – 176 = 0


7x2 – 20x – 32 = 0 (3y – 16)(y + 11) = 0
(7x + 8)(x – 4) = 0 y = 16/3, - 11
X = - 8/7, 4 x>y

8y2 – 17y – 21 = 0 89. Ans. (D)


(8y + 7)(y – 3) = 0 25x2 – 35x + 12 = 0
y = - 7/8, 3 (5x – 4)(5x – 3) = 0
No Relation Established x = 4/5, 3/5

86. Ans. (C) 25y2 – 65y + 36 = 0


x2 + 11x + 30 = 0 (5y – 4)(5y – 9) = 0
(x + 5)(x + 6) = 0 y = 4/5, 9/5
x = - 5, - 6 x≤y

8y2 – 14y + 3 = 0 90. Ans. (E)


(4y – 1)(2y – 3) = 0 x2 – x – 56 = 0
y = 1/4, 3/2 (x – 8)(x + 7) = 0
x<y x = - 7, 8

87. Ans. (D) 8y2 + 69y – 27 = 0


4x2 + 63x – 16 = 0 (8y – 3)(y + 9) = 0
(4x – 1)(x + 16) = 0 y = 3/8, - 9
x = 1/4, - 16 No Relation Established

https://youtu https://instagra
https ://ww
be.com/chan w.fa cebook.
m.com/aashish https://
arorasocial(?)
31
nel/UCYa4_Jr
Orf8R5Kz2uO
com/a ashis utm_medium = t.me/st
ha rorasocial copy_link
tccXQ udified
91. Ans. (C) 4y2 + 35y – 99 = 0
3x2 + 14x + 15 = 0 (4y – 9)(y + 11) = 0
(3x + 5)(x + 3) = 0 y = 9/4, - 11
x = - 5/3, - 3 x>y

3y2 + 7y + 4 = 0 95. Ans. (B)


(3y + 4)(y + 1) = 0 x2 + 12x + 35 = 0
y = - 4/3, - 1 (x + 5)(x + 7) = 0
x<y x = - 5, - 7

92. Ans. (C) y2 + 16y + 63 = 0


x2 + 18x + 77 = 0 (y + 7)(y + 9) = 0
(x + 11)(x + 7) = 0 y = - 7, - 9
x = - 11, - 7 x≥y

5y2 – 52y + 20 = 0 96. Ans. (C)


(5y – 2)(y – 10) = 0 10x2 + 29x + 21 = 0
y = 2/5, 10 (5x + 7)(2x + 3) = 0
x<y x = - 7/5, - 3/2

93. Ans. (E) 77y2 + 136y + 60 = 0


x2 – 24x + 143 = 0 (11y + 10)(7y + 6) = 0
(x – 11)(x – 13) = 0 y = - 10/11, - 6/7
x = 11, 13 x<y

2y2 – 49y + 299 = 0 97. Ans. (D)


(2y – 23)(y – 13) = 0 x2 + 32x + 255 = 0
y = 23/2, 13 (x + 15)(x + 17) = 0
x = y or No Relation Established x = - 15, - 17

94. Ans. (A) y2 + 28y + 195 = 0


3x2 – 61x + 240 = 0 (y + 15)(y + 13) = 0
(x – 15)(3x – 16) = 0 y = - 15, - 13
x = 15, 16/3 x≤y

https://youtu https://instagra
https ://ww
be.com/chan w.fa cebook.
m.com/aashish https://
arorasocial(?)
32
nel/UCYa4_Jr
Orf8R5Kz2uO
com/a ashis utm_medium = t.me/st
ha rorasocial copy_link
tccXQ udified
98. Ans. (E) 20y2 – 53y + 35 = 0
8x2 – 71x + 56 = 0 (5y – 7)(4y – 5) = 0
(8x – 7)(x – 8) = 0 y = 7/5, 5/4
x = 7/8, 8 x>y

8y2 – 39y + 28 = 0 102. Ans. (B)


(8y – 7)(y – 4) = 0 5x2 – 49x + 36 = 0
y = 7/8, 4 (5x – 4)(x – 9) = 0
x = y or No Relation Established x = 4/5, 9

99. Ans. (D) 25y2 – 35y + 12 = 0


x2 + 19x + 88 = 0 (5y – 4)(5y – 3) = 0
(x + 8)(x + 11) = 0 y = 4/5, 3/5
x = - 8, - 11 x≥y

y2 + 15y + 56 = 0 103. Ans. (D)


(y + 7)(y + 8) = 0 x2 + 23x + 132 = 0
y = - 7, - 8 (x + 11)(x + 12) = 0
x≤y x = - 11, - 12

100. Ans. (A) y2 + 21y + 110 = 0


12x2 – 127x + 323 = 0 (y + 10)(y + 11) = 0
(4x – 17)(3x – 19) = 0 y = - 10, - 11
x = 17/4, 19/3 x≤y

20y2 – 123y + 180 = 0 104. Ans. (A)


(4y – 15)(5y – 12) = 0 56x2 – 181x + 143 = 0
y = 15/4, 12/5 (8x – 11)(7x – 13) = 0
X>y x = 11/8, 13/7

101. Ans. (A) 5y2 + 14y – 24 = 0


9x2 – 45x + 56 = 0 (5y – 6)(y + 4) = 0
(3x – 8)(3x – 7) = 0 y = 6/5, - 4
x = 8/3, 7/3 x>y

https://youtu https://instagra
https ://ww
be.com/chan w.fa cebook.
m.com/aashish https://
arorasocial(?)
33
nel/UCYa4_Jr
Orf8R5Kz2uO
com/a ashis utm_medium = t.me/st
ha rorasocial copy_link
tccXQ udified
105. Ans. (C) 4y2 + 33y – 27 = 0
15x2 – 149x + 342 = 0 (4y – 3)(y + 9) = 0
(5x – 18)(3x – 19) = 0 y = 3/4, - 9
x = 18/5, 19/3 No Relation Established

4y2 – 96y + 567 = 0 109. Ans. (B)


(2y – 21)(2y – 27) = 0 x2 – 12√2x + 70 = 0
y = 21/2, 27/2 (x – 5√2)(x – 7√2) = 0
x<y x = 5√2, 7√2

106. Ans. (D) y2 – 9√2y + 40 = 0


x2 – 15x + 56 = 0 (y – 4√2)(y – 5√2) = 0
(x – 8)(x – 7) = 0 y = 4√2, 5√2
x = 8, 7 x≥y

y2 – 16y + 63 = 0 110. Ans. (C)


(y – 7)(y – 9) = 0 x2 + 63x + 992 = 0
y = 7, 9 (x + 31)(x + 32) = 0
x≤y x = - 31, - 32

107. Ans. (C) y2 + 55y + 756 = 0


44x2 – 69x + 27 = 0 (y + 28)(y + 27) = 0
(11x – 9)(4x – 3) = 0 y = - 27, - 28
x = 9/11, 3/4 x<y

10y2 – 51y + 54 = 0 111. Ans. (B)


(2y – 3)(5y – 18) = 0 x2 – 36x + 323 = 0
y = 3/2, 18/5 (x – 17)(x – 19) = 0
x<y x = 17, 19

108. Ans. (E) y2 – 33y + 272 = 0


2x2 + 13x – 7 = 0 (y – 16)(y – 17) = 0
(2x – 1)(x + 7) = 0 y = 16, 17
x = 1/2, - 7 x≥y

https://youtu https://instagra
https ://ww
be.com/chan w.fa cebook.
m.com/aashish https://
arorasocial(?)
34
nel/UCYa4_Jr
Orf8R5Kz2uO
com/a ashis utm_medium = t.me/st
ha rorasocial copy_link
tccXQ udified
112. Ans. (A) y2 – 24y + 128 = 0
20x2 – 63x + 49 = 0 (y – 8)(y – 16) = 0
(4x – 7)(5x – 7) = 0 y = 8, 16
x = 7/4, 7/5 No Relation Established

77y2 – 108y + 36 = 0 116. Ans. (A)


(7y – 6)(11y – 6) = 0 72x2 – 161x + 90 = 0
y = 6/7, 6/11 (8x – 9)(9x – 10) = 0
x>y x = 9/8, 10/9

113. Ans. (A) 48y2 – 82y + 35 = 0


30x2 + 49x + 20 = 0 (8y – 7)(6y – 5) = 0
(5x + 4)(6x + 5) = 0 y = 7/8, 5/6
x = - 4/5, - 5/6 x>y

20y2 + 49y + 30 = 0 117. Ans. (C)


(4y + 5)(5y + 6) = 0 6x2 – 79x + 255 = 0
y = - 5/4, - 6/5 (3x – 17)(2x – 15) = 0
x>y x = 17/3, 15/2

114. Ans. (E) 9y2 – 180y + 896 = 0


x2 + 44x + 483 = 0 (3y – 28)(3y – 32) = 0
(x + 21)(x + 23) = 0 y = 28/3, 32/3
x = - 21, - 23 x<y

y2 + 55y + 744 = 0 118. Ans. (D)


(y + 21)(y + 24) = 0 x2 + 30x + 221 = 0
y = - 21, - 24 (x + 13)(x + 17) = 0
x = y or No Relation Established x = - 13, - 17

115. Ans. (E) y2 + 25y + 156 = 0


x2 – 22x + 105 = 0 (y + 12)(y + 13) = 0
(x – 7)(x – 15) = 0 y = - 12, - 13
x = 7, 15 x≤y

https://youtu https://instagra
https ://ww
be.com/chan w.fa cebook.
m.com/aashish https://
arorasocial(?)
35
nel/UCYa4_Jr
Orf8R5Kz2uO
com/a ashis utm_medium = t.me/st
ha rorasocial copy_link
tccXQ udified
119. Ans. (E) 15y2 + 41y + 28 = 0
9x2 – 52x + 35 = 0 (5y + 7)(3y + 4) = 0
(9x – 7)(x – 5) = 0 y = - 7/5, - 4/3
x = 7/9, 5 x<y

7y2 – 61y + 40 = 0 123. Ans. (A)


(7y – 5)(y – 8) = 0 10x2 – 61x + 72 = 0
y = 5/7, 8 (5x – 8)(2x – 9) = 0
No Relation Established x = 8/5, 9/2

120. Ans. (D) 48y2 – 82y + 35 = 0


x2 – 40x + 399 = 0 (8y – 7)(6y – 5) = 0
(x – 19)(x – 21) = 0 y = 7/8, 5/6
x = 19, 21 x>y

y2 – 44y + 483 = 0 124. Ans. (B)


(y – 21)(y – 23) = 0 x2 + 35x + 306 = 0
y = 21, 23 (x + 17)(x + 18) = 0
x≤y x = - 17, - 18

121. Ans. (D) y2 + 38y + 360 = 0


x2 – 42x + 437 = 0 (y + 18)(y + 20) = 0
(x – 19)(x – 23) = 0 y = - 18, - 20
x = 19, 23 x≥y

y2 – 47y + 552 = 0 125. Ans. (A)


(y – 23)(y – 24) = 0 56x2 – 89x + 35 = 0
y = 23, 24 (7x – 5)(8x – 7) = 0
x≤y x = 5/7, 7/8

122. Ans. (C) 35y2 – 29y + 6 = 0


6x2 + 67x + 165 = 0 (7y – 3)(5y – 2) = 0
(3x + 11)(2x + 15) = 0 y = 3/7, 2/5
x = - 11/3, - 15/2 x>y

https://youtu https://instagra
https ://ww
be.com/chan w.fa cebook.
m.com/aashish https://
arorasocial(?)
36
nel/UCYa4_Jr
Orf8R5Kz2uO
com/a ashis utm_medium = t.me/st
ha rorasocial copy_link
tccXQ udified
126. Ans. (C) 3y2 – 16y + 21 = 0
x2 + 10x + 24 = 0 (3y – 7)(y – 3) = 0
(x + 4)(x + 6) = 0 y = 7/3, 3
x = - 4, - 6 No Relation Established

y2 – 15y + 56 = 0 130. Ans. (C)


(y – 7)(y – 8) = 0 x2 + 10x + 24 = 0
y = 7, 8 (x + 4)(x + 6) = 0
x<y x = - 4, - 6

127. Ans. (A) y2 + 5y + 6 = 0


8x2 + 26x + 21 = 0 (y + 3)(y + 2) = 0
(2x + 3)(4x + 7) = 0 y = - 2, - 3
x = - 3/2, - 7/4 x<y

8y2 + 38y + 45 = 0 131. Ans. (E)


(4y + 9)(2y + 5) = 0 5x2 + 13x – 28 = 0
y = - 9/4, - 5/2 (5x – 7)(x + 4) = 0
x>y x = 7/5, - 4

128. Ans. (A) 8y2 – 51y + 18 = 0


10x2 – 27x + 18 = 0 (8y – 3)(y – 6) = 0
(5x – 6)(2x – 3) = 0 y = 3/8, 6
x = 6/5, 3/2 No Relation Established

32y2 – 20y + 3 = 0 132. Ans. (E)


(8y – 3)(4y – 1) = 0 20x2 – 79x + 66 = 0
y = 3/8, 1/4 (5x – 6)(4x – 11) = 0
x>y x = 6/5, 11/4

129. Ans. (E) y2 – 2y – 48 = 0


4x2 – 23x + 28 = 0 (y + 6)(y – 8) = 0
(4x – 7)(x – 4) = 0 y = - 6, 8
x = 7/4, 4 No Relation Established

https://youtu https://instagra
https ://ww
be.com/chan w.fa cebook.
m.com/aashish https://
arorasocial(?)
37
nel/UCYa4_Jr
Orf8R5Kz2uO
com/a ashis utm_medium = t.me/st
ha rorasocial copy_link
tccXQ udified
133. Ans. (E)
22x2 – 23x + 6 = 0
(11x – 6)(2x – 1) = 0
x = 6/11, 1/2

63y2 – 100y + 32 = 0
(9y – 4)(7y – 8) = 0
y = 4/9, 8/7
No Relation Established

134. Ans. (B)


7x2 – 101x + 42 = 0
(7x – 3)(x – 14) = 0
x = 3/7, 14

49y2 – 35y + 6 = 0
(7y – 3)(7y – 2) = 0
y = 3/7, 2/7
x≥y

135. Ans. (C)


5x2 – 34x + 24 = 0
(5x – 4)(x – 6) = 0
x = 4/5, 6

y2 – 19y + 88 = 0
(y – 8)(y – 11) = 0
y = 8, 11
x<y

https://youtu https://instagra
https ://ww
be.com/chan w.fa cebook.
m.com/aashish https://
arorasocial(?)
38
nel/UCYa4_Jr
Orf8R5Kz2uO
com/a ashis utm_medium = t.me/st
ha rorasocial copy_link
tccXQ udified
https://t.mhttps/t.me
e/studified/studified

https://instagram.com/aashisharora
social?utm_medium = copy_link

https://youtube.com/channel
/UCYa4_JrOrf8R5Kz2uOtccXQ

https://www.facebook.
com/aashisharorasocial

https://youtu https://instagra
https ://ww
be.com/chan w.fa cebook.
m.com/aashish https://
arorasocial(?)
39
nel/UCYa4_Jr
Orf8R5Kz2uO
com/a ashis utm_medium = t.me/st
ha rorasocial copy_link
tccXQ udified
In each of these questions a number is missing in the series. Find out the
missing number.
इनभें से प्रत्मे क प्रश्न श्ख
रॊ रा भें एक सॊख्मा रुप्त है, रुप्त सॊख्मा ऻात कीजजए।

1) 25, 27, 39, 75, 155, ?


(A) 345 (B) 295 (C) 305 (D) 325 (E) None of these

2) 11, 22, 77, 396, ?, 25058


(A) 2726 (B) 2748 (C) 2783 (D) 2756 (E) None of these

3) 168, 211, 233, 256, 292, ?


(A) 376 (B) 366 (C) 346 (D) 356 (E) None of these

4) 12, 42, 264, 3192, ?, 3679584


(A) 76826 (B) 76748 (C) 76428 (D) 76656 (E) None of these

5) 69, 88, 114, 152, 205, ?, 358


(A) 274 (B) 276 (C) 278 (D) 272 (E) None of these

6) 485, 496, 518, 551, ?, 650


(A) 203 (B) 205 (C) 207 (D) 209 (E) None of these

7) 1, 2, 5, 16, 59, ?
(A) 242 (B) 230 (C) 228 (D) 225 (E) 220

8) 2670, ?, 84, 18, 4, 1


(A) 440 (B) 240 (C) 520 (D) 560 (E) 610

9) 17, 36, 93, ?, 321, 492


(A) 132 (B) 168 (C) 188 (D) 172 (E) 124

10) 89, 91, 97, 109, ?, 159


(A) 128 (B) 129 (C) 118 (D) 124 (E) 139

11) 64, 76, ?, 107, 128, 154


(A) 100 (B) 85 (C) 90 (D) 104 (E) None of these

https://youtu https://instagra
https ://ww
be.com/chan w.fa cebook.
m.com/aashish https://
arorasocial(?)
40
nel/UCYa4_Jr
Orf8R5Kz2uO
com/a ashis utm_medium = t.me/st
ha rorasocial copy_link
tccXQ udified
12) 101, 105, 114, 139, ?, 309
(A) 360 (B) 240 (C) 160 (D) 188 (E) None of these

13) ?, 93, 281, 1127, 5639, 33839


(A) 45 (B) 46 (C) 49 (D) 64 (E) None of these

14) 16, 20, ?, 9, -116, -80


(A) 7 (B) - 7 (C) 27 (D) 45 (E) None of these

15) 114, 115, 232, 699, ?, 14005


(A) 2800 (B) 1780 (C) 2350 (D) 2300 (E) None of these

16) 36, 37, ?, 243, 988, 4965


(A) 78 (B) 74 (C) 68 (D) 44 (E) None of these

17) ?, 21, 42, 105, 315, 1102.5


(A) 16 (B) 10.5 (C) 7 (D) 14 (E) None of these

18) 360, 288, 168, 120, ?


(A) 81 (B) 48 (C) 35 (D) 63 (E) None of these

19) 12, 24, 44, ?, 116, 172


(A) 74 (B) 88 (C) 78 (D) 56 (E) None of these

20) 114, ?, 119, 135, 199, 455


(A) 111 (B) 57 (C) 115 (D) 118 (E) None of these

21) 43, ?, 73, 91, 111, 133


(A) 87 (B) 51 (C) 57 (D) 64 (E) None of these

22) 24, 30, ?, 39, 51, 57


(A) 38 (B) 36 (C) 33 (D) 44 (E) None of these

23) 2, 12, 30, 58, ?, 152


(A) 98 (B) 85 (C) 74 (D) 38 (E) None of these

24) ?, 36, 90, 270, 945, 3780


(A) 11 (B) 18 (C) 23 (D) 24 (E) None of these

25) 144, 72, 216, 54, ?, 45


(A) 111 (B) 180 (C) 150 (D) 270 (E) None of these

https://youtu https://instagra
https ://ww
be.com/chan w.fa cebook.
m.com/aashish https://
arorasocial(?)
41
nel/UCYa4_Jr
Orf8R5Kz2uO
com/a ashis utm_medium = t.me/st
ha rorasocial copy_link
tccXQ udified
26) 8, 45, 184, ?, 1112, 1113
(A) 555 (B) 455 (C) 655 (D) 965 (E) None of these

27) 45, ?, 73, 108, 157, 220


(A) 55 (B) 62 (C) 52 (D) 56 (E) None of these

28) 108, 112, ?, 146, 195, 316


(A) 115 (B) 118 (C) 121 (D) 124 (E) None of these

29) ?, 130, 60, 30, 6, 2


(A) 150 (B) 170 (C) 190 (D) 210 (E) None of these

30) 9, 27, 72, ?, 336, 560


(A) 144 (B) 168 (C) 178 (D) 184 (E) None of these

31) 22.5, 27.5, ?, 60.5, 92.5, 137.5


(A) 39.5 (B) 37.5 (C) 38.5 (D) 40.5 (E) None of these

32) 12, 39, 120, 363, 1092, ?


(A) 3269 (B) 3379 (C) 2279 (D) 3279 (E) None of these

33) 88, 188, 44, ?, -16, 308


(A) 240 (B) 245 (C) 266 (D) 215 (E) None of these

34) 19, 28, ?, 121, 247, 464


(A) 51 (B) 56 (C) 84 (D) 32 (E) None of these

35) 240, 342, 380, ?, 552


(A) 511 (B) 522 (C) 506 (D) 544 (E) None of these

36) 36, 18, ?, 27, 54, 135


(A) 18 (B) 22 (C) 33 (D) 24 (E) None of these

37) 121, 122, 130, 157, 221, ?


(A) 211 (B) 222 (C) 233 (D) 246 (E) None of these

38) 115, 140, ?, 172, 51, 220


(A) 81 (B) 92 (C) 91 (D) 94 (E) None of these

39) 3, 14, 39, ?, 155, 258


(A) 88 (B) 84 (C) 80 (D) 82 (E) None of these

https://youtu https://instagra
https ://ww
be.com/chan w.fa cebook.
m.com/aashish https://
arorasocial(?)
42
nel/UCYa4_Jr
Orf8R5Kz2uO
com/a ashis utm_medium = t.me/st
ha rorasocial copy_link
tccXQ udified
40) 133, 157, ?, 211, 241, 273
(A) 173 (B) 183 (C) 133 (D) 143 (E) None of these

41) 5, 5, ?, 180, 2880, 72000


(A) 18 (B) 20 (C) 30 (D) 15 (E) None of these

42) 17, 46, 105, 224, ?, 942


(A) 463 (B) 453 (C) 473 (D) 483 (E) None of these

43) 140, ?, 147, 157, 174, 200


(A) 111 (B) 122 (C) 133 (D) 142 (E) None of these

44) ?, 1017, 1044, 980, 1105, 889


(A) 1125 (B) 1250 (C) 1025 (D) 1425 (E) None of these

45) 45, 135, ?, 840, 849, 9339


(A) 130 (B) 140 (C) 150 (D) 160 (E) None of these

46) 1610, 1614, ?, 1630, 1644, 1666


(A) 1520 (B) 1620 (C) 1460 (D) 1860 (E) None of these

47) 186, 301, 456, ?, 910


(A) 657 (B) 658 (C) 659 (D) 660 (E) None of these

48) 12, 14, ?, 97, 393, 1971


(A) 35 (B) 32 (C) 33 (D) 31 (E) None of these

49) ?, 25, 52, 107, 218, 441


(A) 11 (B) 22 (C) 12 (D) 14 (E) None of these

50) 16, 24, 60, 210, 945, ?


(A) 4155.5 (B) 5197.5 (C) 3254.5 (D) 2755.5 (E) None of these

51) 17, 16, 22, ?, 109, 232


(A) 33 (B) 47 (C) 28 (D) 30 (E) None of these

52) 7, 56, ?, 2352, 11760, 47040


(A) 392 (B) 422 (C) 252 (D) 382 (E) None of these

53) ?, 168, 95, 48, 21, 8


(A) 321 (B) 324 (C) 333 (D) 273 (E) None of these

https://youtu https://instagra
https ://ww
be.com/chan w.fa cebook.
m.com/aashish https://
arorasocial(?)
43
nel/UCYa4_Jr
Orf8R5Kz2uO
com/a ashis utm_medium = t.me/st
ha rorasocial copy_link
tccXQ udified
54) 185, 1146, 122, 1211, 55, ?
(A) 1280 (B) 1540 (C) 1458 (D) 1244 (E) None of these

55) 15, 47, 144, 437, ?, 3965


(A) 1337 (B) 1352 (C) 1318 (D) 1348 (E) None of these

56) 775, ?, 289, 208, 235, 226


(A) 46 (B) 685 (C) 345 (D) 258 (E) None of these

57) ?, 21, 33, 69, 149, 299


(A) 6 (B) 15 (C) 19 (D) 16 (E) None of these

58) 172, 297, 640, ?, 2700, 4897


(A) 1256 (B) 1254 (C) 1369 (D) 1254 (E) None of these

59) 17, 36, 75, 155, 317, ?


(A) 545 (B) 475 (C) 785 (D) 645 (E) None of these

60) ?, 65, 26, 15.6, 12.48, 12.48


(A) 325 (B) 515 (C) 345 (D) 275 (E) None of these

61) 35, 315, ?, 11025, 33075, 33075


(A) 175 (B) 2205 (C) 3045 (D) 1255 (E) None of these

62) 19, ?, -76, 2325, - 175, 2426


(A) 2228 (B) 2453 (C) 2473 (D) 2483 (E) None of these

63) 8, 20, 70, 315, 1732.5, ?


(A) 11188.25 (B) 12245.25 (C) 11254.25 (D) 11261.25 (E) None of these

64) 17, ?, 22, 31, 47, 72


(A) 18 (B) 19 (C) 20 (D) 21 (E) None of these

65) ?, 43, 134, 408, 1231, 3701


(A) 15 (B) 14 (C) 13 (D) 12 (E) None of these

66) 26, 27.5, 31, ?, 44, 53.5


(A) 36.25 (B) 33.5 (C) 36.5 (D) 40.5 (E) None of these

67) 12, ?, 18, 26, 58, 298


(A) 16 (B) 10.5 (C) 7 (D) 14 (E) None of these

https://youtu https://instagra
https ://ww
be.com/chan w.fa cebook.
m.com/aashish https://
arorasocial(?)
44
nel/UCYa4_Jr
Orf8R5Kz2uO
com/a ashis utm_medium = t.me/st
ha rorasocial copy_link
tccXQ udified
68) 244, 240, ?, 217, 198, 174
(A) 211 (B) 231 (C) 225 (D) 218 (E) None of these

69) 16, 28, 40, ?, 64, 76


(A) 52 (B) 58 (C) 78 (D) 56 (E) None of these

70) 21, 32, 53, 85, 138, ?


(A) 181 (B) 248 (C) 235 (D) 223 (E) None of these

71) 28, ?, 118, 238, 478, 958


(A) 87 (B) 58 (C) 57 (D) 64 (E) None of these

72) 36, 54, 74, 102, ?, 146


(A) 138 (B) 136 (C) 133 (D) 122 (E) None of these

73) 6, 6, 12, ?, 96, 216


(A) 12 (B) 24 (C) 34 (D) 36 (E) None of these

74) 9, 27, 45, ?, 81, 99


(A) 63 (B) 68 (C) 58 (D) 74 (E) None of these

75) 143, ?, 195, 226, 255, 290


(A) 111 (B) 180 (C) 170 (D) 270 (E) None of these

76) 16, 21, ?, 48, 74, 111


(A) 37 (B) 31 (C) 27 (D) 34 (E) None of these

77) 50, 100, 25, ?, 12.5, 400


(A) 175 (B) 150 (C) 200 (D) 250 (E) None of these

78) 16, ?, 28, 40, 68, 108


(A) 12 (B) 24 (C) 18 (D) 10 (E) None of these

79) 381, 421, 463, 507, ?, 601


(A) 511 (B) 553 (C) 523 (D) 568 (E) None of these

80) 12, 24, ?, 360, 2520, 27720


(A) 48 (B) 36 (C) 72 (D) 84 (E) None of these

81) 91, 73, ?, 43, 31, 21


(A) 65 (B) 57 (C) 59 (D) 40 (E) None of these

https://youtu https://instagra
https ://ww
be.com/chan w.fa cebook.
m.com/aashish https://
arorasocial(?)
45
nel/UCYa4_Jr
Orf8R5Kz2uO
com/a ashis utm_medium = t.me/st
ha rorasocial copy_link
tccXQ udified
82) ?, 19, 23.5, 30.5, 41, 56
(A) 16 (B) 10.5 (C) 7 (D) 14 (E) None of these

83) 96, 12, 144, 18, ?, 27


(A) 18 (B) 54 (C) 235 (D) 216 (E) None of these

84) 214, ?, 179, 54, -289, -1620


(A) 196 (B) 343 (C) 206 (D) 256 (E) None of these

85) 19, 40, 63, 88, 115, ?


(A) 161 (B) 144 (C) 205 (D) 196 (E) None of these

86) ?, 290, 323, 362, 399, 442


(A) 238 (B) 250 (C) 267 (D) 255 (E) None of these

87) 24, 36, 126, 693, 5197.5, ?


(A) 49376.25 (B)49376.5 (C)49736.25 (D) 49673.5 (E) None of these

88) 108, 162, 243, 364.5, ?, 820.125


(A) 564.25 (B) 546.75 (C) 546.25 (D) 546.125 (E) None of these

89) 16, 32, 24, ?, 26, 27


(A) 63 (B) 68 (C) 28 (D) 74 (E) None of these

90) 19, ?, 18, 323, 17, 288


(A) 380 (B) 180 (C) 360 (D) 604 (E) None of these

91) 991, 1000, ?, 1243, - 53, 5131


(A) 1100 (B) 964 (C) 850 (D) 819 (E) None of these

92) 36, 39, 45, 46, ?, 52


(A) 45 (B) 48 (C) 44 (D) 40 (E) None of these

93) ?, 120, 840, 84, 1092, 68.25


(A) 240 (B) 600 (C) 480 (D) 900 (E) None of these

94) 30, 33, 39, ?, 57, 69


(A) 44 (B) 45 (C) 51 (D) 46 (E) None of these

95) ?, 6859, 20, 8000, 21, 9261


(A) 19 (B) 17 (C) 33 (D) 83 (E) None of these

https://youtu https://instagra
https ://ww
be.com/chan w.fa cebook.
m.com/aashish https://
arorasocial(?)
46
nel/UCYa4_Jr
Orf8R5Kz2uO
com/a ashis utm_medium = t.me/st
ha rorasocial copy_link
tccXQ udified
96) 300, ?, 299, 363, 338, 554
(A) 350 (B) 309 (C) 308 (D) 325 (E) None of these

97) ?, 38, 114, 570, 3990, 43890


(A) 19 (B) 17 (C) 33 (D) 83 (E) None of these

98) 642, 654, 669, ?, 705, 717


(A) 690 (B) 700 (C) 699 (D) 689 (E) None of these

99) ?, 68, 92, 140, 236, 428


(A) 44 (B) 45 (C) 48 (D) 56 (E) None of these

100) 72, ?, 110, 132, 156, 182


(A) 99 (B) 80 (C) 85 (D) 90 (E) None of these

101) 18480, ?, 240, 48, 16, 8


(A) 1680 (B) 1560 (C) 480 (D) 1440 (E) None of these

102) ?, 509, 950, 1479, 2104, 2833


(A) 149 (B) 154 (C) 148 (D) 155 (E) None of these

103) 38, 88, 189, 392, 799, ?


(A) 1548 (B) 1654 (C) 1545 (D) 1614 (E) None of these

104) 18, 19, ?, 62, 434, 443


(A) 38 (B) 57 (C) 76 (D) 26 (E) None of these

105) 16, 40, 200, ?, 15000, 187500


(A) 1500 (B) 1400 (C) 1300 (D) 1200 (E) None of these

106) 80640, ?, 1440, 240, 48, 12


(A) 16800 (B) 15600 (C) 10080 (D) 14400 (E) None of these

107) 15, 65, 270, 1095, 4400, ?


(A) 16625 (B) 17545 (C) 15455 (D) 17625 (E) None of these

108) 17, 34, 102, 510, ?, 39270


(A) 3577 (B) 3570 (C) 3587 (D) 3502 (E) None of these

109) 115, 1139, 2228, ?, 4609, 5905


(A) 3550 (B) 3472 (C) 3578 (D) 3384 (E) None of these

https://youtu https://instagra
https ://ww
be.com/chan w.fa cebook.
m.com/aashish https://
arorasocial(?)
47
nel/UCYa4_Jr
Orf8R5Kz2uO
com/a ashis utm_medium = t.me/st
ha rorasocial copy_link
tccXQ udified
110) 1751, 1753, 1758, ?, 1785, 1813
(A) 1784 (B) 1781 (C) 1784 (D) 1774 (E) None of these

111) 17, 1009, 1939, 2809, ?, 4377


(A) 3621 (B) 3685 (C) 3345 (D) 3258 (E) None of these

112) 145, ?, 230, 315, 434, 621


(A) 151 (B) 162 (C) 179 (D) 196 (E) None of these

113) ?, 25, 40, 75, 152, 295


(A) 16 (B) 17 (C) 18 (D) 19 (E) None of these

114) 4896, 4080, ?, 2730, 2184, 1716


(A) 3256 (B) 3360 (C) 3369 (D) 3254 (E) None of these

115) 980, ?, 964, 956, 960, 958


(A) 1000 (B) 948 (C) 1200 (D) 964 (E) None of these

116) 8, 52, 286, 1287, 4504.5, ?


(A) 11261.25 (B)12361.25 (C)10261.25 (D)12261.25 (E) None of these

117) 1342, ?, 2210, 2730, 3390


(A) 1516 (B) 1624 (C) 1716 (D) 1966 (E) None of these

118) ?, 14, 26, 46, 76, 118


(A) 6 (B) 7 (C) 8 (D) 9 (E) None of these

119) 27, 9, 6, 6, 8, ?
(A) 25/3 (B) 40/3 (C) 29/3 (D) 38/3 (E) None of these

120) 115, ?, 145, 175, 217, 283


(A) 125 (B) 127 (C) 118 (D) 120 (E) None of these

121) 19, 43, 93, ?, 401, 815


(A) 165 (B) 185 (C) 195 (D) 205 (E) None of these

122) 12, ?, 88, 360, 1792, 10760


(A) 32 (B) 24 (C) 28 (D) 36 (E) None of these

123) 245, 974, 190, ?, 131, 1092


(A) 1030 (B) 1031 (C) 1032 (D) 1033 (E) None of these

https://youtu https://instagra
https ://ww
be.com/chan w.fa cebook.
m.com/aashish https://
arorasocial(?)
48
nel/UCYa4_Jr
Orf8R5Kz2uO
com/a ashis utm_medium = t.me/st
ha rorasocial copy_link
tccXQ udified
124) 32, 80, ?, 500, 1250, 3125
(A) 100 (B) 200 (C) 250 (D) 300 (E) None of these

125) ?, 6930, 2310, 462, 66, 6


(A) 13560 (B) 13450 (C) 13546 (D) 13860 (E) None of these

126) 12, 28, ?, 112, 192, 448


(A) 48 (B) 50 (C) 52 (D) 54 (E) None of these

127) ?, 48, 100, 180, 294, 448


(A) 14 (B) 16 (C) 18 (D) 20 (E) None of these

128) 16, 20, 48, 160, 672, ?


(A) 3542 (B) 3424 (C) 3524 (D) 3624 (E) None of these

129) - 254, 258, ?, 131, 6, 70


(A) -75 (B) -85 (C) -65 (D) -45 (E) None of these

130) 29160, 1944, 162, ?, 3, 1


(A) 21 (B) 24 (C)18 (D) 12 (E) None of these

131) 12, 23, 36, 53, ?, 103


(A) 75 (B) 90 (C) 68 (D) 95 (E) None of these

132) 14, ?, 9, 25, 0, 36


(A) 19 (B) 17 (C) 18 (D) 13 (E) None of these

133) 11, 38, ?, 99, – 117, 226


(A) -26 (B) 34 (C) -45 (D) 45 (E) None of these

134) ?, 30, 95, 386, 1937, 11630


(A) 13 (B) 15 (C) 28 (D) 16 (E) None of these

135) 12, 14, 19, 30, 53, ?


(A) 99 (B) 100 (C) 105 (D) 110 (E) None of these

136) 4, ?, 9, 18, 14, 23


(A) 27 (B) 16 (C) 13 (D) 15 (E) None of these

137) ?, 26, 81, 328, 1645, 9876


(A) 19 (B) 17 (C) 18 (D) 12 (E) None of these

https://youtu https://instagra
https ://ww
be.com/chan w.fa cebook.
m.com/aashish https://
arorasocial(?)
49
nel/UCYa4_Jr
Orf8R5Kz2uO
com/a ashis utm_medium = t.me/st
ha rorasocial copy_link
tccXQ udified
138) 24, 72, 36, ?, 54, 162
(A) 27 (B) 18 (C) 108 (D) 145 (E) None of these

139) 31, 37, 47, 65, ?, 165


(A) 93 (B) 99 (C) 98 (D) 96 (E) None of these

140) ?, 432, 414, 387, 351, 306


(A) 499 (B) 456 (C) 441 (D) 729 (E) None of these

https://youtu https://instagra
https ://ww
be.com/chan w.fa cebook.
m.com/aashish https://
arorasocial(?)
50
nel/UCYa4_Jr
Orf8R5Kz2uO
com/a ashis utm_medium = t.me/st
ha rorasocial copy_link
tccXQ udified
1. Ans. (C) 6. Ans. (D)
25 + 12 + 13 = 27 485 + 11 = 496
27 + 22 + 23 = 39 496 + 22 = 518
39 + 32 + 33 = 75 518 + 33 = 551
75 + 42 + 43 = 155 551 + 44 = 595
155 + 52 + 53 = 305 595 + 55 = 650

2. Ans. (C) 7. Ans. (B)


11 × 1 + 11 = 22 1×4– 2=2
22 × 3 + 11 = 77 2×4– 3=5
77 × 5 + 11 = 396 5 × 4 – 4 = 16
396 × 7 + 11 = 2783 16 × 4 – 5 = 59
2783 × 11 + 11 = 25058 59 × 4 – 6 = 230

3. Ans. (B) 8. Ans. (A)


211 – 168 = 43 2670 ÷ 6 – 5 = 440
253 – 211 = 22 = 43 × 0.5 + 0.5 440 ÷ 5 – 4 = 84
256 – 233 = 23 = 22 × 1 + 1 84 ÷ 4 – 3 = 18
292 – 256 = 36 = 23 × 1.5 + 1.5 18 ÷ 3 – 2 = 4
366 – 292 = 74 = 36 × 2 + 2 4÷2– 1=1

4. Ans. (D) 9. Ans. (C)


(12 × 3 + 6) × 20 = 42 17 + 19 × 1 = 36
(42 × 3 + 6) × 21 = 264 36 + 19 × 3 = 93
(254 × 3 + 6) × 22 = 3192 93 + 19 × 5 = 188
(3192 × 3 + 6) × 23 = 76656 188 + 19 × 7 = 321
(76656 × 3 + 6) × 24 = 3679584 321 + 19 × 9 = 492

5. Ans. (A) 10. Ans. (B)


88 – 69 = 19 89 + 1 × 2 = 91
114 – 88 = 26 = 19 + 7 × 1 91 + 2 × 3 = 97
152 – 114 = 38 = 26 + 6 × 2 97 + 3 × 4 = 109
205 – 152 = 53 = 38 + 5 × 3 109 + 4 × 5 = 129
274 – 205 = 69 = 53 + 4 × 4 129 + 5 × 6 = 159

https://youtu https://instagra
https ://ww
be.com/chan w.fa cebook.
m.com/aashish https://
arorasocial(?)
51
nel/UCYa4_Jr
Orf8R5Kz2uO
com/a ashis utm_medium = t.me/st
ha rorasocial copy_link
tccXQ udified
11. Ans. (C) 16. Ans. (A)
76 – 64 = 12 36 × 1 + 12 = 37
90 – 76 = 14 (12 + 2) 37 × 2 + 22 = 78
107 – 90 = 17 (14 + 3) 78 × 3 + 32 = 243
128 – 107 = 21 (17 + 4) 243 × 4 + 42 = 988
154 – 128 = 26 (21 + 5) 988 × 5 + 52 = 4965

12. Ans. (D) 17. Ans. (D)


101 + 22 = 105 14 × 1.5 = 21
105 + 32 = 114 21 × 2 = 42
114 + 52 = 139 42 × 2.5 = 105
139 + 72 = 188 105 × 3 = 315
188 + 112 = 309 315 × 3.5 = 1102.5

13. Ans. (B) 18. Ans. (B)


46 × 2 + 1 = 93 192 – 1 = 360
93 × 3 + 2 = 281 172 – 1 = 289
281 × 4 + 3 = 1127 132 – 1 = 168
1127 × 5 + 4 = 5639 112 – 1 = 120
5639 × 6 + 5 = 33839 72 – 1 = 48

14. Ans. (B) 19. Ans. (A)


16 + 22 = 20 12 + 3 × 4 = 24
20 – 33 = - 7 24 + 4 × 5 = 44
- 7 + 42 = 9 44 + 5 × 6 = 74
9 – 53 = - 116 74 + 6 × 7 = 116
- 116 + 62 = - 80 116 + 7 × 8 = 172

15. Ans. (A) 20. Ans. (C)


114 × 1 + 1 = 115 114 + 20 = 115
115 × 2 + 2 = 232 115 + 22 = 119
232 × 3 + 3 = 699 119 + 24 = 135
699 × 4 + 4 = 2800 135 + 26 = 199
2800 × 5 + 5 = 14005 199 + 28 = 455

https://youtu https://instagra
https ://ww
be.com/chan w.fa cebook.
m.com/aashish https://
arorasocial(?)
52
nel/UCYa4_Jr
Orf8R5Kz2uO
com/a ashis utm_medium = t.me/st
ha rorasocial copy_link
tccXQ udified
21. Ans. (C) 26. Ans. (A)
43 + 14 = 57 8 × 5 + 5 = 45
57 + 16 = 73 45 × 4 + 4 = 184
73 + 18 = 91 184 × 3 + 3 = 555
91 + 20 = 111 555 × 2 + 2 = 1112
111 + 22 = 133 1112 × 1 + 1 = 1113

22. Ans. (C) 27. Ans. (C)


24 + 9 = 33 45 + 7 × 1 = 52
33 + 18 = 51 52 + 7 × 3 = 73
73 + 7 × 5 = 108
30 + 9 = 39 108 + 7 × 7 = 157
39 + 18 = 57 157 + 7 × 9 = 220

23. Ans. (A) 28. Ans. (C)


2 + 32 + 1 = 12 108 + 22 = 112
12 + 42 + 2 = 30 112 + 32 = 121
30 + 52 + 3 = 58 121 + 52 = 146
58 + 62 + 4 = 98 146 + 72 = 195
98 + 72 + 5 = 152 195 + 112 = 316

24. Ans. (B) 29. Ans. (D)


18 × 2 = 36 (210 + 50) ÷ 2 = 130
36 × 2.5 = 90 (130 + 50) ÷ 2 = 90
90 × 3 = 270 (90 + 50) ÷ 2 = 70
270 × 3.5 = 945 (70 + 50) ÷ 2 = 60
945 × 4 = 3780 (60 + 50) ÷ 2 = 55

25. Ans. (D) 30. Ans. (B)


144 ÷ 2 = 72 9 × 9 ÷ 3 = 27
72 × 3 = 216 27 × 8 ÷ 3 = 72
216 ÷ 4 = 54 72 × 7 ÷ 3 = 168
54 × 5 = 270 168 × 6 ÷ 3 = 336
270 ÷ 6 = 45 336 × 5 ÷ 3 = 560

https://youtu https://instagra
https ://ww
be.com/chan w.fa cebook.
m.com/aashish https://
arorasocial(?)
53
nel/UCYa4_Jr
Orf8R5Kz2uO
com/a ashis utm_medium = t.me/st
ha rorasocial copy_link
tccXQ udified
31. Ans. (A) 36. Ans. (A)
22.5 + 32 – 4 = 27.5 36 × 0.5 = 18
27.5 + 42 – 4 = 39.5 18 × 1 = 18
39.5 + 52 – 4 = 60.5 18 × 1.5 = 27
60.5 + 62 – 4 = 92.5 27 × 2 = 54
92.5 + 72 – 4 = 137.5 54 × 2.5 = 135

32. Ans. (D) 37. Ans. (D)


12 × 3 + 3 = 39 121 + 12 = 122
39 × 3 + 3 = 120 122 + 23 = 130
120 × 3 + 3 = 363 130 + 32 = 157
363 × 3 + 3 = 1092 157 + 43 = 221
1092 × 3 + 3 = 3279 221 + 52 = 246

33. Ans. (A) 38. Ans. (C)


88 + 102 = 188 115 + 52 = 140
188 – 122 = 44 140 – 72 = 91
44 + 142 = 240 91 + 92 = 172
240 – 162 = - 16 172 – 112 = 51
- 16 + 182 = 308 51 + 132 = 220

34. Ans. (B) 39. Ans. (B)


19 + 23 + 1 = 28 13 + 12 + 1 = 3
28 + 33 + 1 = 56 23 + 22 + 2 = 14
56 + 43 + 1 = 121 33 + 32 + 3 = 39
121 + 53 + 1 = 247 43 + 42 + 4 = 84
247 + 63 + 1 = 464 53 + 52 + 5 = 155
63 + 62 + 6 = 258
35. Ans. (C)
162 – 16 = 240 40. Ans. (B)
182 + 18 = 342 122 – 11 = 133
202 – 20 = 380 132 – 12 = 157
222 + 22 = 506 142 – 13 = 183
242 – 24 = 552 152 – 14 = 211
162 – 15 = 241
172 – 16 = 273

https://youtu https://instagra
https ://ww
be.com/chan w.fa cebook.
m.com/aashish https://
arorasocial(?)
54
nel/UCYa4_Jr
Orf8R5Kz2uO
com/a ashis utm_medium = t.me/st
ha rorasocial copy_link
tccXQ udified
41. Ans. (B) 46. Ans. (B)
5 × 12 = 5 1610 + 2 × 2 = 1614
5 × 22 = 20 1614 + 2 × 3 = 1620
20 × 32 = 180 1620 + 2 × 5 = 1630
180 × 42 = 2880 1630 + 2 × 7 = 1644
2880 × 52 = 72000 1644 + 2 × 11 = 1666

42. Ans. (A) 47. Ans. (A)


17 × 2 + 12 = 46 63 – 62 + 6 = 186
46 × 2 + 13 = 105 73 – 72 + 7 = 301
105 × 2 + 14 = 224 83 – 82 + 8 = 456
224 × 2 + 15 = 463 93 – 92 + 9 = 657
463 × 2 + 16 = 942 103 – 102 + 10 = 910

43. Ans. (D) 48. Ans. (D)


140 + 12 + 1 = 142 12 × 1 + 2 = 14
142 + 22 + 1 = 147 14 × 2 + 3 = 31
147 + 32 + 1 = 157 31 × 3 + 4 = 97
157 + 42 + 1 = 174 97 × 4 + 5 = 393
174 + 52 + 1 = 200 393 × 5 + 6 = 971

44. Ans. (C) 49. Ans. (C)


1025 – 23 = 1017 12 × 2 + 1 = 25
1017 + 33 = 1044 25 × 2 + 2 = 52
1044 – 43 = 980 52 × 2 + 3 = 107
980 + 53 = 1105 107 × 2 + 4 = 218
1105 – 63 = 889 218 × 2 + 5 = 441

45. Ans. (B) 50. Ans. (B)


45 × 3 = 135 16 × 1.5 = 24
135 + 5 = 140 24 × 2.5 = 60
140 × 7 = 840 60 × 3.5 = 210
840 + 9 = 849 210 × 4.5 = 945
849 × 11 = 9339 945 × 5.5 = 5197.5

https://youtu https://instagra
https ://ww
be.com/chan w.fa cebook.
m.com/aashish https://
arorasocial(?)
55
nel/UCYa4_Jr
Orf8R5Kz2uO
com/a ashis utm_medium = t.me/st
ha rorasocial copy_link
tccXQ udified
51. Ans. (B) 56. Ans. (A)
17 + 13 – 2 = 16 775 – 36 = 46
16 + 23 – 2 = 22 46 + 35 = 289
22 + 33 – 2 = 47 289 – 34 = 208
47 + 43 – 2 = 109 208 + 33 = 235
109 + 53 – 2 = 232 235 – 32 = 226

52. Ans. (A) 57. Ans. (C)


7 × 8 = 56 19 + 12 + 13 = 21
56 × 7 = 392 21 + 22 + 23 = 33
392 × 6 = 2352 33 + 32 + 33 = 69
2352 × 5 = 11760 69 + 42 + 43 = 149
11760 × 4 = 47040 149 + 52 + 53 = 299

53. Ans. (D) 58. Ans. (C)


63 + 72 + 8 = 273 172 + 53 = 297
53 + 62 + 7 = 168 297 + 73 = 640
43 + 52 + 6 = 95 640 + 93 = 1369
33 + 42 + 5 = 48 1369 + 113 = 2700
23 + 32 + 4 = 21 2700 + 133 = 4897

54. Ans. (A) 59. Ans. (D)


185 + 312 = 1146 17 × 2 + 2 = 36
1146 – 322 = 122 36 × 2 + 3 = 75
122 + 332 = 1211 75 × 2 + 5 = 155
1211 – 342 = 55 155 × 2 + 7 = 317
55 + 352 = 1280 317 × 2 + 11 = 645

55. Ans. (C) 60. Ans. (A)


15 × 3 + 2 = 47 325 × 1 ÷ 5 = 65
47 × 3 + 3 = 144 65 × 2 ÷ 5 = 26
144 × 3 + 5 = 437 26 × 3 ÷ 5 = 15.6
437 × 3 + 7 = 1318 15.6 × 4 ÷ 5 = 12.48
1318 × 3 + 11 = 3965 12.48 × 5 ÷ 5 = 12.48

https://youtu https://instagra
https ://ww
be.com/chan w.fa cebook.
m.com/aashish https://
arorasocial(?)
56
nel/UCYa4_Jr
Orf8R5Kz2uO
com/a ashis utm_medium = t.me/st
ha rorasocial copy_link
tccXQ udified
61. Ans. (B) 66. Ans. (C)
35 × 9 = 315 27.5 – 26 = 1.5
315 × 7 = 2205 31 – 27.5 = 3.5 (1.5 + 2)
2205 × 5 = 11025 36.5 – 31 = 5.5 (3.5 + 2)
11025 × 3 = 33075 44 – 36.5 = 7.5 (5.5 + 2)
33075 × 1 = 33075 53.5 – 44 = 9.5 (7.5 + 2)

62. Ans. (A) 67. Ans. (D)


19 + 472 = 2228 12 × 1 + 2 = 14
2228 – 482 = - 76 14 × 1 + 4 = 18
- 76 + 492 = 2325 18 × 1 + 8 = 26
2325 - 502 = - 175 26 × 2 + 6 = 58
- 175 + 512 = 2426 58 × 5 + 9 = 298

63. Ans. (D) 68. Ans. (B)


8 × 2.5 = 20 244 – 240 = 4
20 × 3.5 = 70 240 – 231 = 9 (4 + 5)
70 × 4.5 = 315 231 – 217 = 14 (9 + 5)
315 × 5.5 = 1732.5 217 – 198 = 19 (14 + 5)
1732.5 × 6.5 = 11261.25 198 – 174 = 24 (19 + 5)

64. Ans. (A) 69. Ans. (A)


17 + 12 = 18 28 × 2 – 16 = 40
18 + 22 = 22 2 × 40 - 28 = 52
22 + 32 = 31 2 × 52 – 40 = 64
31 + 42 = 47 2 × 64 – 52 = 76
47 + 52 = 72
70. Ans. (D)
65. Ans. (C) 21 + 32 = 53
13 × 3 + 4 = 43 32 + 53 = 85
43 × 3 + 5 = 134 53 + 85 = 138
134 × 3 + 6 = 408 85 + 138 = 223
408 × 3 + 7 = 1231
1231 × 3 + 8 = 3701

https://youtu https://instagra
https ://ww
be.com/chan w.fa cebook.
m.com/aashish https://
arorasocial(?)
57
nel/UCYa4_Jr
Orf8R5Kz2uO
com/a ashis utm_medium = t.me/st
ha rorasocial copy_link
tccXQ udified
71. Ans. (B) 76. Ans. (B)
28 × 2 + 2 = 58 16 + 22 + 1 = 21
58 × 2 + 2 = 118 21 + 32 + 1 = 31
118 × 2 + 2 = 238 31 + 42 + 1 = 48
238 × 2 + 2 = 478 48 + 52 + 1 = 74
478 × 2 + 2 = 958 74 + 62 + 1 = 111

72. Ans. (D) 77. Ans. (C)


36 + 3 × 6 = 54 50 × 2 = 100
54 + 5 × 4 = 74 100 ÷ 4 = 25
74 + 7 × 4 = 102 25 × 8 = 200
102 + 10 × 2 = 122 200 ÷ 16 = 12.5
122 + 12 × 2 = 146 12.5 × 32 = 400

73. Ans. (D) 78. Ans. (A)


6 + 13 – 1 = 6 16 + 12 = 28
6 + 23 – 2 = 12 12 + 28 = 40
12 + 33 – 3 = 36 28 + 40 = 68
36 + 43 – 4 = 96 40 + 68 = 108
96 + 53 – 5 = 216
79. Ans. (B)
74. Ans. (A) 381 + 40 = 421
9 + 18 = 27 421 + 42 = 463
27 + 18 = 45 463 + 44 = 507
45 + 18 = 63 507 + 46 = 553
63 + 18 = 81 553 + 47 = 601
81 + 18 = 99
80. Ans. (C)
75. Ans. (C) 12 × 2 = 24
170 – 143 = 27 24 × 3 = 72
195 – 170 = 25 (27 – 2) 72 × 5 = 360
226 – 195 = 31 (25 + 6) 360 × 7 = 2520
255 – 226 = 29 (31 – 2) 2520 × 11 = 27720
290 – 255 = 35 (29 + 6)

https://youtu https://instagra
https ://ww
be.com/chan w.fa cebook.
m.com/aashish https://
arorasocial(?)
58
nel/UCYa4_Jr
Orf8R5Kz2uO
com/a ashis utm_medium = t.me/st
ha rorasocial copy_link
tccXQ udified
81. Ans. (B) 86. Ans. (D)
91 – 18 = 73 323 – 255 = 68
73 – 16 = 57 399 – 323 = 76 (68 + 8)
57 – 14 = 43
43 – 12 = 31 362 – 290 = 72
31 – 10 = 21 442 – 362 = 80 (72 + 8)

82. Ans. (A) 87. Ans. (A)


19 – 16 = 3 24 × 1.5 = 36
23.5 – 19 = 4.5 (3 + 1.5) 36 × 3.5 = 126
30.5 – 23.5 = 7 (4.5 + 2.5) 126 × 5.5 = 693
41 – 30.5 = 10.5 (7 + 3.5) 693 × 7.5 = 5197.5
56 – 41 = 15 (10.5 + 4.5) 5197.5 × 9.5 = 49376.25

83 Ans. (D) 88. Ans. (B)


96 ÷ 8 = 12 108 × 1.5 = 162
12 × 12 = 144 162 × 1.5 = 243
144 ÷ 8 = 18 243 × 1.5 = 364.5
18 × 12 = 216 364.5 × 1.5 = 546.75
216 ÷ 8 = 27 546.75 × 1.5 = 820.125

84. Ans. (C) 89. Ans. (C)


214 - 23 = 206 (16 + 32) ÷ 2 = 24
206 – 33 = 179 (32 + 24) ÷ 2 = 28
179 – 53 = 54 (24 + 28) ÷ 2 = 26
54 – 73 = - 289 (28 + 26) ÷ 2 = 27
- 289 – 113 = 1620
90. Ans. (C)
85. Ans. (B) 192 – 1 = 360
19 + 21 = 40 182 – 1 = 323
40 + 23 = 63 172 – 1 = 288
63 + 25 = 88
88 + 27 = 115
115 + 29 = 144

https://youtu https://instagra
https ://ww
be.com/chan w.fa cebook.
m.com/aashish https://
arorasocial(?)
59
nel/UCYa4_Jr
Orf8R5Kz2uO
com/a ashis utm_medium = t.me/st
ha rorasocial copy_link
tccXQ udified
91. Ans. (B) 96. Ans. (C)
991 + 32 = 1000 300 + 23 = 308
1000 – 62 = 964 308 – 32 = 299
964 + 122 = 1108 299 + 43 = 363
1108 – 242 = 532 363 – 52 = 338
532 + 482 = 2836 338 + 63 = 554

92. Ans. (B) 97. Ans. (A)


36 + (6 – 3) = 39 19 × 2 = 38
39 + (9 – 3) = 45 38 × 3 = 114
45 + (5 – 4) = 46 114 × 5 = 570
46 + (6 – 4) = 48 570 × 7 = 3990
48 + (8 – 4) = 52 3990 × 11 = 43890

93. Ans. (C) 98. Ans. (A)


480 ÷ 4 = 120 642 + (6 + 4 + 2) = 654
120 × 7 = 840 654 + (6 + 5 + 4) = 669
840 ÷ 10 = 84 669 + (6 + 6 + 9) = 690
84 × 13 = 1092 690 + (6 + 9 + 0) = 705
1092 ÷ 16 = 68.25 705 + (7 + 0 + 5) = 717

94. Ans. (C) 99. Ans. (D)


30 + (3 + 0) = 33 56 + 12 × 20 = 68
33 + (3 + 3) = 39 68 + 12 × 21 = 92
39 + (3 + 9) = 51 92 + 12 × 22 = 140
51 + (5 + 1) = 57 140 + 12 × 23 = 236
57 + (5 + 7) = 69 236 + 12 × 34 = 428

95. Ans. (A) 100. Ans. (D)


193 = 6859 72 + 18 = 90
203 = 8000 90 + 20 = 110
213 = 9261 110 + 22 = 132
132 + 24 = 156
156 + 26 = 182

https://youtu https://instagra
https ://ww
be.com/chan w.fa cebook.
m.com/aashish https://
arorasocial(?)
60
nel/UCYa4_Jr
Orf8R5Kz2uO
com/a ashis utm_medium = t.me/st
ha rorasocial copy_link
tccXQ udified
101. Ans. (A) 106. Ans. (C)
18480 ÷ 11 = 1680 80640 ÷ 8 = 10080
1680 ÷ 7 = 240 10080 ÷ 7 = 1440
240 ÷ 5 = 48 1440 ÷ 6 = 240
48 ÷ 3 = 16 240 ÷ 5 = 48
16 ÷ 2 = 8 48 ÷ 4 = 12

102. Ans. (C) 107. Ans. (D)


148 + 192 = 509 15 × 4 + 5 = 65
509 + 212 = 950 65 × 4 + 10 = 270
950 + 232 = 1479 270 × 4 + 15 = 1095
1479 + 252 = 2104 1095 × 4 + 20 = 4400
2104 + 272 = 2833 4400 × 4 + 25 = 17625

103. Ans. (D) 108. Ans. (B)


38 × 2 + 12 = 88 17 × 2 = 34
88 × 2 + 13 = 189 34 × 3 = 102
189 × 2 + 14 = 392 102 × 5 = 510
392 × 2 + 15 = 799 510 × 7 = 3570
799 × 2 + 16 = 1614 3570 × 11 = 39270

104. Ans. (B) 109. Ans. (D)


18 + 1 = 19 115 + 322 = 1139
19 × 3 = 57 1139 + 332 = 2228
57 + 5 = 62 2228 + 342 = 3384
62 × 7 = 434 3384 + 352 = 4609
434 + 9 = 442 4609 + 362 = 5905

105. Ans. (A) 110. Ans. (A)


16 × 2.5 = 40 1751 + 2 = 1753
40 × 5 = 200 1753 + 2 + 3 = 1758
200 × 7.5 = 1500 1758 + 2 + 3 + 5 = 1768
1500 × 10 = 15000 1768 + 2 + 3 + 5 + 7 = 1785
15000 × 12.5 = 187500 1785 + 2 + 3 + 5 + 7 + 11 = 1813

https://youtu https://instagra
https ://ww
be.com/chan w.fa cebook.
m.com/aashish https://
arorasocial(?)
61
nel/UCYa4_Jr
Orf8R5Kz2uO
com/a ashis utm_medium = t.me/st
ha rorasocial copy_link
tccXQ udified
111. Ans. (A) 116. Ans. (A)
17 + 312 + 31 = 1009 8 × 6.5 = 52
1009 + 302 + 30 = 1939 52 × 5.5 = 286
1939 + 292 + 29 = 2809 286 × 4.5 = 1287
3809 + 282 + 28 = 3621 1287 × 3.5 = 4504.5
3621 + 272 + 27 = 4377 450.5 × 2.5 = 11261.25

112. Ans. (C) 117. Ans. (C)


145 + 17 × 2 = 179 113 + 11 = 1342
179 + 17 × 3 = 230 123 – 12 = 1716
230 + 17 × 5 = 315 133 + 13 = 2210
315 + 17 × 7 = 434 143 – 14 = 2730
434 + 17 × 11 = 612 153 + 15 = 3390

113. Ans. (D) 118. Ans. (C)


19 + 2 × 3 = 25 8 + 2 × 3 = 14
25 + 3 × 5 = 40 14 + 3 × 4 = 26
40 + 5 × 7 = 75 26 + 4 × 5 = 46
75 + 7 × 11 = 152 46 + 5 × 6 = 76
152 + 11 × 13 = 295 76 + 6 × 7 = 118

114. Ans. (B) 119. Ans. (B)


4896 – 4080 = 816 27 × 1/3 = 9
4080 – 3360 = 720 (816 – 96) 9 × 2/3 = 6
3360 – 2730 = 630 (720 – 90) 6 × 3/3 = 6
2730 – 2184 = 546 (630 – 84) 6 × 4/3 = 8
2184 – 1716 = 468 (546 – 78) 8 × 5/3 = 40/3

115. Ans. (B) 120. Ans. (B)


980 – 25 = 948 115 + 6 × 2 = 127
948 + 24 = 964 127 + 6 × 3 = 145
964 – 23 = 956 145 + 6 × 5 = 175
956 + 22 = 960 175 + 6 × 7 = 217
960 – 21 = 958 217 + 6 × 11 = 283

https://youtu https://instagra
https ://ww
be.com/chan w.fa cebook.
m.com/aashish https://
arorasocial(?)
62
nel/UCYa4_Jr
Orf8R5Kz2uO
com/a ashis utm_medium = t.me/st
ha rorasocial copy_link
tccXQ udified
121. Ans. (C) 126. Ans. (A)
19 × 2 + 5 = 43 21 × 2 + 4 = 28
43 × 2 + 7 = 93 28 × 2 – 8 = 48
93 × 2 + 9 = 195 48 × 2 + 16 = 112
195 × 2 + 11 = 401 112 × 2 – 32 = 192
401 × 2 + 13 = 815 192 × 2 + 64 = 448

122. Ans. (A) 127. Ans. (C)


12 × 2 + 23 = 32 48 – 18 = 30
32 × 3 – 23 = 88 100 – 48 = 52 = (30 + 22)
88 × 4 + 23 = 360 180 – 100 = 80 (52 + 28)
360 × 5 – 23 = 1792 294 – 180 = 114 (80 + 34)
1792 × 6 + 23 = 10760 448 – 294 = 154 (114 + 40)

123. Ans. (B) 128. Ans. (B)


245 + 272 = 974 16 × 1 + 22 = 20
974 – 282 = 190 20 × 2 + 23 = 48
190 + 292 = 1031 48 × 3 + 24 = 160
1031 – 302 = 131 160 × 4 + 25 = 672
131 + 313 = 1092 672 × 5 + 26 = 3424

124. Ans. (B) 129. Ans. (B)


32 × 2.5 = 80 - 254 + 83 = 258
80 × 2.5 = 200 258 - 73 = - 85
200 × 2.5 = 500 - 85 + 63 = 131
500 × 2.5 = 1250 131 – 53 = 6
1250 × 2.5 = 3125 6 + 43 = 70

125. Ans. (D) 130. Ans. (C)


13860 ÷ 2 = 6930 29160 ÷ 15 = 1944
6930 ÷ 3 = 2310 1944 ÷ 12 = 162
2310 ÷ 5 = 462 162 ÷ 9 = 18
462 ÷ 7 = 66 18 ÷ 6 = 3
66 ÷ 11 = 6 3÷3= 1

https://youtu https://instagra
https ://ww
be.com/chan w.fa cebook.
m.com/aashish https://
arorasocial(?)
63
nel/UCYa4_Jr
Orf8R5Kz2uO
com/a ashis utm_medium = t.me/st
ha rorasocial copy_link
tccXQ udified
131. Ans. (A) 136. Ans. (C)
23 – 13 = 10 4 + 32 = 13
36 – 23 = 13 (11 + 3) 13 – 22 = 9
53 – 36 = 17 (13 + 4) 9 + 32 = 18
75 – 53 = 22 (17 + 5) 18 – 22 = 14
103 – 75 = 28 (22 + 6) 14 + 32 = 23

132. Ans. (C) 137. Ans. (D)


14 + 22 = 18 12 × 2 + 2 = 26
18 – 33 = 9 26 × 3 + 3 = 81
9 + 42 = 25 81 × 4 + 4 = 328
25 – 52 = 0 328 × 5 + 5 = 1645
0 + 62 = 36 1645 × 6 + 6 = 9876

133. Ans. (A) 138. Ans. (C)


11 + 33 = 38 24 × 3 = 72
38 – 43 = - 26 72 ÷ 2 = 36
- 26 + 53 = 99 36 × 3 = 108
99 – 63 = - 117 108 ÷ 2 = 54
- 117 + 73 = 226 54 × 3 = 162

134. Ans. (A) 139. Ans. (B)


13 × 2 + 4 = 30 37 – 31 = 6
30 × 3 + 5 = 95 47 – 37 = 10 (6 + 22)
95 × 4 + 6 = 386 65 – 47 = 18 (10 + 23)
386 × 5 + 7 = 1937 99 – 65 = 34 (18 + 24)
1937 × 6 + 8 = 11630 165 – 99 = 66 (34 + 25)

135. Ans. (B) 140. Ans. (C)


14 – 12 = 2 441 – 9 × 1 = 432
19 – 14 = 5 (2 + 3 × 20) 432 – 9 × 2 = 414
30 – 19 = 11 (5 + 3 × 21) 414 – 9 × 3 = 387
53 – 30 = 23 (11 + 3 × 22) 387 – 9 × 4 = 351
100 – 53 = 47 (23 + 3 × 23) 351 – 9 × 5 = 306

https://youtu https://instagra
https ://ww
be.com/chan w.fa cebook.
m.com/aashish https://
arorasocial(?)
64
nel/UCYa4_Jr
Orf8R5Kz2uO
com/a ashis utm_medium = t.me/st
ha rorasocial copy_link
tccXQ udified
In each of these questions a number is wrong in the series) Find out the
wrong number.
इन प्रश्नों भें प्रत्मे क भें श्ॊखरा भें एक सॊख्मा गरत है। गरत सॊख्मा फतामे ।

1) 1, 8, 25, 61, 121, 211


(A) 1 (B) 8 (C) 61 (D) 121 (E) 25

2) 19, 76, 266, 798, 1995, 4112


(A) 4112 (B) 76 (C) 19 (D) 798 (E) 266

3) 263, 526, 477, 693, 668, 732


(A) 526 (B) 668 (C) 477 (D) 263 (E) 732

4) 4, 16, 46, 63, 76, 87


(A) 16 (B) 46 (C) 27 (D) 76 (E) 87

5) 1, 2, 6, 27, 92, 485


(A) 1 (B) 18 (C) 2 (D) 27 (E) 485

6) 1.5, 2, 12, 72, 576, 5760


(A) 12 (B) 2 (C) 1.5 (D) 576 (E) None of these

7) 109, 111, 116, 127, 144, 173


127 (B) 111 (C) 109 (D) 173 (E) None of these

8) 297, 298, 290, 318, 253, 379


(A) 298 (B) 290 (C) 318 (D) 253 (E) None of these

9) 21, 30, 37, 55, 85, 131, 197


(A) 131 (B) 37 (C) 55 (D) 30 (E) None of these

10) 9, 10, 11, 13, 19, 41, 163


(A) 10 (B) 113 (C) 41 (D) 9 (E) None of these

11) 179, 192, 207, 220, 243, 264, 287


(A) 192 (B) 207 (C) 220 (D) 243 (E) None of these

https://youtu https ://ww https://instag https://


be.com/chan w.fa cebook. ram.com/aas
nel/UCYa4_Jr com/a ashis hisharorasoci t.me/st
65 Orf8R5Kz2uO al?utm_medi
tccXQ
ha rorasocial
um=copy_link udified
12) 137, 150, 167, 186, 210, 234, 263
(A) 137 (B) 167 (C) 210 (D) 234 (E) None of these

13) 263, 260, 266, 261, 269, 254, 272


(A) 263 (B) 261 (C) 269 (D) 272 (E) None of these

14) 191, 195, 203, 219, 251, 317, 443


(A) 317 (B) 251 (C) 203 (D) 191 (E) None of these

15) 211, 234, 264, 301, 345, 398, 454


(A) 211 (B) 264 (C) 345 (D) 398 (E) None of these

16) 237, 241, 252, 275, 324, 445, 614


(A) 252 (B) 275 (C) 445 (D) 237 (E) None of these

17) 187, 206, 227, 245, 267, 282, 307


(A) 187 (B) 307 (C) 245 (D) 282 (E) None of these

18) 259, 264, 274, 291, 313, 354, 404


(A) 259 (B) 274 (C) 313 (D) 404 (E) None of these

19) 4, 11, 30, 69, 128, 219


(A) 4 (B) 30 (C) 219 (D) 69 (E) None of these

20) 37, 379, 590, 718, 781, 807, 814


(A) 590 (B) 718 (C) 807 (D) 37 (E) None of these

21) 79, 79, 158, 479, 1896, 9480


(A) 79 (B) 158 (C) 9480 (D) 1896 (E) 479

22) 32, 16.5, 17.5, 35, 152, 1224, 1960


(A) 35 (B) 1960 (C) 17.5 (D) 152 (E) 1224

23) 193, 197, 208, 240, 335, 614, 1470


(A) 193 (B) 208 (C) 240 (D) 614 (E) 1470

24) 243, 244, 235, 260, 215, 292, 171


(A) 171 (B) 215 (C) 235 (D) 244 (D) 243

25) 43, 44, 48, 75, 93, 216, 252


(A) 43 (B) 48 (C) 93 (D) 75 (E) 252

https://youtu https://instagra
https ://ww
be.com/chan w.fa cebook.
m.com/aashish https://
arorasocial(?)
66
nel/UCYa4_Jr
Orf8R5Kz2uO
com/a ashis utm_medium = t.me/st
ha rorasocial copy_link
tccXQ udified
26) 17, 19, 41, 128, 513, 2571, 15433
(A) 128 (B) 17 (C) 19 (D) 15433 (E) 513

27) 304, 146, 68, 30, 14, 4, 1


(A) 304 (B) 68 (C) 14 (D) 1 (E) 146

28) 3, 5, 13, 64, 430, 4725, 61419


(A) 3 (B) 13 (C) 430 (D) 61419 (E) 64

29) 19, 76, 80, 320, 326, 1296, 1300


(A) 76 (B) 326 (C) 1300 (D) 19 (E) 80

30) 124, 32, 17, 18, 39, 149, 1193


(A) 124 (B) 17 (C) 149 (D) 39 (D) 32

31) 23, 240, 366, 435, 459, 468, 470


(A) 470 (B) 459 (C) 435 (D) 240 (E) None of these

32) 1.5, 4, 6, 26, 144, 3456


(A) 1.5 (B) 26 (C) 3456 (D) 4 (E) None of these

33) 5, 4, 6, 15, 58, 275, 1644


(A) 1644 (B) 4 (C) 15 (D) 58 (E) None of these

34) 237.25, 475, 476, 240, 68, 16, 17


(A) 68 (B) 476 (C) 237.25 (D) 17 (E) None of these

35) 37, 54, 88, 143, 217, 322, 458


(A) 37 (B) 458 (C) 322 (D) 143 (E) None of these

36) 143, 146, 161, 196, 261, 358, 501


(A) 143 (B) 161 (C) 261 (D) (E) None of these

37) 111, 119, 110, 231, 186, 1517, 1348


(A) 119 (B) 231 (C) 186 (D) 1348 (E) None of these

38) 39, 30, 42, 97, 360, 175, 10476


(A) 97 (B) 1755 (C) 39 (D) 39 (E) None of these

https://youtu https://instagra
https ://ww
be.com/chan w.fa cebook.
m.com/aashish https://
arorasocial(?)
67
nel/UCYa4_Jr
Orf8R5Kz2uO
com/a ashis utm_medium = t.me/st
ha rorasocial copy_link
tccXQ udified
1. Ans. (B) 1.5 × 2 = 3
1+1× 6=7 3 × 4 = 12
7 + 2 × 9 = 25 12 × 6 = 72
25 + 3 × 12 = 61 72 × 8 = 576
61 + 4 × 15 = 121 576 × 10 = 5760
121 + 5 × 18 = 211
7. Ans. (D)
2. Ans. (A) 109 + 2 = 111
19 × 4 = 76 111 + 5 = 116
76 × 3.5 = 266 116 + 11 = 127
266 × 3 = 798 127 + 17 = 144
798 × 2.5 = 1995 144 + 23 = 167
1995 × 2 = 3990
8. Ans. (A)
3. Ans. (D) 297 + (13 + 1) = 299
443 + 83 = 526 299 – (23 + 1) = 290
526 – 72 = 477 290 + (33 + 1) = 318
477 + 63 = 693 318 – (43 + 1) = 252
693 – 52 = 668 253 + (53 + 1) = 379
668 + 43 = 732
9. Ans. (D)
4. Ans. (A) 27 – 21 = 6
4 + 23 = 27 37 – 27 = 10 (6 + 4 × 1)
27 + 19 = 46 55 – 37 = 18 (10 + 4 × 2)
46 + 17 = 63 85 – 55 = 30 (18 + 4 × 3)
63 + 13 = 76 131 – 85 = 46 (30 + 4 × 4)
76 + 11 = 87 197 – 131 = 66 (46 + 4 × 5)

5. Ans. (C) 10. Ans. (C)


4 × 1 + 12 = 5 10 – 9 = 1
5 × 2 – 22 = 6 11 – 10 = 1 (1 × 1)
6 × 3 + 32 = 27 13 – 11 = 2 (1 × 2)
27 × 4 – 42 = 92 19 – 13 = 6 (2 × 3)
92 × 5 + 52 = 485 43 – 19 = 24 (6 × 4)
163 – 43 = 120 (24 × 5)
6. Ans. (B)

https://youtu https ://ww https://instag https://


be.com/chan w.fa cebook. ram.com/aas
nel/UCYa4_Jr com/a ashis hisharorasoci t.me/st
68 Orf8R5Kz2uO al?utm_medi
tccXQ
ha rorasocial
um=copy_link udified
11. Ans. (C) 301 + 44 = 345
179 + 13 = 192 345 + 51 = 396
192 + 15 = 207 396 + 58 = 454
207 + 17 = 224
224 + 19 = 243 16. Ans. (A)
243 + 21 = 264 237 + 22 = 241
264 + 23 = 287 241 + 32 = 250
250 + 52 = 275
12. Ans. (C) 275 + 72 = 324
137 + 13 = 150 324 + 112 = 445
150 + 17 = 167 445 + 132 = 614
167 + 19 = 186
186 + 23 = 209 17. Ans. (C)
209 + 25 = 234 206 – 187 = 19
234 + 29 = 263 227 – 206 = 21 (19 + 2 = 21)
244 – 227 = 17 (21 – 4 = 17)
13. Ans. (B) 267 – 244 = 23 (17 + 6 = 23)
263 – 3 = 260 282 – 267 = 15 (23 – 8 = 15)
260 + 6 = 266 307 – 282 = 25 (15 + 10 = 25)
266 – 9 = 257
257 + 12 = 269 18. Ans. (C)
269 – 15 = 254 259 + (22 + 1) = 264
254 + 18 = 272 264 + (32 + 1) = 274
274 + (42 + 1) = 291
14. Ans. (A) 291 + (52 + 1) = 317
191 + 4 × 20 = 195 317 + (62 + 1) = 354
195 + 4 × 21 = 203 354 = (72 + 1) = 404
203 + 4 × 22 = 219
219 + 4 × 23 = 251 19. Ans. (D)
251 + 4 × 24 = 315 13 + 3 = 4
315 + 4 × 25 = 443 23 + 3 = 11
33 + 3 = 30
15. Ans. (D) 43 + 3 = 67
211 + 23 = 234 53 + 3 = 128
234 + 30 = 264 63 + 3 = 219
264 + 37 = 301

https://youtu https ://ww https://instag https://


be.com/chan w.fa cebook. ram.com/aas
nel/UCYa4_Jr com/a ashis hisharorasoci t.me/st
69 Orf8R5Kz2uO al?utm_medi
tccXQ
ha rorasocial
um=copy_link udified
20. Ans. (A) 260 – 72 = 211
37 + (73 – 1) = 379 211 + 92 = 292
379 + (63 – 1) = 594 292 – 112 = 171
594 + (53 – 1) = 718
718 + (43 – 1) = 781 25. Ans. (C)
781 + (33 – 1) = 807 43 + 13 = 44
807 + (23 – 1) = 814 44 + 22 = 48
48 + 33 = 75
21. Ans. (E) 75 + 42 = 91
79 × 1 = 79 91 + 53 = 216
79 × 2 = 158 216 + 62 = 252
158 × 3 = 474
474 × 4 = 1896 26. Ans. (A)
1986 × 5 = 9480 17 × 1 + 2 = 19
19 × 2 + 3 = 41
22. Ans. (A) 41 × 3 + 4 = 127
32 × 0.5 + 0.5 = 16.5 127 × 4 + 5 = 513
16.5 × 1 + 1 = 17.5 513 × 5 + 6 = 2571
17.5 × 2 + 2 = 37 2571 × 6 + 7 = 15433
37 × 4 + 4 = 152
152 × 8 + 8 = 1224 27. Ans. (C)
1224 × 16 + 16 = 19600 304 ÷ 2 – 6 = 146
146 ÷ 2 – 5 = 68
23. Ans. (D) 68 ÷ 2 – 4 = 30
197 – 193 = 4 30 ÷ 2 – 3 = 12
208 – 197 = 11 (4 × 3 – 1) 12 ÷ 2 – 2 = 4
240 – 208 = 32 (11 × 3 – 1) 4÷2– 1=1
335 – 240 = 95 (32 × 3 – 1)
619 – 335 = 284 (95 × 3 – 1) 28. Ans. (E)
1470 – 619 = 851 (284 × 3 – 1) 3×2– 1=5
5 × 3 – 2 = 13
24. Ans. (B) 13 × 5 – 3 = 62
243 + 12 = 244 62 × 7 – 4 = 430
244 – 32 = 235 430 × 11 – 5 = 4725
235 + 52 = 260 4725 × 13 – 6 = 61419

https://youtu https ://ww https://instag https://


be.com/chan w.fa cebook. ram.com/aas
nel/UCYa4_Jr com/a ashis hisharorasoci t.me/st
70 Orf8R5Kz2uO al?utm_medi
tccXQ
ha rorasocial
um=copy_link udified
29. Ans. (B) (56 – 1) × 5 = 275
19 × 4 = 76 (275 – 1) × 6 = 1644
76 + 4 = 80
80 × 4 = 320 34. Ans. (A)
320 + 4 = 324 237.25 ÷ 0.5 + 0.5 = 475
324 × 4 = 1296 475 ÷ 1 + 1 = 476
1296 + 4 = 1300 476 ÷ 2 + 2 = 240
240 ÷ 4 + 4 = 64
30. Ans. (D) 64 ÷ 8 + 8 = 16
124 × 0.25 + 1 = 32 16 ÷ 16 + 16 = 17
32 × 0.5 + 1 = 17
17 × 1 + 1 = 18 35. Ans. (D)
18 × 2 + 1 = 37 54 – 37 = 17
37 × 4 + 1 = 149 88 – 54 = 34 (17 + 17)
149 × 8 + 1 = 1193 141 – 88 = 53 (34 + 19)
217 – 141 = 76 (53 + 23)
31. Ans. (C) 322 – 217 = 105 (23 + 29)
23 + (63 + 1) = 240 458 – 322 = 136 (29 + 31)
240 + (53 + 1) = 366
366 + (43 + 1) = 431 36. Ans. (C)
431 + (33 + 1) = 459 143 + 1 × 3 = 146
459 + (23 + 1) = 468 146 + 3 × 5 = 161
468 + (13 + 1) = 470 161 + 5 × 7 = 196
196 + 7 × 9 = 259
32. Ans. (B) 259 + 9 × 11 = 358
1.5 × 4 = 6 358 + 11 × 13 = 501
4 × 6 = 24
6 × 24 = 144 37. Ans. (B)
24 × 144 = 3456 111 + 23 = 119
119 – 32 = 110
33. Ans. (D) 110 + 53 = 235
(5 – 1) × 1 = 4 235 – 72 = 186
(4 – 1) × 2 = 6 186 + 113 = 1517
(6 – 1) × 3 = 15 1517 – 132 = 1348
(15 – 1) × 4 = 56

https://youtu https ://ww https://instag https://


be.com/chan w.fa cebook. ram.com/aas
nel/UCYa4_Jr com/a ashis hisharorasoci t.me/st
71 Orf8R5Kz2uO al?utm_medi
tccXQ
ha rorasocial
um=copy_link udified
38. Ans. (A)
39 × 1 – 9 = 30
30 × 2 – 18 = 42
42 × 3 – 27 = 99
99 × 4 – 36 = 360
360 × 5 – 45 = 1755
1755 × 6 – 54 = 10476

https://youtu https ://ww https://instag https://


be.com/chan w.fa cebook. ram.com/aas
nel/UCYa4_Jr com/a ashis hisharorasoci t.me/st
72 Orf8R5Kz2uO al?utm_medi
tccXQ
ha rorasocial
um=copy_link udified
https://t.mhttps/t.me
e/studified/studified

https://instagram.com/aashisharora
social?utm_medium = copy_link

https://youtube.com/channel
/UCYa4_JrOrf8R5Kz2uOtccXQ

https://www.facebook.
com/aashisharorasocial

https://youtu https://instagra
https ://ww
be.com/chan w.fa cebook.
m.com/aashish https://
arorasocial(?)
73
nel/UCYa4_Jr
Orf8R5Kz2uO
com/a ashis utm_medium = t.me/st
ha rorasocial copy_link
tccXQ udified
1. A and B started a business by investing certain sum in the ratio 8 :
7; respectively for 5 years. If 25% of the total profit is donated in an
orphanage and A's share is Rs. 1760, then find the profit generated
from the business.
A औय B ने एक ननजश्चत याशि को क्रभि् 8 : 7 के अनुऩात भें 5 वर्षों के शरए ननवे ि
कयके एक व्मवसाम िुरू ककमा। मदद कु र राब का 25% एक अनाथारम भें दान कय
ददमा जाता है औय A का दहस्सा 1760 रुऩमे है, तो व्मवसाम से उत्ऩन्न राब ऻात
कीजजए।
(A) Rs. 3600 (B) Rs. 5200 (C) Rs. 4400
(D) Rs. 6800 (E) None of these

2. A person purchased petrol at Rs. 8, Rs. 10 and Rs. 16 per litre in


three successive years. If he spent Rs. 3600 each year on petrol, then
find the approximate average cost (per litre) paid by him in the given
three successive years.
एक व्मजक्त ने रगाताय तीन वर्षों भें 8 रुऩमे , 10 रुऩमे औय 16 रुऩमे प्रनत रीटय ऩय
ऩे ट्रोर खयीदा। मदद वह प्रत्मे क वर्षष ऩे ट्रोर ऩय 3600 रुऩमे खचष कयता है, तो ददए गए
तीन क्रशभक वर्षों भें उसके द्वाया बुगतान की गई अनुभाननत औसत रागत (प्रनत रीटय)
ऻात कीजजए।
(A) Rs. 9 (B) Rs. 10 (C) Rs. 11
(D) Rs. 12 (E) None of these

3. A seller purchased certain number of mobiles at prices ranging from


Rs. 1800 to Rs. 2200 and sold them at price ranging from Rs. 1950 to
Rs. 2400. Find the maximum profit that can be earned by selling 8
such mobiles.
एक ववक्रेता ने 1800 रुऩमे से 2200 रुऩमे तक की कीभतों ऩय कुछ भोफाइर खयीदे औय
उन्हें 1950 रुऩमे से 2400 रुऩमे के फीच भें फे च ददमा। ऐसे 8 भोफाइर फे चकय
अचधकतभ राब ऻात कीजजए।
(A) Rs. 5400 (B) Rs. 4800 (C) Rs. 3500
(D) Rs. 5200 (E) None of these

4. The average of 18 observations calculated by a student was 72.50,


but when he again went through his observations, he realized that he
took the values ​of the three observations as 46.5, 72.5 and 52.25
instead of 52.75, 42.5 and 88.5 respectively and also realized that the
total observations taken by him were 17. Find the original average of
all the observations taken by him.
https://youtu https://instagra
https ://ww
be.com/chan w.fa cebook.
m.com/aashish https://
arorasocial(?)
74
nel/UCYa4_Jr
Orf8R5Kz2uO
com/a ashis utm_medium = t.me/st
ha rorasocial copy_link
tccXQ udified
एक छात्र द्वाया ऩरयकशरत 18 प्रे ऺणों का औसत 72.50 था, रे ककन जफ उसने कपय से
अऩने प्रे ऺणों को दे खा, तो उसने भहसूस ककमा कक उसने तीन अवरोकनों के भूल्मों को
क्रभि् 52.75, 42.5 औय 88.5 के फजाम 46.5, 72.5 औय 52.25 के रूऩ भें शरमा
औय मह बी ने भहसूस ककमा कक उसके द्वाया शरए गए कु र प्रे ऺण 17 थे। उसके द्वाया
शरए गए सबी प्रे ऺणों का भूर औसत ऻात कीजजए।
(A) 80.5 (B) 82.75 (C) 77.5
(D) 77.5 (E) None of these

5. Amir can do a work as fast as Rahul and Mohit can do it by


working together. If Rahul alone can complete the work in 18 hours
and Mohit in 24 hours, then find the time taken by Amir to complete
the work.
आशभय एक काभ को उतनी ही ते ज ी से कय सकते हैं जजतनी ते ज ी से याहुर औय भोदहत
शभरकय काभ कयके कय सकते हैं । मदद याहुर अके रे कामष को 18 घॊटे भें औय भोदहत
24 घॊटे भें ऩूय ा कय सकता है, तो आशभय द्वाया कामष को ऩूय ा कयने भें शरमा गमा सभम
ऻात कीजजए।
(A) 8 5/3 hour (B) 8 hour (C) 6 hour
(D) 10 2/7 hour (E) None of these

6. The simple interest received on a certain sum is Rs. 360 less than
the sum invested. If the sum was invested at 14% p.a. for 5 years,
then find the simple interest received.
एक ननजश्चत याशि ऩय प्राप्त साधायण ब्माज ननवे ि की गई याशि से 360 रुऩमे कभ है।
मदद याशि को 14% प्रनत वर्षष की दय से 5 वर्षों के शरए ननवे ि ककमा गमा था, तो प्राप्त
साधायण ब्माज ऻात कीजजए।
(A) Rs. 720 (B) Rs. 840 (C) Rs. 1440
(D) Rs. 1220 (E) None of these

7. A boat can travel a distance of 128 km downstream and 72 km


upstream in 7 hours. If ratio of speed of boat in still water to speed of
stream is 7 : 1 respectively, then find the time taken by the boat to
cover 384 km downstream as well upstream?
एक नाव धाया के अनुकूर 128 ककभी औय धाया के प्रनतकूर 72 ककभी की दयू ी 7 घॊटे भें
तम कय सकती है। मदद िाॊत जर भें नाव की गनत का धाया की गनत से अनुऩात क्रभि्
7 : 1 है, तो नाव द्वाया धाया के प्रनतकू र 384 ककभी की दयू ी तम कयने भें रगने वारा
सभम ऻात कीजजए।
(A) 16 hours (B) 24 hours (C) 20 hours
(D) 28 hours (E) None of these

8. A rectangular field cost Rs. 192 for leveling at 50 paisa per square
meter. If the ratio of length to breadth is 8 : 3. Find the breadth of
field?
50 ऩैसे प्रनत वगष भीटय की दय से सभतर कयने के शरए एक आमताकाय खेत की कीभत
https://instagra
https://youtu https ://ww
be.com/chan w.fa cebook.
m.com/aashish https://
arorasocial(?)
75
nel/UCYa4_Jr
Orf8R5Kz2uO
com/a ashis utm_medium = t.me/st
ha rorasocial copy_link
tccXQ udified
192 रुऩमे है। मदद रॊफाई औय चौडाई का अनुऩात 8 : 3 है। भैदान की चौडाई ऻात
कीजजए।
(A) 10 m (B) 9 m (C) 8m
(D) 12 m (E) None of these

9. In how many ways a group of 5 men and 2 women be made out of


total of 7 men and 3 women?
कु र 7 ऩुरुर्षों औय 3 भदहराओॊ भें से ककतने प्रकाय से 5 ऩुरुर्षों औय 2 भदहराओॊ का एक
सभूह फनामा जाता है?
(A) 45 (B) 90 (C) 63
(D) 105 (E) None of these

10. Rakesh sees a train passing over a bridge of length 2 km. The
length of train is half of the length of bridge. If the train passes the
bridge in 3 minutes then find the speed of the train?
याकेि एक ट्रे न को 2 ककभी रॊफे ऩुर के ऊऩय से गुज यते हुए दे खता है। ट्रे न की रॊफाई
ऩुर की रॊफाई की आधी है। मदद ट्रे न ऩुर को 3 शभनट भें ऩाय कयती है तो ट्रे न की गनत
ऻात कीजजए।
(A) 45 km/hr (B) 60 km/hr (C) 75 km/hr
(D) 30 km/hr (E) None of these

11. Arnav started a business with an investment of Rs. 6000 and after
‘T' years Anish joins him with Rs. 7000. After ‘T' more years Arnav
withdraws Rs. 2500 and at the end of 12 years, the ratio of profit
share of Arnav to Anish is 1 : 1 respectively, then what is the value of
‘T'?
अनषव ने 6000 रुऩमे के ननवे ि के साथ एक व्मवसाम िुरू ककमा औय ‘T' वर्षों के फाद
अनीि 7000 रुऩमे के साथ उससे जु ड गमा। ' T ' अनतरयक्त वर्षों के फाद अनषव ने
2500 रुऩमे ननकारे औय 12 वर्षष के अॊत भें अनषव औय अनीि के राबाॊि का अनुऩात 1
: 1 है, तो ' T ' का भान क्मा है?
(A) 3 (B) 2.5 (C) 3.5
(D) Cannot be determined (E) None of these

12. When the efficiency of Rajan is increased by 42.84%, he can


complete a work in 14 days lesser than the number of days taken by
him to complete the same work when his efficiency is decreased by
28.56%. The number of days required by Rajan to complete the work
with his original efficiency is :
जफ याजन की दऺता भें 42.84% की वरवि होती है, तो वह उसी कामष को ऩूय ा कयने भें
उसके द्वाया शरए गए ददनों की सॊख्मा से 14 ददनों से कभ सभम भें कामष ऩूया कय
सकता है, जफ उसकी दऺता 28.56% कभ होती है। याजन द्वाया अऩनी भूर दऺता के
साथ कामष को ऩूय ा कयने के शरए आवश्मक ददनों की सॊख्मा है :

https://youtu https://instagra
https ://ww
be.com/chan w.fa cebook.
m.com/aashish https://
arorasocial(?)
76
nel/UCYa4_Jr
Orf8R5Kz2uO
com/a ashis utm_medium = t.me/st
ha rorasocial copy_link
tccXQ udified
(A) 25 days (B) 20 days (C) 15 days
(D) 10 days (E) None of these

13. While travelling with speed 63 km/h, a bike takes 5 min more than
actual time to cover 'D + 26' km and with speed 72 km/h, it takes 10
min less than actual time to cover the same distance, then find the
value of 'D'.
63 ककभी/घॊटा की गनत से मात्रा कयते सभम, एक फाइक 'D + 26' ककभी की दयू ी तम
कयने भें वास्तववक सभम से 5 शभनट अचधक रे ती है औय 72 ककभी/घॊटा की गनत से
सभान दयू ी तम कयने भें वास्तववक सभम से 10 शभनट कभ रे ती है, तो 'D' का भान
ऻात कीजजए।
(A) 126 (B) 156 (C) 200
(D) 100 (E) None of these

14. 16 litres mixture of petrol and water is mixed with 26 litres of


another mixture that contains petrol and water in the ratio 9 : 4
respectively. Resultant mixture contains petrol and water in the ratio 9
: 5 respectively, then what is the difference between the quantities of
petrol and water in the first mixture?
16 रीटय ऩे ट्रोर औय ऩानी के शभश्ण को 26 रीटय दस ू ये शभश्ण भें शभरामा जाता है,
जजसभें ऩे ट्रोर औय ऩानी का अनुऩात 9 : 4 है। अॊनतभ शभश्ण भें ऩे ट्रोर औय ऩानी का
अनुऩात 9 : 5 है, तो ऩहरे शभश्ण भें ऩे ट्रोर औय ऩानी की भात्रा के फीच का अॊतय
ककतना है?
(A) 2 (B) 7 (C) 9
(D) Cannot be determined (E) None of these

15. The diagonals of a rhombus are 16 cm and 12 cm respectively.


Find the side of a square whose area is 100 cm² more than the area
of rhombus.
एक सभचतुबुषज के ववकणष क्रभि् 16 से भी औय 12 से भी हैं । उस वगष की बुज ा ऻात
कीजजए, जजसका ऺेत्रपर सभचतुबुषज के ऺेत्रपर से 100 वगष से भी अचधक है।
(A) 8 cm (B) 16 cm (C) 12 cm
(D) 14 cm (E) None of these

16. From his total amount, Rajeev gave 33.33% to Jayati. Rajeev gave
16.66% from his initial amount to Jyona and Jayati gave 42% of the
amount received from Rajeev to Jyona. If Jyona receives total Rs.
11040, then what is the total amount given by Rajeev to Jayati?
याजीव ने अऩनी कु र धनयाशि भें से 33.33% जमनत को ददमा। याजीव ने अऩनी
प्रायॊ शबक धनयाशि का 16.66% ज्मोना को ददमा औय जमनत ने याजीव से प्राप्त धनयाशि
का 42% ज्मोना को ददमा। मदद ज्मोना को कु र 11040 रुऩमे शभरते हैं , तो याजीव
द्वाया जमनत को दी गई कु र धनयाशि ककतनी है?

https://youtu https://instagra
https ://ww
be.com/chan w.fa cebook.
m.com/aashish https://
arorasocial(?)
77
nel/UCYa4_Jr
Orf8R5Kz2uO
com/a ashis utm_medium = t.me/st
ha rorasocial copy_link
tccXQ udified
(A) Rs. 6000 (B) Rs. 15000 (C) Rs. 12000
(D) Rs. 16000 (E) None of these

17. A shopkeeper purchases an item for Rs. 5500 and marked it up by


18.18%. He sold it after giving two successive discounts of 20% and
Rs. 200 respectively. Had he interchanged both the discounts, what
would have been the change in the profit amount?
एक दक ु ानदाय एक वस्तु को 5500 रुऩमे भें खयीदता है औय उस ऩय 18.18% अचधक
भूल्म वरवि कयता है। उसने इसे क्रभि् 20% औय 200 रुऩमे की दो क्रशभक छूट दे क य
फे च ददमा। मदद उसने दोनों छू टों को आऩस भें फदर ददमा होता, तो राब की धनयाशि भें
ककतना ऩरयवतषन होता?
(A) Rs. 56 (B) Rs. 40 (C) Rs. 66
(D) Rs. 50 (E) None of these

18. The average price of the items A, B, C and D is Rs. 51 while the
average price of the items A, C, D, E and F is Rs. 40. If the average
price of the items B, E and F is Rs. 44, then find the price of item B.
वस्तुओ ॊ A, B, C औय D का औसत भल् ू म 51 रुऩमे है, जफकक वस्तुओ ॊ A, C, D, E
औय F का औसत भूल्म 40 रुऩमे है। मदद वस्तु B, E औय F का औसत भूल्म 44
रुऩमे है, तो वस्तु B का भूल्म ऻात कीजजए।
(A) 88 (B) 72 (C) 60
(D) 68 (E) None of these

19. Quantity 1 : Mehul purchased 150 shirts from a whole seller at the
rate of 300 each. He spends Rs. 4200 on packaging. He marked the
price of each shirts at Rs. 500. He offers 20% discount on marked
price, then what is the profit percentage earned by him?
Quantity II : The ratio of monthly income of Rajat and Komal is 2 : 3
and ratio of monthly income of Rajat and Akash is 3 : 5. If Komal 's
monthly income is Rs. 9889789, then monthly income of Komal is what
percentage of the monthly income of Akash?
भात्रा। : भे हुर ने एक थोक ववक्रे ता से 300 प्रत्मे क की दय से 150 कभीज खयीदी। वह
ऩैकेजजॊग ऩय 4200 रुऩमे खचष कयता है। उसने प्रत्मे क कभीज की कीभत 500 रु. अॊककत
ककमा। वह अॊककत भूल्म ऩय 20% की छूट प्रदान कयता है, तो उसके द्वाया अजजषत राब
प्रनतित क्मा है?
भात्रा ॥ : यजत औय कोभर की भाशसक आम का अनुऩात 2 : 3 है तथा यजत औय
आकाि की भाशसक आम का अनुऩात 3 : 5 है। मदद कोभर की भाशसक आम
98,897,89 रु. है, तो कोभर की भाशसक आम आकाि की भाशसक आम का ककतने
प्रनतित है?
(A)Quantity I > Quantity II (B) Quantity I < Quantity II
(C) Quantity I ≥ Quantity II (D) Quantity I ≤ Quantity II
(E) Quantity I = Quantity II

https://youtu https://instagra
https ://ww
be.com/chan w.fa cebook.
m.com/aashish https://
arorasocial(?)
78
nel/UCYa4_Jr
Orf8R5Kz2uO
com/a ashis utm_medium = t.me/st
ha rorasocial copy_link
tccXQ udified
20. Quantity I : Akram invested Rs. 25000, Ben invested Rs. 36,000
and Chavi invested Rs. 45000 for 6 months, 5 months and 4 months
respectively. The profit made by Chavi is Rs. 1980. Find the value of
total profit.
Quantity II : Find the interest received on a loan of Rs. 27360 at
8.33% per annum compounded annually for 2 years.
भात्रा I : अकयभ ने 25000 रुऩमे का ननवे ि ककमा, फे न ने 36,000 रुऩमे का ननवे ि
ककमा औय छवव ने 45000 रुऩमे का ननवे ि ककमा क्रभि् 6 भहीने , 5 भहीने औय 4
भहीने के शरए। छवव द्वाया अजजषत राब 1980 रुऩमे है। कुर राब का भान ऻात
कीजजए।
भात्रा II : 27360 रु के ऋण ऩय 8.33% प्रनत वर्षष की दय से 2 वर्षों के शरए वावर्षषक
रूऩ से प्राप्त चक्रवरवि ब्माज ऻात कीजजए।
(A)Quantity I > Quantity II (B) Quantity I < Quantity II
(C) Quantity I ≥ Quantity II (D) Quantity I ≤ Quantity II
(E) Quantity I = Quantity II

21. An article is sold at a discount of 15% and marked by Rs. 200


above its cost price. Had it been marked 26% more than its cost price
and given the same discount as before, a profit of Rs. 58 could have
been earned. How much profit would have actually been earned?
एक वस्तु को 15% की छू ट ऩय फे चा जाता है औय उसके क्रम भूल्म से 200 रुऩमे
अचधक अॊककत ककमा जाता है। मदद उसे इसके क्रम भूल्म से 26% अचधक अॊककत ककमा
जाता औय ऩहरे की तयह सभान छू ट दी जाती, तो 58 रुऩमे का राब अजजषत ककमा जा
सकता था। वास्तव भें ककतना राब अजजषत ककमा होगा?
(A) Rs. 70 (B) Rs. 60 (C) Rs. 75
(D) Rs. 50 (E) Rs. 65

22. A student got 120 marks in English subject and his average marks
in five other subjects taken together is 115. The maximum marks in
each subject is 150. When his marks are taken wrong in English
subject then he has got total 71.67% marks in all the six given
subjects combined, then what is the ratio of his actual marks and
wrong marks taken in the same subject English?
एक ववद्माथी ने अॊग्रेज ी ववर्षम भें 120 अॊक प्राप्त ककमा औय सॊमुक्त रूऩ से शरए गए
ऩाॊच अन्म ववर्षमों भें उसके औसत अॊक 115 हैं । प्रत्मे क ववर्षम भें अचधकतभ अॊक 150
हैं । जफ उसके अॊक अॊग्रेज ी ववर्षम भें गरत शरए गए हो तो उसने सॊमुक्त रूऩ से सबी
छ् ददए गए ववर्षमों भें कु र 71.67% अॊक प्राप्त ककमे हैं , तो सभान ववर्षम अॊग्रेज ी भें
शरए गए उसके वास्तववक अॊक औय गरत अॊक का अनुऩात ककतना है?
(A) 11 : 8 (B) 12 : 7 (C) 9 : 8
(D) 8 : 11 (E) Cannot be determined

23. A mixture of 300 litres of milk and water contains 30% water. If W
litres of water is added to the mixture in order https://instagra
to increase the
https://youtu https ://ww
be.com/chan w.fa cebook.
m.com/aashish https://
arorasocial(?)
79
nel/UCYa4_Jr
Orf8R5Kz2uO
com/a ashis utm_medium = t.me/st
ha rorasocial copy_link
tccXQ udified
percentage of water by 20% and W litres of mixture is taken out and
mixed with W/3 litres of milk, then what will be the ratio of milk to
water in the new mixture?
300 रीटय दध ू औय ऩानी के एक शभश्ण भें 30% ऩानी है। मदद W रीटय ऩानी को
शभश्ण भें इस तयह से शभरामा जाता है कक ऩानी की 20% प्रनतित वरवि हो जाती है
औय W रीटय शभश्ण को ननकारा जाता है औय W/3 रीटय दध ू के साथ शभचश्त ककमा
जाता है, तो नए शभश्ण भें दध
ू औय ऩानी का अनुऩात क्मा होगा?
(A) 4 : 3 (B) 2 : 7 (C) 5 : 3
(D) 1 : 1 (E) Cannot be determined

24. Puneet invested Rs. 12000 at the rate of simple interest of r% and
got a total interest of Rs. 5400 after 3 years If he had invested the
same amount at the same rate of compound interest for 2 years, then
what percent the compound interest earned by him would be less/more
than the simple interest received by him?
ऩुनीत ने 12000 रुऩमे r% की साधायण ब्माज की दय ऩय ननवे ि ककमे औय उसे 3 वर्षों
के फाद कु र 5400 रुऩमे ब्माज के प्राप्त हुए। मदद उसने 2 वर्षों के शरए सभान दय ऩय
चक्रवरवि ब्माज ऩय सभान याशि का ननवे ि ककमा हो, तो उसके द्वाया प्राप्त चक्रवरवि
ब्माज, उसके द्वाया प्राप्त साधायण ब्माज से ककतने प्रनतित कभ/अचधक होगा?
(A) 27.65% (B) 26.67% (C) 28.33%
(D) 29.45% (E) Rs. 65

25. 10 year ago, the average age of Raj, Dimple, Chandan and Sonu
is 27 years and the ratio of the present ages of Raj and Dimple is 11
: 7. The age of Dimple after 24 years is equal to the sum of the ages
of Chandan and Sonu 12 years ago. What is the average age of
Dimple and Sonu after four years?
10 वर्षष ऩहरे , याज, डडॊऩर, चॊदन औय सोनू की औसत आमु 27 वर्षष है तथा याज औय
डडॊऩर की वतषभान आमु का अनुऩात 11 : 7 है। 24 वर्षष के फाद डडॊऩर की आमु, 12
वर्षष ऩहरे चॊदन औय सोनु की आमु के मोग के फयाफय है। चाय वर्षो के फाद डडॊऩर औय
सोनू की औसत आमु क्मा है?
(A) 24 (B) 30 (C) 34
(D) 22 (E) Cannot be determined

26. The length of train P is 25% less than that of rail Q. Rail P and
rail Q are running at the same speed and rail P can cross a 40 meter
long bridge in 8 seconds while rail Q can cross a 30 meter long
bridge in 10 seconds. If train P crosses train Q in T second while
running in opposite direction, then what is the value of T?
ट्रे न P की रॊफाई ये र Q की रॊफाई से 25% कभ है। ये र P औय ये र Q सभान गनत से
चर यहे हैं औय ये र P 40 भीटय रॊफे ऩुर को 8 से कॊड भें ऩाय कय सकती है जफकक ये र
Q 30 भीटय रॊफे ऩुर को 8 से कॊड भें ऩाय कय सकती है। मदद ट्रे न P ववऩयीत ददिा भें
दौडते हुए ट्रे न Q को T से कॊड भें ऩाय कयती है, तो T का भान क्मा है?
https://instagra
https://youtu https ://ww
be.com/chan w.fa cebook.
m.com/aashish https://
arorasocial(?)
80
nel/UCYa4_Jr
Orf8R5Kz2uO
com/a ashis utm_medium = t.me/st
ha rorasocial copy_link
tccXQ udified
(A) 8 sec (B) 9 sec (C) 10 sec
(D) CND (E) None of these

27. A circle whose circumference is equal to the sum of the perimeters


of a rectangle and a square. The ratio of the length of the rectangle
and the side of the square is 9 : 7 and the area of ​the rectangle is 64
cm² less than the area of ​the square. If the side of the square is 40%
more than the breadth of the rectangle, then find the circumference of
that circle?
एक वरत्त जजसकी ऩरयचध एक आमत औय एक वगष के ऩरयभाऩों के मोग के फयाफय होती
है। आमत की रॊफाई औय वगष की बुज ा का अनुऩात 9 : 7 है औय आमत का ऺेत्रपर
वगष के ऺेत्रपर से 64 से भी² कभ है। मदद वगष की बुज ा आमत की चौडाई से 40%
अचधक है, तो उस वत्र त की ऩरयचध ऻात कीजजए।
(A) 224 cm (B) 112 cm (C) 336 cm
(D) 448 cm (E) None of these

28. Average weight of person P and Q is 11 kg less than that of


person R. The ratio of the weights of person P and the sum of the
weights of persons R and S together is 1 : 3 and the weights of
persons P and S are 24 kg and 34 kg respectively.
Quantity 1 : Weight of person Q.
Quantity 2 : Average weight of person P, R and S.
व्मजक्त P औय Q का औसत वजन व्मजक्त R के वजन से 11 ककग्रा कभ है। व्मजक्त
P के वजन तथा सॊमुक्त तौय ऩय व्मजक्त R औय S के वजनों के मोग का अनुऩात 1 :
3 है तथा व्मजक्त P औय S का वजन क्रभि् 24 ककग्रा औय 34 ककग्रा है।
भात्रा 1 : व्मजक्त Q का वजन।
भात्रा 2 : व्मजक्त P, R औय S का औसत वजन।
(A) Quantity 1 = Quantity 2 (B) Quantity 1>Quantity 2
(C) Quantity 1<Quantity 2 (D) Quantity 1≥Quantity 2
(E) None of these

29. A boat can travel 64 km downstream in 8 hours and 52 km


upstream in 13 hours Find the difference between the distance covered
by the boat in downstream in 10 hours. and the distance covered by
the boat in still water in 4 hours.
एक नाव धाया के अनुकूर ददिा भें 8 घॊटे भें 64 ककभी औय धाया के प्रनतकूर ददिा भें
13 घॊटे भें 52 ककभी की दयू ी तम कय सकती है। नाव द्वाया धाया के अनुकू र ददिा भें
10 घॊटे भें तम की गई दयू ी औय नाव द्वाया जस्थय ऩानी भें 4 घॊटे भें तम की गई दयू ी
के फीच का अॊतय ऻात कयें ।
(A) 58 km (B) 56 km (C) 60 km
(D) 54 km (E) None of these

30. The number of days taken by B to do a piece of work is 12 days


https://instagra
https://youtu https ://ww
be.com/chan w.fa cebook.
m.com/aashish https://
arorasocial(?)
81
nel/UCYa4_Jr
Orf8R5Kz2uO
com/a ashis utm_medium = t.me/st
ha rorasocial copy_link
tccXQ udified
more than the number of days taken by A. After working jointly for 9
days they found that 1/5th of the work was left. In how many days can
A alone finish the whole work?
एक काभ को कयने भें B द्वाया शरए गए ददनों की सॊख्मा, A द्वाया शरए गए ददनों की
सॊख्मा से 12 ददन अचधक हैं । 9 ददन सॊमुक्त रूऩ से काभ कयने के फाद उन्हें ऻात हुआ
कक काभ का 1/5 बाग िे र्ष था। A अके रे सम्ऩूणष काभ को ककतने ददनों भें सभाप्त कय
सकता है?
(A) 12 (B) 18 (C) 24
(D) 30 (E) None of these

31. In an election 15% of the voters on the voters’ list did not cast
votes and 150 voters cast their ballot papers blank. There were only
two candidates. The winner was supported by 55% of all voters in the
list and he got 900 votes more than his rival. The number of voters on
the list was :
एक चुनाव भें भतदाता सूची के 15% भतदाताओॊ ने वोट नहीॊ डारा औय 150
भतदाताओॊ ने अऩने भतऩत्र खारी डारे । के वर दो उम्भीदवाय थे। ववजे ता को सूची भें
सबी भतदाताओॊ के 55% का सभथषन प्राप्त था औय उसे अऩने प्रनतद्वॊद्वी से 900 भत
अचधक शभरे । सूची भें भतदाताओॊ की सॊख्मा थी :
(A) 3000 (B) 7500 (C) 2500
(D) 4500 (E) None of these

32. The prices of two chairs are in the ratio 5 : 7. If the price of the
first chair be increased by 25% and that of the second by Rs. 14, the
original ratio remains the same. The original price of the second chair
is :
दो कुशसषमों के भूल्म 5 : 7 के अनुऩात भें हैं । मदद ऩहरी कुसी की कीभत भें 25% औय
दसू यी की कीभत भें 14 रुऩमे की वरवि की जाती है, तो भूर अनुऩात वही यहता है। दस ू यी
कुसी का भूर भूल्म है :
(A) Rs. 88 (B) Rs. 48 (C) Rs. 56
(D) Rs. 40 (E) None of these

33. A shopkeeper sells an article at a profit of 25%. If he want to


mark a certain amount on the article and gives a discount of 30%, the
profit of the article is same as previous, then find the mark price of
the article if the cost price of the article is Rs. 798.
एक दुक ानदाय एक वस्तु को 25% के राब ऩय फे चता है। मदद वह वस्तु ऩय एक
ननजश्चत याशि अॊककत कयना चाहता है औय 30% की छूट दे ता है, तो वस्तु का राब
वऩछरे के सभान है, तो वस्तु का अॊककत भूल्म ऻात कीजजए, मदद वस्तु का क्रम भूल्म
798 रुऩमे है
(A) Rs. 1400 (B) Rs. 1490 (C) Rs. 1480
(D) Rs. 1425 (E) None of these

https://youtu https://instagra
https ://ww
be.com/chan w.fa cebook.
m.com/aashish https://
arorasocial(?)
82
nel/UCYa4_Jr
Orf8R5Kz2uO
com/a ashis utm_medium = t.me/st
ha rorasocial copy_link
tccXQ udified
34. A trader buys goods at 12.5 % discount on marked price. If he
wants to make a profitof 14.28% after allowing à discount of 11.11%,
by what percent should his marked price be greater than the original
marked price?
एक व्माऩायी अॊककत भूल्म ऩय 12.5% ​छूट ऩय साभान खयीदता है। मदद वह 11.11%
की छू ट दे ने के फाद 14.28% का राब कभाना चाहता है, तो उसका अॊककत भूल्म भूर
अॊककत भूल्म से ककतने प्रनतित अचधक होना चादहए?
(A) 12.5% (B) 15% (C) 16.66%
(D) 20% (E) None of these

35. The area of a rectangular field is 480 square metres. If the length
is 20 per cent more than the breadth, what is breadth of the
rectangular field?
एक आमताकाय भैदान का ऺेत्रपर 480 वगष भीटय है। मदद रॊफाई, चौडाई से 20 प्रनतित
अचधक है, तो आमताकाय खेत की चौडाई क्मा है?
(A) 25 m (B) 16 m (C) 25 m
(D) 20 m (E) None of these

36. Two vessels contain a mixture of milk and water. The content of
the milk is 60% in the first vessel and 30% in the second vessel. In
what ratio must the mixtures from the first and the second vessel be
taken to form a mixture containing 50% milk?
दो फतषनों भें दध
ू औय ऩानी का शभश्ण है। ऩहरे फतषन भें दध
ू की भात्रा 60% औय दस ू ये
फतषन भें 30% है। 50% दध ू वारा शभश्ण फनाने के शरए ऩहरे औय दस ू ये फतषन के
शभश्ण को ककस अनुऩात भें शरमा जाना चादहए?
(A) 1 : 3 (B) 1 : 4 (C) 2 : 1
(D) 4 : 5 (E) None of these

37. In a school there was sufficient food for 150 children for 30 days.
After 25 days 120 children left the fort. For how many extra days will
the rest of the food last for the remaining children?
एक स्कू र भें 150 फच्चों के शरए 30 ददनों के शरए ऩमाषप्त बोजन था। 25 ददनों के
फाद 120 फच्चे ककरे से चरे गए। फचा हुआ बोजन फचे हुए फच्चों के शरए ककतने
अनतरयक्त ददनों तक चरे गा?
(A) 18 days (B) 16 days (C) 20 days
(D) 15 days (E) None of these

38. Shivam invested Rs. 16500 in a scheme A which offered simple


interest at the rate of R% for two years. After two years he withdrew
the principle amount plus interest and invested the entire amount in
another scheme B, which offers simple interest at the rate of 25% for
two years. If he got total interest of Rs. 10560 after two years, find
value of R. https://instagra
https://youtu https ://ww
be.com/chan w.fa cebook.
m.com/aashish https://
arorasocial(?)
83
nel/UCYa4_Jr
Orf8R5Kz2uO
com/a ashis utm_medium = t.me/st
ha rorasocial copy_link
tccXQ udified
शिवभ ने एक मोजना A भें 16500 रुऩमे का ननवे ि ककमा, जजसभें दो सार के शरए
R% की दय से साधायण ब्माज ददमा गमा था। दो वर्षों के फाद उसने भूर याशि औय
ब्माज को वाऩस रे शरमा औय ऩयू ी याशि को दस
ू यी मोजना B भें ननवे ि कय ददमा, जो दो
सार के शरए 25% की दय से साधायण ब्माज प्रदान कयती है। मदद उसे दो वर्षष फाद
10560 रुऩमे का कुर ब्माज शभरता है, तो R का भूल्म ऻात कयें ।
(A) 18% (B) 16% (C) 15%
(D) 14% (E) None of these

39. Ram is 12.5% less efficient than Shyam & both complete the work
together in 20 days. If another person karan takes 5 days more than
Ram and Shyam together to complete the same work. Then find in
how many days all three complete the work together?
याभ, श्माभ से 12.5% ​क भ कुिर है औय दोनों शभरकय कामष को 20 ददनों भें ऩयू ा कयते
हैं । मदद कोई अन्म व्मजक्त कयण सभान कामष को ऩूय ा कयने भें याभ औय श्माभ से
शभराकय 5 ददन अचधक रे ता है, तो ऻात कीजजए, कक तीनों शभरकय कामष को ककतने
ददनों भें ऩूया कयें गे?
(A) 11(1/9) days (B) 12 days (C) 16(1/3) days
(D) 7 days (E) None of these

40. There are 8 red balls, 4 blue balls and 5 green balls in a bag.
Five balls are chosen at random. What is the probability of their being
2 red balls, 1 blue ball and 2 green balls?
एक थैरे भें 8 रार गेंदें, 4 नीरी गेंदें औय 5 हयी गेंदें हैं । ऩाॊच गेंदों को मादृजच्छक रूऩ
से चुना जाता है। उनके 2 रार गेंद, 1 नीरी गेंद औय 2 हयी गेंद होने की प्रानमकता
क्मा है?
(A) 40/221 (B) 25/443 (C) 31/223
(D) 143/82 (E) None of these

41. When a shopkeeper sold an article at 3/8th of the marked price,


the loss percentage was 25%. The article should be sold at what
fraction of marked price to gain 50%.
जफ एक दुक ानदाय ने एक वस्तु को अॊककत भूल्म के 3/8वें दहस्से ऩय फे चा, तो हानन
प्रनतित 25% था। वस्तु को 50% राब प्राप्त कयने के शरए अॊककत भूल्म के ककस अॊि
ऩय फे चा जाना चादहए।
(A) 4/5 (B) 2/3 (C) 3/4
(D) 5/4 (E) None of these

42. Train P crosses a pole and platform in 12 seconds and 14


seconds. The length of platform is 146.4 metre. What will be the
length of train Q if it is equal to the sum of the half of the length of
train P and thrice the length of the platform?
ट्रे न P एक ऩोर औय प्रे टपॉभष को 12 से कॊड औय 14 से कॊड भें ऩाय कयती है। प्रे टपाभष
की रॊफाई 146.4 भीटय है। ट्रे न Q की रॊफाई क्मा होगी मदद मह ट्रे न P की रॊफाई के
https://instagra
https://youtu https ://ww
be.com/chan w.fa cebook.
m.com/aashish https://
arorasocial(?)
84
nel/UCYa4_Jr
Orf8R5Kz2uO
com/a ashis utm_medium = t.me/st
ha rorasocial copy_link
tccXQ udified
आधे औय प्रे टपॉभष की रॊफाई के तीन गुना के मोग के फयाफय है?
(A) 876.4 m (B) 878.4 m (C) 880.4 m
(D) 882.4 m (E) None of these

43. A liter of ethanol weighs a kilogram and a liter of another liquid


weighs 1500 gram. A mixture of the two weigh 1350 gram/liter. Find
the ratio of volume of ethanol and liquid respectively in the new
mixture.
एक रीटय इथेनॉर का वजन एक ककरोग्राभ है औय एक रीटय दस ू ये तयर का वजन
1500 ग्राभ है। दोनों के शभश्ण का वजन 1350 ग्राभ/रीटय है। नए शभश्ण भें क्रभि्
इथेनॉर औय तयर के आमतन का अनुऩात ऻात कीजजए।
(A) 3 : 7 (B) 4 : 3 (C) 3 : 4
(D) 7 : 3 (E) None of these

44. Ramesh can eat a 5 star chocolate in 15 days and Suresh is 25%
less efficient than Ramesh. They started eating the same chocolate on
alternative days starting with Ramesh. In how many days the chocolate
will be finished?
यभे ि एक 5 स्टाय चॉकरे ट 15 ददनों भें खा सकता है औय सुये ि, यभे ि से 25% कभ
कुिर है। उसने यभे ि से िुरू कयते हुए वे एकान्तय ददनों ऩय चॉकरे ट खाते है। ककतने
ददनों भें चॉकरे ट्स ख़तभ हो जाएॊगी?
(A) 19 (B) 17 (C) 16
(D) 20 (E) None of these

45. A sum of money at compound interest amounts to Rs. 5070 in 2


years and Rs. 6591 in 3 years. Find the principal.
एक याशि चक्रवरवि ब्माज ऩय 2 वर्षष भें 5070 रुऩमे औय 3 वर्षों भें 6591 रुऩमे हो जाती
है। याशि ककतनी है?
(A) Rs. 3900 (B) Rs. 4000 (C) Rs. 3000
(D) Rs. 4200 (E) None of these

46. Upadesh's present age is 28(4/7)% more than what he was 8 year
ago. After how many years will 125% of his age will be equal to his
age 4 year ago?
उऩदे ि की वतषभान आमु 8 वर्षष ऩहरे की आमु से 28(4/7)% अचधक है। ककतने वर्षष फाद
उसकी आमु का 125% उसकी 4 वर्षष ऩूवष की आमु के फयाफय हो जाएगा?
(A) 12 years (B) 4 years (C) 8 years
(D) 6 years (E) None of these

47. Ananya started a business with an investment equal to 4/7 times


that of Aaliya, who joins her after 8 months. Ananya left after 4 month
of joining Alia. If at the end of 1(1/2) year, the profit earned by Aliya
is Rs. 2450, then what is the total profit received by both of them?
https://instagra
https://youtu https ://ww
be.com/chan w.fa cebook.
m.com/aashish https://
arorasocial(?)
85
nel/UCYa4_Jr
Orf8R5Kz2uO
com/a ashis utm_medium = t.me/st
ha rorasocial copy_link
tccXQ udified
अनन्मा ने आशरमा, जो 8 भहीने फाद उससे जु डती है, के ननवे ि के 4/7 गुना के फयाफय
ननवे ि के साथ एक व्मवसाम िुरू ककमा। आशरमा से जु डने के 4 भहीने फाद अनन्मा
चरी गई। मदद 1(1/2) वर्षष के अॊत भें , आशरमा को प्राप्त राब 2450 रुऩमे है, तो उन
दोनों को प्राप्त कु र राब ककतना है?
(A) Rs. 4310 (B) Rs. 4130 (C) Rs. 4110
(D) Rs. 4330 (E) None of these

48. In a triangle,base is increased by 7.69%, then by how much %


height should be decreased such that area of triangle remains same?
एक त्रत्रबुज भें , आधाय भें 7.69% की वरवि की जाती है, तो ककतनी% ऊॊचाई कभ की
जानी चादहए ताकक त्रत्रबुज का ऺेत्रपर सभान यहे ?
(A) 7.41% (B) 7.18% (C) 7.25%
(D) 7.14% (E) None of these

49. If A shopkeeper marks his article 71(3/7)% above cost price and
sell it after a discount that he suffers a loss of 57(1/7)%. Find the
discount given.
मदद एक दुक ानदाय अऩनी वस्तु ऩय क्रम भूल्म से 71(3/7)% अचधक अॊककत कयता है
औय छू ट के फाद उसे फे चता है तो उसे 57(1/7)% की हानन होती है। दी गई छू ट ऻात
कीजजए।
(A) 46% (B) 25% (C) 78%
(D) 75% (E) None of these

50. The population of a town decrease by 40% every year. If the


present population of the town is 11880, then what was the population
three years ago?
एक कस्फे की जनसॊख्मा भें प्रनतवर्षष 40% की कभी होती है। मदद कस्फे की वतषभान
जनसॊख्मा 11880 है, तो तीन वर्षष ऩूवष जनसॊख्मा ककतनी थी?
(A) 20000 (B) 55000 (C) 15000
(D) 10000 (E) None of these

51. In a family of 12 adults and some minors, the average


consumption of pulses per head per month is 11.4 kg, while the
average consumption for adult is 16 kg per head and for minors is 9
kg per head. The number of minors in the family will be :
12 वमस्कों औय कुछ अव्मसको के ऩरयवाय भें , प्रनत व्मजक्त प्रनत भाह दार की औसत
खऩत 11.4 ककरोग्राभ है, जफकक वमस्कों के शरए औसत खऩत 16 ककरोग्राभ प्रनत
व्मजक्त औय अव्मसको के शरए प्रनत व्मजक्त 9 ककरोग्राभ है। ऩरयवाय भें अवमस्कों की
सॊख्मा होगी :
(A) 26 (B) 46 (C) 23
(D) 43 (E) None of these

52. A sum of money becomes Rs. 6864 in 4 years at 14% rate of


https://instagra
https://youtu https ://ww
be.com/chan w.fa cebook.
m.com/aashish https://
arorasocial(?)
86
nel/UCYa4_Jr
Orf8R5Kz2uO
com/a ashis utm_medium = t.me/st
ha rorasocial copy_link
tccXQ udified
simple interest and another sum of money becomes Rs. 9078 in 6
years at 13% rate of simple interest. What is the total of both of the
sums.
एक धनयाशि साधायण ब्माज की 14% की दय ऩय 4 वर्षो भें 6864 रु हो जाती है औय
अन्म याशि साधायण ब्माज की 13% की दय ऩय 6 वर्षो भें 9078 रु हो जाती है। दोनों
धनयाशिमो का मोग ऻात कये ।
(A) Rs. 9000 (B) Rs. 9550 (C) Rs. 9500
(D) Rs. 9800 (E) None of these

53. Arpit bought a bundle of cloth of length 50m and width 1/5m. What
more would be the cost of a bundle of same cloth of length 75m and
width 2/3m at the rate of Rs. 36 per m2?
अवऩषत ने 50 भीटय रॊफाई औय 1/5 भीटय चौडाई के कऩडे का एक फॊडर खयीदा। 36
रूऩमे प्रनत वगष भीटय की दय से 75 भीटय रॊफाई औय 2/3 भीटय चौडाई के सभान कऩडे
के एक फॊडर का रागत भूल्म ककतना अचधक होगा?
(A) Rs. 1040 (B) Rs. 1440 (C) Rs. 1480
(D) Rs. 1065 (E) None of these

54. Priya obtained 76% marks in an examination. Riya obtained 16


marks more than Priya and obtained 84% marks in that examination. If
Richa obtained 160 marks in this examination then what is her
percentage score?
वप्रमा ने एक ऩयीऺा भें 76% अॊक प्राप्त ककए। रयमा ने वप्रमा से 16 अॊक अचधक प्राप्त
ककए औय उस ऩयीऺा भें 84% अॊक प्राप्त ककए। मदद ऋचा ने इस ऩयीऺा भें 160 अॊक
प्राप्त ककए तो उसका प्रनतित अॊक क्मा है?
(A) 80% (B) 40% (C) 36%
(D) 45% (E) None of these

55. Akbar’s present age is 20% less than Birbal’s present age. 8 years
later, Akbar will be 6/7 times as old as Birbal, what is the present age
of Birbal?
अकफय की वतषभान आमु फीयफर की वतषभान आमु से 20% कभ है। 8 वर्षष फाद अकफय
की आमु फीयफर से 6/7 गुना होगी, फीयफर की वतषभान आमु क्मा है?
(A) 24 years (B) 16 years (C) 20 years
(D) 26 years (E) None of these

56. Deepti earned 30% as share from profit and Stuti earned 35% as
share from profit, rest was earned by Shruti. If Deepti earned for 3
months, Stuti for 4 months and Shruti for 2 months and Deepti’s
investment was Rs. 37000, then what is the difference between
investment of Stuti and Shruti?
दीजप्त ने राब से अॊि के रूऩ भें 30% अजजषत ककमा औय स्तुनत ने राब से अॊि के रूऩ
भें 35% अजजषत ककमा, िे र्ष को श्ुनत ने अजजषत ककमा। मददhttps://instagra
दीजप्त ने 3 भहीने के शरए,
https://youtu https ://ww
be.com/chan w.fa cebook.
m.com/aashish https://
arorasocial(?)
87
nel/UCYa4_Jr
Orf8R5Kz2uO
com/a ashis utm_medium = t.me/st
ha rorasocial copy_link
tccXQ udified
स्तुनत ने 4 भहीने के शरए औय श्ुनत ने 2 भहीने के शरए औय दीजप्त ने 37000 रु का
ननवे ि ककमा तो स्तुनत औय श्ुनत की ननवे शित याशि भें ककतना अॊतय है?
(A) Rs. 56000 (B) Rs. 42000 (C) Rs. 7000
(D) Rs. 5600 (E) None of these

57. The cost price of mixture of two acids A and B is Rs. 42 per liter.
The cost price if 5 liters of acid A is Rs. 30 per liter. If the cost price
of acid B is Rs. 50 per liter then how many liters of acid B should be
mixed?
दो अम्र A औय B के शभश्ण का क्रम भूल्म 42 रुऩमे प्रनत रीटय है। मदद 5 रीटय
अम्र A का क्रम भूल्म 30 रुऩमे प्रनत रीटय है। मदद अम्र B का क्रम भूल्म 50 रुऩमे
प्रनत रीटय है तो अम्र B ककतने रीटय शभरामा जाना चादहए?2/3 भीटय चौडाई के
सभान कऩडे के एक फॊडर का रागत भल् ू म ककतना अचधक होगा?
(A) 7.5 L (B) 6 L (C) 8 L
(D) 6.5 L (E) None of these

58. Upstream speed of boat and stream speed are in the ratio 3 : 1
respectively and the difference between the downstream speed and
speed of boat is 6 km/hr. if speed of boat is increased by 6km/hr, then
what is the time taken by boat to cover 288 km downstream?
नाव की धाया के प्रनतकू र गनत औय धाया की गनत का अनुऩात क्रभि् 3 : 1 है औय
धाया के अनुकूर गनत औय नाव की गनत के फीच का अॊतय 6 ककभी/घॊटा है। मदद नाव
की गनत भें 6 ककभी/घॊटा की वरवि कय दी जाती है, तो नाव द्वाया धाया के अनुकू र 288
ककभी की दयू ी तम कयने भें ककतना सभम रगेगा?.
(A) 8 hours (B) 4 hours (C) 10 hours
(D) 14 hours (E) None of these

59. Pipe A and pipe B can fill a tank in 4 hours and 6 hours
respectively. However there is a leak in the tank which can empty the
half tank in 2 hours. If pipes A and B are opened together then after
how much times will the tank be filled?
ऩाइऩ A औय ऩाइऩ B एक टैं क को क्रभि् 4 घॊटे औय 6 घॊटे भें बय सकते हैं । हाराॊकक
टैं क भें एक रयसाव है जो 2 घॊटे भें आधा टैं क खारी कय सकता है। मदद ऩाइऩ A औय
B को एक साथ खोर ददमा जाए तो टॊक ी ककतने सभम फाद बये गी?
(A) 8 hours (B) 4 hours (C) 10 hours
(D) 14 hours (E) None of these

60. Arman sold 4/7 of a share at a profit at 42% and the rest at a
loss of 14%. If he got an overall profit of Rs. 324, what is the actual
cost of the share?
अयभान ने एक िे मय का 4/7 दहस्सा 42% के राब ऩय औय िे र्ष को 14% की हानन ऩय
फे चा। मदद उसे कुर शभराकय रु. 324 का राब हुआ, िे मय की वास्तववक रागत क्मा
है? https://instagra
https://youtu https ://ww
be.com/chan w.fa cebook.
m.com/aashish https://
arorasocial(?)
88
nel/UCYa4_Jr
Orf8R5Kz2uO
com/a ashis utm_medium = t.me/st
ha rorasocial copy_link
tccXQ udified
(A) Rs. 3600 (B) Rs. 5400 (C) Rs. 1800
(D) Rs. 1880 (E) None of these

61. The length of a cuboid is 4 m greater than its breadth and height
is 4 m. if the total surface area of the cuboid is 248 m2 then what is
the sum of lengths of all its edges.
एक घनाब की रॊफाई उसकी चौडाई से 4 भीटय अचधक है औय ऊॊचाई 4 भीटय है। मदद
घनाब का कु र ऩष्र ठीम ऺेत्रपर 248 वगष भीटय है, तो इसके सबी ककनायों की रॊफाई का
मोग क्मा है?
(A) 40 m (B) 80 m (C) 36 m
(D) 58 m (E) None of these

62. Average cost price of two items A and B is Rs. 380. A and B are
sold at 15% and 19% profit respectively and total selling price of A
and B is Rs. 881.6, then find the cost price of item A?
दो वस्तुओ ॊ A औय B का औसत क्रम भूल्म रु. 380 है। A औय B क्रभि् 15% औय
19% राब ऩय फे चे जाते हैं औय A औय B का कु र त्रफक्री भूल्म रु. 881.6 है, तो वस्तु
A का क्रम भूल्म ऻात कीजजए।
(A) Rs. 590 (B) Rs. 570 (C) Rs. 490
(D) Rs. 516 (E) None of these

63. A mixture contains 70% oil and remaining water. By some method,
when 15 liter oil is replaced by water, then % of oil becomes 30%.
What is the initial quantity of oil?
एक शभश्ण भें 70% ते र औय िे र्ष ऩानी है। ककसी ववचध से , जफ 15 रीटय ते र को
ऩानी से फदर ददमा जाता है, तो ते र का % 30% हो जाता है। ते र की प्रायॊ शबक भात्रा
क्मा है?
(A) 26.25 L (B) 25.26 L (C) 18.25 L
(D) 32.5 L (E) None of these

64. Anil and Mukesh started a business with initial capital Rs. 1400
and Rs. 2000 respectively. After 3 months, Anil adds Rs. 600 more
and Mukesh withdraws Rs. 400 from his capital. If the end of 7
months from the start, the total profit was Rs. 650 then find the
difference between the profit share of Mukesh and Anil?
अननर औय भुकेि ने क्रभि् 1400 रु. औय 2000 रु. की प्रायम्बक ऩूॉज ी के साथ एक
व्मवसाम िुरू ककमा। 3 भहीनो के फाद अननर, 600 रु. अनतरयक्त जोडता है औय भुके ि
अऩनी ऩूॉज ी से 400 रु. ननकरता है। मदद प्रायम्ब से 7 भहीनो के अॊत भें अननर औय
भुके ि का कु र राब 6150 रु. है, तो दोनों के राब दहस्से का अॊतय ऻात कये ।
(A) Rs. 40 (B) Rs. 80 (C) Rs. 50
(D) Rs. 70 (E) None of these

65. The salary of Raj and Simran are in the ratio 5 : 2 and their
https://instagra
https://youtu https ://ww
be.com/chan w.fa cebook.
m.com/aashish https://
arorasocial(?)
89
nel/UCYa4_Jr
Orf8R5Kz2uO
com/a ashis utm_medium = t.me/st
ha rorasocial copy_link
tccXQ udified
expenditure are in the ratio 4 : 1. if both of them save Rs. 1000 then
find double of Raj’s salary.
याज औय शसभयन का वे तन 5 : 2 के अनुऩात भें है औय उनका व्मम 4 : 1 के अनुऩात
भें है। मदद वे दोनों 1000 रुऩमे फचाते हैं , तो याज के वे तन का दोगुना ऻात कीजजए।
(A) Rs. 4000 (B) Rs. 8000 (C) Rs. 10000
(D) Rs. 15000 (E) None of these

66. P and Q can do a work in 8 days, Q and R can do the same


work in 12 days and P, Q, R together can do the work in 6 days. In
how many days P and R can do the whole work?
P औय Q एक काभ को 8 ददनों भें कय सकते हैं , Q औय R उसी काभ को 12 ददनों भें
कय सकते हैं औय P, Q, R शभरकय उस काभ को 6 ददनों भें कय सकते हैं । P औय R
ऩयू े काभ को ककतने ददनों भें कय सकते हैं ?
(A) 8 days (B) 10 days (C) 6 days
(D) 12 days (E) None of these

67. In an examination, 10% passed with distinction, 28% passed in 1st


class and 14% passed in 2nd class. If remaining 96 students failed in
examination, what is the total number of students passed with
distinction.
एक ऩयीऺा भें 10% डडजस्टॊक्िन के साथ, 28% प्रथभ श्ेणी भें औय 14% द्ववतीम श्ेणी
भें उत्तीणष हुए। मदद िे र्ष 96 ववद्माथी ऩयीऺा भें अनुत्तीणष हो जाते हैं , तो ऩयीऺा भें
डडजस्टॊक्िन के साथ उत्तीणष हुए ववद्माचथषमों की कु र सॊख्मा ककतनी है?
(A) 20 (B) 56 (C) 28
(D) 102 (E) None of these

68. Two trains of same speeds with lengths in ratio 6 : 11 completely


cross each other in 34 seconds. The longer train can cross a man
running at platform with a speed of 10m/s in the same direction of the
train, in 55 seconds. What is the length of longer train?
सभान गनत की दो ट्रे नें जजनकी रॊफाई 6 : 11 के अनुऩात भें हैं , एक दस ू ये को 34 से कॊड
भें ऩूय ी तयह से ऩाय कयती हैं । रॊफी ट्रे न सभान ददिा भें 10 भीटय/से केंड की गनत से
प्रे टपॉभष ऩय दौड यहे एक व्मजक्त को 55 से कॊड भें ऩाय कय सकती है। रॊफी ट्रे न की
रॊफाई क्मा है?
(A) 2200 m (B) 2400 m (C) 2875 m
(D) 1331 m (E) None of these

69. A sum of Rs.(X – 1000) is lent you at 10% per annum simple
interest for 12 years and another sum of Rs.(3X + 1000) is lent out at
15% per annum simple interest for 16 years. If total interest earned is
Rs. 43200, then what is the value of X?
12 वर्षो के शरए (X – 1000) रु की धनयाशि 10% प्रनत वर्षष साधायण ब्माज की दय ऩय
उधाय दी जाती है औय 16 वर्षो के शरए (3X + 1000) रु https://instagra
की अन्म धनयाशि 15% प्रनत
https://youtu https ://ww
be.com/chan w.fa cebook.
m.com/aashish https://
arorasocial(?)
90
nel/UCYa4_Jr
Orf8R5Kz2uO
com/a ashis utm_medium = t.me/st
ha rorasocial copy_link
tccXQ udified
वर्षष साधायण ब्माज ऩय उधय दी जाती है। मदद अजजषत कु र ब्माज 43200 रु है, तो X
का भान ककतना है?
(A) Rs. 5000 (B) Rs. 2000 (C) Rs. 4000
(D) Rs. 6000 (E) None of these

70. Average age of 6 sons of a family is 8 year. Average age of son


together with their parents is 22 years. If the father is older than the
mother by 16 year, then the age of mother is?
एक ऩरयवाय के 6 ऩुत्रों की औसत आमु 8 वर्षष है। अऩने भाता – वऩता के साथ ऩुत्र की
औसत आमु 22 वर्षष है। मदद वऩता, भाता से 16 वर्षष फडा है, तो भाता की आमु है?
(A) 72 years (B) 80 years (C) 56 years
(D) 62 years (E) None of these

71. If Pankaj and Sahil started a company by investing Rs. 32500 and
Rs. 22500, as Pankaj is an active partner he invests additional Rs.
18500 from the part of his salary and Shubham joined them after 8
months and he invests Rs. 35500, then what is the ratio of
investments of Pankaj, Sahil and Shubham after 1 year?
मदद ऩॊक ज औय सादहर ने 32500 रुऩमे औय 22500 रुऩमे का ननवे ि कयके एक कॊऩनी
िुरू की, क्मोंकक ऩॊकज एक सकक्रम बागीदाय है, तो वह अऩने वे तन के दहस्से से
अनतरयक्त 18500 रुऩमे का ननवे ि कयता है औय िुबभ 8 भहीने फाद उनसे जु ड जाता है
औय वह 35500 रुऩमे का ननवे ि कयता है, तो 1 वर्षष फाद ऩॊकज, सादहर औय िुबभ के
ननवे ि का अनुऩात क्मा है?
(A) 306 : 147 : 71 (B) 306 : 135 : 71 (C) 306 : 175 : 71
(D) 306 : 157 : 71 (E) None of these

72. Raju invested money in a scheme which claims to provide 28% per
annum simple interest but it actually provides 15% per annum
compound interest. If the difference between Raju's actual and
expected interest after 2 years is Rs. 3562.5, what was the money
Raju invested?
याज ू ने एक ऐसी मोजना भें धन का ननवे ि ककमा जो 28% प्रनत वर्षष साधायण ब्माज
प्रदान कयने का दावा कयती है रे ककन मह वास्तव भें 15% प्रनत वर्षष चक्रवरवि ब्माज
प्रदान कयती है। मदद 2 वर्षष फाद याज ू के वास्तववक औय अऩे क्षऺत ब्माज के फीच का
अॊतय 3562.5 रुऩमे है, तो याज ू ने ककतना ऩैसा ननवे ि ककमा था?
(A) Rs. 12600 (B) Rs. 15500 (C) Rs. 15000
(D) Rs. 12000 (E) None of these

73. From a well shuffled deck of 52 card, 3 cards are chosen at


random without replacement.
Quantity 1 : What is the probability that all the three cards chosen are
Black?
Quantity 2 : 4/51. https://instagra
https://youtu https ://ww
be.com/chan w.fa cebook.
m.com/aashish https://
arorasocial(?)
91
nel/UCYa4_Jr
Orf8R5Kz2uO
com/a ashis utm_medium = t.me/st
ha rorasocial copy_link
tccXQ udified
52 काडों के एक अच्छी तयह से पेयफदर ककए गए डे क से , त्रफना प्रनतस्थाऩन के
मादृजच्छक रूऩ से 3 काडष चुने जाते हैं ।
भात्रा 1 : इसकी क्मा प्रानमकता है कक चन ु े गए तीनों ऩत्ते कारे हैं ?
भात्रा 2 : 4/51
(A) Quantity I > Quantity II (B) Quantity I ≥ Quantity II
(C) Quantity I < Quantity II (D) Quantity I ≤ Quantity II
(E) None of these

74. In a family, three daughters were born for every 5 years one after
the other. If the ratio of present ages of father and the eldest daughter
is 9 : 5 and the average of the present ages of the 3 daughters is 15
years, what will be the ratio of the ages of the father and the
youngest daughter after 7 years?
एक ऩरयवाय भें हय 5 सार भें एक के फाद एक तीन फे दटमों का जन्भ होता है। मदद
वऩता औय सफसे फडी ऩुत्री की वतषभान आमु का अनुऩात 9 : 5 है औय 3 फे दटमों की
वतषभान आमु का औसत 15 वर्षष है, तो 7 वर्षष फाद वऩता औय सफसे छोटी ऩुत्री की आमु
का अनुऩात क्मा होगा?
(A) 43 : 17 (B) 17 : 43 (C) 43 : 18
(D) 18 : 43 (E) None of these

75. Quantity 1 : Total cost of a Mobile and A.C. is Rs. 45000 in 2015.
If the cost of the mobile increased by 44.44% and the cost of A.C.
increases by 16% then what will be the total cost of both the items in
year 2016?
Quantity 2 : 70000.
भात्रा 1 : 2015 भें एक भोफाइर औय एसी की कु र रागत 45000 रुऩमे है। मदद
भोफाइर की रागत भें 44.44% की वरवि होती है औय एसी की रागत भें 16% की वरवि
होती है, तो वर्षष 2016 भें दोनों वस्तुओ ॊ की कु र रागत क्मा होगी?
भात्रा 2 : 70000.
(A) Quantity I > Quantity II (B) Quantity I ≥ Quantity II
(C) Quantity I < Quantity II (D) Quantity I ≤ Quantity II
(E) None of these

76. Three pipes P, Q and R can fill a tank completely 15 minutes, 18


minutes, and 36 minutes respectively. The tank is empty, and all the
three pipes are opened. All the three pipes are used to fill three
different liquids in such a way that pipe P fills Roohafza, pipe Q fills
Cold – drink and pipe R fills Soda.
Quantity 1 : After 12 Minutes quantity of soda in tank
Quantity 2 : After 10 minutes quantity of Roohafza in tank.
तीन ऩाइऩ P, Q औय R एक टैं क को क्रभि् 15 शभनट, 18 शभनट औय 36 शभनट भें
ऩूयी तयह से बय सकते हैं । टैं क खारी है, औय तीनों ऩाइऩ खोर ददए गए हैं । तीनों
ऩाइऩों का उऩमोग तीन अरग – अरग तयर ऩदाथष बयने कhttps://instagra
े शरए इस प्रकाय ककमा जाता
https://youtu https ://ww
be.com/chan w.fa cebook.
m.com/aashish https://
arorasocial(?)
92
nel/UCYa4_Jr
Orf8R5Kz2uO
com/a ashis utm_medium = t.me/st
ha rorasocial copy_link
tccXQ udified
है कक ऩाइऩ P रूहफ्जा बयता है, ऩाइऩ Q कोल्ड – डरॊक बयता है औय ऩाइऩ R सोडा
बयता है।
भात्रा 1 : 12 शभनट के फाद टैं क भें सोडा की भात्रा
भात्रा 2 : 10 शभनट के फाद टैं क भें रूहपजा की भात्रा।
(A) Quantity I > Quantity II (B) Quantity I ≥ Quantity II
(C) Quantity I < Quantity II (D) Quantity I ≤ Quantity II
(E) None of these

77. A rectangular board is decorated by pasting ‘P' square shaped


colored papers of 154 cm2 surface area each on it. Perimeter of top
surface of rectangular board is 660 cm and its sides are in the ratio 7
: 8 respectively. What is the value of ‘P'?
एक आमताकाय फोडष को 154 से भी2 सतह ऺेत्रपर वारे 'P' वगष आकाय के यॊ गीन कागज
चचऩका कय सजामा जाता है। आमताकाय फोडष की ऊऩयी सतह का ऩरयभाऩ 660 से भी है
औय इसकी बुज ाएॉ क्रभि् 7 : 8 के अनुऩात भें हैं । 'P' का भान क्मा है?
(A) 166 (B) 154 (C) 176
(D) 174 (E) None of these

78. Marked price and cost price of a Motorbike is in the ratio 7 : 12


and marked price and selling price is in the ratio 3 : 2. If the
shopkeeper had a loss of Rs. 1320, then find the discount given?
एक भोटयफाइक का अॊककत भूल्म औय रागत भूल्म 7 : 12 के अनुऩात भें है औय
अॊककत भूल्म औय त्रफक्री भूल्म 3 : 2 के अनुऩात भें है। मदद दुक ानदाय को रु. 1320 का
नुक्सान हुआ, तो दी गई छू ट ऻात कीजजए।
(A) Rs. 420 (B) Rs. 460 (C) Rs. 440
(D) Rs. 400 (E) None of these

79. Rice costing Rs. 25/kg mixed with the rice costing Rs. 32/kg. If the
mixture is sold at Rs. 30/kg, it makes the seller profit of 11.11%, in
what ratio were the two varieties of rice mixed?
25 रु./ककग्रा की रागत वारे चावर को 32 रु./ककग्रा वारे चावर के साथ शभरामा जाता
है, मदद शभश्ण को 30 रु./ककग्रा ऩय फे चा जाता है, तो वह ववक्रे ता को 11.11% का राब
होता है, चावर की दो ककस्भो को ककस अनुऩात भें शभरामा गमा था?
(A) 2 : 5 (B) 5 : 2 (C) 4 : 3
(D) 3 : 4 (E) None of these

80. A Boat can go 35 km downstream and 45 km upstream in 14


hours and it can go 14 km downstream and 15 km upstream in 5
hours,find the time taken by boat in covering 186 km in still water?
एक नाव धाया के अनुकूर 35 ककभी औय धाया के प्रनतकूर 45 ककभी 14 घॊटे भें जा
सकती है औय मह 14 ककभी धाया के अनुकू र औय 15 ककभी धाया के प्रनतकू र 5 घॊटे भें
जा सकती है, िाॊत ऩानी भें 186 ककभी की दयू ी तम कयने भें नाव द्वाया शरमा गमा
सभम ऻात कयें ? https://instagra
https://youtu https ://ww
be.com/chan w.fa cebook.
m.com/aashish https://
arorasocial(?)
93
nel/UCYa4_Jr
Orf8R5Kz2uO
com/a ashis utm_medium = t.me/st
ha rorasocial copy_link
tccXQ udified
(A) 25 hours (B) 17 hours (C) 41 hours
(D) 31 hours (E) None of these

81. Ram and Shyam invest Rs. 2100 and Rs. 3100 for different
number of months. The sum of their time periods is 8 months. If
Ram’s share out of Rs. 19800 is Rs. 10500, then for how many
months Shyam has invested?
याभ औय श्माभ अरग – अरग सॊख्मा भें भहीनों के शरए क्रभि : 2100 रु औय 3100
रु. का ननवे ि कयते हैं । उनके ननवे ि की सभमववचध का मोग 8 भहीने है। मदद 19800
रु. के राब भें से याभ का दहस्सा 10500 रु. है, तो श्माभ ने ककतने भहीनो के शरए
अऩनी याशि का ननवे ि ककमा?
(A) 4 months (B) 3 months (C) 2 months
(D) 5 months (E) None of these

82. Sum of present ages of A and B is 68 years and sum of present


ages of B and C is 56 years. If average age of A, B and C is 37
years then find the ratio of present ages of A, B and C?
A औय B की वतषभान आमु का मोग 68 वर्षष है औय B औय C की वतषभान आमु का
मोग 56 वर्षष है। मदद A, B औय C की औसत आमु 37 वर्षष है, तो A, B औय C की
वतषभान आमु का अनुऩात ऻात कीजजए।
(A) 43 : 13 : 55 (B) 65 : 13 : 43 (C) 55 : 13 : 43
(D) 55 : 15 : 43 (E) None of these

40% of Ramu’s monthly salary spent on studies, 34% is spent on


shopping and 25% of the remaining is spent on travel, then he is left
with Rs. 3900, find the monthly salary of Nobita whose monthly salary
is 25% more than Ramu’ salary?
याभू के भाशसक वे तन का 40% ऩढाई ऩय खचष ककमा जाता है, 34% खयीदायी ऩय खचष
ककमा जाता है औय िे र्ष का 25% मात्रा ऩय खचष ककमा जाता है, तो उसके ऩास 3900 रु.
फच जाते हैं । नोत्रफता का भाशसक वे तन ऻात कीजजए, जजसका भाशसक वे तन याभू के
वे तन से 25% अचधक है?
(A) Rs. 18000 (B) Rs. 14200 (C) Rs. 25000
(D) Rs. 20000 (E) None of these

84. A man distributed Rs. 37840 among four organization P, Q, R, S


such that P got Rs. 160 less than Q who got Rs. 10000 and the ratio
of amount received by R and S is 8 : 7. What is the average share of
P and S?
एक आदभी ने 37840 रु. चाय सॊगठन P, Q, R, S भें इस प्रकाय फाॊटे है कक P को Q
से 160 रु. कभ शभरे जजसे 10000 रु. शभरे है औय R औय S को प्राप्त याशि का
अनुऩात 8 : 7 है। P औय S का औसत दहस्सा ककतना है?
(A) Rs. 8640 (B) Rs. 9120 (C) Rs. 9880
(D) Rs. 8740 (E) None of these https://instagra
https://youtu https ://ww
be.com/chan w.fa cebook.
m.com/aashish https://
arorasocial(?)
94
nel/UCYa4_Jr
Orf8R5Kz2uO
com/a ashis utm_medium = t.me/st
ha rorasocial copy_link
tccXQ udified
85. Two pipes P and Q can fill a tank in 27 minutes and 36 minutes.
If both the pipes are opened simultaneously after how much time Q
should be closed so that tank is full in 18 minutes?
दो ऩाइऩ P औय Q एक टैं क को 27 शभनट औय 36 शभनट भें बय सकते हैं । मदद दोनों
ऩाइऩों को एक साथ खोर ददमा जाए तो Q को ककतने सभम फाद फॊद कय दे ना चादहए
ताकक टॊक ी 18 शभनट भें बय जाए?
(A) 12 minutes (B) 13 minutes (C) 14 minutes
(D) 15 minutes (E) None of these

86. A boat can cover 126 km upstream in 31.5 hours and speed of
current is 2 km/hr. find the time taken by boat in covering 192 km
downstream and 48 km upstream?
एक नाव धाया के प्रनतकू र 126 ककभी की दयू ी 31.5 घॊटे भें तम कय सकती है औय धाया
की गनत 2 ककभी/घॊटा है। नाव द्वाया धाया के अनुकूर 192 ककभी औय धाया के प्रनतकूर
48 ककभी की दयू ी तम कयने भें ककतना सभम रगता है?
(A) 36 hours (B) 34 hours (C) 32 hours
(D) 35 hours (E) None of these

87. Two blends of a commodity costing Rs. 28 and Rs. 36 per kg are
mixed in the ratio 3 : 5 by weight. If 12.5% of the mixture sold at Rs.
35 per kg and remaining at 40 per kg, then find the profit percent
earned?
28 रु. औय 36 रु. प्रनत ककग्रा रुऩमे की रागत वारी वस्तु के दो शभश्ण बाय के अनुसाय
3 : 5 के अनुऩात भें शभरामा जाता है। मदद शभश्ण का 12.5 प्रनतित रु. 35 प्रनत
ककग्रा औय िे र्ष 40 प्रनत ककग्रा, तो अजजषत राब प्रनतित ऻात कीजजए।
(A) 19.31% (B) 15.31% (C) 14.21 %
(D) 18.31% (D) None of these

88. What is the difference between the compound interests on Rs.


12000 for 1 year at 18% per annum compounded yearly and half
yearly?
1 वर्षष के शरए 12000 रु. 18% चक्रवरवि ब्माज की दय से प्राप्त वावर्षषक औय अधषवावर्षषक
ब्माज भें क्मा अॊतय होगा?
(A) Rs. 95.5 (B) Rs. 102.6 (C) Rs. 97.2
(D) Rs. 89.5 (E) None of these

89. P can complete 80% of a work in 32 days and Q takes 12.5%


more time than P to complete the same work. Find the time taken by
P and Q together to complete 85% of the work?
P एक कामष के 80% को 32 ददनों भें ऩूया कय सकता है औय Q उसी काभ को ऩूया
कयने भें P की तुरना भें 12.5% अचधक सभम रे ता है। 85% काभ को ऩूय ा कयने के
शरए दोनों P औय Q द्वाया शरमा गमा सभम ऻात कयें ?

https://youtu https://instagra
https ://ww
be.com/chan w.fa cebook.
m.com/aashish https://
arorasocial(?)
95
nel/UCYa4_Jr
Orf8R5Kz2uO
com/a ashis utm_medium = t.me/st
ha rorasocial copy_link
tccXQ udified
(A) 16 days (B) 18 days (C) 19 days
(D) 20 days (E) None of these

90. Two cars travel form city P to city Q at a speed of 57 km/hr and
152 Km/hr. if one takes 5 hours less than the other then find the
distance between P and Q?
दो कायें िहय P से िहय Q तक 57 ककभी/घॊटा औय 152 ककभी/घॊटा की गनत से मात्रा
कयती हैं । मदद एक दस
ू ये से 5 घॊटे कभ रे ता है, तो P औय Q के फीच की दयू ी ऻात
कीजजए।
(A) 496 km (B) 456 km (C) 546 km
(D) 556 km (E) None of these

91. If upstream speeds of two boats P and Q are in the ratio 4 : 5


and their downstream speeds are in the ratio 7 : 8, then find that the
speed of Boat Q is approximately what percent more than the speed
of boat P in still water?
मदद दो नावों P औय Q की धाया के प्रनतकू र गनत 4 : 5 के अनुऩात भें है औय उनकी
अनुप्रवाह गनत 7 : 8 के अनुऩात भें है, तो ऻात कीजजए, कक नाव Q की गनत िाॊत जर
भें नाव P की गनत से रगबग ककतने प्रनतित अचधक है?
(A) 9.09% (B) 36.36% (C) 18.18%
(D) 27.27% (E) None of these

92. The ratio of the present ages of Mother and Son is 7 : 2 and the
ratio of the ages of son and father 8 years from now is 1 : 3. if the
sum of the present ages of father and mother is 81 years what is the
present age of son?
भाता औय ऩुत्र की वतषभान आमु का अनुऩात 7 : 2 है औय अफ से 8 वर्षष फाद ऩुत्र औय
वऩता की आमु का अनुऩात 1 : 3 है। मदद वऩता औय भाता की वतषभान आमु का मोग
81 वर्षष है तो ऩुत्र की वतषभान आमु क्मा है?
(A) 12 years (B) 8 years (C) 10 years
(D) 14 years (D) None of these

93. Sachin and Amit alone can complete a piece of work in 12 days
and 16 days respectively. They started working on alternate days with
Sachin starting first. After 5 days, Bhole replaced both Sachin and
Amit and started working alone. If the remaining work gets completed
in 10 days, what is the part of total work completed by Bhole alone in
one day?
सचचन औय अशभत अकेरे एक कामष को क्रभि् 12 ददन औय 16 ददन भें ऩूय ा कय सकते
हैं । उन्होंने सचचन को ऩहरे िुरू कयने के साथ वैक जल्ऩक ददनों भें काभ कयना िुरू
ककमा। 5 ददनों के फाद, बोरे ने सचचन औय अशभत दोनों को फदर ददमा औय अकेरे
काभ कयना िुरू कय ददमा। मदद िे र्ष कामष 10 ददनों भें ऩूया हो जाता है, तो अकेरे बोरे
द्वाया एक ददन भें ऩूये ककए गए कु र कामष का ककतना बागhttps://instagra
है?
https://youtu https ://ww
be.com/chan w.fa cebook.
m.com/aashish https://
arorasocial(?)
96
nel/UCYa4_Jr
Orf8R5Kz2uO
com/a ashis utm_medium = t.me/st
ha rorasocial copy_link
tccXQ udified
(A) 1/24 (B) 1/28 (C) 1/36
(D) 1/16 (E) None of these

94. The amount when principal of Rs. 3A earns compound interest at


the rate of 16(2/3)% per annum in 2 years is 6050 greater than the
amount when a principal of Rs. A earns compound interest at the rate
of 25% per annum in 2 years, what is the value of A?
3A रु. के भर ू धन ऩय 2 वर्षो भें 16(2/3)% प्रनत वर्षष की दय से चक्रववर ि ब्माज अजजषत
कयने ऩय प्राप्त धनयाशि, उस धनयाशि से 6050 रु. अचधक है, जो A रु. के भूरधन ऩय
2 वर्षो भें 25% प्रनत वर्षष की दय से चक्रवरवि ब्माज द्वाया अजजषत की जाती है, तो A का
भान ककतना है?
(A) Rs. 2400 (B) Rs. 3600 (C) Rs. 1800
(D) Rs. 2000 (E) None of these

95. Two numbers P and Q are in the ratio 8 : 11 respectively. P is


14.28% more than R and Q is 8.33% less than S. if S is 70 more than
R, then what is the average of P and R taken together?
दो सॊख्माएॉ P औय Q क्रभि् 8 : 11 के अनुऩात भें हैं । P, R से 14.28% अचधक है
औय Q, S से 8.33% कभ है। मदद S, R से 70 अचधक है, तो P औय R को शभराकय
औसत क्मा है?
(A) 105 (B) 110 (C) 102
(D) 108 (E) None of these

96. If A has invested for 8 more months than B and C has invested
for half as much time as B, ratio of their investment is 7 : 11 : 18
respectively, profit at the end of the year is Rs. 218 and C’s share out
of it is Rs. 54. for how many months has C invested his money?
मदद A ने B से 8 भहीने अचधक ननवे ि ककमा है औय C ने B से आधे सभम के शरए
ननवे ि ककमा है, उनके ननवे ि का अनुऩात क्रभि् 7 : 11 : 18 है, वर्षष के अॊत भें राब
218 रु. है औय इसभें से C का दहस्सा 54 रु. है, तो C ने ककतने भहीनों के शरए
अऩना ऩैसा ननवे ि ककमा है?
(A) 3 months (B) 2 months (C) 5 months
(D) 4 months (D) None of these

97. Shyam and Raju went to a shop and bought two TV each at
different cost. Cost price of TV bought by Shyam is Rs. 5000 more
than the cost price of TV bought by Raju. Shopkeeper earned 14.28%
on selling both the TV together which the shopkeeper bought for Rs.
14000. At what cost did Shyam buy TV?
श्माभ औय याज ू एक दुक ान ऩय गए औय अरग – अरग कीभत ऩय दो टीवी खयीदे ।
श्माभ द्वाया खयीदे गए टीवी का क्रम भूल्म याज ू द्वाया खयीदे गए टीवी के क्रम भूल्म से
5000 रु. अचधक था। दोनों टीवी को एक साथ फे चने ऩय दुक ानदाय ने 14.28% राब
अजजषत ककमा जजसे दुक ानदाय ने रु. 14000 भें खयीदा था, तो श्माभ ने टीवी ककस कीभत
https://instagra
https://youtu https ://ww
be.com/chan w.fa cebook.
m.com/aashish https://
arorasocial(?)
97
nel/UCYa4_Jr
Orf8R5Kz2uO
com/a ashis utm_medium = t.me/st
ha rorasocial copy_link
tccXQ udified
ऩय खयीदा?
(A) Rs. 5000 (B) Rs. 5500 (C) Rs. 10500
(D) Rs. 15000 (E) None of these

98. A train of 140m in length crosses a pole in 4 sec and speed of a


car is 50% less than the speed of train, if the car passes a bridge in
10 seconds then find the time taken by train to pass that bridge?
140 भीटय रॊफाई की एक ट्रे न 4 से कॊड भें एक ऩोर को ऩाय कयती है औय काय की गनत
ट्रे न की गनत से 50% कभ है मदद काय 10 से कॊड भें एक ऩुर को ऩाय कयती है तो उस
ऩुर को ऩाय कयने के शरए ट्रे न द्वाया शरमा गमा सभम ऻात कयें ?
(A) 4 seconds (B) 5 seconds (C) 6 seconds
(D) 9 seconds (E) None of these

99. P, Q and R can complete a piece of work in 12, 15, 20 days


respectively. They started the work together but P left work before 6
days of completion and Q also left the work two days after P left. In
how many days was the work completed?
P, Q औय R एक कामष को क्रभि् 12, 15, 20 ददनों भें ऩूया कय सकते हैं । उन्होंने
एक साथ काभ िुरू ककमा रे ककन P ऩूय ा होने के 6 ददन ऩहरे काभ छोड दे ता है औय Q
बी P के जाने के दो ददन फाद काभ छोड दे ता है। कामष ककतने ददनों भें ऩूया हुआ?
(A) 10.8 days (B) 8 days (C) 6 days
(D) 8.8 days (E) None of these

100. The average age of 10 teachers in a school is 35 years. A


teacher among them of age 45 years left the school whereas another
teacher whose age was 42 years joined the school. The average age
of teachers in school now is?
एक ववद्मारम भें 10 शिऺकों की औसत आमु 35 वर्षष है। उनभें से एक 45 वर्षष आमु के
शिऺक ने स्कूर छोड ददमा जफकक एक अन्म शिऺक जजसकी उम्र 42 वर्षष थी, स्कूर भें
िाशभर हो गमा। अफ ववद्मारम भें शिऺकों की औसत आमु है?
(A) 33.7 years (B) 32.7 years (C) 34.7 years
(D) 36.7 years (D) None of these

101. A boat row 222 km upstream and 245 km downstream in 81


hours if the speed of boat in still water is 18.75% more than speed of
stream, then find the speed of stream?
एक नाव 222 ककभी धाया के प्रनतकू र औय 245 ककभी धाया के अनुकू र कु र 81 घॊटे भें
चरती है मदद िाॊत ऩानी भें नाव की गनत धाया की गनत से 18.75% अचधक है, तो
धाया की गनत ऻात कीजजए।
(A) 14 km/hr (B) 16 km/hr (C) 20 km/hr
(D) 18 km/hr (E) None of these

102. Amitav is 31.25% more efficient than Bhanupriya, if Amitav alone


https://instagra
https://youtu https ://ww
be.com/chan w.fa cebook.
m.com/aashish https://
arorasocial(?)
98
nel/UCYa4_Jr
Orf8R5Kz2uO
com/a ashis utm_medium = t.me/st
ha rorasocial copy_link
tccXQ udified
can complete the whole work in 32 days, then together they can
complete what percent of work in 21 days?
अशभताव बानुवप्रमा से 31.25% अचधक कु िर है, अगय अशभताव अके रे ऩयू े काभ को 32
ददनों भें ऩूय ा कय सकता है, तो वे शभरकय काभ का ककतना प्रनतित 21 ददनों भें ऩूय ा
कयें गे?
(A) 112.625% (B) 110.625% (C) 116.625%
(D) 116.625% (E) None of these

103. In a hostel there are ‘X’ students initially and the per student
expenditure is Rs. 35. The next day number of students increased by
15 and the per student expenditure is decreased by 4 but over all
expenditure is increased by Rs. 165. what is the value of ‘X’?
एक छात्रावास भें प्रायॊ शबक रूऩ से 'X' छात्र हैं औय प्रनत छात्र व्मम रु. 35. अगरे ददन
छात्रों की सॊख्मा भें 15 की वरवि हुई औय प्रनत छात्र व्मम भें 4 की कभी आई, रे ककन
कु र व्मम भें 165 रुऩमे की वरवि हुई। 'X' का भान क्मा है?
(A) 40 (B) 60 (C) 45
(D) 75 (E) None of these

104. A 365 ml of mixture contains milk and water. When 35 ml water


and 25 ml milk added to it, then the milk become 41⅔% of water. Find
the initial quantity of milk?
एक 365 शभरीरीटय शभश्ण भें दध ू औय ऩानी होता है। जफ इसभें 35 शभरी ऩानी औय
25 शभरी दध ू शभरामा जाता है, तो शभश्ण भें दध
ू ऩानी का 41⅔% होगा। दधू की
प्रायॊ शबक भात्रा ऻात कीजजए।(
(A) 100 ml (B) 120 ml (C) 140 ml
(D) 80 ml (D) None of these

105. Marked price of an item is ‘A’ and that item is sold at 15%
discount. If the item is sold at (A – 225), then what is the discount
percent if the item is sold for Rs. 900?
एक वस्तु का अॊककत भूल्म 'A' है औय उस वस्तु को 15% की छूट ऩय फे चा जाता है।
मदद वस्तु को (A – 225) ऩय फे चा जाता है, तो वस्तु को 900 रु भें फे चने ऩय छू ट
प्रनतित क्मा है?
(A) 40% (B) 45% (C) 50 %
(D) 55% (E) None of these

106. The perimeter of two squares P and Q are in ratio 7 : 8


respectively and sum of their areas is 16272 cm2. What is the area of
square whose side is equal to the average of sides of square P and
Q?
दो वगों P औय Q के ऩरयभाऩ क्रभि् 7 : 8 के अनुऩात भें हैं औय उनके ऺेत्रपरों का
मोग 16272 से भी2 है। उस वगष का ऺेत्रपर क्मा है जजसकी बुज ा वगष P औय Q की
बुज ाओॊ के औसत के फयाफय है? https://instagra
https://youtu https ://ww
be.com/chan w.fa cebook.
m.com/aashish https://
arorasocial(?)
99
nel/UCYa4_Jr
Orf8R5Kz2uO
com/a ashis utm_medium = t.me/st
ha rorasocial copy_link
tccXQ udified
(A) 8100 cm² (B) 10000 cm² (C) 4900 cm²
(D) 6400 cm² (E) None of these

107. Mahesh was told that he will get 25% hike on his current salary
but due to pandemic he got only 19% hike, due to which his annual
increment is decreased by Rs. 62208, then find the current monthly
salary of Mahesh?
भहे ि को फतामा गमा था कक उनके वतषभान वे तन भें 25% की ववर ि होगी, रे ककन
भहाभायी के कायण उसे केवर 19% की वरवि शभरी, जजसके कायण उनकी वावर्षषक वे तन
वरवि भें 62208 रु. की कभी आमी, तो भहे ि का वतषभान भाशसक वे तन ऻात कीजजए।
(A) Rs. 72000 (B) Rs. 85500 (C) Rs. 86400
(D) Rs. 92500 (E) None of these

108. Meenakshi taken a loan of certain amount from a bank at 30%


per annum compound interest for 2 years and lent the whole amount
to Sheenu at 20% SI for 5 years and earns Rs. 7874 find the
borrowed amount by Meenakshi?
भीनाऺी ने एक फैंक से 2 सार के शरए 30% प्रनत वर्षष चक्रवरवि ब्माज ऩय एक ननजश्चत
याशि का ऋण शरमा औय ऩूय ी याशि को 20% साधायण ब्माज ऩय 5 सार के शरए िीनू
को उधाय ददमा औय इस रे नदे न भें 7874 रुऩमे कभाती है। भीनाऺी द्वाया उधाय री गई
याशि ऻात कीजजए।
(A) Rs. 18400 (B) Rs. 25400 (C) Rs. 20400
(D) Rs. 24400 (E) None of these

109. P, Q and R entered into a partnership such that ratio of


investment of P to Q is 7 : 8 and ratio of investment of Q to R is 13
: 7. if ratio of their period of investment is 3 : 2 : 5 and total profit
earned by them is Rs. 19025 then find average of profit share of P
and R?
P, Q औय R को एक साझेदायी भें इस प्रकाय दजष ककमा गमा है कक P से Q के ननवे ि
का अनुऩात 7 : 8 है औय Q से R के ननवे ि का अनुऩात 13 : 7 है। मदद उनके ननवे ि
की अवचध का अनुऩात 3 : 2 : 5 है औय उनके द्वाया अजजषत कु र राब 19025 रु. है,
तो P औय R के राब दहस्से का औसत ऻात कीजजए।
(A) Rs. 6902.5 (B) Rs. 6922.5 (C) Rs. 6912.5
(D) Rs. 6952.5 (E) None of these

110. The respective ratio of the ages of Manish, Soumya and Avantika
before 3 years was 7 : 5 : 9. After 2 years, the respective ratio of the
ages of Manish and Soumya will be 13 : 10. find the sum of the
present ages of Avantika and Soumya?
3 वर्षष ऩहरे भनीर्ष, सौम्मा औय अवॊनतका की आमु का अनुऩात क्रभि् 7 : 5 : 9 था।
2 वर्षष फाद, भनीर्ष औय सौम्मा की आमु का अनुऩात क्रभि् 13 : 10 होगा। अवॊनतका
औय सौम्मा की वतषभान आमु का मोग ऻात कीजजए। https://instagra
https://youtu https ://ww
be.com/chan w.fa cebook.
m.com/aashish https://
arorasocial(?)
100
nel/UCYa4_Jr
Orf8R5Kz2uO
com/a ashis utm_medium = t.me/st
ha rorasocial copy_link
tccXQ udified
(A) 48 years (B) 44 years (C) 45 years
(D) 42 years (E) None of these

111. Hannu drives his bike at a speed of 70 km/hr and reaches his
office at 10 : 00 AM. If he drives at 60 km/hr he will reach his office
at 10 : 45 AM. At what speed he should drive to reach at 8 : 30 AM?
हन्नू अऩनी फाइक 70 ककभी/घॊटा की गनत से चराता है औय सुफह 10 : 00 फजे अऩने
कामाषरम ऩहुॊचता है। मदद वह 60 ककभी/घॊटा की गनत से राइव कयता है तो वह सुफह
10 : 45 फजे अऩने कामाषरम ऩहुॊच जाएगा। सुफह 8 : 30 फजे ऩहुॉचने के शरए उसे
ककस गनत से गाडी चरानी चादहए?
(A) 75 km/hr (B) 105 km/hr (C) 85 km/hr
(D) 115 km/hr (E) None of these

112. Sakshi and Anuj borrowed the same sum of money from a
borrower at the same rate of interest compounded annually for 3 years
and 4 years respectively. Amount paid by Sakshi at the end of two
years is Rs. 15650 and amount paid by Anuj at the end of 3 years is
Rs. 20345, then what is the rate of interest?
साऺी औय अनुज ने एक उधायकताष से सभान धनयाशि को क्रभि् 3 वर्षष औय 4 वर्षष के
शरए सभान वावर्षषक चक्रवरवि ब्माज दय ऩय उधाय शरमा। साऺी द्वाया दो वर्षष के अॊत भें
बुगतान की गई याशि 15650 रुऩमे है औय अनुज द्वाया 3 वर्षों के अॊत भें बुगतान की
गई याशि 20345 रुऩमे है, तो ब्माज की दय क्मा है?
(A) 40% (B) 25% (C) 30%
(D) 15% (E) None of these

113. A and B can finish a job in 15 hours while B and C can finish
the same job in 18 hours if C finish the whole job in 30 hours then
what part of the job will be done by A alone in 15 hours?
A औय B एक काभ को 15 घॊटे भें ऩूया कय सकते हैं जफकक B औय C उसी काभ को
18 घॊटे भें ऩूय ा कय सकते हैं मदद C ऩूये काभ को 30 घॊटे भें ऩूय ा कयता है, तो A
अकेरे 15 घॊटे भें काभ का ककतना दहस्सा ऩूय ा कये गा?
(A) 4/7 (B) 7/12 (C) 2/3
(D) 3/8 (E) None of these

114. Ramesh and Suresh entered in a partnership with investments of


Rs. 31500 and Rs. 37500 respectively. At the end of one year they
received Rs. X as profit. If 30% of the profit goes to charity and
Suresh received Rs. 2100 then what is the value of X?
यभे ि औय सुये ि ने क्रभि् 31500 रुऩमे औय 37500 रुऩमे के ननवे ि के साथ एक
साझेदायी भें प्रवे ि ककमा। एक वर्षष के अॊत भें उन्हें राब के रूऩ भें X रुऩमे प्राप्त हुए।
मदद राब का 30% दान भें जाता है औय सुये ि को 2100 रुऩमे प्राप्त होते हैं , तो X का
भान क्मा है?

https://youtu https://instagra
https ://ww
be.com/chan w.fa cebook.
m.com/aashish https://
arorasocial(?)
101
nel/UCYa4_Jr
Orf8R5Kz2uO
com/a ashis utm_medium = t.me/st
ha rorasocial copy_link
tccXQ udified
(A) Rs. 5520 (B) Rs. 5250 (C) Rs. 2550
(D) Rs. 5250 (E) None of these

115. P sold a mobile to Q at 41⅔% loss and Q sold it to R at 28.56%


profit. If P had sold it to R at 58⅓% profit directly, then R would have
purchased the mobile at what percentage more than the price at which
he actually purchased it?
P ने Q को 41⅔% हानन ऩय एक भोफाइर फे चा औय Q ने इसे R को 28.56% राब
ऩय फे च ददमा। मदद P ने इसे सीधे 58⅓% राब ऩय R को फे च ददमा होता, तो R ने
उस भूल्म से ककतने प्रनतित अचधक ऩय भोफाइर खयीदा होगा जजस ऩय उसने वास्तव भें
इसे खयीदा था?
(A) 111.11% (B) 81.11% (C) 91.11%
(D) 121.11% (E) None of these

116. The ratio of age of Manish before 2 years to the age of Mohan
after 4 years is 6 : 5 and the ratio of present age of Mohan and Sunil
is 7 : 4, the average of present age of Manish, Mohan and Sunil is
649/21 year, then what is the present age of Manish?
2 वर्षष ऩहरे भनीर्ष की आमु का 4 वर्षष फाद भोहन की आमु से अनुऩात 6 : 5 है औय
भोहन औय सुनीर की वतषभान आमु का अनुऩात 7 : 4 है, भनीर्ष, भोहन औय सुनीर की
वतषभान आमु का औसत 649/21 वर्षष है, तो भनीर्ष की वतषभान आमु क्मा है?
(A) 44 years (B) 45 years (C) 46 years
(D) 43 years (E) None of these

117. A shopkeeper has two types of Wheat, the first type costs Rs. 48
per kg and the cost of second type is not known. To gain 15% on the
whole, he mixed 15 kg of the first type of wheat with 36 kg of second
type of wheat and sold the mixture at Rs. 69. By what percentage is
the cost of second typed wheat more than the cost of first typed
wheat?
एक दुक ानदाय के ऩास दो प्रकाय का गेहूॊ होता है, ऩहरे प्रकाय की रागत 48 रुऩमे प्रनत
ककरो है औय दसू यी प्रकाय की रागत ऻात नहीॊ है। कु र शभराकय 15% प्राप्त कयने के
शरए, उसने ऩहरे प्रकाय के 15 ककरोग्राभ गेहूॊ को 36 ककरोग्राभ दस ू ये प्रकाय के गेहूॊ के
साथ शभरामा औय शभश्ण को 69 रुऩमे ऩय फे च ददमा। दस ू ये प्रकाय के गेहूॊ की रागत
ऩहरे की रागत से ककतने प्रनतित अचधक है ?
(A) Rs. 16 (B) Rs. 17 (C) Rs. 18
(D) Rs. 19 (E) None of these

118. In a school, the average age of students is 8 years, and the


average age of 16 teachers is 36 years. If average age of combined
group of teachers and students is 12, then find the total number of
students in school?
एक ववद्मारम भें छात्रों की औसत आमु 8 वर्षष है औय 16 https://instagra
शिऺकों की औसत आमु 36
https://youtu https ://ww
be.com/chan w.fa cebook.
m.com/aashish https://
arorasocial(?)
102
nel/UCYa4_Jr
Orf8R5Kz2uO
com/a ashis utm_medium = t.me/st
ha rorasocial copy_link
tccXQ udified
वर्षष है। मदद शिऺकों औय छात्रों के सॊमुक्त सभूह की औसत आमु 12 है, तो स्कू र भें
छात्रों की कुर सॊख्मा ऻात कीजजए।
(A) 98 (B) 96 (C) 88
(D) 106 (E) None of these

119. A jetski goes upstream at a speed of 36 km/hr and comes back


the same distance at the speed of 48 km/hr. find the average speed of
the for the total journey?
एक जे टस्की 36 ककभी/घॊटा की गनत से धाया के प्रनतकूर जाता है औय 48 ककभी/घॊटा
की गनत से सभान दयू ी ऩय वाऩस आता है। कु र मात्रा की औसत गनत ऻात कीजजए।
(A) 188/7 (B) 144/7 (C) 288/7
(D) 166/7 (E) None of these

120. A person invested one fourth of the sum of Rs. 30000 at a


certain rate of simple interest and the rest at 4% per annum high rate
if the total interest received at the end of two years is Rs. 10800, then
find the rate percent?
एक व्मजक्त ने 30000 रु. की याशि का एक चौथाई साधायण ब्माज की एक ननजश्चत दय
ऩय ननवे ि ककमा औय िे र्ष 4% प्रनत वर्षष उच्च दय ऩय ननवे ि ककमा, मदद दो वर्षों के अॊत
भें प्राप्त कुर ब्माज 10800 रु. तो दय प्रनतित ऻात कीजजए।
(A) 14% (B) 18% (C) 16%
(D) 15% (E) None of these

121. Two items P and Q are sold at 31.25% and 58⅓% profit. The
selling price of both the articles is same and difference between their
CP is Rs. 208 then find the SP of article?
दो वस्तुओ ॊ P औय Q को 31.25% औय 58⅓% के राब ऩय फे चा जाता है। दोनों
वस्तुओ ॊ का ववक्रम भूल्म सभान है औय उनके क्रम भूल्म के फीच का अॊतय 208 रु. है,
तो वस्तु का SP ऻात कीजजए।
(A) Rs. 1596 (B) Rs. 1842 (C) Rs. 1648
(D) Rs. 1452 (E) None of these

122. A sphere of diameter 24 cm is melted and reformed into 3


spheres, one of which has radius 6 cm and one has radius 8 cm.
what is the radius of third sphere?
24 से भी व्मास के एक गोरे को वऩघराकय 3 गोरे भें फदर ददमा जाता है, जजनभें से
एक की त्रत्रज्मा 6 से भी औय एक की त्रत्रज्मा 8 से भी होती है। तीसये गोरे की त्रत्रज्मा क्मा
है?
(A) 15 cm (B) 12 cm (C) 18 cm
(D) 10 cm (E) None of these

123. Piyush is 26⅔% more efficient than Sonu. When Piyush start
working alone, he completes 20% of work in 6 https://instagra
days. If Sonu and
https://youtu https ://ww
be.com/chan w.fa cebook.
m.com/aashish https://
arorasocial(?)
103
nel/UCYa4_Jr
Orf8R5Kz2uO
com/a ashis utm_medium = t.me/st
ha rorasocial copy_link
tccXQ udified
piyush starts together then find the total time taken to complete the
work?
ऩीमर्ष
ू सोनू से 26⅔% अचधक कु िर है। जफ ऩीमर्ष ू अके रे काभ कयना िुरू कयता है, तो
वह 6 ददनों भें 20% काभ ऩूय ा कयता है। मदद सोनू औय ऩीमूर्ष एक साथ िुरू कयते हैं ,
तो कामष को ऩूया कयने भें शरमा गमा कुर सभम ऻात कीजजए।
(A) 16 14/17 days (B) 15 13/17 days (C) 16 13/17 days
(D) 15 14/17 days (E) None of these

124. The probability of P eating food is 2/3 and the probability of Q


eating food is ¾. If both were asked to eat food, then what is the
probability that the food has been eaten?
P के खाना खाने की प्रानमकता 2/3 है औय Q के खाना खाने की प्रानमकता ¾ है। मदद
दोनों को खाना खाने के शरए कहा गमा, तो इसकी क्मा प्रानमकता है कक खाना खा शरमा
गमा है?
(A) 11/12 (B) 12/13 (C) 13/16
(D) 15/17 (E) None of these

125. Suman and Shashi entered into a partnership together and


invested Rs. 1400 and Rs. 2000. After 6 months Suman invested Rs.
600 more and Shashi withdrew Rs. 800 and after 3 more months
Rajesh entered into the partnership and invested Rs. 4800. the profit
at the end of the year is Rs. 56160. Then find the profit of Shashi?
सुभन औय िशि ने एक साथ साझेदायी भें प्रवे ि ककमा औय 1400 रु. औय 2000 रु. का
ननवे ि ककमा। 6 भहीने फाद सुभन ने 600 रु. अचधक का ननवे ि ककमा औय िशि ने
800 रु. ननकर शरए। 3 औय भहीनों के फाद याजे ि ने साझेदायी भें प्रवे ि ककमा औय
4800 रु. का ननवे ि ककमा। वर्षष के अॊत भें राब 56160 रु. है, तो िशि का राब ऻात
कीजजए।
(A) Rs. 19524 (B) Rs. 18025 (C) Rs. 19968
(D) Rs. 17814 (E) None of these

126. A car travels to a place Q from place P, it travelled 30% of total


distance 50 km/hr, 3/5th of total distance at 75 km/hr and it travelled
remaining distance at 40 km/hr, then in the whole journey, what is the
average speed of car?
एक काय स्थान P से Q स्थान तक जाती है, कु र दयू ी का 30% बाग 50 ककभी/घॊटा
की गनत से , कुर दयू ी का 3/5 बाग 75 ककभी/घॊटा की गनत से तम कयती है औय िे र्ष
दयू ी 40 ककभी/घॊटा की गनत से तम कयती है, तो ऩूय ी मात्रा भें काय की औसत गनत क्मा
है?
(A) 60 20/33 km/hr (B) 61 20/33 km/hr (C) 62 20/33 km/h
(D) 63 20/33 km/hr (E) None of these

127. The ratio of present age of Ramesh and present age of Suresh is
9 : 11, after 16 years, the ratio of age of Ramesh and Suresh will be
https://instagra
https://youtu https ://ww
be.com/chan w.fa cebook.
m.com/aashish https://
arorasocial(?)
104
nel/UCYa4_Jr
Orf8R5Kz2uO
com/a ashis utm_medium = t.me/st
ha rorasocial copy_link
tccXQ udified
17 : 19, then what is the present age of Kirti if Kirti’s age and Suresh
age is in the ratio 13 : 11?
यभे ि की वतषभान आमु औय सुये ि की वतषभान आमु का अनुऩात 9 : 11 है, 16 वर्षष फाद
यभे ि औय सुये ि की आमु का अनुऩात 17 : 19 होगा, तो कीनतष की वतषभान आमु क्मा
है, मदद कीनतष की आमु औय सुयेि की आमु 13 : 11 के अनुऩात भें है?
(A) 28 years (B) 27 years (C) 26 years
(D) 25 years (E) None of these

128. A boat rows ‘A’ km upstream and returns back in total 12.5 hours.
Speed of Boat is 35 km/hr which is 5 times of speed of stream, then
find the value of ‘A’?
एक नाव कु र 12.5 घॊटे भें धाया के प्रनतकू र 'A' ककभी जाती है औय वाऩस रौटती है।
नाव की गनत 35 ककभी/घॊटा है जो धाया की गनत का 5 गन ु ा है, तो 'A' का भान ऻात
कीजजए।
(A) 220 km (B) 210 km (C) 185 km
(D) 195 km (E) None of these

129. Weight of P and Q is in the ratio 7 : 11 respectively and the


respective ratio of weight of P and R is 9 : 8 if the sum of the
weights of Q and R is 155 kg then find the weight of P?
P औय Q का वजन क्रभि् 7 : 11 के अनुऩात भें है औय P औय R के वजन का
सॊफॊचधत अनुऩात 9 : 8 है मदद Q औय R के वजन का मोग 155 ककग्रा है तो P का
वजन ऻात कीजजए।
(A) 68 kg (B) 63 kg (C) 65 kg
(D) 69 kg (E) None of these

130. A 90 kg mixture of wheat and impurity contains 46⅔% wheat in it.


What quantity of pure wheat must be added to the mixture so that it
now contains 62.5% of pure wheat in it?
गेहूॊ औय शभरावट के 90 ककरो के शभश्ण भें 46⅔% गेहूॊ है। शभश्ण भें ककतनी भात्रा भें
िुि गेहूॊ शभरामा जाना चादहए ताकक अफ इसभें 62.5% िुि गेहूॊ हो?
(A) 42 kg (B) 35 kg (C) 45 kg
(D) 38 kg (E) None of these

131. Average age of Mani and Shilpa after 3 years is 38 years and
average age of Shilpa and Rani is 33 years. If average age of Rani
and Kusum before 3 year is 37.5 years and present age of Mani is 48
then find the present age of Kusum?
3 वर्षष फाद भणण औय शिल्ऩा की औसत आमु 38 वर्षष है औय शिल्ऩा औय यानी की
औसत आमु 33 वर्षष है। मदद 3 वर्षष ऩहरे यानी औय कुसुभ की औसत आमु 37.5 वर्षष है
औय भणण की वतषभान आमु 48 है, तो कु सुभ की वतषभान आमु ऻात कीजजए।
(A) 35 years (B) 36 years (C) 37 years
(D) 38 years (E) None of these https://instagra
https://youtu https ://ww
be.com/chan w.fa cebook.
m.com/aashish https://
arorasocial(?)
105
nel/UCYa4_Jr
Orf8R5Kz2uO
com/a ashis utm_medium = t.me/st
ha rorasocial copy_link
tccXQ udified
132. A alone started a business with investment of Rs. 13500 and
after ‘P’ years B joins him with investment of Rs. 2500 more than that
of A. If out of total profit Rs. 1450 at the end of the year Share of A
is Rs. 810, then what is the value of ‘P’?
A अकेरे 13500 रुऩमे के ननवे ि के साथ एक व्मवसाम िुरू कयता है औय 'P' वर्षों के
फाद B उससे 2500 रुऩमे अचधक के ननवे ि के साथ जु ड जाता है। मदद वर्षष के अॊत भें
कुर राब भें से 1450 रुऩमे A का दहस्सा 810 रुऩमे है, तो 'P' का भान क्मा है?
(A) 8 (B) 10 (C) 6
(D) 4 (E) None of these

133. A certain sum is to be divided between two person P and Q, P


received 25% more than the average sum received by both of them. If
Q received Rs. 3900 less than P, then what is the value of certain
sum?
एक ननजश्चत याशि को दो व्मजक्त P औय Q के फीच ववबाजजत ककमा जाना है, P को
उन दोनों द्वाया प्राप्त औसत याशि से 25% अचधक प्राप्त हुआ। मदद Q को P से 3900
रु. कभ शभरे है, तो ननजश्चत याशि का भान क्मा है?
(A) Rs. 14500 (B) Rs. 15600 (C) Rs. 18400
(D) Rs. 16500 (E) None of these

134. Karan appeared in a SSC exam. When Karan got 28% marks
then he failed by 18 marks and when Karan scored 38% marks then
he scored 22 marks more than passing marks, then what is the
difference between passing marks and maximum marks is?
कयण SSC ऩयीऺा भें उऩजस्थत हुए। जफ कयण को 28% अॊक शभरे तो वह 18 अॊक ों
से अनुत्तीणष हो गमा औय जफ कयण ने 38% अॊक प्राप्त ककए तो उसने उत्तीणष अॊक ों से
22 अॊक अचधक प्राप्त ककए, तो उत्तीणष अॊकों औय अचधकतभ अॊकों के फीच का अॊतय
क्मा है?
(A) 245 (B) 250 (C) 270
(D) 285 (E) None of these

135. Price of wheat and rice is Rs. 57 and Rs. 64 respectively. when
the mixture of these rice and wheat is sold at Rs. 85 then there is a
profit of 41⅔% profit. In what ratio rice and wheat mixed?
गेहूॊ औय चावर की कीभत क्रभि : 57 रुऩमे औय 64 रुऩमे है। जफ इन चावर औय गेहूॊ
के शभश्ण को 85 रुऩमे भें फे चा जाता है तो 41⅔% राब का राब होता है। चावर औय
गेहूॊ को ककस अनुऩात भें शभरामा गमा?
(A) 4 : 3 (B) 3 : 4 (C) 7 : 8
(D) 8 : 7 (E) None of these

136. Speed of the boat in still water is 26⅔% more than speed of
stream. The boat can travel 408 km downstream and 16 km upstream
in 8 hour. Find the time taken by the boat in travelling
https://instagra
816 km
https://youtu https ://ww
be.com/chan w.fa cebook.
m.com/aashish https://
arorasocial(?)
106
nel/UCYa4_Jr
Orf8R5Kz2uO
com/a ashis utm_medium = t.me/st
ha rorasocial copy_link
tccXQ udified
downstream and 64 km upstream?
िाॊत जर भें नाव की गनत धाया की गनत से 26⅔% अचधक है। नाव 8 घॊटे भें धाया के
अनुकू र 408 ककभी औय धाया के प्रनतकू र 16 ककभी की मात्रा कय सकती है। नाव द्वाया
धाया के अनुकू र 816 ककभी औय धाया के प्रनतकू र 64 ककभी की मात्रा भें शरमा गमा
सभम ऻात कीजजए।
(A) 14 hr (B) 18.5 hr (C) 20 hr
(D) 25 hr (E) None of these

137. Height of cylinder and radius of the sphere is same and radius of
cylinder is 4 times radius of sphere if volume of sphere is 38808 cm3
then find the volume of cylinder.
फे रन की ऊॉचाई औय गोरे की त्रत्रज्मा सभान है औय फे रन की त्रत्रज्मा गोरे की त्रत्रज्मा
का 4 गनु ा है मदद गोरे का आमतन 38808 से भी3 है तो फे रन का आमतन ऻात
कीजजए।
(A) 475596 cm3 (B) 465696 cm3 (C) 485796 cm3
(D) 425496 cm 3 (E) None of these

138. P, Q and R can complete 40% of work in 12 days, Q and R can


complete the work in 45 days, R alone can complete 25% of the work
in 15 days. What is the part of work done by P and Q in 20 days?
P, Q औय R 40% कामष को 12 ददनों भें ऩूय ा कय सकते हैं , Q औय R 45 ददनों भें
कामष को ऩूया कय सकते हैं , R अकेरे कामष का 25% 15 ददनों भें ऩूया कय सकता है। P
औय Q द्वाया 20 ददनों भें ककए गए कामष का बाग ककतना है?
(A) 2/3rd (B) 1/4rd (C) 1/3rd
(D) 3/5rd (E) None of these

139. A rectangular sheet with dimension 44 × 12 m is rolled into a


cylinder so that the smaller side becomes height of the cylinder. What
is the volume of cylinder so formed?
44 × 12 भीटय आमाभ वारी एक आमताकाय िीट को एक शसरेंडय भें घुभामा जाता है
ताकक छोटी बुज ा शसरेंडय की ऊॊचाई फन जाए। इस प्रकाय फने फे रन का आमतन ककतना
है?
(A) 1848 m3 (B) 1548 m3 (C) 1846 m3
(D) 1748 m 3 (E) None of these

140. Two unit of an item is sold at 5A and 7A profit amount and total
profit percent earned after selling both the item is 45%, then what is
the value ‘A’ if cost price of one unit is Rs. 700?
एक वस्तु की दो इकाइमों को 5A औय 7A की राब याशि ऩय फे चा जाता है औय दोनों
वस्तुओ ॊ को फे चने के फाद अजजषत कुर राब प्रनतित 45% है, तो 'A' का भूल्म क्मा है
मदद एक इकाई का रागत भूल्म 700 रुऩमे है?
(A) 50.4 (B) 51.4 (C) 55.8
(D) 52.5 (E) None of these https://instagra
https://youtu https ://ww
be.com/chan w.fa cebook.
m.com/aashish https://
arorasocial(?)
107
nel/UCYa4_Jr
Orf8R5Kz2uO
com/a ashis utm_medium = t.me/st
ha rorasocial copy_link
tccXQ udified
141. Efficiency of P is 28.56% more than that of Q. P alone can
complete 25% of a work in 7 days while P, Q and R together can
complete whole work in 14 days then in how many days R alone will
complete the work?
P की ऺभता Q की तुरना भें 28.56% अचधक है। P अकेरे एक कामष का 25% 7
ददनों भें ऩूय ा कय सकता है जफकक P, Q औय R शभरकय ऩूये कामष को 14 ददनों भें ऩूय ा
कय सकते हैं , तो R अकेरा उस कामष को ककतने ददनों भें ऩूया कये गा?
(A) 126 days (B) 118 days (C) 140 days
(D) 112 days (E) None of these

142. Neyog, Ankit and Amit started a business with their investment in
the ratio 11 : 9 : 13 respectively. After 4 years Ankit double his
investment, then what will be the difference between profit share of
Neyog and Amit, if profit share of Ankit is Rs. 1890 at the end of 7
years?
ननमोग, अॊककत औय अशभत ने क्रभि् 11 : 9 : 13 के अनुऩात भें अऩने ननवे ि के
साथ एक व्मवसाम िुरू ककमा। 4 वर्षों के फाद अॊककत अऩने ननवे ि को दोगुना कय दे ता
है, तो ननमोग औय अशभत के राब दहस्से के फीच का अॊतय क्मा होगा, मदद अॊककत का
राब 7 सार के अॊत भें दहस्सा 1890 रु. है?
(A) 284 (B) 264 (C) 254
(D) 294 (E) None of these

143. Shanky sold his car at a loss of 43.75% loss. If he had sold it for
Rs. 7168 more, he would have made a percentage profit of 12.25%.
What is the actual selling price of the car?
िैंक ी ने अऩनी काय को 43.75% हानन ऩय फे च ददमा। अगय उसने इसे 7168 रु. ज्मादा
भें फे चा होता, तो वह 12.25% का प्रनतित राब अजजषत कयता। काय का वास्तववक
ववक्रम भूल्म क्मा है?
(A) Rs. 6400 (B) Rs. 7200 (C) Rs. 7800
(D) Rs. 7500 (E) None of these

144. In a school of 1350 students ratio of boys to girls is 8 : 7 and


ratio of boys passed to number of boys failed is 7 : 5, if 130 girls
failed in exam then how much percent less number of boys passed
than number of girls passed in the exam?
1350 छात्रों के एक स्कूर भें रडकों का रडककमों से अनुऩात 8 : 7 है औय उत्तीणष
रडकों का अनुत्तीणष होने वारे रडकों की सॊख्मा से अनुऩात 7 : 5 है, मदद 130
रडककमाॊ ऩयीऺा भें अनुत्तीणष हुई हैं , तो ऩयीऺा भें उत्तीणष होने वारे रडकों की सॊख्मा
उत्तीणष रडककमों की सॊख्मा से ककतने प्रनतित कभ है ?
(A) 5 : 1 : 12 (B) 5 : 12 : 1 (C) 1 : 12 : 5
(D) 15 : 21 : 12 (E) None of these

145. What is the normal time taken by Aditya (in minutes) to reach his
https://instagra
https://youtu https ://ww
be.com/chan w.fa cebook.
m.com/aashish https://
arorasocial(?)
108
nel/UCYa4_Jr
Orf8R5Kz2uO
com/a ashis utm_medium = t.me/st
ha rorasocial copy_link
tccXQ udified
destination if it is known that when he reduces his speed by 46 2/3%
then he reaches his office 28 minutes late?
आददत्म द्वाया अऩने गॊतव्म तक ऩहुॉचने भें (शभनटों भें ) साभान्म सभम क्मा है मदद मह
ऻात हो कक जफ वह अऩनी गनत 46 2/3% कभ कयता है तो वह अऩने कामाषरम भें 28
शभनट दे यी से ऩहुॉचता है?
(A) 18% (B) 16% (C) 14%
(D) 12% (E) None of these

146. What is the normal time taken by Aditya (in minutes) to reach his
destination if it is known that when he reduces his speed by 46 2/3%
then he reaches his office 28 minutes late?
आददत्म द्वाया अऩने गॊतव्म तक ऩहुॉचने भें (शभनटों भें ) साभान्म सभम क्मा है मदद मह
ऻात हो कक जफ वह अऩनी गनत 46 2/3% कभ कयता है तो वह अऩने कामाषरम भें 28
शभनट दे य ी से ऩहुॉचता है?
(A) 20 minutes (B) 40 minutes (C) 36 minutes
(D) 32 minutes (E) None of these

147. 96 liters of mixture P contains milk and water. 12 liters of milk


and 15 liters of water is added into the mixture P. the milk in final
mixture is 5% more than water in final mixture. Find the ratio of milk
and water in initial mixture?
96 रीटय शभश्ण P भें दध ू औय ऩानी है। 12 रीटय दध
ू औय 15 रीटय ऩानी शभश्ण P
भें शभरामा जाता है। अॊनतभ शभश्ण भें दध
ू अॊनतभ शभश्ण भें ऩानी से 5% अचधक है।
प्रायॊ शबक शभश्ण भें दध
ू औय ऩानी का अनुऩात ऻात कीजजए।
(A) 12 : 13 (B) 13 : 12 (C) 17 : 15
(D) 15 : 17 (E) None of these

148. The ratio of A’s age after 5 years and B’s present age is 7 : 3
and the ratio of A’s age 4 years ago and C’s age 2 years ago is 5 :
4. if B is 3 years younger than A, then find the sum of the ages of A
and C?
5 वर्षष फाद A की आमु औय B की वतषभान आमु का अनुऩात 7 : 3 है औय 4 वर्षष ऩूवष
A की आमु औय 2 वर्षष ऩहरे C की आमु का अनुऩात 5 : 4 है। मदद B, A से 3 वर्षष
छोटा है, तो A औय C की आमु का मोग ऻात कीजजए।
(A) 14 years (B) 15 years (C) 16 years
(D) 17 years (E) None of these

149. Speed of boat in still water is 13.33% more than speed of stream,
if the boat takes 18 hour to cover 384 km downstream and 12 km
upstream, then find the time taken by boat to cover 768 km
downstream and 72 km upstream?
िाॊत जर भें नाव की गनत धाया की गनत से 13.33% अचधक है, मदद नाव धाया के
अनुकू र 384 ककभी औय धाया के प्रनतकू र 12 ककभी की दयू https://instagra
ी तम कयने भें 18 घॊटे का
https://youtu https ://ww
be.com/chan w.fa cebook.
m.com/aashish https://
arorasocial(?)
109
nel/UCYa4_Jr
Orf8R5Kz2uO
com/a ashis utm_medium = t.me/st
ha rorasocial copy_link
tccXQ udified
सभम रे ती है, तो नाव द्वाया 768 ककभी धाया के अनुकू र औय 72 ककभी धाया के
प्रनतकूर तम कयने भें ककतना सभम रगता है?
(A) 40 hours (B) 50 hours (C) 60 hours
(D) 70 hours (E) None of these

150. The radius of the cylinder is equal to the height of the cone and
radius of cone is half of the height of the cylinder, if height of the
cylinder is 14 m and volume of cylinder is 7436 m3 then find the
volume of the cone?
फे रन की त्रत्रज्मा िॊकु की ऊॉचाई के फयाफय है औय िॊकु की त्रत्रज्मा फे रन की ऊॉचाई की
आधी है, मदद फे रन की ऊॉचाई 14 m औय फे रन का आमतन 7436 m3 है तो िॊकु का
आमतन ऻात कीजजए।
(A) 767.33 m3 (B) 687.33 m3 (C) 667.33 m3
(D) 697.33 m3 (E) None of these

151. Priyanshi has 320 skirts, out of which she sells some skirts at
28% profit and the rest at 32% profit. If her overall profit percentage is
30.5%, then find the difference between the number of skirts sold at
28% and 32% profit.
वप्रमाॊिी के ऩास 320 स्कटष हैं , जजनभें से कुछ स्कटष वह 28% राब ऩय औय िे र्ष 32%
राब ऩय फे चती है। मदद उसका कु र राब प्रनतित 30.5% है, तो 28% ऩय फे ची गई
स्कटों की सॊख्मा औय 32% राब ऩय फे चीॊ गमी स्कटो के फीच का अॊतय ऻात कीजजए।
(A) 90 (B) 60 (C) 80
(D) 40 (E) None of these

152. Akasa and Pratik invested their amounts in the ratio 2 : 5, Pratik
and Nishant invested their amount in the ratio 6 : 11 and Nishant and
Vishal invested their amount in the ratio 12 : 13. If the profit share of
Pratik is Rs. 14400 then find the difference between the profit share of
Akasa and Nishant.
अकासा औय प्रतीक ने अऩनी याशि को 2 : 5 के अनुऩात भें ननवे ि ककमा, प्रतीक औय
ननिाॊत ने अऩनी याशि को 6 : 11 के अनुऩात भें ननवे ि ककमा औय ननिाॊत औय वविार
ने अऩनी याशि को 12 : 13 के अनुऩात भें ननवे ि ककमा। मदद प्रतीक का राब दहस्सा
रु. 14400 तो अकासा औय ननिाॊत के राब दहस्से के फीच का अॊतय ऻात कीजजए।
(A) Rs. 20640 (B) Rs. 20850 (C) Rs. 20880
(D) Rs. 20890 (E) None of these

153. The difference between the simple interest earned for 3 years and
the compound interest earned for 2 years on the same principal
amount when both are invested at same rate of interest of 30% per
annum is Rs. 1260, then find the principal amount.
3 वर्षों के शरए अजजषत साधायण ब्माज औय सभान भूरधन ऩय 2 वर्षों के शरए अजजषत
चक्रवरवि ब्माज के फीच का अॊतय, जफ दोनों को 30% प्रनत https://instagra
वर्षष की सभान ब्माज दय ऩय
https://youtu https ://ww
be.com/chan w.fa cebook.
m.com/aashish https://
arorasocial(?)
110
nel/UCYa4_Jr
Orf8R5Kz2uO
com/a ashis utm_medium = t.me/st
ha rorasocial copy_link
tccXQ udified
ननवे ि ककमा जाता है, 1260 रूऩमे है, तो भूरधन ऻात कीजजए।
(A) Rs. 6040 (B) Rs. 6550 (C) Rs. 6000
(D) Rs. 6065 (E) None of these

154. Average marks obtained by Asif, Aman and Bhanu in Maths is 53


and average marks obtained by Aman, Bhanu and Chetan in same
subject is 72. If the sum of marks obtained by Asif and Chetan is 63,
then how many marks obtained by Chetan?
आशसप, अभन औय बानु द्वाया गणणत भें प्राप्त औसत अॊक 53 हैं औय अभन, बानु
औय चेतन द्वाया उस ही ववर्षम भें प्राप्त औसत अॊक 72 हैं । मदद आशसप औय चेतन
द्वाया प्राप्त अॊकों का मोग 63 है, तो चेतन ने ककतने अॊक प्राप्त ककए?
(A) 80 (B) 40 (C) 3
(D) 60 (E) None of these

155. The distance cover by a motorboat in upstream in 6 hours is 12


km less than distance covered by the motorboat in downstream in 5
hours. If the speed of the stream is 2 km/hr, then what will be the
speed of motorboat in still water?
एक भोटयफोट द्वाया धाया के प्रनतकू र 6 घॊटे भें तम की गई दयू ी, धाया के अनुकू र
भोटयफोट द्वाया 5 घॊटे भें तम की गई दयू ी से 12 ककभी कभ है। मदद धाया की गनत 2
ककभी/घॊटा है, तो तो िाॊत जर भें भोटयफोट की गनत क्मा होगी?
(A) 10 km/hr (B) 12 km/hr (C) 20 km/hr
(D) 26 km/hr (E) None of these

156. Find the total surface area of a rectangular tank whose sum of
length, breadth and height is 30 cm and length of body diagonal is 20
cm.
एक आमताकाय टैं क का कु र ऩरष्ठीम ऺेत्रपर ऻात कीजजए, जजसकी रॊफाई, चौडाई औय
ऊॊचाई का मोग 30 से भी औय ववकणष की रॊफाई 20 से भी है।
(A) 560 cm² (B) 500 cm² (C) 420 cm²
(D) 520 cm² (E) None of these

157. Population ( male, female and children) of a village is 68000 out


of which children are 20% and ratio of female and male is 7 : 9
respectively. If out of the total population 30% are illiterate and ratio of
illiterate female, male and children is 5 : 18 : 7, then find the total
literate male and female.
एक गाॉव की जनसॊख्मा (ऩुरुर्ष, भदहरा औय फच्चे) 68000 है जजसभें से फच्चे 20% हैं
औय भदहरा औय ऩुरुर्ष का अनुऩात क्रभि् 7 : 9 है। मदद कु र जनसॊख्मा भें से 30%
ननयऺय हैं औय ननयऺय भदहरा, ऩुरुर्ष औय फच्चों का अनुऩात 5 : 18 : 7 है, तो कुर
साऺय ऩुरुर्ष औय भदहरा की सॊख्मा ऻात कये ।
(A) 12240 (B) 18360 (C) 20400
(D) 38760 (E) None of these https://instagra
https://youtu https ://ww
be.com/chan w.fa cebook.
m.com/aashish https://
arorasocial(?)
111
nel/UCYa4_Jr
Orf8R5Kz2uO
com/a ashis utm_medium = t.me/st
ha rorasocial copy_link
tccXQ udified
158. To complete a work, P worked alone for 5 days and Q worked
alone for some time. If P and Q together can complete the whole work
in 10 days and P alone can complete the same work in 12.5 days,
then for how many days did B work?
एक कामष को ऩूया कयने के शरए, P ने अकेरे 5 ददनों तक कामष ककमा औय Q ने कुछ
सभम के शरए अकेरे कामष ककमा। मदद P औय Q शभरकय ऩूये कामष को 10 ददनों भें
ऩूया कय सकते हैं औय P अकेरा उसी कामष को 12.5 ददनों भें ऩूया कय सकता है, तो B
ने ककतने ददनों तक कामष ककमा?
(A) 30 days (B) 10 days (C) 24 days
(D) 16 days (E) None of these

159. A mixture contains 43.75% ethanol and remaining water. When


40.4 liter ethanol is removed from the mixture, then the ratio of ethanol
and water becomes 3 : 5 respectively. What is the initial quantity of
water in the mixture?
एक शभश्ण भें 43.75% इथेनॉर औय िे र्ष ऩानी है। जफ शभश्ण भें 40.4 रीटय इथेनॉर
ननकारा गमा है, तो इथेनॉर औय ऩानी का अनुऩात क्रभि् 3 : 5 हो जाता है। शभश्ण
भें ऩानी की प्रायॊ शबक भात्रा क्मा है?
(A) 272.25 L (B) 277.5 L (C) 227.25 L
(D) 227.5 L (E) None of these

160. After 6 years, sum of age of Radha and Raj is 4 years less than
the sum of age of Bhumika and Bhavishya at that time and Raj is 4
years younger than Bhumika. If ratio of present ages of Radha and
Bhumika is 5 : 6 and present age of Bhavishya is 30 years, then find
the total present age of Radha and Raj together?
6 वर्षष फाद, याधा औय याज की आमु का मोग उस सभम बूशभका औय बववष्म की आमु
के मोग से 4 वर्षष कभ है औय याज, बूशभका से 4 वर्षष छोटा है। मदद याधा औय बूशभका
की वतषभान आमु का अनुऩात 5 : 6 है औय बववष्म की वतषभान आमु 30 वर्षष है, तो
याधा औय याज की कु र वतषभान आमु ऻात कीजजए।
(A) 62 years (B) 64 years (C) 63 years
(D) 60 years (E) None of these

161. Rahul invested Rs.(X + 1000) and Pankaj invested Rs.(X + 500)
in a partnership. After 3 months, Rahul withdraw Rs. 900 and Pankaj
withdraw Rs. 700 from their initial investment. At the end of 7 months
the ratio of their profit is 8 : 7 then find the initial investment of Rahul.
याहुर ने (X + 1000) रुऩमे का ननवे ि ककमा औय ऩॊकज ने एक साझेदायी भें (X +
500) रुऩमे का ननवे ि ककमा। 3 भहीने फाद याहुर ने 900 रुऩमे औय ऩॊक ज ने 700
रुऩमे अऩने िुरुआती ननवे ि से ननकार शरए। 7 भहीने के अॊत भें उनके राब का
अनुऩात 8 : 7 है तो याहुर का प्रायॊ शबक ननवे ि ऻात कीजजए।
(A) Rs. 7800 (B) Rs. 7600 (C) Rs. 3800
(D) Rs. 3600 (E) None of these https://instagra
https://youtu https ://ww
be.com/chan w.fa cebook.
m.com/aashish https://
arorasocial(?)
112
nel/UCYa4_Jr
Orf8R5Kz2uO
com/a ashis utm_medium = t.me/st
ha rorasocial copy_link
tccXQ udified
162. The ratio of simple interest on a sum at R% rate of interest per
annum for 12 years to the compound interest on 50% more sum at
30% rate of interest, compounded annually for 2 years is 50 : 69
respectively. Then find the rate?
ककसी याशि ऩय 12 वर्षों के शरए प्रनत वर्षष ब्माज की R% दय ऩय साधायण ब्माज का
50% अचधक याशि का 2 वर्षो भें 30% वावर्षषक चक्रववर ि ब्माज का अनुऩात क्रभि् 50 :
69 है, तो ब्माज दय ऻात कये ।
(A) 6.5% (B) 6.25% (C) 6 %
(D) 6.75% (E) None of these

163. A bag X has some quantity of sugar and bag Y has 70 kg of


salt. 40% of sugar from bag X is taken out and mixed with same
quantity of salt from bag Y. If the remaining quantity of salt in bag Y
is 70% of its actual quantity then what is the initial quantity of sugar
in bag X?
एक फैग X भें कुछ भात्रा भें चीनी है औय फैग Y भें 70 ककरो नभक है। फैग X से
40% चीनी ननकार री जाती है औय फैग Y से सभान भात्रा भें नभक शभरा ददमा जाता
है। मदद फैग Y भें नभक की िे र्ष भात्रा इसकी वास्तववक भात्रा का 70% है, तो फैग X
भें चीनी की प्रायॊ शबक भात्रा क्मा है?
(A) 52.75 kg (B) 42.5 kg (C) 52.5 kg
(D) 42.75 kg (E) None of these

164. Efficiency of Billu is 58.33% less than the combined efficiency of


Billu and Champa. Champa is 3.5 times efficient as Arundhati.
Arundhati and Billu takes 10 days to complete the same work. How
many days Champa will take to complete the work?
त्रफल्रू की ऺभता त्रफल्रू औय चॊऩा की सॊमुक्त दऺता से 58.33 प्रनतित कभ है। चॊऩा
अरुॊ धनत की तुरना भें 3.5 गुना कु िर है। अरुॊ धनत औय त्रफल्रू सभान कामष को ऩूय ा
कयने भें 10 ददन का सभम रे ते हैं । चॊऩा को काभ ऩूया कयने भें ककतने ददन रगेंगे?
(A) 90/29 days (B) 45/8 days (C) 90 days
(D) 10 days (E) None of these

165. 8 years ago, the ratio of age of Amit to Ashish was 7 : 8


respectively. Amit’s age after 9 years will be equal to age of Ashish
after 1 year. Find the age of Ashish after 5 years.
8 वर्षष ऩहरे , अशभत औय आिीर्ष की आमु का अनुऩात क्रभि् 7 : 8 था। 9 वर्षष फाद
अशभत की आमु 1 वर्षष फाद आिीर्ष की आमु के फयाफय होगी। 5 वर्षष फाद आिीर्ष की
आमु ऻात कीजजए।
(A) 77 years (B) 73 years (C) 68 years
(D) 79 years (E) None of these

166. A train has 12 bogies. The length of each bogie of the train is
10.5 m and the length of the engine of train is https://instagra
20 m. Distance
https://youtu https ://ww
be.com/chan w.fa cebook.
m.com/aashish https://
arorasocial(?)
113
nel/UCYa4_Jr
Orf8R5Kz2uO
com/a ashis utm_medium = t.me/st
ha rorasocial copy_link
tccXQ udified
between each bogie is 2 m and the distance between engine and first
bogie is 2 m. If the train is moving with a speed of 34 m/s then how
long it will take to cross a platform of length 170 m.
एक ट्रे न भें 12 फोचगमाॊ होती हैं । ट्रे न की प्रत्मे क फोगी की रॊफाई 10.5 भीटय औय ट्रे न के
इॊजन की रॊफाई 20 भीटय है। प्रत्मे क फोगी के फीच की दयू ी 2 भीटय है औय इॊजन औय
ऩहरी फोगी के फीच की दयू ी 2 भीटय है। मदद ट्रे न 34 भीटय/से केंड की गनत से चर यही
है तो 170 भीटय रॊफाई के प्रे टपॉभष को ऩाय कयने भें ककतना सभम रगेगा।
(A) 8 sec (B) 10 sec (C) 6 sec
(D) 12 sec (E) None of these

167. Vidhi bought two items A and B together at Rs. 2800. She sold
the items at Rs. 3164 and earned a profit of 11% and 17% on items A
and B respectively. Then find the difference between cost price of both
articles.
ववचध ने दो वस्तु A औय B को एक साथ 2800 रूऩमे भें खयीदा। उसने वस्तुओ को एक
साथ 3164 रूऩमे ऩय फे चा औय वस्तु A औय B ऩय क्रभि् 11% औय 17% का राब
अजजषत ककमा। दोनों वस्तु के क्रम भूल्म के फीच का अॊतय ऻात कये ।
(A) 933.99 (B) 933.33 (C) 954.64
(D) 956.65 (E) None of these

168. In a hostel, there is a total of M students initially and the per


student expenditure is Rs. 30. The next month, the number of students
increased by 15 and per student expenditure decreased by Rs. 5 but
overall expenditure increases by Rs. 75 then find the value of M?
एक छात्रावास भें प्रायॊ ब भें कु र M छात्र हैं औय प्रनत छात्र व्मम रु. 30 हैं । अगरे भहीने ,
छात्रों की सॊख्मा भें 15 की ववर ि हुई औय प्रनत छात्र व्मम भें 5 रुऩमे की कभी आई।
रे ककन कुर व्मम 75 रुऩमे फढ जाता है, तो M का भान ऻात कीजजए।
(A) 40 (B) 65 (C) 70
(D) 60 (E) None of these

169. If a boat can travel 51 km downstream and 21 km upstream in 12


hours and the same boat can cover 17 km downstream and 70 km
upstream in 22 hours. Find the speed of stream.
मदद एक नाव धाया के अनुकू र 51 ककभी औय धाया के प्रनतकू र 21 ककभी की मात्रा
12 घॊटे भें कय सकती है औय वही नाव धाया के अनुकू र 17 ककभी औय धाया के ववऩयीत
70 ककभी की दयू ी 22 घॊटे भें तम कय सकती है। धाया की गनत ऻात कीजजए।
(A) 2.5 km/hr (B) 3.5 km/hr (C) 6.5 km/hr
(D) 8.5 km/hr (E) None of these

170. If A is 5.55% more than B and B is 56.25% less than C then A


is how much % more or less than C?
मदद A, B से 5.55% अचधक है औय B, C से 56.25% कभ है, तो A, C से ककतना%
अचधक मा कभ है? https://instagra
https://youtu https ://ww
be.com/chan w.fa cebook.
m.com/aashish https://
arorasocial(?)
114
nel/UCYa4_Jr
Orf8R5Kz2uO
com/a ashis utm_medium = t.me/st
ha rorasocial copy_link
tccXQ udified
(A) 55% (B) 53.91% (C) 53.81%
(D) 54.81% (E) None of these

171. Cost of fencing a rectangular field of length 21 meters is Rs. 3.8


per meter. If the total cost of fencing the field is Rs. 220.4, then find
the area of the field.
21 भीटय रम्फे एक आमताकाय खेत की फाड रगाने की रागत रु. 3.8 प्रनत भीटय है।
मदद खेत की फाड रगाने की कु र रागत रु. 220.4, तो भैदान का ऺेत्रपर ऻात कीजजए।
(A) 140 m² (B) 168 m² (C) 136 m²
(D) 158 m² (E) None of these

172. Jaya can sell lipsticks on an average of 24 lipsticks per day in a


month. If she sells on an average of 24 lipsticks in the first 8 days
and 28 lipsticks in the next 9 days, then what is the approximate
average number of lipsticks sold by Jaya in the remaining days of the
month?(1 month = 30 days)
जमा एक भहीने भें प्रनतददन औसतन 24 शरऩजस्टक फे च सकती हैं । मदद वह ऩहरे 8
ददनों भें औसतन 24 शरऩजस्टक औय अगरे 9 ददनों भें 28 शरऩजस्टक फे चती है, तो
भहीने के िे र्ष ददनों भें जमा द्वाया फे ची गई शरऩजस्टक की अनुभाननत औसत सॊख्मा क्मा
है?(1 भहीना= 30 ददन)
(A) 21.8 (B) 21.4 (C) 21.2
(D) 21.9 (E) None of these

173. 15 men can do as much work as 24 women can do. If 6 men


and 4 women can complete a certain work in 15 days then ho many
women can complete the same work in 34 days?
15 ऩुरुर्ष उतना काभ कय सकते हैं जजतना 24 भदहराएॊ कय सकती हैं । मदद 6 ऩुरुर्ष औय
4 भदहराएॊ एक ननजश्चत कामष को 15 ददनों भें ऩूय ा कय सकते हैं तो ककतनी भदहराएॊ
उसी कामष को 34 ददनों भें ऩूया कय सकती हैं ?
(A) 5 (B) 9 (C) 6
(D) 8 (E) None of these

174. The ratio of present ages of P and Q is 3 : 4. After ‘z’ years, the
respective ratio of their ages will be 26 : 33. R is 4 years older than
Q and the present age of R is 32 years, then find the value of ‘z’
P औय Q की वतषभान आमु का अनुऩात 3 : 4 है। 'z' वर्षष के फाद, उनकी आमु का
सॊफॊचधत अनुऩात 26 : 33 होगा। R, Q से 4 वर्षष फडा है औय R की वतषभान आमु 32
वर्षष है, तो 'z' का भान ऻात कीजजए।
(A) 5 (B) 8 (C) 6
(D) 9 (E) None of these

175. Rahul and Anjali entered into a partnership with their initial capital
of Rs. 2400 and 3200 respectively. After 3 months, Rahul increased his
https://instagra
https://youtu https ://ww
be.com/chan w.fa cebook.
m.com/aashish https://
arorasocial(?)
115
nel/UCYa4_Jr
Orf8R5Kz2uO
com/a ashis utm_medium = t.me/st
ha rorasocial copy_link
tccXQ udified
investment by Rs. 400 and Anjali decreased her investment by Rs.
400. after further 4 months, Rahul decreased his investment by Rs.
400 and Anjali increased her investment by Rs. 400. find the ratio of
their profits at the end of one year.
याहुर औय अॊज शर ने क्रभि् 2400 रूऩमे औय 3200 रुऩमे की अऩनी प्रायॊ शबक ऩूॊज ी के
साथ एक साझेदायी भें प्रवे ि ककमा। 3 भहीने के फाद, याहुर ने अऩने ननवे ि भें रु. 400
जोडे औय अॊजशर ने अऩने ननवे ि भें रु. 400 घटा ददए। अगरे 4 भहीनों के फाद, याहुर
ने अऩने ननवे ि भें रु. 400 घटाए औय अॊज शर ने अऩने ननवे ि भें रु. 400. जोड शरए।
एक वर्षष के अॊत भें उनके राब का अनुऩात ऻात कीजजए।
(A) 19 : 23 (B) 369 : 304 (C) 396 : 340
(D) 340 : 396 (E) None of these

176. A train travelling at a speed of 21.6 km/hr crosses a pole in 15


seconds. How much time will it take to cross a platform of length 54
m?
21.6 ककभी/घण्टा की चार से चर यही एक ये रगाडी एक खम्बे को 15 से कण्ड भें ऩाय
कयती है। 54 भीटय रॊफे प्रे टपॉभष को ऩाय कयने भें उसे ककतना सभम रगेगा?
(A) 30 sec (B) 24 sec (C) 29 sec
(D) 28 sec (E) None of these

177. Speed of boat in still water is 6.4 km/hr and speed of stream is
0.8 km/hr. The boat covers a certain distance from P to Q upstream in
3 hours and came back to the starting point in 2 hours. Find the total
distance covered.
िाॊत जर भें नाव की गनत 6.4 ककभी/घॊटा है औय धाया की गनत 0.8 ककभी/घॊटा है। नाव
धाया के प्रनतकू र P से Q तक एक ननजश्चत दयू ी 3 घॊटे भें तम कयती है औय 2 घॊटे भें
वाऩस प्रायॊ शबक त्रफॊदु ऩय आ जाती है। तम की गई कुर दयू ी ऻात कीजजए।
(A) 32.4 km (B) 31 km (C) 31.5 km
(D) 31.2 km (E) None of these

178. The marked price of an article is Rs. 320. Anupriya earns Rs. Y
less as profit when she sold the article at 45% discount instead of
20% discount. Find the value of Y.
एक वस्तु का अॊककत भूल्म रु. 320. है। अनुवप्रमा राब के रूऩ भें Y रु. कभ कभाती है
जफ वह वस्तु को 20% छू ट के फजाम 45% छू ट ऩय फे चती है। Y का भान ऻात
कीजजए।
(A) Rs. 80 (B) Rs. 40 (C) Rs. 34
(D) Rs. 32 (E) None of these

179. A sum of money lent at compound interest for 1 year at 20% per
annum compounded annually would fetch Rs. 420 more, if the interest
was payable half yearly. Then find the sum of money.
1 वर्षष के शरए 20% की दय से चक्रवरवि ब्माज ऩय उधाय दीhttps://instagra
गमी याशि से 420. रु
https://youtu https ://ww
be.com/chan w.fa cebook.
m.com/aashish https://
arorasocial(?)
116
nel/UCYa4_Jr
Orf8R5Kz2uO
com/a ashis utm_medium = t.me/st
ha rorasocial copy_link
tccXQ udified
अचधक प्राप्त होते , मदद ब्माज अधषवावर्षषक दे म होता। धनयाशि ऻात कये ।
(A) Rs. 420000 (B) Rs. 4200 (C) Rs. 42000
(D) Rs. 64000 (E) None of these

180. 134 litres of a mixture contains ethanol and water. 30% of the
mixture is used. The quantity of water in the mixture is 36 litres now.
Find the quantity of ethanol that is left in the mixture.
134 रीटय शभश्ण भें इथेनॉर औय ऩानी है। शभश्ण का 30% उऩमोग ककए जाने के फाद
शभश्ण भें ऩानी की भात्रा अफ 36 रीटय है। शभश्ण भें फची हुई इथेनॉर की भात्रा ऻात
कीजजए।
(A) 52.4 L (B) 56.3 L (C) 54.6 L
(D) 57.8 L (E) None of these

181. Avni lent certain amount to Bhavna for 3 years at 10% annual
compound interest an Bhavna lent the whole amount to Shivangi for 3
years at 15% simple interest. If profit amount earned by Bhavna in this
process is Rs. 190.4, then what is the certain amount given by Avni?
अवनन ने 10% वावर्षषक चक्रववर ि ब्माज ऩय बावना को 3 सार के शरए कु छ याशि उधाय
दी, जफकक बावना ने ऩूयी याशि शिवाॊगी को 3 सार के शरए 15% साधायण ब्माज ऩय
उधाय दी। मदद इस प्रकक्रमा भें बावना द्वाया अजजषत राब याशि रु. 190.4 है, तो अवनन
द्वाया दी गई ननजश्चत याशि क्मा है ?
(A) Rs. 1650 (B) Rs. 1400 (C) Rs. 1180
(D) Rs. 1600 (E) None of these

182. Two person Mahira and Paras invested Rs. 7560 and Rs. 9240
respectively in a business for the same period of time. After 3 years,
Paras received Rs. 3421 as profit, then find the profit share of Mahira.
दो व्मजक्त भादहया औय ऩायस ने सभान अवचध के शरए एक व्मवसाम भें क्रभि् 7560
रुऩमे औय 9240 रुऩमे का ननवे ि ककमा। 3 वर्षष फाद ऩायस को राब के रूऩ भें 3421
रुऩमे प्राप्त हुए, तो भादहया का राब दहस्सा ऻात कीजजए।
(A) Rs. 2979 (B) Rs. 2799 (C) Rs. 2977
(D) Rs. 2779 (E) None of these

183. The ratio of cost price of item A to that of item B is 7 : 9


respectively and item A is sold at y% profit and item B is sold at 20%
loss. What would be the value of ‘y’ to gain overall 36% profit?
वस्तु A औय वस्तु B के क्रम भूल्म का अनुऩात क्रभि् 7 : 9 है औय वस्तु A को y%
राब ऩय फे चा जाता है औय वस्तु B को 20% हानन ऩय फे चा जाता है। कु र 36% राब
प्राप्त कयने के शरए 'y' का भान क्मा होगा?
(A) 104 (B) 72 (C) 108
(D) 54 (E) None of these

https://youtu https://instagra
https ://ww
be.com/chan w.fa cebook.
m.com/aashish https://
arorasocial(?)
117
nel/UCYa4_Jr
Orf8R5Kz2uO
com/a ashis utm_medium = t.me/st
ha rorasocial copy_link
tccXQ udified
184. Ratio of present to absent employees in an office in 1st week is
9 : 4 while that in 2nd week is 6 : 1. If total employees in office on
both the weeks are equal, then find the ratio of absent employees on
week 1st to that on week 2nd.
एक कामाषरम भें ऩहरे सप्ताह भें उऩजस्थत व ् अनुऩजस्थत कभषचारयमों का अनुऩात 9 : 4
है जफकक दस ू ये सप्ताह भें 6 : 1 है। मदद दोनों सप्ताहों भें कामाषरम भें कु र कभषचायी
सभान हैं , तो ऩहरे सप्ताह भें अनुऩजस्थत कभषचारयमों का दस ू ये सप्ताह भें अनुऩजस्थत
कभषचारयमों का अनुऩात ऻात कीजजए।
(A) 28 : 13 (B) 13 : 28 (C) 30 : 24
(D) 29 : 31 (E) None of these

185. The number of items in a shop is increased by 26.66% and the


price of item per head is decreased by 18.75%. Then, the total
decrease or increase in total revenue is?
एक दुक ान भें वस्तुओ ॊ की सॊख्मा भें 26.66% की वरवि होती है औय प्रनत वस्तु की
कीभत भें 18.75% की कभी होती है, तो, कुर याजस्व भें कुर कभी मा ववर ि है?
(A) 2.99% (B) 8.87% (C) 2.91 %
(D) 2.94% (E) None of these

186. Geetika takes 25% more time to complete a piece of work than
Chesta. If together Geetika and Chesta take 12 days to complete the
whole work and Ashish’s efficiency is 20% less than that of Chesta’s
efficiency, then in how many days Ashish alone can complete the
whole work?
गीनतका एक कामष को ऩूय ा कयने भें चेष्टा से 25% अचधक सभम रे ती है। मदद गीनतका
औय चेष्टा शभरकय ऩूये कामष को ऩूय ा कयने भें 12 ददन का सभम रे ते हैं औय आिीर्ष की
दऺता चेष्टा की दऺता से 20% कभ है, तो आिीर्ष अकेरे ऩूये कामष को ककतने ददनों भें
ऩूय ा कय सकता है?
(A) 30 days (B) 24 days (C) 42 days
(D) 27 days (E) None of these

187. 45 litres mixture of milk and water in the ratio 5 : 4. In this


mixture, 3 litres of milk and 2 litres of water were added. What more
is required to be added to obtain a mixture containing milk and water
in the ratio 1 : 1?
45 रीटय दध ू औय ऩानी का शभश्ण 5 : 4 के अनुऩात भें है। इस शभश्ण भें 3 रीटय
दध ू औय 2 रीटय ऩानी शभरामा गमा। 1 : 1 के अनुऩात भें दध ू औय ऩानी वारा शभश्ण
प्राप्त कयने के शरए औय क्मा शभराने की आवश्मकता है ?
(A) 8 litre (B) 6 litre (C) 5 litre
(D) 7.5 litre (E) None of these

188. The ratio between the circumference and area of a semicircle is


12 : 77. Find the radius of this semicircle. https://instagra
https://youtu https ://ww
be.com/chan w.fa cebook.
m.com/aashish https://
arorasocial(?)
118
nel/UCYa4_Jr
Orf8R5Kz2uO
com/a ashis utm_medium = t.me/st
ha rorasocial copy_link
tccXQ udified
एक अधषवत्र त की ऩरयचध औय ऺेत्रपर का अनुऩात 12 : 77 है। इस अधषवत्र त की त्रत्रज्मा
ऻात कीजजए।
(A) 21 (B) 14 (C) 7
(D) 10.5 (E) None of these

189. If a bike travels with speed 36 km/hr instead of Y km/hr, then it


will cover 60 km more in same time. If the actual distance covered by
the bike is 480 km, then find the value of Y?
मदद कोई फाइक Y ककभी/घॊटा के फजाम 36 ककभी/घॊटा की गनत से मात्रा कयती है, तो
वह उसी सभम भें 60 ककभी अचधक दयू ी तम कये गी। मदद फाइक द्वाया तम की गई
वास्तववक दयू ी 480 ककभी है, तो Y का भान ऻात कीजजए।
(A) 32 (B) 16 (C) 38
(D) 36 (E) None of these

190. The average of the present age of A, B and C is 29 1/3 and the
ratio of A's age 4 years ago, B's age 1 year ago and C's age 5 years
hence is 2 : 3 : 3 respectively. What is the sum of present age of B
and C?
A, B औय C की वतषभान आमु का औसत 29 1/3 है औय 4 वर्षष ऩहरे A की आमु, 1
वर्षष ऩहरे B की आमु औय 5 वर्षष फाद C की आमु का अनुऩात क्रभि : 2 : 3 : 3 है।
B औय C की वतषभान आमु का मोग क्मा है?
(A) 62 yrs (B) 64 yrs (C) 66 yrs
(D) 60 yrs (E) None of these

191. (X + 5) men can finish a piece of work in (X + 4) days. If the


work is to be completed in 12 days then number of extra workers
required is 25. find the value of X.
(X + 5) ऩुरुर्ष ककसी कामष को (X + 4) ददनों भें सभाप्त कय सकते हैं । मदद कामष को
12 ददनों भें ऩूया कयना है तो आवश्मक अनतरयक्त श्शभकों की सॊख्मा 25 है। X का भान
ऻात कीजजए।
(A) 20 (B) 30 (C) 25
(D) 35 (E) None of these

192. The principal is 100 times the value of rate of simple interest per
annum. If the amount in 2 years becomes Rs. 4800 then find the
double of rate of interest per annum.
भूरधन प्रनत वर्षष साधायण ब्माज की दय के भूल्म का 100 गुना है। मदद 2 वर्षष भें याशि
रु. 4800 हो जाती है, तो वावर्षषक ब्माज दय का दोगुना ऻात कीजजए।
(A) 40% (B) 30% (C) 60%
(D) 80% (E) None of these

193. A boat covers 48 km upstream in 6 hours. The speed of boat in


still water is 20% more than the upstream speed of boat. Find the
https://instagra
https://youtu https ://ww
be.com/chan w.fa cebook.
m.com/aashish https://
arorasocial(?)
119
nel/UCYa4_Jr
Orf8R5Kz2uO
com/a ashis utm_medium = t.me/st
ha rorasocial copy_link
tccXQ udified
time taken by boat to cover 28 km downstream.
एक नाव धाया के प्रनतकूर 48 ककभी की दयू ी 6 घॊटे भें तम कयती है। िाॊत जर भें नाव
की गनत, नाव की धाया के प्रनतकू र गनत से 20% अचधक है। धाया के अनुकू र 28 ककभी
की दयू ी तम कयने भें नाव द्वाया शरमा गमा सभम ऻात कीजजए।
(A) 4 hours (B) 3.2 hours (C) 2.5 hours
(D) 2.75 hours (E) None of these

194. 640 people applied for a job in a company for 2 different posts.
15% of total people did not apply for HR post and 75% of total people
did not apply for engineer post. Then how many people applied for
both posts?
एक कॊऩनी भें 2 अरग – अरग ऩदों ऩय नौकयी के शरए 640 रोगों ने आवे दन ककमा
था। कुर रोगों भें से 15% ने एचआय ऩद के शरए आवे दन नहीॊ ककमा औय कुर रोगों
भें से 75% ने इॊज ीननमय ऩद के शरए आवे दन नहीॊ ककमा, तो ककतने रोगों ने दोनों ऩदों
के शरए आवे दन ककमा?
(A) 64 (B) 480 (C) 96
(D) 68 (E) None of these

195. Ayesha, Simran and Rohit are three cousins. Ayesha is 6 years
younger to Rohit. If ratio between ages of Rohit and Simran 3 years
ago was 11 : 6 and 6 years hence will be 14 : 9, then find the ratio
between age of Ayesha and Rohit 5 years ago.
आमिा, शसभयन औय योदहत तीन चचेये बाई फहन हैं । आमिा योदहत से 6 सार छोटी है।
मदद 3 वर्षष ऩहरे योदहत औय शसभयन की आमु का अनुऩात 11 : 6 था औय 6 वर्षष फाद
14 : 9 होगा, तो 5 वर्षष ऩहरे आमिा औय योदहत की आमु के फीच का अनुऩात ऻात
कीजजए।
(A) 36 : 15 (B) 31 : 25 (C) 25 : 31
(D) 17 : 39 (E) None of these

196. When 2 sacks of rice of average weight 30 kg removes from the


stack of sacks, the average weight of remaining 44 sacks remains
same. Find the average weight.
जफ 30 ककरो के औसत वजन के 2 फोयी चावर को फोरयमों के ढे य से हटा ददमा जाता
है, तो िे र्ष 44 फोरयमों का औसत वजन सभान यहता है। औसत वजन ऻात कीजजए।
(A) 24 (B) 10 (C) 20
(D) 30 (E) None of these

197. Payal purchased ‘y’ cups at Rs. 360/cup and ‘y – 4’ cups at Rs.
450/cup. She sold all the cups at Rs. 558/cup and earns 40% profit.
Then find the value of ‘y’.
ऩामर ने 'y' कऩ 360/कऩ रुऩमे भें खयीदा औय 'y – 4' कऩ रु. 450/कऩ भें ख़यीदा।
उसने सबी कऩ 558/कऩ रुऩमे भें फे चे औय 40% राब अजजषत ककमा। तफ 'y' का भान
ऻात कीजजए। https://instagra
https://youtu https ://ww
be.com/chan w.fa cebook.
m.com/aashish https://
arorasocial(?)
120
nel/UCYa4_Jr
Orf8R5Kz2uO
com/a ashis utm_medium = t.me/st
ha rorasocial copy_link
tccXQ udified
(A) 19 (B) 16 (C) 14
(D) 15 (E) None of these

198. Priyu riding a scooty completes one lap of rectangular field along
its perimeter at a speed of 72 km/hr in 6 seconds. Find the area of
field if ratio between the length and breadth of rectangular field is 11 :
9.
स्कू टी ऩय सवाय वप्रमु 72 ककभी/घॊटा की गनत से 6 से कॊड भें आमताकाय ऺेत्र की ऩरयचध
के साथ का एक चक्कय ऩूया कयता है। मदद आमताकाय खेत की रॊफाई औय चौडाई का
अनुऩात 11 : 9 है, तो खेत का ऺेत्रपर ऻात कीजजए।
(A) 400 m² (B) 652 m² (C) 652 m²
(D) 600 m² (E) None of these

199. The ratio of investment of Ayush, Badal and Chahal is 3 : 9 : 7


respectively. The ratio of investment of time of Ayush, Badal and
Chahal is 4 : 6 : 1 respectively. If there is profit of Rs. 2628 then find
the difference between the profit share of Badal and Chahal.
आमुर्ष, फादर औय चहर के ननवे ि का अनुऩात क्रभि् 3 : 9 : 7 है। आमुर्ष, फादर
औय चहर के सभम के ननवे ि का अनुऩात क्रभि् 4 : 6 : 1 है। अगय 2628 रुऩमे का
राब है, तो फादर औय चहर के राब दहस्से के फीच का अॊतय ऻात कीजजए।
(A) Rs. 1628 (B) Rs. 1629 (C) Rs. 1962
(D) Rs. 1962 (E) None of these

200. If the value of petrol increased by 46.66%. By what %, a person


should decrease its consumption so that the expenditure remains
same?
मदद ऩे ट्रोर के भूल्म भें 46.66% की वरवि हुई। एक व्मजक्त को अऩने उऩबोग भें ककतने
प्रनतित की कभी कयनी चादहए ताकक व्मम सभान यहे ?
(A) 31.81% (B) 33.91% (C) 33.81 %
(D) 34.81% (E) None of these

201. Two shirts P and Q together are purchased for a total of Rs.
2300. If P and Q are sold at 56% and 20% profit respectively and
selling prices of both shirts are same, then the cost price of shirt Q is
what % of cost price of shirt P?
एक साथ दो िटष P औय Q को कुर 2300 रु भें ख़यीदा जाता है। मदद P औय Q को
क्रभि् 56% औय 20% राब ऩय फे चा जाता है औय दोनों कभीजों का ववक्रम भूल्म
सभान है, तो िटष Q का क्रम भूल्म िटष P के रागत भूल्म का ककतना प्रनतित है?
(A) 120% (B) 130% (C) 30%
(D) 135% (E) None of these

202. A sphere of diameter 36 cm is melted and reformed into 3,


spheres one of which has radius 2 cm and onehttps://instagra
has diameter 32
https://youtu https ://ww
be.com/chan w.fa cebook.
m.com/aashish https://
arorasocial(?)
121
nel/UCYa4_Jr
Orf8R5Kz2uO
com/a ashis utm_medium = t.me/st
ha rorasocial copy_link
tccXQ udified
cm then the radius of third sphere is what % less than the radius of
original sphere?
36 से भी व्मास के एक गोरे को वऩघराकय 3 गोरे भें फदर ददमा जाता है, जजनभें से
एक की त्रत्रज्मा 2 से भी औय एक का व्मास 32 से भी है, तो तीसये गोरे की त्रत्रज्मा भूर
गोरे की त्रत्रज्मा से ककतने प्रनतित कभ है?
(A) 40% (B) 30.05% (C) 66.67%
(D) 33.33% (E) None of these

203. Shrey can work 60% as efficient as Ankush. When shrey start
working alone, he completes 30% of the work in 6 days. Find the total
time taken taken by Ankush to complete the whole work alone?
श्ेम, अॊकु ि की तुरना भें 60% दऺता से कामष कय सकता है। जफ श्ेम अकेरे काभ
कयना िुरू कयता है, तो वह 6 ददनों भें 30% काभ ऩयू ा कयता है। अकेरे ऩयू े कामष को
ऩूय ा कयने के शरए अॊकु ि द्वाया शरमा गमा कु र सभम ऻात कीजजए।
(A) 12 days (B) 10 days (C) 15 days
(D) 16 days (E) None of these

204. The probability of X telling a truth is 1/3 and the probability of Y


telling a truth is 3/5. If both were asked to tell the truth, then what is
the probability that the truth has been told?
X के सच फोरने की प्रानमकता 1/3 है औय Y के सच फोरने की प्रानमकता 3/5 है। मदद
दोनों को सच फोरने के शरए कहा गमा, तो क्मा प्रानमकता है कक सत्म कहा गमा है ?
(A) 12/17 (B) 13/15 (C) 11/15
(D) 6/11 (E) None of these

205. Priya and Chaitanya entered into a partnership together and


invested Rs. 2000 and 1400 respectively. After 3 months, Priya
withdrew Rs. 400 and Chaitanya invested Rs. 400 more. After 3 more
months, Pravesh entered into the partnership with Rs. 2400. The profit
at end of one year is Rs. 10350 then find the difference between the
profit share of Priya and Pravesh.
वप्रमा औय चैतन्म ने एक साथ साझेदायी भें प्रवे ि ककमा औय क्रभि् 2000 रु. औय
1400 रु. का ननवे ि ककमा। 3 भहीने के फाद, वप्रमा ने 400 रुऩमे ननकार शरए औय
चैतन्म ने 400 रुऩमे का ननवे ि ककमा। 3 औय भहीनों के फाद, प्रवे ि ने 2400 रुऩमे के
साथ साझेदायी भें प्रवे ि ककमा। एक वर्षष के अॊत भें राब 10350 रु. है तो वप्रमा औय
प्रवे ि के राब दहस्से के फीच का अॊतय ऻात कीजजए।
(A) Rs. 1250 (B) Rs. 0 (C) Rs. 1259
(D) Rs. 1125 (E) None of these

206. P can complete 30% of work in 6 days, Q can complete 83.33%


of work in 10 days and R can complete 46.66% of work in 14 days.
The time taken by P and Q together to complete the work is what %
of the time taken by P and R together to complete the work?
https://instagra
https://youtu https ://ww
be.com/chan w.fa cebook.
m.com/aashish https://
arorasocial(?)
122
nel/UCYa4_Jr
Orf8R5Kz2uO
com/a ashis utm_medium = t.me/st
ha rorasocial copy_link
tccXQ udified
P 30% कामष को 6 ददनों भें ऩूय ा कय सकता है, Q 83.33% कामष को 10 ददनों भें ऩूय ा
कय सकता है औय R 46.66% कामष को 14 ददनों भें ऩूया कय सकता है। P औय Q
द्वाया शभरकय कामष को ऩयू ा कयने भें शरमा गमा सभम P औय R द्वाया शभरकय कामष
को ऩूय ा कयने भें शरए गए सभम का ककतना प्रनतित है?
(A) 64.25% (B) 60% (C) 62.25%
(D) 62.5% (E) None of these

207. Ashish sells a cup at Rs. 36, a bottle at Rs. 20 and a spoon at
Rs. 22 at profit of 50%, 25% and 10% respectively. Find the total profit
earned by him when he sells 15 cups, 20 bottles and 10 spoons.
आिीर्ष एक कऩ को रु. 36, एक फोतर रु. 20 औय एक चम्भच रु. 22 ऩय क्रभि्
50%, 25% औय 10% के राब ऩय फे चता है। 15 कऩ, 20 फोतरें औय 10 चम्भच
फे चने ऩय उसके द्वाया अजजषत कुर राब ऻात कीजजए।
(A) 31.81% (B) 31.44% (C) 32.81%
(D) 31.89% (E) None of these

208. A motorcycle travels from Delhi to Alwar. It travelled 25% of the


total distance at 25 km/hr. 2/5th of total distance at 32 km/hr and
remaining at 21 km/hr. then find the average speed of the motorcycle
for the whole journey.
एक भोटयसाइककर ददल्री से अरवय जाती है। मह कु र दयू ी का 25% 25 ककभी/घॊटा
ऩय, कुर दयू ी का 2/5 बाग 32 ककभी/घॊटा ऩय औय िे र्ष 21 ककभी/घॊटा की गनत से से
मात्रा कयता है, तो ऩूय ी मात्रा के शरए भोटयसाइककर की औसत गनत का ऩता रगाएॊ।
(A) 1200/49 km/hr (B) 1200/47 km/hr (C) 1200/46 km/hr
(D) 1000/47 km/hr (E) None of these

209. The ratio of present age of Luv and Kush is 5 : 6. after 4 years
the ratio of age of Luv and Kush will be 29 : 34, then what is the
present age of Lokit if Lokit is 6 years younger than Kush?
रव औय कु ि की वतषभान आमु का अनुऩात 5 : 6 है। 4 वर्षष फाद रव औय कु ि की
आमु का अनुऩात 29 : 34 होगा, तो रोककत की वतषभान आमु क्मा है मदद रोककत कुि
से 6 वर्षष छोटा है?
(A) 26 years (B) 24 years (C) 30 years
(D) 25 years (E) None of these

210. A motorboat rows ‘D’ km upstream and returns back in 16 hours


speed of boat in still water is 20 km/hr which is 3 times more than the
speed of the stream. Find the value of D?
एक भोटयफोट धाया के प्रनतकू र 'D' ककभी जाती है औय 16 घॊटे भें वाऩस रौटती है,
िाॊत ऩानी भें नाव की गनत 20 ककभी/घॊटा है जो धाया की गनत से 3 गुना अचधक है। D
का भान ऻात कीजजए।
(A) 150 km (B) 140 km (C) 160 km
(D) 180 km (E) None of these https://instagra
https://youtu https ://ww
be.com/chan w.fa cebook.
m.com/aashish https://
arorasocial(?)
123
nel/UCYa4_Jr
Orf8R5Kz2uO
com/a ashis utm_medium = t.me/st
ha rorasocial copy_link
tccXQ udified
211. Area of a rhombus is 336 m2 and one of its diagonal is 14 m
long. Find the perimeter of the rhombus.
एक सभचतुबुषज का ऺेत्रपर 336 भी2 है औय इसका एक ववकणष 14 भीटय रॊफा है।
सभचतुबुषज का ऩरयभाऩ ऻात कीजजए।
(A) 100 m (B) 130 m (C) 150 m
(D) 135 m (E) None of these

212.The ratio between the number of days that will be taken by P and
Q to complete 78 units of work alone is 5 : 8. If both start working
together, then the work will be completed in 3 days. Find the number
of days taken by Q to complete the work.
P औय Q द्वाया अकेरे 78 मूननट कामष को ऩूय ा कयने भें रगने वारे ददनों की सॊख्मा के
फीच का अनुऩात 5 : 8 है। मदद दोनों एक साथ कामष कयना प्रायॊ ब कयते हैं , तो कामष 3
ददनों भें ऩूय ा हो जाएगा। Q द्वाया कामष को ऩूय ा कयने भें शरए गए ददनों की सॊख्मा ऻात
कीजजए।
(A) 3.9 days (B) 9.5 days (C) 7.8 days
(D) 7 days (E) None of these

213. In a building there are 360 apartments, 58.33% of them are in A


complex, 25% in B complex, 60% of remaining in C complex. Find the
number of apartments in 4th complex i.e. D complex.
एक इभायत भें 360 अऩाटषभेंट हैं , उनभें से 58.33% A कॉम्प्रे क्स भें हैं , 25% B
कॉम्प्रे क्स भें हैं , फाकी का 60% C कॉम्प्रे क्स भें हैं । चौथे कॉम्प्रे क्स मानी D
कॉम्प्रे क्स भें अऩाटषभेंट की सॊख्मा ऻात कये ।
(A) 12 (B) 10 (C) 15
(D) 24 (E) None of these

214. A certain sum of money when deposited at 20% per annum


compound interest for 2 years gives interest of Rs. 4326 more when
the same sum is invested at 10% per annum simple interest for 3
years. Find the principal.
एक ननजश्चत याशि को 10% प्रनत वर्षष साधायण ब्माज ऩय 3 वर्षों के शरए ननवे ि ककमा
जाता है औय जफ साभान याशि को 20% प्रनत वर्षष की दय से 2 वर्षष के शरए चक्रवरवि
ब्माज ऩय जभा की जाती है, तो 4326 रुऩमे का अचधक ब्माज शभरता है। वप्रॊशसऩर
खोजें।
(A) Rs. 36000 (B) Rs. 30900 (C) Rs. 3090
(D) Rs. 306000 (E) None of these

215. If 240 ml solution containing milk and water in the ratio 3 : 1,


replaced twice by 80 ml water, then what will be the ratio of milk and
water in the final mixture?
मदद 240 शभरीरीटय घोर भें दध
ू औय ऩानी का अनुऩात 3 : 1 है, तो उसे दो फाय 80
शभरीरीटय ऩानी से फदर ददमा जाता है, तो अॊनतभ शभश्ण https://instagra
भें दध ू औय ऩानी का अनुऩात
https://youtu https ://ww
be.com/chan w.fa cebook.
m.com/aashish https://
arorasocial(?)
124
nel/UCYa4_Jr
Orf8R5Kz2uO
com/a ashis utm_medium = t.me/st
ha rorasocial copy_link
tccXQ udified
क्मा होगा?
(A) 2 : 3 (B) 3 : 4 (C) 1 : 2
(D) 3 : 2 (E) None of these

216. A and B start a business by investing Rs. 24000 and by Rs.


12000 respectively. B withdraws his total money after 4 months.
Another partner C joins with A 2 months after B left with Rs. 36000. If
B joins back in 9th month with the same amount and total profit after
1 year is Rs. 67500 then find the difference between profit share of A
and B.
A औय B क्रभि् 24000 औय 12000 रुऩमे का ननवे ि कयके एक व्मवसाम िुरू कयते
हैं । 4 भहीने फाद B अऩना कु र ऩैसा ननकारता है। एक अन्म साथी C, B के जाने के 2
भहीने फाद A के साथ 36000 रु. ननवे ि कयता है। मदद 9वें भहीने भें B सभान याशि के
साथ वाऩस िाशभर होता है औय 1 वर्षष के फाद कु र राब 67500 रु. है तो A औय B
के राब दहस्से के फीच का अॊतय ऻात कीजजए।
(A) Rs. 16000 (B) Rs. 13500 (C) Rs. 26002
(D) Rs. 21600 (E) None of these

217. A bike covers X km in 30 min. and (X – 4) km in 45 min. The


average speed of bike for the whole journey is 36 km/hr. Find the
value of X.
एक फाइक 30 शभनट भें X ककभी औय 45 शभनट भें (X – 4) ककभी की दयू ी तम कयती
है। ऩूय ी मात्रा भें फाइक की औसत गनत 36 ककभी/घॊटा है। X का भान ऻात कयें ।
(A) 24.5 (B) 26 (C) 20.5
(D) 22 (E) None of these

218. X sold a bag to Y at 58.33% loss and Y sold it to Z at 60%


profit. If X had sold it to Z directly at 33.33% profit, then Z would have
purchased the bag at what % more or less than the price at which he
actually purchased it?
X ने एक फैग Y को 58.33 प्रनतित हानन ऩय फे चा औय Y ने इसे Z को 60% राब
ऩय फे चा। मदद X ने इसे सीधे 33.33% राब ऩय Z को फे च ददमा होता, तो Z उस फैग
को उस वऩसे से ककतने प्रनतित अचधक मा कभ ऩय खयीदता, जजस ऩय उसने वास्तव भें
इसे खयीदा था?
(A) 120% (B) 100% (C) 10 %
(D) 12% (E) None of these

219. A shopkeeper has 2 types of pulses, the first type costs Rs. 36
per kg and the cost of second type is not known. To gain 70% on the
whole, he mixed 40 kg of first type pulse with 20 kg of 2nd type pulse
and the sold the mixture at Rs. 68 per kg. find the cost price of 2nd
type.
एक दुक ानदाय के ऩास 2 प्रकाय की दारें होती हैं , ऩहरे प्रकाय की दार की कीभत रु. 36
https://instagra
https://youtu https ://ww
be.com/chan w.fa cebook.
m.com/aashish https://
arorasocial(?)
125
nel/UCYa4_Jr
Orf8R5Kz2uO
com/a ashis utm_medium = t.me/st
ha rorasocial copy_link
tccXQ udified
प्रनत ककरो औय दस
ू ये प्रकाय की कीभत ऻात नहीॊ है। कु र शभराकय 70% राब प्राप्त
कयने के शरए, उसने ऩहरी प्रकाय की 40 ककरोग्राभ दार को 20 ककरोग्राभ दसू यी प्रकाय
की दार के साथ शभरामा औय शभश्ण को रु. 68 प्रनत ककग्रा ऩय फे च ददमा। दसू ये प्रकाय
की दार का रागत भूल्म ऻात कीजजए।
(A) 46 (B) 24 (C) 48
(D) 36 (E) None of these

220. The average of A, B and C is 39 and the average of B, C and D


is 54. If D is 62.5% more than A then find the average of B and C.
A, B औय C का औसत 39 है औय B, C औय D का औसत 54 है। मदद D, A से
62.5% अचधक है, तो B औय C का औसत ऻात कीजजए।
(A) 22.25 (B) 22.75 (C) 22.5
(D) CND (E) None of these

221. Three persons P, Q and R started a business with initial


investments of Rs. 2700, Rs. 1550 and Rs. 3500 respectively. After 8
months, they added another amount of Rs. 300, Rs. 350 and Rs. 1000
to their investment. After another 4 months they withdrew Rs. 1200,
Rs. 400 and Rs. 1650 respectively from their new investment. If P
received Rs. 20400 as his profit then what is the average profit after
16 months?
तीन व्मजक्त P, Q औय R ने 2700 रु. 1550 रु. औय 3500 रु. के प्रायॊ शबक ननवे ि
साथ एक व्मवसाम िुरू ककमा। 8 भहीने के फाद, उन्होंने 300 रु. 350 रु. औय 1000
रु. की याशि ननफे ि भें जोड दी। औय 4 भहीने के फाद उन्होंने उनके नए ननवे ि से
क्रभि् 1200 रु. 400 रु. औय 1650 रु. ननकार शरए। मदद P को उसके राब के रूऩ
भें 20400 रु. शभरे तो 16 भहीने के फाद औसत राब क्मा है?
(A) Rs. 22700 (B) Rs. 21700 (C) Rs. 23700
(D) Rs. 20700 (E) None of these

222. Pankaj alone can complete a work in 36 days. Salim is 33.33%


more efficient than Pankaj and Amit takes 22.22% more days than
Pankaj to complete the work alone. If all the three worked together for
10 days and left, then find the fraction of remaining work?
ऩॊक ज अकेरा एक कामष को 36 ददनों भें ऩूय ा कय सकता है। सरीभ ऩॊक ज से 33.33%
अचधक कु िर है औय अशभत अके रे काभ को ऩूय ा कयने भें ऩॊक ज से 22.22% अचधक
ददन रे ता है। मदद तीनों एक साथ 10 ददनों के शरए काभ कयते हैं औय छोड दे ते हैं , तो
िे र्ष कामष का अॊि ऻात कयें ?
(A) 37/297 (B) 37/295 (C) 37/299
(D) 37/293 (E) None of these

223. A mixture contains Salt and Sugar in the ratio of 5 : 3 and


another mixture contains them in the ratio 3 : 11 respectively. How
much quantity of the second mixture must be mixed with 32 kg of the
https://instagra
https://youtu https ://ww
be.com/chan w.fa cebook.
m.com/aashish https://
arorasocial(?)
126
nel/UCYa4_Jr
Orf8R5Kz2uO
com/a ashis utm_medium = t.me/st
ha rorasocial copy_link
tccXQ udified
first so that the resultant mixture may contain equal quantities of Salt
and Sugar?
एक शभश्ण भें नभक औय चीनी का अनुऩात 5 : 3 है औय दस ू ये शभश्ण भें क्रभि् 3 :
11 का अनुऩात है। दसू ये शभश्ण की ककतनी भात्रा ऩहरे के 32 ककग्रा के साथ शभचश्त
की जानी चादहए ताकक ऩरयणाभी शभश्ण भें नभक औय चीनी की सभान भात्रा हो सके ?
(A) 11 kg (B) 12 kg (C) 13 kg
(D) 14 kg (E) None of these

224. A certain amount is invested in Bank P for 5 years at X% simple


interest. If the same amount is invested in Bank Q for 8 years at (X +
6)% simple interest, then the ratio between the simple interest received
from Bank P and Bank Q is 1 : 4. find the rate of interest offered in
Bank Q?
एक ननजश्चत याशि फैंक P भें 5 वर्षष के शरए X% साधायण ब्माज ऩय ननवे ि की जाती
है। मदद सभान याशि फैंक Q भें 8 वर्षों के शरए (X + 6)% साधायण ब्माज ऩय ननवे ि
की जाती है, तो फैंक P औय फैंक Q से प्राप्त साधायण ब्माज के फीच का अनुऩात 1 :
4 है। फैंक Q भें दी जाने वारी ब्माज दय ऻात कीजजए।
(A) 15% (B) 10% (C) 12 %
(D) 11% (E) None of these

225. A book is sold at a gain of X%. If it was sold for Rs. P more,
there would have been a gain of (X + 7)%. Find the value of P if the
cost price of book is Rs. 1100.
एक ककताफ को X% के राब ऩय फे चा जाता है। अगय इसे P रु. अचधक भें फे चा गमा
होता तो (X + 7)% का राब होता। मदद ऩुस्तक का क्रम भूल्म 1100 रु. है तो P का
भान ऻात कीजजए।
(A) 88 (B) 66 (C) 55
(D) 77 (E) None of these

226. The speed of a boat is 46 km/hr which is 43.75% more than the
speed of stream. Boat covers A km downstream and (A – 12) km
upstream in 78 hour. Find the value of A?
एक नाव की गनत 46 ककभी/घॊटा है जो धाया की गनत से 43.75% अचधक है। नाव धाया
के अनुकू र A ककभी औय धाया के प्रनतकू र (A – 12) ककभी 78 घॊटे भें तम कयती है। A
का भान ऻात कीजजए।
(A) 936 (B) 824 (C) 854
(D) 954 (E) None of these

227. 10 years ago, the sum of age of A, B and C is 76 years and


after 4 years form present the sum of age of B and C will become 54
years, then what is the age of A after 12 years?
10 वर्षष ऩहरे A, B औय C की आमु का मोग 76 वर्षष है औय वतषभान से 4 वर्षष के फाद
B औय C की आमु का मोग 54 वर्षष हो जाएगा, तो 12 वर्षष फाद A की आमु क्मा है?
https://instagra
https://youtu https ://ww
be.com/chan w.fa cebook.
m.com/aashish https://
arorasocial(?)
127
nel/UCYa4_Jr
Orf8R5Kz2uO
com/a ashis utm_medium = t.me/st
ha rorasocial copy_link
tccXQ udified
(A) 55 years (B) 48 years (C) 38 years
(D) 72 years (E) None of these

228. The monthly salary of A is Rs. 17500 less than that of B and the
monthly salary of C is Rs. 8500 less than that of A. if the monthly
salary of B is 130% more than that of C, then what is the ratio of the
monthly salary of A, B and C?
A का भाशसक वे तन B से 17500 रु. कभ है औय C का भाशसक वे तन A से 8500 रु.
कभ है। मदद B का भाशसक वे तन C से 130% अचधक है, तो A, B औय C के भाशसक
वे तन का अनुऩात क्मा है?
(A) 45 : 57 : 96 (B) 57 : 92 : 40 (C) 40 : 57 : 85
(D) 57 : 92 : 65 (E) None of these

229. Area of a square is 3136 m2. The length of the rectangle is


14.28% less than the length of side of square and breadth of rectangle
is 33.33% less than the length of the rectangle. Find the perimeter of
the rectangle?
एक वगष का ऺेत्रपर 3136 भी2 है। आमत की रॊफाई वगष की बुज ा की रॊफाई से
14.28% कभ है औय आमत की चौडाई आमत की रॊफाई से 33.33% कभ है। आमत
का ऩरयभाऩ ऻात कीजजए।
(A) 160 m (B) 170 m (C) 150 m
(D) 140 m (E) None of these

230. The speed of a car is 413% more than that of Bike. To cover a
X distance bike take 2.5 hours more than car and speed of car is 68
km/hr then find the value of X?
एक काय की गनत फाइक की गनत से 41% अचधक है। एक x दयू ी को फाइक को तम
कयने भें काय से 2.5 घॊटे अचधक रगते हैं औय काय की गनत 68 ककभी/घॊटा है तो x का
भान ऻात कीजजए।
(A) 456 km (B) 384 km (C) 408 km
(D) 420 km (E) None of these

231. Sum of lengths of train P and Q is 1488 m. while moving in the


opposite directiontrain Q crosses train P in 24 seconds and the speed
of train Q is 58% more than speed of train P. What is the speed of
train P?
ट्रे न P औय Q की रॊफाई का मोग 1488 भीटय है। ववऩयीत ददिा भें चरते सभम ट्रे न Q
ट्रे न P को 24 से कॊड भें ऩाय कयती हैं औय ट्रे न Q की ट्रे न P की गनत से 581/3 %
अचधक है। ट्रे न P की गनत क्मा है?
(A) 24 m/s (B) 20 m/s (C) 28 m/s
(D) 62.5% (E) None of these

232. The total age of 17 students in a class is https://instagra


219. Two teachers of
https://youtu https ://ww
be.com/chan w.fa cebook.
m.com/aashish https://
arorasocial(?)
128
nel/UCYa4_Jr
Orf8R5Kz2uO
com/a ashis utm_medium = t.me/st
ha rorasocial copy_link
tccXQ udified
age 87 and 55 joined the class. What will be the average age of class
including those two teachers?
एक कऺा भें 17 छात्रों की कु र आमु 219 है। 87 औय 55 वर्षष के दो शिऺक कऺा भें
िाशभर हुए। उन दो शिऺकों को शभराकय कऺा की औसत आमु क्मा होगी?
(A) 18 years (B) 19 years (C) 16 years
(D) 15 years (E) None of these

233. Three years ago, Ram’s age is 1/4th of is father’s age. Ram’s
sister is 3 years older than him. 5 years hence, the age of Ram’s
father is 160% of age of Ram’s sister. Find the present age of Ram.
तीन वर्षष ऩहरे , याभ की आमु वऩता की आमु की 1/4 है। याभ की फहन उससे 3 वर्षष फडी
है। 5 वर्षष फाद, याभ के वऩता की आमु याभ की फहन की आमु का 160% है। याभ की
वतषभान आमु ऻात कीजजए।
(A) 6 years (B) 7 years (C) 8 years
(D) 9 years (E) None of these

234. Manish bought 72 articles at Rs. 18 each. If he can sell 46 article


at Rs. 24 each and the remaining article at Rs. 10 each, then what is
the overall profit percent earned by Manish?
भनीर्ष ने 72 वस्तुएॊ 18 रु. प्रनत इकाई की दय ऩय खयीदीॊ। मदद वह 46 वस्तु को 24
रु. प्रनत इकाई की दय ऩय औय िे र्ष वस्तु 10 रु. प्रनत इकाई की दय ऩय फे चता है, तो
भनीर्ष द्वाया अजजषत कुर राब प्रनतित ककतना है?
(A) 7.2% (B) 8.2% (C) 6.2 %
(D) 5.2% (E) None of these

235. An item is marked above 53 – % above its cost price and then a
discount of Rs. 465.2 is given on its marked price. If cost price of 38
items is equal to selling price of 30 items, then find the marked price
of item?
एक वस्तु ऩय उसके क्रम भूल्म को 53 % अचधक अॊककत ककमा जाता है औय कपय उसके
अॊककत भूल्म ऩय 465.2 रु की छूट दी जाती है। मदद 38 वस्तुओ ॊ का क्रम भूल्म 30
वस्तुओ ॊ के ववक्रम भूल्म के फयाफय है, तो वस्तु का अॊककत भूल्म ऻात कीजजए।
(A) Rs. 2674.9 (B) Rs. 27754.9 (C) Rs. 2876.9
(D) Rs. 2977.9 (E) None of these

236. Sunil, Anil and Pammi entered into a partnership and invested Rs.
25750, Rs. 17525 and Rs. 19725. After 5 months Sunil, Anil and
Pammi withdrew Rs. 1250, Rs. 1625 and Rs. 1375. If at the end of the
year profit of Anil is Rs. 7957, then find the total profit of them?
सुनीर, अननर औय ऩम्भी ने एक साझेदायी भें प्रवे ि ककमा औय 25750 रु. 17525 रु.
औय 19725 रु. का ननवे ि ककमा। 5 भहीने फाद सुनीर, अननर औय ऩम्भी ने 1250 रु.
1625 रु. औय 1375 रु. ननकार शरए। मदद वर्षष के अॊत भें अननर का राब 7957 रु.
है, तो उनका कु र राब ऻात कीजजए। https://instagra
https://youtu https ://ww
be.com/chan w.fa cebook.
m.com/aashish https://
arorasocial(?)
129
nel/UCYa4_Jr
Orf8R5Kz2uO
com/a ashis utm_medium = t.me/st
ha rorasocial copy_link
tccXQ udified
(A) Rs. 25450 (B) Rs. 29050 (C) Rs. 32454
(D) Rs. 26540 (E) None of these

237. Length and breadth of a rectangle is in the ratio of 12 : 7, if the


length is increased by 41% and breadth is increased by 28.56% then
the area is increased by 276 m², then find the perimeter of original
rectangle.
एक आमत की रॊफाई औय चौडाई का अनुऩात 12 : 7 है, मदद रॊफाई भें 41% की ववर ि
की जाती है औय चौडाई भें 28.56% की वरवि की जाती है, तो ऺेत्रपर भें 276m की
वरवि होती है, तो भूर आमत का ऩरयभाऩ जाते कीजजए।
(A) 68 m (B) 70 m (C) 74 m
(D) 74 m (E) None of these

238. P takes 3A days to complete a work while Q takes 7A/3 days to


complete the same work. If they started work together, they will
complete 40% of the work in 42 days, then find the value of A?
P एक कामष को ऩूय ा कयने भें 3A ददन रे ता है जफकक Q उसी कामष को ऩूय ा कयने भें
7A/3 ददन रे ता है। मदद वे एक साथ काभ कयना िुरू कयते हैं , तो वे 42 ददनों भें काभ
का 40% ऩूय ा कयें गे, तो A का भान ऻात कीजजए।
(A) 80 (B) 50 (C) 60
(D) 30 (E) None of these

239. Rajat invested certain amount for 3 years at 12% simple interest.
Had he invested the same amount for 2 years at 16% compound
interest annually, he would have received Rs. 108 more, find the sum
invested by Rajat.
यजत ने 12% साधायण ब्माज ऩय 3 वर्षों के शरए ननजश्चत याशि का ननवे ि ककमा। मदद
उसने सभान याशि को 2 वर्षों के शरए 162/3% वावर्षषक चक्रवरवि ब्माज ऩय ननवे ि ककमा
होता, तो उसे 108 रु. अचधक प्राप्त होते , यजत दवाया ननवे ि की गई याशि ऻात
कीजजए।
(A) Rs. 84200 (B) Rs. 98200 (C) Rs. 86200
(D) Rs. 97200 (E) None of these

240. To cover a 384 km Downstream and 32 km upstream a boat


takes 28 hours and speed of boat is 13.33% more than the speed of
stream, then find the time taken by boat in covering 1032 km
downstream and 96 km upstream?
384 ककभी धाया के अनुकूर औय 32 ककभी धाया के ववरुि कवय कयने के शरए एक नाव
28 घॊटे रे ती है औय नाव की गनत धाया की गनत से 13.33% अचधक है, तो 1032
ककभी धाया के अनुकूर औय 96 ककभी धाया के ववरुि को कवय कयने भें नाव द्वाया
शरमा गमा सभम ऻात कयें ?
(A) 80.25 hours (B) 85.25 hours (C) 60.25 hours
(D) 70.25 hours (E) None of these https://instagra
https://youtu https ://ww
be.com/chan w.fa cebook.
m.com/aashish https://
arorasocial(?)
130
nel/UCYa4_Jr
Orf8R5Kz2uO
com/a ashis utm_medium = t.me/st
ha rorasocial copy_link
tccXQ udified
241. Amit bought a motorcycle and spent Rs. 18500 on its repairing. If
he sold the motorcycle at Rs. 85652 and earned a profit of 33%, then
find the price at which he bought the motorcycle?
अशभत ने एक भोटयसाइककर खयीदी औय 18500 रु. इसकी भयम्भत ऩय खचष ककए।
अगय उसने भोटयसाइककर को 85652 रु. भें फे चा औय 33% का राब अजजषत ककमा, तो
उसने भोटयसाइककर को ककस भूल्म ऩय खयीदा था?
(A) Rs. 45900 (B) Rs. 48600 (C) Rs. 45800
(D) Rs. 47500 (E) None of these

242. After 7 years, the age of Pankaj will be 28.56% less than age of
Salim. The ratio of the age of Pankaj and Salim 6 years ago is 6 : 11.
what will be the age of Salim after 8 years?
7 वर्षष फाद ऩॊक ज की आमु सरीभ की आमु से 28.56 प्रनतित कभ होगी। 6 वर्षष ऩव
ू ष
ऩॊक ज औय सरीभ की आमु का अनुऩात 6 : 11 है। 8 वर्षष फाद सरीभ की आमु क्मा
होगी?
(A) 48 years (B) 24 years (C) 36 years
(D) 42 years (E) None of these

243. The perimeter of a rectangle is 4 times the radius of the circle.


The area of circle is 1386 m2. If the breadth of the rectangle is 20 m,
then find the length of the rectangle.
एक आमत का ऩरयभाऩ वरत्त की त्रत्रज्मा का 4 गुना है। वरत्त का ऺेत्रपर 1386 भी2 है।
मदद आमत की चौडाई 20 भीटय है, तो आमत की रॊफाई ऻात कीजजए।
(A) 26 m (B) 22 m (C) 24 m
(D) 20 m (E) None of these

244. A sum of Rs.(P + 12500) at 15% per annum compounded


annually for two years is Rs.(2P + 6030), then what is the value of P?
(P + 12500) रु. की याशि 15% प्रनतवर्षष की दय से दो वर्षों के शरए वावर्षषक रूऩ से
चक्रवरचधत ककमे जाने ऩय (2P + 6030) रु. प्राप्त होता है, तो P का भान क्मा है?
(A) Rs. 15500 (B) Rs. 15250 (C) Rs. 15270
(D) Rs. 15285 (E) None of these

245. P, Q and R working alone can do a work in 45 days, 30 days


and 60 days. For completing the whole work together, their total wages
is Rs. 2275, find the share of R?
P, Q औय R अके रे कामष कयते हुए ककसी कामष को 45 ददन, 30 ददन औय 60 ददन भें
ऩूय ा कय सकते हैं । ऩूये काभ को एक साथ ऩूय ा कयने के शरए, उनकी कु र भजदयू ी 2275
रु. है, R का दहस्सा ऻात कीजजए।
(A) Rs. 475 (B) Rs. 625 (C) Rs. 450
(D) Rs. 535 (E) None of these

https://youtu https://instagra
https ://ww
be.com/chan w.fa cebook.
m.com/aashish https://
arorasocial(?)
131
nel/UCYa4_Jr
Orf8R5Kz2uO
com/a ashis utm_medium = t.me/st
ha rorasocial copy_link
tccXQ udified
246. A mixture of rice and wheat in the ratio 1 : 7. if ratio of cost of
rice to that of mixture of rice and wheat is 11 : 32, then find the ratio
of cost of rice to that of wheat?
चावर औय गेहूॊ का शभश्ण 1 : 7 के अनुऩात भें है। मदद चावर तथा चावर औय गेहूॊ के
शभश्ण की रागत का अनुऩात 11 : 32 है, तो चावर की रागत का गेहूॊ की रागत से
अनुऩात ऻात कीजजए।
(A) 3 : 11 (B) 11 : 3 (C) 15 : 7
(D) 7 : 15 (E) None of these

247. Manish and Chunnu started a shop. The investment of Manish is


56.25% more than investment of Chunnu and time period of Chunnu’s
investment is 37.5% more than that of Manish’s investment. If
difference between their profit is Rs. 4440, then find the profit share of
Chunnu?
भनीर्ष औय चुन्नू ने एक दुक ान िुरू की। भनीर्ष का ननवे ि चुन्नू के ननवे ि से 56.25%
अचधक है औय चुन्नू के ननवे ि की सभमावचध भनीर्ष के ननवे ि से 37.5% अचधक है।
मदद उनके राब के फीच का अॊतय रु. 4440, तो चुन्नू का राब दहस्सा ऻात कीजजए।
(A) Rs. 32000 (B) Rs. 34650 (C) Rs. 33640
(D) Rs. 32560 (E) None of these

248. The average number of students in School P, Q and R is 169


while the average number of students in P and Q is 124. if the
average of number of students in School R and S is 179 then find the
number of students in School S?
स्कू र P, Q औय R भें छात्रों की औसत सॊख्मा 169 है जफकक P औय Q भें छात्रों की
औसत सॊख्मा 124 है। मदद स्कू र R औय S भें छात्रों की औसत सॊख्मा 179 है, तो
स्कूर S भें छात्रों की सॊख्मा ऻात कीजजए।
(A) 89 (B) 99 (C) 104
(D) 95 (E) None of these

249. When a shopkeeper gives a discount of 13% on marked price of


an item he gains 27%, if the cost price of the product increased by
10% and selling price still remains same then find the profit percent?
जफ एक दुक ानदाय ककसी वस्तु के अॊककत भूल्म ऩय 13% की छू ट दे ता है तो उसे 27%
का राब होता है, मदद उत्ऩाद की रागत भूल्म भें 10% की वरवि होती है औय त्रफक्री
भूल्म अबी बी वही यहता है तो राब प्रनतित ऻात कीजजए।
(A) 12.5% (B) 14.5% (C) 15.45 %
(D) 18.18% (E) None of these

250. The speed of two boats P and Q in still water is 19 km/h and 21
km/h and the speed of the stream is 5 km/h. What is the total time
taken by boat P to cover a distance of 231 km upstream and 507 km
downstream by boat Q? https://instagra
https://youtu https ://ww
be.com/chan w.fa cebook.
m.com/aashish https://
arorasocial(?)
132
nel/UCYa4_Jr
Orf8R5Kz2uO
com/a ashis utm_medium = t.me/st
ha rorasocial copy_link
tccXQ udified
िाॊत जर भें दो नावों P औय Q की गनत 19 ककभी/घॊटा औय 21 ककभी/घॊटा है औय
धाया की गनत 5 ककभी/घॊटा है। नाव P द्वाया धाया के प्रनतकूर 231 ककभी औय नाव Q
द्वाया धाया के अनुकू र 507 ककभी की दयू ी तम कयने भें कु र ककतना सभम रगता है?
(A) 32.5 hour (B) 34.5 hour (C) 36 hour
(D) 38 hours (E) None of these

251. Akshay started a business with an investment of 12500 and after


P years Prashant joins him with Rs. 7500. After P years more, Akshay
withdrew Rs. 2500 and at the end of 20 years ratio of profit share of
Akshay to Prashant is 46 : 21, then what is the values of P?
अऺम ने 12500 रु. के ननवे ि के साथ एक व्मवसाम िुरू ककमा औय P वर्षों के फाद
प्रिाॊत उसके साथ 7500 रु. के साथ िाशभर हो गमा। P वर्षष अचधक के फाद, अऺम ने
2500 रु. ननकारे है औय 20 वर्षष के अॊत भें अऺम औय प्रिाॊत के राब दहस्से का
अनुऩात 46 : 21 है, तो P का भान क्मा है?
(A) 5 (B) 6 (C) 7
(D) 8 (E) None of these

252. When the efficiency of a man is increased by 46%, he can


complete a work in 18 days lesser than the number of days taken by
him to complete the work when his efficiency is decreased by 13.33%.
The number of days taken by the man to complete work with his
original efficiency?
जफ एक व्मजक्त की दऺता भें 462/3% की वरवि होती है, तो वह उसी कामष को ऩूय ा
कयने भें उसके द्वाया शरए गए ददनों की सॊख्मा से 18 ददनों से काभ सभम भें कामष ऩूय ा
कय सकता है, जफ उसकी दऺता भें 13.33% की कभी हो जाती है। आदभी द्वाया अऩनी
भूर दऺता के साथ काभ ऩूय ा कयने भें रगने वारे ददनों की सॊख्मा?
(A) 32 2/15 days (B) 36 2/15 days (C) 38 2/15 days
(D) 37 2/15 days (E) None of these

253. While travelling with a speed of 68 km/hr, a bike takes 20 minute


more than actual time to cover A km and with speed of 102 km/Hr, it
takes 12 min less than actual time to cover the same distance, find
the value of A.
68 ककभी/घॊटा की गनत से मात्रा कयते सभम, एक फाइक A ककभी की दयू ी तम कयने भें
वास्तववक सभम से 20 शभनट अचधक रे ती है औय 102 ककभी/घॊटा की गनत से , सभान
दयू ी को तम कयने भें वास्तववक सभम से 12 शभनट कभ रे ती है, तो A का भान ऻात
कीजजए।
(A) 105.8 km (B) 107.8 km (C) 108.8 km
(D) 110.8 km (E) None of these

254. 42 L mixture of milk and water is mixed with 56 L of another


mixture that contains milk and water in the ratio 17 : 11. Final mixture
contains milk and water in the ratio 27 : 22, then what is the
https://instagra
https://youtu https ://ww
be.com/chan w.fa cebook.
m.com/aashish https://
arorasocial(?)
133
nel/UCYa4_Jr
Orf8R5Kz2uO
com/a ashis utm_medium = t.me/st
ha rorasocial copy_link
tccXQ udified
difference between the quantities of milk and water in the first
mixture?
दध
ू औय ऩानी के 42 रीटय शभश्ण को 56 रीटय दस ू ये शभश्ण भें शभरामा जाता है
जजसभें दध
ू औय ऩानी का अनुऩात 17 : 11 है। अॊनतभ शभश्ण भें दध ू औय ऩानी का
अनुऩात 27 : 22 है, तो ऩहरे शभश्ण भें दध
ू औय ऩानी की भात्रा के फीच का अॊतय
ककतना है?
(A) 4 L (B) 2 L (C) 5 L
(D) 6 L (E) None of these

255. The side and diagonal of a rhombus are 13 cm and 10 cm


respectively. Find the side of the square whose area is 24 cm2 more
than the area of rhombus?
एक सभचतुबुषज की बुज ा औय ववकणष क्रभि् 13 से भी औय 10 से भी हैं । उस वगष की
बुज ा ऻात कीजजए, जजसका ऺेत्रपर सभचतुबुषज के ऺेत्रपर से 24 से भी2 अचधक है?
(A) 10 cm (B) 9 cm (C) 14 cm
(D) 12 cm (E) None of these

256. From his total amount, Amit gave 28% to Soniya. Amit gave 12%
from his initial amount to Prerna and Soniya gave 20% of the amount
received from Amit to Prerna. If Prerna receives Rs. 2728, then what
is total amount given by Amit to Soniya?
अऩनी कुर याशि भें से अशभत ने सोननमा को 28% ददमा। अशभत ने अऩनी प्रायॊ शबक
याशि का 12% प्रे य णा को ददमा औय सोननमा ने अशभत से प्राप्त याशि का 20% प्रे य णा
को ददमा। अगय प्रे य णा को कु र 2728 रु. शभरते हैं तो अशभत द्वाया सोननमा को दी गई
कु र याशि ककतनी है?
(A) Rs. 4340 (B) Rs. 4250 (C) Rs. 4346
(D) Rs. 4220 (E) None of these

257. A shopkeeper purchases a mobile for Rs. 10500 and marked it up


by 53%. He sold it after giving two successive discounts of Rs. 880
and 20%. Had he interchanged both the discounts, what would have
been the change in the profit amount?
एक दुक ानदाय एक भोफाइर 10500 रु. भें खयीदता है औय उस ऩय 531/3% का
भाकषअऩ कयता है। उसने इसे 880 रु. औय 20% की दो क्रभागत छू ट दे क य फे च ददमा।
मदद उसने दोनों छू टों को आऩस भें फदर ददमा होता, तो राब की याशि भें क्मा ऩरयवतषन
होता?
(A) Rs. 145 (B) Rs. 194 (C) Rs. 176
(D) Rs. 185 (E) None of these

258. The average age of A, B, C and D is 52 while the average age


of A, B, C, E and F is 48, if the average age of D, E and F is 42,
then find the age of D.
A, B, C औय D की औसत आमु 52 है जफकक A, B, C,https://instagra
E औय F की औसत आमु
https://youtu https ://ww
be.com/chan w.fa cebook.
m.com/aashish https://
arorasocial(?)
134
nel/UCYa4_Jr
Orf8R5Kz2uO
com/a ashis utm_medium = t.me/st
ha rorasocial copy_link
tccXQ udified
48 है, मदद D, E औय F की औसत आमु 42 है, तो D की आमु ऻात कीजजए।
(A) 77 years (B) 94 years (C) 71 years
(D) 65 years (E) None of these

259. P invested Rs. 17500, Q invested Rs. 21400 and R invested Rs.
12300 for 4 months, 5 months and 8 months respectively. The profit
made by R is Rs. 6888, find total profit.
P ने 17500 रु. का ननवे ि ककमा, Q ने 21400 रु. का ननवे ि ककमा औय R ने 12300
रु. का ननवे ि ककमा क्रभि् 4 भहीने , 5 भहीने औय 8 भहीने के शरए। R द्वाया अजजषत
राब 6888 रु. है, कु र राब ऻात कीजजए।
(A) Rs. 19278 (B) Rs. 19528 (C) Rs. 19547
(D) Rs. 19485 (E) None of these

260. Rs. P is invested in a bank for 3 years at 15% per annum


compounded annually and received Rs. 4167 as interest. Find the
interest received by the same sum if it is invested for 5 years at
18.5% simple interest.
P रु. को एक फैंक भें 3 वर्षष के शरए 15% वावर्षषक चक्रवरवि ब्माज की दय से ननवे ि
ककमा जाता है औय ब्माज के रूऩ भें 4167 रु. प्राप्त होता है, मदद इसे 5 वर्षष के शरए
18.5 प्रनतित साधायण ब्माज ऩय ननवे ि ककमा जाता है तो उसी याशि से प्राप्त ब्माज
ऻात कीजजए।
(A) Rs. 7600 (B) Rs. 8200 (C) Rs. 7500
(D) Rs. 7400 (E) None of these

261. The average price of 17 item is decreased by Rs. 4 when the


item worth Rs.______ is replaced with the item worth Rs. 1751.
17 वस्तुओ ॊ का औसत भूल्म 4 रुऩमे कभ हो जाता है जफ ______ रुऩमे की वस्तु को
1751 रुऩमे की वस्तु से फदर ददमा जाता है।
(A) Rs. 1458 (B) Rs. 1845 (C) Rs. 1819
(D) Rs. 1745 (E) None of these

262. The ratio of the present age of Rohit and Kanika is 5 : 2 and the
ratio of present ages of Monu and Rohit is 7 : 3. the sum of the
present ages of Rohit, Kanika and Monu is 168 years. what is the
present age of Kanika?
योदहत औय कननका की वतषभान आमु का अनुऩात 5 : 2 है औय भोनू औय योदहत की
वतषभान आमु का अनुऩात 7 : 3 है। योदहत, कननका औय भोनू की वतषभान आमु का
मोग 168 वर्षष है। कननका की वतषभान आमु क्मा है?
(A) 20 years (B) 18 years (C) 16 years
(D) 14 years (E) None of these

263. While moving with a speed of P km/hr, a train crosses a pole in


24 seconds. If the speed of the train decreaseshttps://instagra
by 5 m/s, then it takes
https://youtu https ://ww
be.com/chan w.fa cebook.
m.com/aashish https://
arorasocial(?)
135
nel/UCYa4_Jr
Orf8R5Kz2uO
com/a ashis utm_medium = t.me/st
ha rorasocial copy_link
tccXQ udified
3 seconds more to cross the pole. What is the value of P?
एक ट्रे न P ककभी/घॊटा की गनत से चरते हुए एक खम्बे को 24 से कॊड भें ऩाय कयती है।
मदद ये रगाडी की गनत 5 भी/से कभ हो जाती है, तो वह खम्बे को ऩाय कयने भें 3
से क ण्ड अचधक रे ती है। P का भान क्मा है?
(A) 164 (B) 156 (C) 154
(D) 162 (E) None of these

264. A, B and C started a business together. Seven times the


investment of A is equal to the four times the investment of B is equal
to the five times the investment of C. at the end of one year, if the
profit earned by C is Rs. 10192 then find the total profit earned by
them.
A, B औय C ने शभरकय एक व्माऩाय िुरू ककमा। A के ननवे ि का सात गन ु ा, B के
ननवे ि के चाय गुना के फयाफय है, C के ननवे ि के ऩाॊच गुना के फयाफय है। एक वर्षष के
अॊत भें , मदद C द्वाया अजजषत राब 10192 रु. है, तो उनके द्वाया अजजषत कु र राब
ऻात कीजजए।.
(A) Rs. 30212 (B) Rs. 31542 (C) Rs. 32546
(D) Rs. 30145 (E) None of these

265. P had a total of 660 apples initially. P gave 53(1/3)% of his total
number of apples to Q, who gave 25% of the number of apples that P
got to R. If R gave 27.27% of the number of apples that Q got to S,
then how many apples did S get from R?
P के ऩास िुरू भें कु र 660 से फ थे। P ने अऩने से फों की कु र सॊख्मा का 53(1/3)% Q
को ददमा, जजसने P से प्राप्त से फों की सॊख्मा का 25% R को ददमा। मदद R ने Q से
प्राप्त से फों की सॊख्मा का 27.27% S को ददमा, तो S को R से ककतने से फ प्राप्त हुए?
(A) 54 (B) 48 (C) 24
(D) 27 (E) None of these

266. Sumit marked a laptop 58¹% above the purchased price. While
selling the laptop he offered a discount of 30% to his friend and sold
it for Rs. 1995, find the price at which Sumit purchased the laptop?
सुशभत ने एक रैऩटॉऩ ऩय खयीदे गए भूल्म से 581/3% अचधक अॊककत ककमा। रैऩटॉऩ
फे चते सभम उसने अऩने दोस्त को 30% की छू ट की ऩे िकि की औय इसे 1995 रु. भें
फे च ददमा, सुशभत ने रैऩटॉऩ ककस कीभत ऩय खयीदी था?
(A) Rs. 1800 (B) Rs. 1450 (C) Rs. 1600
(D) Rs. 1650 (E) None of these

267. In a mixture A of 12 litre, ratio of milk to water is 7 : 5, when 48


litre of mixture B of milk and water is mixed with mixture A, ratio of
mixture of milk and water becomes 1 : 5 find the percentage of milk in
mixture B?
12 रीटय के शभश्ण A भें दध
ू औय ऩानी का अनुऩात 7 : https://instagra
5 है, जफ 48 रीटय शभश्ण
https://youtu https ://ww
be.com/chan w.fa cebook.
m.com/aashish https://
arorasocial(?)
136
nel/UCYa4_Jr
Orf8R5Kz2uO
com/a ashis utm_medium = t.me/st
ha rorasocial copy_link
tccXQ udified
B दधू औय ऩानी को शभश्ण A भें शभरामा जाता है, तो दधू औय ऩानी के शभश्ण का
अनुऩात 1 : 5 हो जाता है शभश्ण B भें दध
ू का प्रनतित ऻात कीजजए।
(A) 5.25% (B) 5.5% (C) 6.25%
(D) 7% (E) None of these

268. Speed of boat in still water is 415% more than the speed of
stream. The boat covers 348 km downstream and 35 km upstream in
19 hour, find the time taken by boat in covering 406 km downstream
and 62.5 km upstream?
िाॊत जर भें नाव की गनत धाया की गनत से 412/3% अचधक है। नाव धाया के अनुकू र
348 ककभी औय धाया के प्रनतकूर 35 ककभी की दयू ी 19 घॊटे भें तम कयती है, नाव
द्वायों धाया के अनुकू र 406 ककभी औय धाया के प्रनतकू र 62.5 ककभी की दयू ी तम कयने
भें ककतना सभम रगती है?
(A) 24.5 hr (B) 26.5 hr (C) 25.5 hr
(D) 32.5 hr (E) None of these

269. A sum amounts to be Rs. 2200 at 12.5% rate of simple interest


per annum in 3 years, find the amount received when the same sum
invested at same rate compounded annually for 2 years?
P के ऩास कुर 220 से फ हैं । P के ऩास Q से 46(2/3)% अचधक से फ हैं औय R के
ऩास Q से 73(1/3)% अचधक से फ हैं । मदद R ने अऩने ऩास के से फों का 9/13वाॊ दहस्सा
खामा है, तो उसके द्वाया अबी नहीॊ खाए गए से फों की सॊख्मा ऻात कीजजए।
(A) 50 (B) 60 (C) 70
(D) 80 (E) None of these

274. The average price of 42 bottles is Rs. 172. if the average price of
21 bottles is Rs.(P + 18) and the average price of remaining bottles is
Rs.(Q – 12), then find the value of P + Q.
42 फोतरों का औसत भूल्म 172 रु. है। मदद 21 फोतरों का औसत भूल्म (P + 18) रु.
है औय िे र्ष फोतरों का औसत भूल्म (Q – 12) रु. है, तो P + Q का भान ऻात
कीजजए।
(A) 338 (B) 345 (C) 365
(D) 367 (E) None of these

275. 4 years ago. ratio of age of Nirmal and Pooja is 18 : 23. after 4
years, ratio of their ages will be 4 : 5. Laado is as much as younger
to Pooja as she is older to Nirmal. Find the ratio of present age of
Laado and Pooja.
4 वर्षष ऩहरे , ननभषर औय ऩूज ा की आमु का अनुऩात 18 : 23 था। 4 वर्षष फाद, उनकी
आमु का अनुऩात 4 : 5 होगा। राडो ऩूज ा से उतनी ही छोटी है, जजतनी वह ननभषर से
फडी है। राडो औय ऩूज ा की वतषभान आमु का अनुऩात ऻात कीजजए।
(A) 48 : 53 (B) 43 : 48 (C) 43 : 51
(D) 43 : 47 (E) None of these https://instagra
https://youtu https ://ww
be.com/chan w.fa cebook.
m.com/aashish https://
arorasocial(?)
137
nel/UCYa4_Jr
Orf8R5Kz2uO
com/a ashis utm_medium = t.me/st
ha rorasocial copy_link
tccXQ udified
276. The ratio of time taken by a boat to cover 45 km upstream to the
time taken by it to cover 90 km downstream is 11 : 9. if the speed of
stream is 1.3 km/hr, then find the time taken by the boat to cover 462
km in downstream.
एक नाव द्वाया धाया के प्रनतकूर 45 ककभी की दयू ी तम कयने भें शरए गए सभम का
धाया के अनुकू र 90 ककभी की दयू ी तम कयने भें शरए गए सभम का अनुऩात 11 : 9
है। मदद धाया की गनत 1.3 ककभी/घॊटा है, तो नाव द्वाया धाया के अनुकूर 462 ककभी
की दयू ी तम कयने भें शरमा गमा सभम ऻात कीजजए।
(A) 140 hr (B) 135 hr (C) 105 hr
(D) 115 hr (E) None of these

277. Sonu bought 75 articles and sold all of them at a loss, the loss
is equal to the cost price of X articles. If he had the loss of 46%, then
the value of X is?
सोनू ने 75 वस्तुएॉ खयीदीॊ औय उन सबी को हानन ऩय फे च ददमा, हानन X वस्तुओ ॊ के
क्रम भूल्म के फयाफय है। मदद उसे 46/3% की हानन होती है, तो X का भान है?
(A) 35 (B) 42 (C) 28
(D) 49 (E) None of these

278. Half of a sum of money was invested at 15% rate of simple


interest for 2 years and rest of the sum is invested at 13.33% rate of
compound interest. If the total interest received after two years is Rs.
1578, then find the sum?
एक याशि का आधा 2 वर्षष के शरए साधायण ब्माज की 15% दय ऩय ननवे ि ककमा गमा
औय िे र्ष याशि को चक्रवरवि ब्माज की 13.33% दय ऩय ननवे ि ककमा गमा। मदद दो वर्षष
फाद प्राप्त कु र ब्माज 1578 रु. है, तो याशि ऻात कीजजए।
(A) Rs. 7000 (B) Rs. 7200 (C) Rs. 4900
(D) Rs. 5400 (E) None of these

279. To complete a task, P is 413% more efficient than Q. if P works


for the first 15 days and rest of the work is completed by Q in 5 days.
In how much time will Q alone will complete the 60% of the work?
एक कामष को ऩूया कयने के शरए, P, Q से 41 /3% अचधक कुिर है। मदद P ऩहरे 15
ददनों के शरए कामष कयती है औय िे र्ष कामष ) द्वाया 5 ददनों भें ऩूय ा ककमा जाता है। Q
अकेरे 60% कामष को ककतने सभम भें ऩूय ा कये गा?
(A) 12.75 days (B) 15.75 days (C) 16.75 days
(D) 18.75 days (E) None of these

280. A75 L of mixture contains 46 % water and rest is milk. If 20% of


mixture is taken out and replaced with same quantity of water and the
same process is repeated once more. What is the quantity of water in
final mixture?
एक 75 रीटय शभश्ण भें 46/3% ऩानी है औय िे र्ष दध
ू है। https://instagra
मदद शभश्ण का 20%
https://youtu https ://ww
be.com/chan w.fa cebook.
m.com/aashish https://
arorasocial(?)
138
nel/UCYa4_Jr
Orf8R5Kz2uO
com/a ashis utm_medium = t.me/st
ha rorasocial copy_link
tccXQ udified
ननकार शरमा जाता है औय ऩानी की सभान भात्रा के साथ फदरे ददमा जाता है औय उसी
प्रकक्रमा को एक फाय कपय दोहयामा जाता है। अॊनतभ शभश्ण भें ऩानी की भात्रा ककतनी है ?
(A) 50.4 L (B) 51.4 L (C) 55.8L
(D) 49.4 L (E) None of these

281. Average of girl and boy child in village A and B are 4000 and
6000 respectively. If ‘x’ boys and ‘x + 300’ girls move from village A to
B, then the average of village B is increased by 450. Then what is the
new average of village A?
गाॉव A औय B भें रडकी औय रडके का औसत क्रभि् 4000 औय 6000 है। मदद 'x‘
रडके औय 'x + 300' रडककमाॊ गाॉव A से B भें चरे जाते हैं , तो गाॉव B के औसत भें
450 की वरवि होती है, तो गाॉव A का नमा औसत क्मा है?
(A) 4450 (B) 4500 (C) 3550
(D) 3500 (E) None of these

282. Avantika bought an item at a price more than Rs. 55 but less
than Rs. 65. She sold the item at the price whose value is obtained
by reversing the positions of the digits of the value of cost price. The
profit earned was Rs. 27. At what price did Avantika sold the item?
अवॊनतका ने एक वस्तु को 55 रुऩमे से अचधक की कीभत ऩय खयीदा रे ककन 65 रुऩमे से
कभ कीभत ऩय ख़यीदा। उसने वस्तु को उस भूल्म ऩय फे चा जजसका भूल्म क्रम भूल्म के
भूल्म के अॊक ों के स्थान को उरटने ऩय प्राप्त होता है। अजजषत राब रु. 27 है। अवॊनतका
ने वस्तु को ककस कीभत ऩय फे चा?
(A) Rs. 85 (B) Rs. 75 (C) Rs. 57
(D) Rs. 58 (E) None of these

283. ‘x’ years ago, the ratio of ages of Ram and Piya was 4 : 5
respectively. ‘3x + 3’ years later, this ratio will be 9 : 10 respectively.
If the difference between their ages is 3 years, then find what is the
value of ‘x’?
'x' वर्षष ऩहरे , याभ औय वऩमा की आमु का अनुऩात क्रभि् 4 : 5 था। '3x + 3' सार
फाद मह अनुऩात क्रभि : 9 : 10 होगा। मदद उनकी आमु के फीच का अॊतय 3 वर्षष है,
तो 'x' का भान क्मा है?
(A) 3 (B) 6 (C) 5
(D) 4 (E) None of these

284. A bike covers 84 km at a speed of (y – 5) km/hr in some time. If


the bike travels at a speed of y km/hr, it would cover 60 km more in
the same time. Find the value of y.
एक फाइक कुछ सभम भें (y – 5) ककभी/घॊटा की गनत से 84 ककभी की दयू ी तम कयती
है। मदद फाइक y ककभी/घॊटा की गनत से मात्रा कयती है, तो मह सभान सभम भें 60
ककभी अचधक दयू ी तम कये गी। y का भान ऻात कीजजए।

https://youtu https://instagra
https ://ww
be.com/chan w.fa cebook.
m.com/aashish https://
arorasocial(?)
139
nel/UCYa4_Jr
Orf8R5Kz2uO
com/a ashis utm_medium = t.me/st
ha rorasocial copy_link
tccXQ udified
(A) 10 (B) 15 (C) 7
(D) 12 (E) None of these

285. 12 men or 14 boys can do a piece of work in 15 days. In how


many days can 4 boys and 3 men together complete a work as 5
times as before?
12 आदभी मा 14 रडके एक काभ को 15 ददनों भें कय सकते हैं । 4 रडके औय 4
आदभी शभरकय एक काभ जो ऩहरे से 5 गन
ु ा है ककतने ददनों भें ऩयू ा कय सकते हैं ?
(A) 140 (B) 160 (C) 125
(D) 105 (E) None of these

286. A, B, C and D entered into a partnership such that A invested an


amount equal to one – fourteenth of total investment, B invested an
amount equal to one – fifteenth of total investment, C invested an
amount equal to one – fourth of total investment and D invested an
amount equal to one – sixth of total investment. If their total profit is
Rs. 58250 and each of them invested for 2 years, then what is the
difference between the profit share of A and D?
A, B, C औय D ने एक साझेदायी भें प्रवे ि ककमा जैसे कक A ने कु र ननवे ि के चौदहवें
दहस्से के फयाफय याशि का ननवे ि ककमा, B ने कुर ननवे ि के ऩॊद्रहवें दहस्से के फयाफय
याशि का ननवे ि ककमा, C ने कु र ननवे ि के एक – चौथाई के फयाफय याशि का ननवे ि
ककमा औय D ने कुर ननवे ि के एक – छठे के फयाफय याशि का ननवे ि ककमा। मदद उनका
कु र राब रु. 58250 औय उनभें से प्रत्मे क ने 2 वर्षष के शरए ननवे ि ककमा, तो A औय
D के राब दहस्से भें ककतना अॊतय है?
(A) Rs. 16000 (B) Rs. 13500 (C) Rs. 20000
(D) Rs. 10000 (E) None of these

287. Shivam gets Rs. 880 after investing at Rs. Y at 20% per annum
SI for 6 years and Rs. 464 after investing Rs. X at 15% per annum SI
for 3 years. Find the value of (Y – X).
शिवभ ् को 6 वर्षष के शरए, 20% प्रनत वर्षष साधायण ब्माज ऩय Y रूऩमे का ननवे ि कयने
के फाद 880 रूऩमे औय 3 वर्षष के शरए, 15% प्रनत वर्षष साधायण ब्माज ऩय X रूऩमे का
ननवे ि कयने के फाद 464 रूऩमे प्राप्त होते है, तो (Y – X) का भान ऻात कीजजए।
(A) 85 (B) 75 (C) 80
(D) 90 (E) None of these

288. If price of rice is increased by 14% and the expenditure of a


family is increased by 26% then the consumption of rice for the family
increased by :
मदद चावर की कीभत भें 14% की वरवि होती है औय एक ऩरयवाय के खचष भें 26% की
ववर ि होती है, तो ऩरयवाय के शरए चावर की खऩत भें ककतनी ववर ि होती है :
(A) 10.68% (B) 10.77% (C) 10.52%
(D) 10.35 (E) None of these https://instagra
https://youtu https ://ww
be.com/chan w.fa cebook.
m.com/aashish https://
arorasocial(?)
140
nel/UCYa4_Jr
Orf8R5Kz2uO
com/a ashis utm_medium = t.me/st
ha rorasocial copy_link
tccXQ udified
289. Perimeter of a square and rectangle are in the ratio 3 : 1
respectively. Length of rectangle is 2 cm more than its breadth and
half of side of square. What is the difference between area of square
and rectangle?
एक वगष औय आमत का ऩरयभाऩ क्रभि् 3 : 1 के अनुऩात भें है। आमत की रॊफाई
उसकी चौडाई से 2 से भी अचधक औय वगष की बुज ा की आधी है। वगष औय आमत के
ऺेत्रपर भें क्मा अॊतय है?
(A) 33 cm² (B) 24 cm² (C) 30 cm²
(D) 25 cm² (E) None of these

290. Ratio of number of men to that of female in apartment C is 4 : 7


while ratio of men in apartment C to that in apartment D is 3 : 4 and
ratio of number of females in apartment D to apartment C is 5 : 6.
Find the ratio of females to that of males in apartment D.
अऩाटषभेंट C भें ऩुरुर्षों की सॊख्मा का भदहराओॊ की सॊख्मा से अनुऩात 4 : 7 है जफकक
अऩाटषभेंट C भें ऩुरुर्षों का अऩाटषभेंट D भें ऩुरुर्षों का अनुऩात 3 : 4 है औय अऩाटषभेंट D
भें भदहराओॊ की सॊख्मा का अऩाटषभेंट C से अनुऩात 5 : 6 है। अऩाटषभेंट D भें भदहराओॊ
का ऩुरुर्षों से अनुऩात ऻात कीजजए।
(A) 35 : 32 (B) 32 : 37 (C) 32 : 35
(D) 25 : 37 (E) None of these

291. Akansha started a business with Rs. 24000 and after 6 months,
Prateek joined him by investing 83.33% more than what Akansha
invested. After some time Chirayu joined the business with Akansha
and Prateek. At the end of one year, out of a total profit of Rs. 79500,
Chirayu got Rs. 10500 as his share. Find after how much time Chirayu
joined the business from the start of the business, if Chirayu’s capital
was Rs. 42000.
आकाॊऺा ने 24000 रुऩमे के साथ एक व्मवसाम िुरू ककमा औय 6 भहीने के फाद,
आकाॊिा ने जजतना ननवे ि ककमा, उससे 83.33% अचधक ननवे ि कयके प्रतीक उसके साथ
जु ड गमा। कुछ सभम फाद चचयामु आकाॊिा औय प्रतीक के साथ व्माऩाय भें िाशभर हो
गमा। एक वर्षष के अॊत भें , 79500 के कु र राब भें से , चचयामु को रु 10500 दहस्से के
रूऩ भें शभरे । ऻात कीजजए, कक चचयामु व्मवसाम की िुरुआत से ककतने सभम फाद
व्मवसाम भें िाशभर हुआ, मदद चचयामु की ऩूॊज ी रु. 42000 है।
(A) 8 (B) 10 (C) 6
(D) 9 (E) None of these

292. Bhumika gave 36% of his monthly income to Priyanshi, 16% to


Chesta and 85% of remaining to Himanshi. If she is now left with Rs.
7200, then find the monthly income of Bhumika.
बूशभका ने अऩनी भाशसक आम का 36% वप्रमाॊिी को, 16% चेस्टा को औय िे र्ष का
85% दहभाॊिी को ददमा। अगय उसके ऩास अफ 7200 रुऩमे फचे हैं तो बूशभका की
भाशसक आम ऻात कीजजए। https://instagra
https://youtu https ://ww
be.com/chan w.fa cebook.
m.com/aashish https://
arorasocial(?)
141
nel/UCYa4_Jr
Orf8R5Kz2uO
com/a ashis utm_medium = t.me/st
ha rorasocial copy_link
tccXQ udified
(A) Rs. 100000 (B) Rs. 1000 (C) Rs. 140000
(D) Rs. 1400 (E) None of these

293. The perimeter of right angled isosceles triangle is 40.8 cm. what
is the area of the triangle?(Take √2 = 1.4)
सभकोण सभद्ववफाहु त्रत्रबुज का ऩरयभाऩ 40.8 से भी है। त्रत्रबुज का ऺेत्रपर क्मा है?(√2
= 1.4 रीजजए)
(A) 60 cm² (B) 100 cm² (C) 72 cm²
(D) 54 cm² (E) None of these

294. Speed of three bikes P, Q and R is in the ratio 6 : 9 : 5


respectively. Bike P covers 288 km in 12 hours. What is the total
distance covered by bike Q in 5 hours and the distance covered by
bike R in 4 hours?
तीन फाइक P, Q औय R की गनत क्रभि् 6 : 9 : 5 के अनुऩात भें है। फाइक P 12
घॊटे भें 288 ककभी की दयू ी तम कयती है। फाइक Q द्वाया 5 घॊटे भें तम की गई कुर
दयू ी औय फाइक R द्वाया 4 घॊटे भें तम की गई दयू ी ककतनी है?
(A) 240 km (B) 280 km (C) 120 km
(D) 120 km (E) None of these

295. The ratio between the present ages of Aakash and Badal is 8 : 5.
the average age of Aakash after 6 years and age of Badal before 3
years is equal to 3 less than present age of Aakash. Find the present
age of Badal.
आकाि औय फादर की वतषभान आमु का अनुऩात 8 : 5 है। 6 वर्षष फाद आकाि आमु
औय 3 वर्षष से ऩहरे फादर की आमु का औसत आकाि की वतषभान आमु से 3 कभ के
फयाफय है। फादर की वतषभान आमु ऻात कीजजए।
(A) 15 years (B) 10 years (C) 40 years
(D) 24 years (E) None of these

296. A sum becomes 6 times in 6 years at a certain rate of simple


interest. Find the time in which the same amount will be 11 times at
the same rate of interest.
एक याशि साधायण ब्माज की एक ननजश्चत दय से 6 वर्षों भें 6 गुना हो जाती है। वह
सभम ऻात कीजजए, जजसभें सभान याशि सभान ब्माज दय ऩय 11 गुना हो जाएगी।
(A) 12 years (B) 13 years (C) 14 years
(D) 16 years (E) None of these

297. P and Q together can do a piece of work in 24 days and R


alone can complete same work in 36 days. When P and R worked
together for first 8 days then the remaining work is completed by Q in
40 days. In how many days P will complete the whole work alone?
P औय Q शभरकय एक काभ को 24 ददनों भें कय सकते हैंhttps://instagra
औय R अकेरा उसी काभ
https://youtu https ://ww
be.com/chan w.fa cebook.
m.com/aashish https://
arorasocial(?)
142
nel/UCYa4_Jr
Orf8R5Kz2uO
com/a ashis utm_medium = t.me/st
ha rorasocial copy_link
tccXQ udified
को 36 ददनों भें ऩूया कय सकता है। जफ P औय R ऩहरे 8 ददनों के शरए एक साथ
कामष कयते हैं तो िे र्ष कामष Q द्वाया 40 ददनों भें ऩयू ा ककमा जाता है। P अके रे ऩयू े कामष
को ककतने ददनों भें ऩूया कये गा?
(A) 34 (B) 38 (C) 36
(D) 31 (E) None of these

298. Average age of 50 students is 180 cm and average age of all 30


female students is 160 cm. What is the average height of male
students?
50 छात्रों की औसत आमु 180 से भी है औय सबी 30 भदहरा छात्रों की औसत आमु
160 से भी है। ऩुरुर्ष छात्रों की औसत ऊॊचाई क्मा है?
(A) 240 (B) 210 (C) 265
(D) 214 (E) None of these

299. If the speed of stream is 4km/hr and speed of boat in still water
is 6 km/hr and the boat takes 24 hours more to go upstream than to
go downstream the same distance. Find the distance.
मदद धाया की गनत 4 ककभी/घॊटा है औय िाॊत ऩानी भें नाव की गनत 6 ककभी/घॊटा है औय
नाव को धाया के प्रनतकू र जाने भें सभान दयू ी ऩय जाने की तुरना भें 24 घॊटे अचधक
रगते हैं । दयू ी ऻात कीजजए।
(A) 60 km (B) 40 km (C) 30 km
(D) 25 km (E) None of these

300. Ratio of number of boys to girls in class 7th is 8 : 9 and the


ratio of boys and girls in class 7th to class 8th is 4 : 7 and 8 : 3.
Find the ratio of girls to boys in class 8th.
कऺा 7वीॊ भें रडकों की सॊख्मा का रडककमों से अनुऩात 8 : 9 है औय कऺा 7वीॊ से
कऺा 8वीॊ भें रडकों औय रडककमों का अनुऩात 4 : 7 औय 8 : 3 है। कऺा 8 भें
रडककमों का रडकों से अनुऩात ऻात कीजजए।
(A) 27 : 112 (B) 112 : 25 (C) 27 : 115
(D) 27 : 115 (E) None of these

https://youtu https://instagra
https ://ww
be.com/chan w.fa cebook.
m.com/aashish https://
arorasocial(?)
143
nel/UCYa4_Jr
Orf8R5Kz2uO
com/a ashis utm_medium = t.me/st
ha rorasocial copy_link
tccXQ udified
1. Ans. (C) 7. Ans. (D)
Total profit of A & B Speed of Boat in still water = 7x
= 1760 × 15/8 = 3300 Rs. km/h, Speed of stream = x km/h
Total Profit generated 72/(7x – x) + 128/(7x + x) = 7
= 3300/0.75 = 4400 Rs. 72/6x + 128/8x = 7
12/x + 16/x = 7
2. Ans. (B) 28/x = 7
Quantity of Petrol purchased x=4
= 3600/8 + 3600/10 + 3600/16 Speed of Boat = 7 × 4 = 28 km/h,
= 450 + 360 + 225 = 1035 L Speed of stream = 4 km/h
Average cost (per litre) Time required to cover 384 km in
= 3 × 3000/1035 = 10 Rs. U/S & D/S both
= 384/(28 – 4) + 384/(28 + 4)
3. Ans. (B) = 16 + 12 = 28 hours
Maximum Profit
= 8 × (2400 – 1800) = 4800 Rs. 8. Ans. (D)
Area of Field = 192/0.5 = 384 m2
4. Ans. (C) Length = 8x m, Width = 3x m
Actual Average 8x × 3x = 384
= [18 × 72.5 – (46.5 + 72.5 + 52.25) + 24x2 = 384
(52.75 + 42.5 + 88.5)]/17 x2 = 16
= (1305 – 171.25 + 183.75)/17 x=4
= 1317.5/17 = 77.5 Breadth of field = 3 × 4 = 12 m

5. Ans. (D) 9. Ans. (C)


Rahull  18 4 Number of ways of making group
} 72 { = 7C5 × 3C 2 = 21 × 3 = 63
Mohit  24 3
Efficiency of Amir = 4 + 3 10. Ans. (B)
Time taken by Amir to complete the Length of Bridge = 2000 m
work = 72/7 = 10(2/7) Hours Length of Train = 1000 m
Speed of Train
6. Ans. (B) = (2000 + 1000)/3 × 60
Interest received = x Rs. = 50/3 m/s = 50/3 × 18/5 = 60 km/h
(x + 360) × 0.14 × 5 = x
0.7x + 360 × 0.7 = x 11. Ans. (C)
0.3x = 252 x = 840 Rs. Investment of Arnav
= 6000 × 2Thttps://instagra
+ 3500 × (12 – 2T)
https://youtu https ://ww
be.com/chan w.fa cebook.
m.com/aashish https://
arorasocial(?)
144
nel/UCYa4_Jr
Orf8R5Kz2uO
com/a ashis utm_medium = t.me/st
ha rorasocial copy_link
tccXQ udified
= 5000T + 42000 Rs. Difference, Petrol – Water
Investment of Anish =9–7=2L
= 7000 × (12 – T) Rs.
(5000T + 42000)/7000(12 – T) = 1/1 15. Ans. (D)
5T + 42 = 7 × 12 – 7T Area of Rhombus
12T = 84 – 42 = 42 = 16 × 12/2 = 96 cm2
T = 3.5 Area of Square
= 96 + 100 = 196 cm2
12. Ans. (B) Side of Square = 14 cm
Actual Efficiency = 7,
Increased Efficiency = 7 × 10/7 = 10 16. Ans. (C)
Decreased Efficiency = 7 × 5/7 = 5 Total income of Rajeev = x Rs.
x × 5 = (x – 14) × 10 By Rajeev, Amount given to Jayati =
x = 2x – 28 x/3, Amount given to Jyona = x/6
x = 28 Total amount to Jyona
With original efficiency, work = x/6 + x/3 × 0.42 = 11040
complete in = (28 × 5)/7 = 20 days x/6 + 7x/50 = 11040
23x/75 = 11040
13. Ans. (D) x = 36000 Rs.
Actual time required Amount given to Jayati
= (D + 26)/63 – 5/60 = (D + 26)/72 + = 36000/3 = 12000 Rs.
10/60
(D + 26)/63 – (D + 26)/72 = 15/60 17. Ans. (B)
(D + 26)/504 = 1/4 CP of item = 5500 Rs.
D + 26 = 126 MP of Item = 5500 × 13/11 = 6500
D = 100 Rs.
SP in 1st case
14. Ans. (A) = 6500 × 0.8 – 200 = 5000 Rs.
In 2nd mixture, SP in 2nd case
Petrol = 26 × 9/13 = 18 L = (6500 – 200) × 0.8 = 5040 Rs.
Water = 26 × 4/13 = 8 L Difference between profit
In Final mixture, = 5040 – 5000 = 40 Rs.
Petrol = 42 × 9/14 = 27 L
Water = 42 × 5/14 = 15 L 18. Ans. (D)
In 1st mixture, Price of Item B
Petrol = 27 – 18 = 9 L = (4 × 51 – 5 × 40 + 3 × 44)/2
Water = 15 – 8 = 7 L = (204 – 200 + 132)/2 = 136/2 = 68
https://youtu https://instagra
https ://ww
be.com/chan w.fa cebook.
m.com/aashish https://
arorasocial(?)
145
nel/UCYa4_Jr
Orf8R5Kz2uO
com/a ashis utm_medium = t.me/st
ha rorasocial copy_link
tccXQ udified
19. Ans. (B) = 850 – 800 = 50 Rs.
Equivalent cost of each shirt
= 300 + 4200/150 22. Ans. (B)
= 300 + 28 = 328 Rs. Wrong marks in English
SP of shirt = 500 × 0.8 = 400 Rs. = 6 × 150 × 43/60 – 5 × 115
P% = (400 – 328) × 100/328 = 21.95% = 645 – 575 = 70
 Quantity 1 Actual Marks : Wrong Marks
= 120 : 70 = 12 : 7
Ratio of Income, Rajat : Komal :
Akash = 6 : 9 : 10 23. Ans. (C)
% of monthly income of Komal with In 300 L, Milk = 0.7 × 300 = 120 L,
monthly income of Akash Water = 90 L
= 9 × 100/10 = 90%  Quantity 2 After adding W L water,
Quantity 1 < Quantity 2 % of Water = 30 + 20 = 50%
(300 + W) × 0.5 = 90 + W
20. Ans. (A) 150 + 0.5W = 90 + 0.5W
Profit Ratio, Akram : Ben : Chavi 0.5W = 60
= (25 × 6) : (36 × 5) : (45 × 4) W = 120 L
=5:6:6 In W = 120 L mixture, Milk = 60 L,
Total Profit = 1980 × 17/6 = 5610 Water = 60 L
Rs. Quantity 1 After adding W/3 = 40 L Milk in W L
new mixture
CI = 27360 × [(13/12)2 – 1] Milk : Water = (60 + 40) : 60
= 27360 × (169 – 144)/144 = 100 : 60 = 5 : 3
= 190 × 25 = 4750 Rs. Quantity 2
Quantity 1 > Quantity 2 24. Ans. (C)
r = (5400 × 100)/(12000 × 3) = 15%
21. Ans. (D) CI = 12000 × (1.152 – 1)
CP = x Rs., MP = (x + 200) Rs. = 12000 × 0.3225 = 3870 Rs.
SP = 0.85(x + 200) CI will be less than SI by
Discount = (5400 – 3780) × 100/5400 = 28.33%
= 0.15(x + 200) = 0.15x + 30 Rs.
1.26x – (0.15x + 30) = x + 58 25. Ans. (E)
1.26x – 0.15x – x = 58 + 30 Present age,
0.11x = 88 Raj = 11x year, Dimple = 7x year
x = 800 Present Age, Chandan + Sonu
Actual Profit = 0.85(800 + 200) – 800 = (7x + 24) + 2 × 12 = 7x + 48 year
= 0.85 × 1000 – 800 11x + 7x + (7x + 48) = 4 × (27 + 10)
https://instagra
https://youtu https ://ww
be.com/chan w.fa cebook.
m.com/aashish https://
arorasocial(?)
146
nel/UCYa4_Jr
Orf8R5Kz2uO
com/a ashis utm_medium = t.me/st
ha rorasocial copy_link
tccXQ udified
25x = 148 – 48 = 100 = 2 × (38 – 11) – 24 = 54 – 24 = 30 kg
x=4 Quantity 2, Average of P, R & S
Present Age of Dimple = 7 × 4 = 28 = (24 + 38 + 34)/3 = 96/3 = 32 kg
year, Present Age of Sonu = Data Quantity 1 < Quantity 2
Inadequate, Can’t be determined
29. Ans. (B)
26. Ans. (A) D/S speed = 64/8 = 8 km/h
Length of Train Q = 4x m U/S speed = 52/13 = 4 km/h
Length of Train P = 3x m Speed of Boat in still water
Speed = (3x + 40)/8 = (4x + 30)/10 = (8 + 4)/2 = 6 km/h
5(3x + 40) = 4(4x + 30) 10 × 8 – 4 × 6 = 80 – 24 = 56 km
15x + 200 = 16x + 120
x = 80 30. Ans. (B)
Speed = (3 × 80 + 40)/8 Time taken by A = x days, Time
= 280/3 = 35 m/s taken by B = (x + 12) days
Train P cross Train Q while running 9 × (1/x + 1/(x + 12)) = 4/5
in opposite direction in T 45(x + 12 + x) = 4x(x + 12)
= (4 × 80 + 3 × 80)/(35 + 35) 45(2x + 12) = 4x2 + 48x
= (7 × 80)/70 = 8 sec. 90x + 540 = 4x2 + 48x
4x2 – 42x – 540 = 0
27. Ans. (A) 2(2x + 15)(x – 18) = 0
Side of Square = 7x cm x = 18 days
Length of Rectangle = 9x cm,
Breadth of Rectangle = 5x cm 31. Ans. (A)
7x × 7x – 9x × 5x = 64 Total votes = x
49x2 – 45x2 = 4x2 = 64 0.85x – 150 = 0.55x + (0.55x – 900)
x2 = 16 0.85x – 150 = 1.1x – 900
x = 4 cm 0.25x = 750
Perimeter of Circle x = 3000
= 4 × (7 × 4) + 2 × (9 × 4 + 5 × 4)
= 4 × 28 + 2 × 56 32. Ans. (C)
= 112 + 112 = 224 cm Price of 1st chair = 5x Rs.
Price of 2nd chair = 7x Rs.
28. Ans. (C) (5x × 1.25)/(7x + 14) = 5/7
P : (R + S) = 24 : (R + 34) = 1 : 3 1.25x = x + 2
3 × 24 = R + 34 0.25x = 2
R = 72 – 34 = 38 kg x=8
Quantity 1, Weight of Q chair = 7 × 8 = 56 Rs.
Price of 2ndhttps://instagra
https://youtu https ://ww
be.com/chan w.fa cebook.
m.com/aashish https://
arorasocial(?)
147
nel/UCYa4_Jr
Orf8R5Kz2uO
com/a ashis utm_medium = t.me/st
ha rorasocial copy_link
tccXQ udified
33. Ans. (D) 25 × (1 + R/50) = 16/0.5
MP of Article 1 + R/50 = 32/25
= 798 × 1.25/0.7 = 1425 Rs. R/50 = 7/25
R = 14%
34. Ans. (A)
Actual MP = 100 Rs. 39. Ans. (A)
CP for trader = 87.5 Rs. Efficiency of Shyam = 1
MP by Trader Efficiency of Ram = 0.875
= 87.5 × (8/7)/(8/9) = 112.5 Rs. Efficiency of Karan
New MP is greater than Original MP = 20 × (1 + 0.875)/25
by = 112.5 – 100 = 12.5% = 4 × 1.875/5 = 1.5
Together they all complete the work
35. Ans. (D) in = 20 × 1.875/(1 + 0.875 + 1.5)
Breadth = x m, Length = 1.2x m = 37.5/3.375 = 11 (1/9) days
1.2x × x = 480
x2 = 400 40. Ans. (A)
x = 20 m Number of ways getting 5 balls
= 17C5 = 6188
36. Ans. (C) Number of ways getting 2 Red
Let x unit of 1st mixture & y unit of balls, 1 Blue ball & 2 Green balls
2nd mixture is taken then = 8C2 × 4C1 × 5C2 = 28 × 4 × 10 = 1120
Milk in final mixture Required Probability
= 0.6x + 0.3y = 0.5(x + y) = 1120/6188 = 40/221
0.6x – 0.5x = 0.5y – 0.3y
0.1x = 0.2y 41. Ans. (C)
x:y=2 :1 MP = 80 Rs., SP = 30 Rs.
CP = 30/0.75 = 40 Rs.
37. Ans. (C) At 50% profit, SP = 1.5 × 40 = 60 Rs.
Let remaining food last for x extra Fraction with MP = 60/80 = 3/4
days
150 × 30 = 150 × 25 + 30 × (5 + x) 42. Ans. (B)
450 = 375 + 15 + 3x Speed of Train
3x = 60 = 146.4/(14 – 12) = 73.2 m/s
x = 20 days Length of Train P
= 73.2 × 12 = 878.4 m
38. Ans. (D) Length of Train Q
16500 × (1 + R × 2/100) = 10560/(0.25 = 878.4/2 + 3 × 146.4
× 2) = 439.2 + 439.2 = 878.4 m
https://instagra
https://youtu https ://ww
be.com/chan w.fa cebook.
m.com/aashish https://
arorasocial(?)
148
nel/UCYa4_Jr
Orf8R5Kz2uO
com/a ashis utm_medium = t.me/st
ha rorasocial copy_link
tccXQ udified
43. Ans. (A) 47. Ans. (B)
In New mixture Profit Ratio, Ananya : Alia
Volume of Ethanol = x L = (12 × 4) : (10 × 7) 48 : 70 = 24 : 35
Volume of Another Liquid = y L Total profit
1000x + 1500y = 1350(x + y) = 2450 × 59/35 = 4130 Rs
350x = 150y
x : y = 150 : 350 = 3 : 7 48. Ans. (D)
Decrease in Height = x%
44. Ans. (B) (1 + 1/13) × (1 – x/100) = 1
Efficiency of Ramesh 14/13 × (1 – x/100) = 1
= 5/15 = 1/3 chocolate/day, 1 – x/100 = 13/14
Efficiency of Suresh 1 – 13/14 = x/100
= 0.75 × 1/3 = 1/4 chocolate/day x = 100/14 = 7.14%
In 2 days they eat
= 1/3 + 1/4 = 7/12 chocolate 49. Ans. (D)
In 16 days CP = 7x Rs., MP = 12x Rs.
= 8 × 7/12 = 4 + 2/3 chocolate SP = 3x Rs.
Remaining chocolates eaten by Discount%
Ramesh on 17th days = (12x – 3x) × 10/12x = 75%
So Chocolates will be finished in
= 17 days 50. Ans. (B)
Population, 3 year ago
45. Ans. (C) = 11880/0.63 = 11880/0.216 = 55000
1 + r/100 = 6591/5070 = 1.3
P = 5070/1.32 = 5070/1.69 = 3000 Rs. 51. Ans. (C)
Minors in Family = x
46. Ans. (B) (12 + x) × 11.4 = 12 × 16 + x × 9
Age of Upadesh 12 × 11.4 + 11.4x = 12 × 16 + 9x
8 year Ago = 7x year 11.4x – 9x = 12 × 16 – 12 × 11.4
At Present = 9x year 2.4x = 12 × 4.6
9x – 7x = 8 0.2x = 4.6
2x = 8 x = 23
x=4
Age, 4 year ago 52. Ans. (C)
= 9 × 4 – 4 = 36 – 4 = 32 year 6864/(1 + 0.14 × 4) + 9078/(1 + 0.13 ×
125% of age 4 year ago will be after 6) = 6864/1.56 + 9078/1.78
= 1.25 × 32 – 36 = 40 – 36 = 4 year = 4400 + 5100 = 9500 Rs.
https://youtu https://instagra
https ://ww
be.com/chan w.fa cebook.
m.com/aashish https://
arorasocial(?)
149
nel/UCYa4_Jr
Orf8R5Kz2uO
com/a ashis utm_medium = t.me/st
ha rorasocial copy_link
tccXQ udified
53. Ans. (B) Speed of Boat in U/S = 3x km/h
36 × (75 × 2/3 – 50 × 1/5) Speed of Boat = 3x + x = 4x km/h,
= 36 × (50 – 10) = 36 × 40 = 1440 Rs. D/S speed = 4x + x = 5x km/h
5x – 4x = 6
54. Ans. (A) x = 6 km/h
Maximum Marks Time required by boat to cover 288
= 16/(0.84 – 0.76) = 16/0.08 = 200 km in D/S with increased speed of
% of Richa = (160 × 100)/200 = 80% Boat = 288/[(4 × 6 + 6) + 6]
= 288/36 = 8 hour
55. Ans. (C)
Present Age of Birbal = x year 59. Ans. (B)
Present age of Akbar = 0.8x year A 4 3
8 year later, 0.8x + 8 = 6/7 × (x + 8) B  6 } 12 { 2
7(0.8x + 8) = 6(x + 8) L 4 3
5.6x + 56 = 6x + 48 A & B fill the tank with leakage in
6x – 5.6x = 56 – 48 = 12/[(3 + 2) – 3]= 6 hour
0.4x = 8
x = 20 year 60. Ans. (C)
Total Share = 7x
56. Ans. (C) 4x × 0.42 – 3x × 0.14 = 324
Profit Ratio, Deepti : Stuti : Shruti 1.68x – 0.42x = 324
= (x × 3) : (y × 4) : (z × 2) 1.26x = 324
= 30 : 35 : 35 = 6 : 7 : 7 7x = 1800 Rs.
Investment ratio, Deepti : Stuti :
Shruti = x : y : z = 16 : 7 : 14 61. Ans. (B)
Difference between Investment of Length = x m,
Shruti & Stuti Breadth = (x – 4) m
= 37000 × (14 – 7)/37 = 7000 Rs. Height = 4 m
2 × [x × (x – 4) + x × 4 + (x – 4) × 4]=
57. Ans. (A) 248
5 × 30 + x × 50 = (5 + x) × 42 x2 – 4x + 4x + 4x – 16 = 124
5 × 30 + 50x = 5 × 42 + 42x x2 + 4x – 140
50x – 42x = 5 × 42 – 5 × 30 (x – 10)(x + 14) = 0
8x = 12 × 5 x = 10 m
x = 7.5 L Length = 10 m, Breadth = 6 m,
Height = 4 m
58. Ans. (A) Sum of all edges
Speed of Stream = x km/h = 4 × (10 + 6https://instagra
+ 4) = 4 × 20 = 80 m
https://youtu https ://ww
be.com/chan w.fa cebook.
m.com/aashish https://
arorasocial(?)
150
nel/UCYa4_Jr
Orf8R5Kz2uO
com/a ashis utm_medium = t.me/st
ha rorasocial copy_link
tccXQ udified
62. Ans. (B) = 2 × 5 × 1000 = 10000 Rs.
CP of A = x Rs.
CP of B = 2 × 380 – x = 760 – x Rs. 66. Ans. (A)
1.15x + 1.19(760 – x) = 881.6 P+Q 8 3
1.15x + 904.4 – 1.19x = 881.6 Q+R  12 } 24 { 2
0.04x = 22.8 P + Q + R 6 4
x = 570 Rs. Efficiency of P & R
=2×4– 3–2 =8 –5=3
63. Ans. (A) P & R can complete the work in
Quantity of mixture = x L, = 24/3 = 8 Days
Initially, Oil = 0.7x L, Water = 0.3x L
0.3x = 0.7x – 15 67. Ans. (A)
0.4x = 15 % of Failed Students
x = 37.5 L = 100 – (10 + 28 + 14) = 48%
Initial Quantity of oil Students who passed with
= 0.7 × 37.5 = 26.25 L Distinction = (0.1 × 96)/0.48 = 20

64. Ans. (C) 68. Ans. (A)


Investment of Anil Length of Trains = 6x m & 11x m
= 1400 × 3 + 2000 × 4 Speed of train
= 4200 + 8000 = 12200 Rs. = (6x + 11x)/(2 × 34) = 0.25x m/s
Investment of Mukesh 11x = (0.15x – 10) × 55
= 2000 × 3 + 1600 × 4 x = 0.75x – 50
= 6000 + 6400 = 12400 Rs. 0.25x = 50
Profit Ratio, x = 200
Anil : Mukesh = 122 : 124 = 61 : 62 Length of longer train
Difference between profits = 11 × 200 = 2200 m
= 6150 × (62 – 61)/123 = 50 Rs.
69. Ans. (A)
65. Ans. (C) (X – 1000) × 0.1 × 12 + (3X + 1000) ×
Salary of Raj = 5x Rs. 0.15 × 16 = 43200
Simran = 2x Rs. 1.2(X – 1000) + 2.4(3X + 1000) =
(5x – 1000)/(2x – 1000) = 4/1 43200
5x – 1000 = 4(2x – 1000) 1.2X – 1200 + 7.2X + 2400 = 43200
5x – 1000 = 8x – 4000 8.4X = 43200 – 1200
3x = 3000 8.4X = 42000
x = 1000 X = 5000 Rs.
Double of Raj’s Salary https://instagra
https://youtu https ://ww
be.com/chan w.fa cebook.
m.com/aashish https://
arorasocial(?)
151
nel/UCYa4_Jr
Orf8R5Kz2uO
com/a ashis utm_medium = t.me/st
ha rorasocial copy_link
tccXQ udified
70. Ans. (C) year
Age of Mother = x year Eldest daughter = 15 + 5 = 20 year
Father’s age = x + 16 Father = 20 × 9/5 = 36 year
22 × 8 – 8 × 6 = x + (x + 16) 7 year later,
176 – 48 = 2x + 16 Age of Father : Age of Youngest
2x = 128 – 16 = 112 Daughter
x = 56 years = (36 + 7) : (10 + 7) = 43 : 17

71. Ans. (B) 75. Ans. (C)


Investment of Pankaj In 2015, Let Cost of AC = 9x Rs.
= (32500 + 18500) × 12 = 51000 × 12 Cost of Mobile = (45000 – 9x) Rs.
Investment of Sahil = 22500 × 12 Total cost of both item in 2016
Investment of Shubham = 35500 × 4 = 1.16 × 9x + 13/9 × (45000 – 9x)
Profit Ratio, = 10.44x + (65000 – 13x)
Pankaj : Sahil : Shubham = 65000 – 2.56x = Quantity 1
= (510 × 12) : (225 × 12) : (355 × 4) Quantity 2 = 70000
= 306 : 135 : 71 Quantity 1 < Quantity 2

72. Ans. (C) 76. Ans. (C)


Investment made by Raju Let Total Capacity of Tank
= 3562.5/[0.28 × 2 – (1.152 – 1)] = LCM (15, 18, 36) = 180 L
= 3562.5/(0.56 – 0.3225) Efficiency of,
= 3562.5/0.2375 = 15000 Rs. P = 12 L/minute
Q = 10 L/minute
73. Ans. (A) R = 5 L/minute
Number of ways of selecting 3 card After 12 minute, Quantity of Soda in
= 52C 3 = (52 × 51 × 50)/3! Tank = 12 × 5 = 60 L = Quantity 1
Number of ways of selecting 3 After 10 minute, Quantity of
black card = 26C3 = (26 × 25 × 24)/3! RoohAfza
Probability of selecting 3 black card = 10 × 12 = 120 L = Quantity 2
= (26 × 25 × 24)/(52 × 51 × 50) Quantity 1 < Quantity 2
= 6/51 = Quantity 1
4/51 = Quantity 2 77. Ans. (C)
Quantity 1 > Quantity 2 2 × (7x + 8x) = 660
15x = 330
74. Ans. (A) x = 22 cm
At present, Age of, P = ((7 × 22) × (8 × 22))/154 = 176
Youngest daughter = 15 – 5 = 10 https://instagra
https://youtu https ://ww
be.com/chan w.fa cebook.
m.com/aashish https://
arorasocial(?)
152
nel/UCYa4_Jr
Orf8R5Kz2uO
com/a ashis utm_medium = t.me/st
ha rorasocial copy_link
tccXQ udified
78. Ans. (A) 10500/9300 = 35/31
CP of Article = 36x Rs. 21 × (8 – x) = 35x
MP of Article = 21x Rs. 35x + 21x = 21 × 8
SP of Article = 14x Rs. 56x = 21 × 8
Loss = 36x – 14x = 1320 x = 3 month
22x = 1320
x = 60 Rs. 82. Ans. (C)
Discount = 21x – 14x = 7x Ratio of Present Age of,
= 7 × 60 = 420 Rs. A : B : C = (3 × 37 – 56) : (68 + 56 – 3
× 37) : (3 × 37 – 68) = (111 – 56) :
79. Ans. (B) (124 – 111) : (111 – 68) = 55 : 13 : 43
Let, x kg of 25 Rs./kg rice mix with
y kg of 32 Rs./kg rice. 83. Ans. (C)
(25x + 32y) × 10/9 = 30(x + y) Monthly income of Ram
250x + 320y = 270x + 270y = 3900/(0.75 × (1 – 0.4 – 0.34))
20x = 50y = 5200/0.26 = 20000 Rs.
x : y = 50 : 20 = 5 : 2 Monthly income of Nobita
= 1.25 × 20000 = 25000 Rs.
80. Ans. (D)
U/S Speed = x km/h, 84. Ans. (B)
D/S Speed = y km/h Q get = 10000 Rs.
45/x + 35/y = 14, 15/x + 14/y = 5 P get = 10000 – 160 = 9840 Rs.
(45/x + 35/y) – 3 × (15/x + 14/y) = 14 R & S get = 37840 – (10000 + 9840)
–3 ×5 = 37840 – 19840 = 18000 Rs.
35/y – 42/y = 14 – 15 S get = 18000 × 7/15 = 8400 Rs.
– 7/y = – 1 Average Share of P & S
y = 7 km/h = (9840 + 8400)/2 = 9120 Rs.
15/x = 5 – 14/7 = 5 – 2 = 3
x = 5 km/h 85. Ans. (A)
Speed of Boat in Still Water P  27 4
= (7 + 5)/2 = 6 km/h } 108 {
Time taken by Boat to cover 186 km Q  36 3
in Still water = 186/6 = 31 hours Pipe Q should be closed after
= (108 – 18 × 4)/3
81. Ans. (B) = 36 – 24 = 12 minute
Let Shyam invest for = x month,
Ram invest for = (8 – x) month 86. Ans. (A)
(2100 × (8 – x))/(3100 × x) = U/S speed = 126/31.5 = 4 km/h
https://instagra
https://youtu https ://ww
be.com/chan w.fa cebook.
m.com/aashish https://
arorasocial(?)
153
nel/UCYa4_Jr
Orf8R5Kz2uO
com/a ashis utm_medium = t.me/st
ha rorasocial copy_link
tccXQ udified
Speed of Boat in Still water Distance between P & Q
= 4 + 2 = 6 km/h = 8 × 57 = 456 km
D/S speed = 6 + 2 = 8 km/h
Time required taken to cover 192 91. Ans. (C)
km in D/S & 48 km in U/S is For Boat P, U/S Speed = 4x km/h,
= 192/8 + 48/4 = 24 + 12 = 36 hours D/S Speed = 7y km/h
For Boat Q, U/S Speed = 5x km/h,
87. Ans. (A) D/S Speed = 8y km/h
CP = 28 × 3 + 36 × 5 = 84 + 180 = Speed of Stream = 7y – 4x = 8y – 5x
264 Rs. 5x – 4x = 8y – 7y
SP = (0.125 × 35 + 0.875 × 40) × 8 x=y
= (4.375 + 35) × 8 In Still water
= 39.375 × 8 = 315 Rs. Speed of Boat P = (7x + 4x)/2 = 5.5x
P% = (315 – 264) × 100/264 = 19.31% Speed of Boat Q = (8x + 5x)/2 = 6.5x
In Still water, Speed of Boat Q is
88. Ans. (C) more than that of Boat P by
Rate of interest half yearly = 9% = (6.5x – 5.5x) × 100/5.5x = 18.18%
Difference in Half yearly & Annual
CI = 12000 × (1.092 – 1.18) 92. Ans. (C)
= 12000 × (1.1881 – 1.18) Present age of, Son = 2x year,
= 12000 × 0.0081 = 97.2 Rs. Mother = 7x year,
Father = 3 × (2x + 8) – 8 = 6x + 16
89. Ans. (B) year
P  40 9 7x + (6x + 16) = 81
} 360 { 13x = 65
Q  45 8 x=5
Time required to fill 85% tank by Present age of Son
both pipes = (0.85 × 360)/(9 + 8) = 2x = 2 × 5 = 10 years
= 0.05 × 360 = 18 Days
93. Ans. (D)
90. Ans. (B) Sachine  12 4
Time Taken by P = x hour, Time } 48 {
taken by Q = (x – 5) hour Amit  16 3
57 × x = 152 × (x – 5) Efficiency of Bhole
57x = 152x – 152 × 5 = (48 – 3 × 4 – 2 × 3)/10
95x = 152 × 5 = (48 – 12 – 6)/10 = 30/10 = 3,
19x = 152 In 1 day part of work done by Bhole
x = 8 hour alone = 3/48https://instagra
= 1/16
https://youtu https ://ww
be.com/chan w.fa cebook.
m.com/aashish https://
arorasocial(?)
154
nel/UCYa4_Jr
Orf8R5Kz2uO
com/a ashis utm_medium = t.me/st
ha rorasocial copy_link
tccXQ udified
94. Ans. (A)
3A × (7/6)2 – A × 1.252 = 6050 98. Ans. (D)
3A × 49/36 – 1.5625A = 6050 Speed of Train = 140/4 = 35 m/s
49A – 18.75A = 12 × 6050 Speed of Car = 0.5 × 35 = 17.5 m/s
30.25A = 12 × 6050 Length of Bridge
A = 12 × 200 = 2400 Rs. = 17.5 × 10 = 175 m
Train cross the Bridge in
95. Ans. (A) = (175 + 140)/35 = 9 seconds
P = 8x, Q = 11x, R = 7x, S = 12x
12x – 7x = 70 99. Ans. (D)
5x = 70 P  12 5
x = 14 Q  15 } 60 { 4
Average of P & R R  20 3
= (8 × 14 + 7 × 14)/2 = 15 × 7 = 105 60 = (x – 6) × 5 + (x – 4) × 4 + x × 3
60 = 5x – 30 + 4x – 16 + 3x
96. Ans. (A) 12x = 60 + 46 = 106
Let C invest for = x month Work will complete in x = 8.83 days
B invest for = 2x month
A invest for = (2x + 8) month 100. Ans. (C)
Total Investment New Average = 35 + (42 – 45)/10
= 7 × (2x + 8) + 11 × 2x + 18 × x = 35 – 0.3 = 34.7 years
= 14x + 56 + 22x + 18x = 54x + 56
Investment of C = 18x 101. Ans. (B)
18x/(54x + 56) = 54/218 Speed of Stream = 16x km/h
218x = 3(54x + 56) Speed of Boat = 19 km/h
218x – 162x = 168x 222/(19x – 16x) + 245/(19x + 16x) =
56x = 168 81
X = 3 month 222/3x + 245/35x = 81
74/x + 7/x = 81
97. Ans. (C) 81/x = 81
CP of TV for Raju = x Rs. x=1
CP of TV for Shyam = x + 5000 Rs. Speed of Stream
x + (x + 5000) = 14000 × 8/7 = 16 × 1 = 16 km/hour
2x = 16000 – 5000
2x = 11000 102. Ans. (C)
x = 5500 If Efficiency of Bhanupriya = 16
Shyam buy TV at cost of then Efficiency of Amitav = 21
= 5500 + 5000 = 10500 Rs. = 21 × 32
Total Work https://instagra
https://youtu https ://ww
be.com/chan w.fa cebook.
m.com/aashish https://
arorasocial(?)
155
nel/UCYa4_Jr
Orf8R5Kz2uO
com/a ashis utm_medium = t.me/st
ha rorasocial copy_link
tccXQ udified
Work done by both in 21 days Area = 902 = 8100 cm2
= 21 × (16 + 21) = 21 × 37
% of work done by both in 21 days 107. Ans. (C)
= (37 × 100)/32 = 115.625 % Monthly Salary of Mahesh
= 62208/(12 × (0.25 – 0.19))
103. Ans. (D) = 5184/0.06 = 86400 Rs.
(x + 15) × 31 = x × 35 + 165
31x + 465 = 35x + 165 108. Ans. (B)
4x = 300 Money borrowed
x = 75 = 7874/((1 + 0.2 × 5) – 1.32)
= 7874/(2 – 1.69) = 7874/0.31
104. Ans. (A) = 25400 Rs.
Initially, Milk = x ml
Water = (365 – x) ml 109. Ans. (C)
(x + 25) = (365 – x + 35) × 5/12 Profit Ratio, P : Q : R
12 × (x + 25) = 5 × (400 – x) = (91 × 3) : (104 × 2) : (56 × 5)
12x + 300 = 2000 – 5x = 273 : 208 : 280
17x = 1700 Average of Profit of P & R
x = 100 ml = 19025 × ((273 + 280)/2)/761
= 25 × 276.5 = 6912.5 Rs.
105. Ans. (A)
0.85A = A – 225 110. Ans. (A)
0.15A = 225 3 year ago, Age of,
A = 1500 Rs. Manish = 7x year
Discount% on selling at 900 Rs. Soumya = 5x year
= (1500 – 900) × 100/1500 = 40% Avantika = 9x year
(7x + 5)/(5x + 5) = 13/10
106. Ans. (A) 10(7x + 5) = 13(5x + 5)
Side of square P = 7x cm 70x + 50 = 65x + 65
Side of Square Q = 8x cm 5x = 15
(7x)2 + (8x)2 = 16272 x=3
49x2 + 64x2 = 16272 sum of Present age of, Soumya &
113x2 = 16272 Avantika = (5 × 3 + 3) + (9 × 3 + 3)
x2 = 144 = 18 + 30 = 48 year
x = 12
Side of Square, whose side is equal 111. Ans. (B)
to average of sides of square P & Q Let He took t hour to reach office at
= (7x + 8x)/2 = 7.5 × 12 = 90 cm 70 km/h speed,
https://instagra
https://youtu https ://ww
be.com/chan w.fa cebook.
m.com/aashish https://
arorasocial(?)
156
nel/UCYa4_Jr
Orf8R5Kz2uO
com/a ashis utm_medium = t.me/st
ha rorasocial copy_link
tccXQ udified
70 × t = 60 × (t + 0.75) Manish = (7x + 4) × 6/5 + 2
7t = 6t + 4.5 = 8.4x + 6.8 year
t = 4.5 7x + 4x + (8.4x + 6.8) = 3 × 649/21
To reach the office at 8 : 30 AM (in 19.4x + 6.8 = 649/7
4.5 – 1.5 = 3 hour), his speed 19.4x × 7 = 649 – 6.8 × 7
should be = (70 × 4.5)/3 = 105 km/h 135.8x = 649 – 47.6
135.8x = 601.4
112. Ans. (C) x = 31/7
1 + r/100 = 20345/15650 Present age of Manish
r = (1.3 – 1) × 100 = 30% = 8.4 × 31/7 + 6.8 = 1.2 × 31 + 6.8
= 37.2 + 6.8 = 44 years
113. Ans. (C)
A + B  15 6 117. Ans. (B)
B + C  18 } 90 { 5 (15 × 48 + 36 × x) × 1.15 = (15 + 36) ×
C  30 3 69
Efficiency of A = 6 – 5 + 3 = 4 720 + 36x = 60 × 51
Part of Work done by A in 15 hour 36x = 3060 – 720 = 2340
= (15 × 4)/90 = 2/3 x = 65 Rs./kg
Cost of 2nd type of wheat is more
114. Ans. (A) than 1st type of wheat by
Profit Ratio, Ramesh : Suresh = 65 – 48 = 17 Rs.
= 31500 : 37500 = 21 : 25
X × 0.7 = 2100 × 46/25 118. Ans. (B)
X = 120 × 46 = 5520 Rs. 8 × x + 36 × 16 = 12 × (x + 16)
8x + 576 = 12x + 192
115. Ans. (A) 4x = 384
CP for P = 100 Rs. Number of Students = x = 96
R bought is from Q in
= 100 × 7/12 × 9/7 = 75 Rs. 119. Ans. (C)
R bought it directly from P in Average speed for Journey
= 100 × 19/12 = 475/3 Rs. = (2 × 36 × 48)/(36 + 48)
Now, price of mobile for R will be = (2 × 3 × 48)/7 = 288/7 = km/hour
more than actual price for R by
= (475/3 – 75) × 100/75 = 111.11% 120. Ans. (D)
30000 × 0.25 × 2 × 0.01r + 30000 ×
116. Ans. (A) 0.75 × 2 × 0.01(r + 4) = 10800
Present age of, Mohan = 7x year, 0.25r + 0.75(r + 4) = 18
Sunil = 4x year, r + 3(r + 4) = 72
https://instagra
https://youtu https ://ww
be.com/chan w.fa cebook.
m.com/aashish https://
arorasocial(?)
157
nel/UCYa4_Jr
Orf8R5Kz2uO
com/a ashis utm_medium = t.me/st
ha rorasocial copy_link
tccXQ udified
r + 3r + 12 = 72 Investment of Suman = 1400 × 12 +
4r = 60 600 × 6 = 16800 + 3600 = 20400 Rs.
r = 15% Investment of Shashi = 2000 × 12 –
800 × 6 = 24000 – 4800 = 19200 Rs.
121. Ans. (A) Investment of Rajesh
Let SP of both article = x Rs. = 4800 × 3 = 14400 Rs.
x × 16/21 – x × 12/19 = 208 Profit Ratio,
x × (16 × 19 – 12 × 21) = 208 × 21 × Suman : Shashi : Rajesh
19 = 204 : 192 : 144 = 17 : 16 : 12
x × (304 – 252) = 208 × 399 Profit of Shashi
x × 52 = 208 × 399 = 56160 × 1000/16.5 = 19968 Rs.
x = 4 × 399
x = 1596 Rs. 126. Ans. (A)
Let, Total distance = 1000 km
122. Ans. (D) Average speed of Cars during
Radius of 3rd Sphere journey
= 3√(123 – 63 – 83) = 1000/(300/50 + 600/75 + 100/40)
= 3√(1728 – 216 – 512) = 1000/(6 + 8 + 2.5)
= 3√1000 = 10 cm = 1000/16.5 = 60 (20/33) km/h

123. Ans. (C) 127. Ans. (C)


Time taken by Piyush alone to Present age of,
complete the work = 6/0.2 = 30 Age of Ramesh = 9x year,
days, Age of Suresh = 11x year
Efficiency of Sonu = 15 16 year later,
Efficiency of Piyush = 19x (9x + 16)/(11x + 16) = 17/19
Together they complete the work in 19 × (9x + 16) = 17 × (11x + 16)
= (30 × 19)/(15 + 19) 171x + 304 = 187x + 272
= (30 × 19)/34 = (15 × 19)/17 16x = 32
= 285/17 = 16 (13/17) days x=2
Present age of Kirti
124. Ans. (A) = (11 × 2) × 13/11 = 2 × 13 = 26 year
Probability that food has been
eaten 128. Ans. (B)
= 2/3 × 1/4 + 1/3 × 3/4 + 2/3 + 3/4 Speed of Boat in still water
= 1/6 + 1/4 + 1/2 = 11/12 = 35 km/h
Speed of Stream = 35/5 = 7 km/h
125. Ans. (C) U/S speed = 35 – 7 = 28 km/h
https://instagra
https://youtu https ://ww
be.com/chan w.fa cebook.
m.com/aashish https://
arorasocial(?)
158
nel/UCYa4_Jr
Orf8R5Kz2uO
com/a ashis utm_medium = t.me/st
ha rorasocial copy_link
tccXQ udified
D/S speed = 35 + 7 = 42 km/h 8 = 12 – P
A/28 + A/42 = 12.5 P = 12 – 8 = 4 year
5A/84 = 12.5
A = 2.5 × 84 = 210 km 133. Ans. (B)
Average = x Rs.
129. Ans. (B) P get = 1.25x
Ratio of weight, Q get = 2x – 1.25x = 0.75x
P : Q : R = 63 : 99 : 56 1.25x – 0.75x = 3900
Weight of P 0.5x = 3900
= 155 × 63/(99 + 56) = 63 kg x = 7800
Value of Sum = 2 × 7800 = 15600 Rs.
130. Ans. (D)
Let x kg of Wheat should be added 134. Ans. (C)
90 × 7/15 + x = (90 + x) × 0.625 Maximum Marks = x
6 × 7 + x = 90 × 0.625 + 0.625x Passing Marks
42 + x = 56.25 + 0.625x = 0.28x + 18 = 0.38x – 22
0.375x = 14.25 0.1x = 40
x = 38 kg x = 400
Maximum Marks – Passing Marks
131. Ans. (C) = 400 – (0.28 × 400 + 18)
Present age of Shilpa = 400 – 130 = 270
= 2 × (38 – 3) – 48 = 2 × 35 – 48
= 70 – 48 = 22 year 135. Ans. (A)
Present age of Rani = 2 × 33 – 22 Let x kg of Wheat & y kg of Rice is
= 66 – 22 = 44 year mixed together, then
Present age of Kusum 57x + 64y) × 17/12 = 85(x + y)
= 2 × (37.5 + 3) – 44 57x + 64y = 60(x + y)
= 2 × 40.5 – 44 = 81 – 44 = 37 years 3x = 4y
x:y=4 :3
132. Ans. (D)
Investment of A = 13500 × 12 Rs. 136. Ans. (C)
Investment of B Speed of Stream = 15x km/h
= 16000 × (12 – P) Rs. Speed of Boat in Still water = 19x
(13500 × 12)/(16000 × (12 – P)) = km/h
810/(1450 – 810) D/S Speed = 19x + 15x = 34x km/h
(135 × 12)/(160 × (12 – P)) = 81/64 U/S Speed = 19x – 15x = 4x km/h
64 × 135 × 12 = 81 × 160 × (12 – P) 408/34x + 16/4x = 8
2 × 5 × 12 = 3 × 5 × (12 – P) 12/x + 4/x =https://instagra
16/x = 8
https://youtu https ://ww
be.com/chan w.fa cebook.
m.com/aashish https://
arorasocial(?)
159
nel/UCYa4_Jr
Orf8R5Kz2uO
com/a ashis utm_medium = t.me/st
ha rorasocial copy_link
tccXQ udified
x=2
D/S Speed = 34 × 2 = 68 km/h 141. Ans. (A)
U/S speed = 4 × 2 = 8 km/h Efficiency of Q = 7
Time taken by Boat to cover 816 km Efficiency of P = 9
in D/S & 64 km in U/S = 816/68 + Efficiency of R = x
64/8 = 12 + 8 = 20 hour 4 × 9 × 7 = 14 × (7 + 9 + x)
9 × 2 = 16 + x
137. Ans. (B) x = 18 – 16
Radius of Sphere = x cm x=2
Height of Cylinder = x cm R alone complete the work in
Radius of Cylinder = 4x cm = (4 × 9 × 7)/2 = 126 days
4/3 × π × x3 = 38808
π × x3 = 29106 142. Ans. (D)
Volume of Cylinder Profit Ratio, Niyog : Ankit : Amit
= π × (4x)2 × x = 16 × π × x3 = (11 × 7) : (4 × 9 + 3 × 18) : (13 × 7)
= 16 × 29106 = 465696 cm3 = 77 : 90 : 91
Difference between profit of Niyog
138. Ans. (C) & Amit = 1890 × (91 – 77)/90
P + Q + R 30 6 = 21 × 14 = 294 Rs.
Q+R  45 } 180 { 4
R  60 3 143. Ans. (B)
Efficiency of P & Q together CP of Car = 7168/(1.1225 – 0.5625)
=6–3=3 = 7168/0.65 = 12800 Rs.
Part of work done by P & Q in 20 Actual SP
days = (20 × 3)/180 = 60/180 = 1/3 = 0.5625 × 12800 = 7200 Rs.

139. Ans. (A) 144. Ans. (A)


Radius of formed cylinder Let, Expenditure of Manoj = 30 Rs.
= 44/(2 × 22/7) = 7 m Income of Manoj = 30 × 7/6 = 35 Rs.
Volume of Cylinder = 22/7 × 72 × 12 Ratio of Saving
= 22 × 84 = 1848 m3 Manoj : Dinesh : Sachin
= (35 – 30) : (25 – 24) : (30 – 18)
140. Ans. (D) = 5 : 1 : 12
2 × 700 × 1.45 = 700 × 2 + 5A + 7A
700 × 2.9 – 700 × 2 = 12A 145. Ans. (B)
12A = 700 × 0.9 Total Boys = 1350 × 8/15 = 720,
12A = 630 Girls = 1350 – 720 = 630
A = 52.5 Passed Boys = 720 × 7/12 = 420
https://instagra
https://youtu https ://ww
be.com/chan w.fa cebook.
m.com/aashish https://
arorasocial(?)
160
nel/UCYa4_Jr
Orf8R5Kz2uO
com/a ashis utm_medium = t.me/st
ha rorasocial copy_link
tccXQ udified
Passed Girls = 630 – 130 = 500 149. Ans. (C)
Passed boys are less than passed Speed of Stream = 15x km/h
girls by Speed of Boat in still water = 17x
= (500 – 420) × 100/500 = 16% km/h
384/(17x + 15x) + 12/(17x – 15x) = 18
146. Ans. (D) 384/32x + 12/2x = 18
Actual speed = 15x km/minute, 12/x + 6/x = 18/x = 18
Reduced speed = 8x km/h x=1
D/8x – D/15x = 28 Speed of Stream = 15 × 1 = 15 km/h,
7D/120x = 28 Speed of Boat in still water = 17 × 1
D/x = 120 × 4 = 480 minute = 17 km/h
Normal time taken Time taken to cover 768 km in D/S
= D/15x = 480/15 = 32 minute & 72 km in U/S
= 768/(17 + 15) + 72/(17 – 15)
147. Ans. (C) = 768/32 + 72/2 = 24 + 36 = 60 hour
Initially in mixture P,
Milk = x L, Water = (96 – x) L 150. Ans. (C)
[(96 – x) + 15] × 1.05 = x + 12 Radius of Cylinder = x m
(111 – x) × 1.05 = x + 12 22/7 × x2 × 14 = 7436
116.55 – 1.05x = x + 12 44 × x2 = 7436
2.05x = 104.55 x2 = 169
x = 51 L x = 13 m = Radius of cylinder =
Initially, Milk : Water Height of Cone,
= 51 : (96 – 51) = 51 : 45 = 17 : 15 Radius of Cone = 14/2 = 7 m
Volume of Cone
148. Ans. (B) = 1/3 × 22/7 × 72 × 13 = 667.33 m3
Present age of,
A = 7x – 5 year, B = 3x year, 151. Ans. (C)
(7x – 5) – 3x = 3 Skirts sold at 28% profit = x
4x = 8 Skirt sold at 32% profit = (320 – x)
x=2 x × 28 + (320 – x) × 32 = 320 × 30.5
Present age of A 7x + 2560 – 8x = 2440
= 7 × 2 – 5 = 14 – 5 = 9 year x = 120 = Skirt sold at 28% profit
Present age of C Skirt sold at 32% profit
= (9 – 4) × 4/5 + 2 = 4 + 2 = 6 year = 320 – 120 = 200
Present Age of A + Present Age of Difference = 200 – 120 = 80
C = 9 + 6 = 15 year
https://youtu https://instagra
https ://ww
be.com/chan w.fa cebook.
m.com/aashish https://
arorasocial(?)
161
nel/UCYa4_Jr
Orf8R5Kz2uO
com/a ashis utm_medium = t.me/st
ha rorasocial copy_link
tccXQ udified
152. Ans. (A) Uneducated Male
Profit Ratio, = 68000 × 0.3 × 18/30 = 12240
Akasa : Pratik : Nishant : Vishal Uneducated Female
= 144 : 360 : 660 : 715 = 68000 × 0.3 × 5/30 = 3400
Difference of Profit share of Akasa Educated Male + Educated Female
& Nishant = 14400 × (660 – 144)/360 = (9 × 3400 – 12240) + (7 × 3400 –
= 40 × 516 = 20640 Rs. 3400)
= (30600 – 12240) + (23800 – 3400)
153. Ans. (C) = 18360 + 20400 = 38760
Principal = 1260/(3 × 0.3 – 1.32 + 1)
= 1260/(0.9 – 1.69 + 1) 158. Ans. (A)
= 1260/0.21 = 6000 Rs. P + Q  10 5
} 50 {
154. Ans. (D) P  12.5 4
Marks of Chetan = x Efficiency of Q = 5 – 4 = 1
Marks of Asif = (63 – x) 50 = 4 × 5 + 1 × x
3 × 72 – 3 × 53 = x – (63 – x) x = 50 – 20
3 × 19 = 2x – 63 Q worked for = x = 30 days
2x = 57 + 63
2x = 120 159. Ans. (C)
x = 60 Initial quantity of mixture = 16x L
Ethanol = 7x L, Water = 9x L
155. Ans. (A) (7x – 40.4)/9x = 3/5
Speed of motorboat in still water = 5 × (7x – 40.4) = 3 × 9x
x km/h 35x – 202 = 27x
5 × (x + 2) – 6 × (x – 2) = 12 8x = 202
5x + 10 – 6x + 12 = 12 x = 25.25
x = 10 km/h Initial quantity of water
= 9 × 25.25 = 227.25 L
156. Ans. (B)
Surface area of Tank = 302 – 202 160. Ans. (A)
= 900 – 400 = 500 cm2 Present age of, Radha = 5x year,
Bhumika = 6x year
157. Ans. (D) Bhavishya = 30 year
Male + Female Raj = 6x – 4 year
= 9x + 7x = 0.8 × 68000 6 year later
16x = 54400 (5x + 6) + (6x – 4 + 6) = (6x + 6) + (30
x = 3400 + 6) – 4 https://instagra
https://youtu https ://ww
be.com/chan w.fa cebook.
m.com/aashish https://
arorasocial(?)
162
nel/UCYa4_Jr
Orf8R5Kz2uO
com/a ashis utm_medium = t.me/st
ha rorasocial copy_link
tccXQ udified
5x + 6x – 4 = 6x + 30 – 4 7b = 3.5 × 5
5x = 30 b = 0.5 × 5 = 2.5
x=6 Champa alone complete the work in
Present age of, Radha + Raj = (10 × (1 + 2.5))/3.5
= 5 × 6 + (6 × 6 – 4) = (10 × 3.5)/3.5 = 10 days
= 30 + 32 = 62 years
165. Ans. (A)
161. Ans. (D) 8 year ago, Age of Amit = 7x year
Investment of Rahul = (x + 1000) × 7 Age of Ashish = 8x year
– 900 × 4 = 7x + 3400 (7x + 8) + 9 = (8x + 8) + 1
Investment of Pankaj = (x + 500) × 7 x=8
– 700 × 4 = 7x + 700 Age of Ashish after 5 year
(7x + 3400)/(7x + 700) = 8/7 = (8 × 8 + 8) + 5 = 64 + 13 = 77 year
(7x + 3400) = 8(x + 100)
7x + 3400 = 8x + 800 166. Ans. (B)
x = 2600 Length of Train = 12 × 10.5 + 20 + 11
Initial investment of Rahul × 2 + 2 = 126 + 20 + 22 + 2 = 170 m
= (x + 1000) Time taken by Train to cross the
= (2600 + 1000) = 3600 Rs. platform = (170 + 170)/34 = 10 sec

162. Ans. (B) 167. Ans. (B)


(12 × 0.01 × R)/(1.5 × (1.32 – 1)) = CP of A = x Rs.
50/69 CP of B = (2800 – x) Rs.
(0.08 × R)/(1.69 – 1) = 50/69 x × 0.11 + (2800 – x) × 0.17 = 3164 –
(0.08 × R)/0.69 = 50/69 2800
8 × R = 50 0.11x + 476 – 0.17x = 364
R = 6.25% 0.06x = 112
x = 1866.66 Rs.
163. Ans. (C) Difference between CP
Initial quantity of Sugar in Bag = 1866.66 – (2800 – 1866.66)
= (0.3 × 70)/0.4 = 52.5 kg = 1866.66 – 933.33 = 933.33 Rs.

164. Ans. (D) 168. Ans. (D)


Efficiency of Arundhati = 1 25 × (M + 15) – 30 × M = 75
Efficiency of Champa = 3.5 25M + 375 – 30M = 75
Efficiency of Billu = b 5M = 300
b = (b + 3.5) × 5/12 M = 60
12b = 5b + 3.5 × 5 https://instagra
https://youtu https ://ww
be.com/chan w.fa cebook.
m.com/aashish https://
arorasocial(?)
163
nel/UCYa4_Jr
Orf8R5Kz2uO
com/a ashis utm_medium = t.me/st
ha rorasocial copy_link
tccXQ udified
169. Ans. (A) (6 × 1.6w + 4w) × 15 = 34 × xw
Upstream Speed = x km/h (9.6 + 4) × 15 = 34x
Downstream Speed = y km/h 13.6 × 15 = 34x
21/x + 51/y = 12, 70/x + 17/y = 22 x = 0.4 × 15
(21/x + 51/y) – 3 × (70/x + 17/y) = 12 34 women complete the work in
– 3 × 22 = x = 6 days
21/x – 210/x = 12 – 66
– 189/x = – 54 174. Ans. (A)
x = 3.5 km/h Present age of
17/y = 22 – 70/3.5 = 22 – 20 = 2 R = 32 year, Q = 28 year
y = 8.5 km/h P = 28 × 3/4 = 21 year
Speed of Stream (21 + z)/(28 + z) = 26/33
= (8.5 – 3.5)/2 = 2.5 km/hour 33(21 + z) = 26(28 + z)
693 + 33z = 728 + 26z
170. Ans. (C) 7z = 35
A = B × 19/18, B = C × 7/16 z=5
A : B : C = 133 : 126 : 288
A is less than C by 175. Ans. (A)
= (288 – 133) × 100/288 = 53.81% Investment of Rahul = 2400 × 3 +
2800 × 4 + 2400 × 5 = 7200 + 11200 +
171. Ans. (B) 12000 = 30400 Rs.
Breadth of Field = x m Investment of Anjali = 3200 × 3 +
2 × (21 + x) × 3.8 = 220.4 2800 × 4 + 3200 × 5 = 9600 + 11200 +
21 + x = 29 16000 = 36800 Rs.
x=8m Profit Ratio, Rahul : Anjali
Area of field = 21 × 8 = 168 m2 = 30400 : 36800 = 19 : 23

172. Ans. (C) 176. Ans. (B)


24 × 30 = 24 × 8 + 28 × 9 + x × 13 Speed of train = 21.6 × 5/18 = 6 m/s
13x = 720 – 192 – 252 Length of Train = 15 × 6 = 90 m
13x = 276 Time taken by Train to cross the
Average Lipsticks sold in remaining platform = (90 + 54)/6 = 24 sec
days = x = 21.2
177. Ans. (D)
173. Ans. (C) Total Distance
15m = 24w = (6.4 – 0.8) × 3 + (6.4 + 0.8) × 2
M = 1.6w = 5.6 × 3 + 7.2 × 2
(6m + 4w) × 15 = 34 × xw = 16.8 + 14.4 = 31.2 km
https://youtu https://instagra
https ://ww
be.com/chan w.fa cebook.
m.com/aashish https://
arorasocial(?)
164
nel/UCYa4_Jr
Orf8R5Kz2uO
com/a ashis utm_medium = t.me/st
ha rorasocial copy_link
tccXQ udified
178. Ans. (A) 185. Ans. (C)
Y = 320 × (0.45 – 0.2) Change in Revenue
= 320 × 0.25 = 80 = (19/15 × 13/16 – 1) × 100
= ( 247/240 – 1) × 100
179. Ans. (C) = 700/240 = 35/12 = 2.91%
Principal = 420/(1.12 – 1.2)
= 420/(1.21 – 1.2) 186. Ans. (D)
= 420/0.01 = 42000 Rs. Efficiency of Geetika = x
Efficiency of Chesta = 1.25x
180. Ans. (D) Efficiency of Ashish
Remaining Ethanol in mixture = 1.25x × 0.8 = x
= 0.7 × 134 – 36 = 93.8 – 36 = 57.8 L Ashish alone complete the work in
= 12 × (x + 1.25x)/x
181. Ans. (D) = 12 × 2.25 = 27 days
Amount Given by Avni
= 190.4/((1 + 0.15 × 3) – 1.13) 187. Ans. (B)
= 190.4/(1.45 – 1.331) Quantity required to be added
= 190.4/0.119 = 1600 Rs. = 45 × (5 – 4)/9 + (3 – 2) = 5 + 1 = 6 L

182. Ans. (B) 188. Ans. (A)


Profit Ratio, Mahira : Paras (22/7 × r2)/(2 × (22/7 × r + 2 × r) ) =
= 7560 : 9240 = 9 : 11, 77/12
Profit of Mahira 12 × 22 × r = 2 × 77 × (22 + 2 × 7)
= 3421 × 9/11 = 2799 Rs. 6 × 2 × r = 7 × (22 + 2 × 7)
12 × r = 7 × 36
183. Ans. (C) Radius of Semicircle
7 × (1 + y/100) + 9 × 0.8 = (7 + 9) × = r = 7 × 3 = 21 unit
1.36
7 × (1 + y/100) = 16 × 1.36 – 7.2 189. Ans. (A)
7 × (1 + y/100) = 21.76 – 7.2 = 14.56 Time for which bike travel
(1 + y/100) = 2.08 = (480 + 60)/36 = 15 hour
y = 1.08 × 100 = 108 36 × 15 – Y × 15 = 60
36 – Y = 4
184. Ans. (A) Y = 32 km/h
Absent Employees in 1st week :
Absent Employees in 2nd week 190. Ans. (A)
= 4/13 : 1/7 = (4 × 7) : 13 = 28 : 13 At present, Age of A = 2x + 4 year,
Age of B = https://instagra
3x + 1,
https://youtu https ://ww
be.com/chan w.fa cebook.
m.com/aashish https://
arorasocial(?)
165
nel/UCYa4_Jr
Orf8R5Kz2uO
com/a ashis utm_medium = t.me/st
ha rorasocial copy_link
tccXQ udified
Age of C = 3x – 5 year = 85 + 25 – 100 = 10%
(2x + 4) + (3x + 1) + (3x – 5) = 3 × (29 People who applied for both post
+ 1/3) = 640 × 0.1 = 64
8x = 87 + 1 = 88
X = 11 195. Ans. (C)
Present Age of B + Present Age of 3 year ago, Age of Rohit = 11x year
C = (3 × 11 + 1) + (3 × 11 – 5) Age of Simran = 6x year
= 34 + 28 = 62 year 6 year later, (11x + 9)/(6x + 9) = 14/9
9 × (11x + 9) = 14 × (6x + 9)
191. Ans. (A) 99x + 81 = 84x + 126
(X + 5)(X + 4) = (X + 30) × 12 15x = 45
X2 + 9X + 20 = 12X + 360 x=3
X2 – 3X – 340 = 0 5 year ago
(X + 17)(X – 20) = 0 Age of Ayesha : Age of Rohit
X = 20 = (11 × 3 + 3 – 6 – 5) : (11 × 3 + 3 – 5)
= 25 : 31
192. Ans. (C)
Rate = r, Principal = 100r 196. Ans. (D)
100r × (1 + 2 × r/100) = 4800 8 Average weight = x kg
100r + 2r2 = 4800 46 × x = 30 × 2 + 44 × x
r2 + 50r – 2400 = 0 2 × x = 30 × 2
(r – 30)(r + 80) = 0 x = 30 kg
r = 30
2r = 60% 197. Ans. (B)
{y × 360 + (y – 4) × 450} × 1.4 = (2y –
193. Ans. (C) 4) × 558
U/S speed = 48/6 = 8 km/h (360y + 450y – 1800) × 1.4 = 1116y –
Speed of Boat in Still water 2232
= 1.2 × 8 = 9.6 km/h 1134y – 2520 = 1116y – 2232
Speed of Stream 18y = 288
= 9.6 – 8 = 1.6 km/h y = 16
D/S speed = 9.6 + 1.6 = 11.2 km/h
Time taken by Boat to cover 28 km 198. Ans. (C)
in D/S = 28/11.2 = 2.5 hour Length of Field = 11x m
Breadth of Field = 9x m
194. Ans. (A) 2 × (11x + 9x) = 72 × 5/18 × 6
% of People who applied for both 20x = 20 × 3
post = (100 – 15) + (100 – 75) – 100 x=3 https://instagra
https://youtu https ://ww
be.com/chan w.fa cebook.
m.com/aashish https://
arorasocial(?)
166
nel/UCYa4_Jr
Orf8R5Kz2uO
com/a ashis utm_medium = t.me/st
ha rorasocial copy_link
tccXQ udified
Area of Field = (11 × 3) × (9 × 3) Efficiency of Ankush = 10
= 33 × 27 = 891 m2 Efficiency of Shrey = 6
Time taken by Ankush to complete
199. Ans. (D) the work
Profit Ratio = (6 × 6)/(0.3 × 10) = 12 days
Aayush : Badal : Chahal
= (3 × 4) : (9 × 6) : (7 × 1) 204. Ans. (C)
= 12 : 54 : 7 Probability that Truth has been told
Difference between profit of Badal is = 1/3 × 2/5 + 2/3 × 3/5 + 1/3 × 3/5
& Chahal = 2628 × (54 – 7)/73 = 2/15 + 2/5 + 1/5 = 11/15
= 36 × 47 = 1692 Rs.
205. Ans. (D)
200. Ans. (A) Investment of Priya
1 = (1 + 7/15)(1 – y/100) = 2000 × 12 – 400 × 9
15/22 = 1 – y/100 = 24000 – 3600 = 20400 Rs.
y/100 = 1 – 15/22 = 7/22 Investment of Chetanya
y = 700/22 = 350/11 = 31.81% = 1400 × 12 + 400 × 9
= 16800 + 3600 = 20400 Rs.
201. Ans. (B) Investment of Pravesh
CP of Shirt P = x Rs. = 2400 × 6 = 14400 Rs.
CP of Shirt Q = (2300 – x) Rs. Profit Ratio,
x × 1.56 = (2300 – x) × 1.2 Priya : Chetanya : Pravesh
1.3x = 2300 – x = 204 : 204 : 144 = 17 : 17 : 12
2.3x = 2300 Difference between Profit of Priya &
x = 1000 = CP of Shirt P, Pravesh
CP of Shirt Q = 2300 – 1000 = 1300 = 10350 × (17 – 12)/46 = 1125 Rs.
% of CP of Q with CP of P
= (1300 × 100)/1000 = 130% 206. Ans. (D)
P  20 3
202. Ans. (D) Q  12 } 60 { 5
Radius of 3rd Sphere R  30 2
= 3√(183 – 23 – 163) Time taken by P & Q
= 3√(5832 – 8 – 4096) = 3√1728 = 12 = 60/(3 + 5) = 7.5 days
Radius of 3rd sphere is less than Time taken by P & R
Radius of Original sphere by = 60/(3 + 2) = 12 days
= (18 – 12) × 100/18 = 33.33% % of time taken by P & Q with time
taken by P & R
203. Ans. (A) = 7.5 × 100/12 = 62.5%
https://youtu https://instagra
https ://ww
be.com/chan w.fa cebook.
m.com/aashish https://
arorasocial(?)
167
nel/UCYa4_Jr
Orf8R5Kz2uO
com/a ashis utm_medium = t.me/st
ha rorasocial copy_link
tccXQ udified
207. Ans. (A) Side of Rhombus
CP = 15 × 36/15 + 20 × 20/1.25 + 10 × = √((48/2)2 + (14/2)2) = √(242 + 72)
22/1.1 = 15 × 24 + 20 × 16 + 10 × 20 = √(576 + 49) = √625 = 25 m
= 360 + 320 + 200 = 880 Rs. Perimeter of Rhombus
Profit = 15 × 24 × 0.5 + 20 × 16 × = 4 × 25 = 100 m
0.25 + 10 × 20 × 0.1 = 180 + 80 + 20
= 280 Rs. 212. Ans. (C)
P% = 280 × 100/880 = 31.81% 5p = 8q
3(p + q) = 78
208. Ans. (B) 8q/5 + q = 13q/5 = 26
Let Total Distance q = 10, p = 16
= LCM (25, 32, 21) = 16800 km Time taken by Q to complete the
Average speed work = 78/10 = 7.8 days
= 16800/(4200/25 + 6720/32 +
5880/21) = 16800/(168 + 210 + 280) 213. Ans. (D)
= 16800/658 = 1200/47 = km/h Number of Apartment in complex D
= 360 × (1 – 7/12 – 1/4) × 0.40
209. Ans. (B) = 144 × 1/6 = 24
At present, Age of Love = 5x year,
Age of Kush = 6x year 214. Ans. (B)
4 year later, (5x + 4)/(5x + 4) = 29/34 Principal = 4326/(1.22 – 1 – 0.1 × 3)
17 × (5x + 4) = 29 × (3x + 2) = 4326/(1.44 – 1 – 0.3)
85x + 68 = 87x + 58 = 4326/0.14 = 30900 Rs.
2x = 10
x=5 215. Ans. (C)
Present age of Lokit Initially,
= 6 × 5 – 6 = 30 – 6 = 24 year Milk = 180 ml, Water = 60 ml
1st replacement, Milk : Water
210. Ans. (A) = (180 – 60) : (60 – 20 + 80)
Speed of Stream = 20/4 = 5 km/h = 120 : 120 = 1 : 1
D/(20 + 5) + D/(20 – 5) = 16 2nd replacement, Milk : Water
D/25 + D/15 = 16 = (120 – 40) : (120 – 40 + 80)
8D/75 = 16 = 80 : 160 = 1 : 2
D = 75 × 2 = 150 km
216. Ans. (D)
211. Ans. (A) Investment of A
Other diagonal of Rhombus = 24000 × 12 = 288000 Rs.
= (336 × 2)/14 = 48 m Investmenthttps://instagra
of B
https://youtu https ://ww
be.com/chan w.fa cebook.
m.com/aashish https://
arorasocial(?)
168
nel/UCYa4_Jr
Orf8R5Kz2uO
com/a ashis utm_medium = t.me/st
ha rorasocial copy_link
tccXQ udified
= 12000 × 4 + 12000 × 4 = 96000 Rs. = (3 × 39 – 72)/2 = 22.5
Investment of C
= 36000 × 6 = 216000 Rs. 221. Ans. (D)
Profit Ratio Investment of P
A : B : C = 288 : 96 : 216 = 12 : 4 : 9 = 2700 × 8 + 3000 × 4 + 1800 × 4
Difference between profit of A & B = 21600 + 12000 + 7200 = 40800 Rs.
= 67500 × (12 – 4)/25 = 21600 Rs. Investment of Q
= 1550 × 8 + 1900 × 4 + 1500 × 4
217. Ans. (A) = 12400 + 7600 + 6000 = 26000 Rs.
x + (x – 4) = 36 × 75/60 Investment of R
2x – 4 = 36 × 1.25 = 3500 × 8 + 4500 × 4 + 2850 × 4
2x = 45 + 4 = 49 = 28000 + 18000 + 11400 = 57400 Rs
x = 24.5 Profit Ratio, P : Q : R
= 408 : 260 : 574 = 204 : 130 : 287
218. Ans. (B) Average Profit
Let, CP for X = 12 Rs. = 20400 × 621/204 × 1/3 = 20700 Rs.
CP for Z when he purchased it from
Y = 12 × 5/12 × 1.6 = 8 Rs. 222. Ans. (A)
If Z bought it directly from X, then Time taken by Pankaj = 36 days
CP for Z = 12 × 4/3 = 16 Rs. Time taken by Salim = 27 days
Z would have bought it more than Time taken by Amit = 44 days
the price at which he actually Pankaj  36 33
bought it = (16 – 8) × 100/8 = 100% Salim  27 } 1188 { 44
Amit  44 27
219. Ans. (C) Fraction of remaining work
(40 × 36 + 20 × x) × 1.7 = 68 × (40 + = 1 – 10 × (33 + 44 + 27)/1188
20) = 1 – 10 × 104/1188
144 + 2x = 40 × 6 = 1 – 260/297 = 37/297
2x = 240 – 144
2x = 96 223. Ans. (D)
x = 48 Rs./kg Let Quantity of 2st mixture = 14x kg,
Salt = 3x kg, Sugar = 11x kg
220. Ans. (C) 20 + 3x = 12 + 11x
D – A = 3 × 54 – 3 × 39 8x = 8
1.625A – A = 3 × 15 x=1
0.625A = 15 Quantity of 2nd mixture
A = 72 = 14 × 1 = 14 kg
Average of B & C https://instagra
https://youtu https ://ww
be.com/chan w.fa cebook.
m.com/aashish https://
arorasocial(?)
169
nel/UCYa4_Jr
Orf8R5Kz2uO
com/a ashis utm_medium = t.me/st
ha rorasocial copy_link
tccXQ udified
224. Ans. (B) Ratio of Salary,, A : B : C
(5 × x)/(8 × (x + 6)) = 1/4 = 28500 : (28500 + 17500) : (28500 –
5x = 2(x + 6) 8500) = 285 : 460 : 200 = 57 : 92 : 40
5x = 2x + 12
3x = 12 229. Ans. (A)
x=4 Side of Square = √3136 = 56 m
Rate of Interest offered in Bank Q For Rectangle,
= 4 + 6 = 10% Length = 56 × 6/7 = 48 m
Breadth = 48 × 2/3 = 32 m
225. Ans. (D) Perimeter of Rectangle
P = 1100 × (x + 7)/100 – 1100 × x/100 = 2 × (48 + 32) = 2 × 80 = 160 m
P = 11 × (x + 7) – 11x
P = 11 × 7 = 77 230. Ans. (C)
Speed of Car = 68 km/h,
226. Ans. (A) Speed of Bike = 68 × 12/17 = 48
Speed of Boat = 46 km/h km/h
Speed of Stream = 46/1.4375 = 32 x/48 – x/68 = 2.5
km/h 5x/816 = 2.5
A/(46 + 32) + (A – 12)/(46 – 32) = 78 x = 408 km
A/78 + (A – 12)/14 = 78
A/78 + A/14 – 6/7 = 78 231. Ans. (A)
23A/273 = 552/7 Speed of Train P = 12x m/s
A = 936 Speed of Train Q = 19x m/s
1488 = 24 × (12x + 19x)
227. Ans. (D) 31x = 62
Age of A, After 12 years x=2
= (76 + 10 × 3) – (54 – 4 × 2) + 12 Speed of Train P = 12 × 2 = 24 m/s
= (76 + 30) – (54 – 8) + 12
= 106 – 46 + 12 = 72 232. Ans. (B)
Average age of Class including
228. Ans. (B) teachers = (219 + 87 + 55)/19
Salary of A = x Rs. = 361/19 = 19 year
Salary of B = x + 17500 Rs.
Salary of C = x – 8500 233. Ans. (B)
x + 17500 = 2.3(x – 8500) 3 year ago,
x + 17500 = 2.3x – 19550 Age of Ram = x year,
1.3x = 37050 Age of Father = 4x year
x = 28500 5 year later,https://instagra
Age of Sister
https://youtu https ://ww
be.com/chan w.fa cebook.
m.com/aashish https://
arorasocial(?)
170
nel/UCYa4_Jr
Orf8R5Kz2uO
com/a ashis utm_medium = t.me/st
ha rorasocial copy_link
tccXQ udified
= (x + 3) + 8 = x + 11 year 237. Ans. (D)
Age of Father = 4x + 8 year Length of Rectangle = 12x m,
(x + 11) × 1.6 = 4x + 8 Breadth of Rectangle = 7x m
1.6x + 17.6 = 4x + 8 (12x × 17/12) × (7x × 9/7) – 12x × 7x
2.4x = 9.6 = 276
x=4 17x × 9x – 12x × 7x = 276
Present age of Ram = 4 + 3 = 7 year 153x2 – 84x2 = 276
69x2 = 276
234. Ans. (D) x2 = 4
CP of Articles = 72 × 18 = 1296 Rs. x=2
SP of Articles = 46 × 24 + 26 × 10 Perimeter of Rectangle = 2 × (12x +
= 1104 + 260 = 1364 Rs. 7x) = 2 × 19 × 2 = 76 m
P%
= (1364 – 1296) × 100/1296 = 5.2% 238. Ans. (A)
P  3A 7/3
235. Ans. (A) } 7A {
CP of Item = 15x Rs. Q  7A/3 3
MP of Item = 23x Rs. 0.4 × 7A = 42 × (7/3 + 3)
SP of Item = 23x – 465.2 Rs. 2.8A = 42 × 16/3 = 224
15x × 38 = (23x – 465.2) × 30 A = 80
19x = 23x – 465.2
4x = 465.2 239. Ans. (D)
x = 116.3 Let sum invested is = x Rs.
MP of Item = 23 × 116.3 = 2674.9 Rs. x × (7/6)2 – x × (1 + 3 × 0.12) = 108
x × 49/36 – 1.36x = 108
236. Ans. (B) x/900 = 108
Investment of Sunil x = 108 × 900 = 97200 Rs.
= 25750 × 12 – 1250 × 7 = 300250
Investment of Anil 240. Ans. (A)
= 17525 × 12 – 1625 × 7 = 198925 Speed of Stream = 15x km/h,
Investment of Pammi Speed of Boat in still water = 17x
= 19725 × 12 – 1375 × 7 = 227075 km/h
Profit Ratio, Sunil : Anil : Pammi 384/(17x + 15x) + 32/(17x – 15x) = 28
= 300250 : 198925 : 227075 384/32x + 32x/2x = 28
= 12010 : 7957 : 9083 12/x + 16/x = 28
Total Profit 28/x = 28
= 7957 × 29050/7957 = 29050 Rs. x=1
Speed of Stream = 15 × 1 = 15 km/h
https://instagra
https://youtu https ://ww
be.com/chan w.fa cebook.
m.com/aashish https://
arorasocial(?)
171
nel/UCYa4_Jr
Orf8R5Kz2uO
com/a ashis utm_medium = t.me/st
ha rorasocial copy_link
tccXQ udified
Speed of Boat in still water (P + 12500) × 1.152 = 2P + 6030
= 17 × 1 = 17 km/h (P + 12500) × 232 = (2P + 6030) × 202
Time taken to covering 1032 km in 529P + 6612500 = 800P = 2412000
D/S & 9 km in U/S 271P = 4200500
= 1032/(17 + 15) + 96/(17 – 15) P = 15500 Rs.
= 1032/32 + 96/2
= 32.25 + 48 = 80.25 hours 245. Ans. (D)
P  45 4
241. Ans. (A) Q  30 } 180 { 6
CP of Motorcycle = x Rs. R  60 3
(x + 18500) × 1.33 = 85652 Wage of R = 2275 × 3/13 = 525 Rs.
x + 18500 = 64400
x = 45900 Rs. 246. Ans. (B)
Cost of Rice = x Rs.
242. Ans. (C) Cost of Wheat = y Rs.
7 year later, (1 × x)/(1 × x + 7 × y) = 11/32
Age of Salim = 7x years 32x = 11(x + 7y)
Age of Pankaj = 5x years 32x – 11x = 77y
6 year ago, 21x = 77y
(5x – 13)/(7x – 13) = 6/11 x/y = 77/21 = 11/3
11(5x – 13) = 6(7x – 13) x : y = 11 : 3
55x – 143 = 42x – 78
13x = 65 247. Ans. (D)
x=5 Profit Ratio, Manish : Chunnu
8 year later, = (25 × 8) : (16 × 11) = 25 : 22
Age of Salim = 7 × 5 + 1 = 36 year Profit Share of Chunnu
= 4440 × 22/3 = 32560 Rs.
243. Ans. (B)
Radius of Circle = r 248. Ans. (B)
22/7 × r2 = 1386 Students in School R
r2 = 441 = 169 × 3 – 124 × 2 = 507 – 248 = 259
r = 21 m Students in School S
Length of Rectangle = x m = 179 × 2 – 259 = 358 – 259 = 99
2 × (x + 20) = 4 × 21
x + 20 = 42 249. Ans. (C)
x = 22 m For Product, MP = 100 Rs.
SP = 0.87 × 100 = 87 Rs.
244. Ans. (A) CP = 87/1.27 = 8700/127 Rs.
https://instagra
https://youtu https ://ww
be.com/chan w.fa cebook.
m.com/aashish https://
arorasocial(?)
172
nel/UCYa4_Jr
Orf8R5Kz2uO
com/a ashis utm_medium = t.me/st
ha rorasocial copy_link
tccXQ udified
New CP Actual time taken
= 1.1 × 8700/1.27 = 9570/127 Rs. = A/68 – 20/60 = A/102 + 12/60
P% A/68 – A/102 = 1/5 + 1/3
= (87 – 9570/127) × 100/(9570/127) A/204 = 8/15
= 15.45% A = 108.8 km

250. Ans. (C) 254. Ans. (B)


Time taken by Boat P + Time taken In 1st mixture,
by Boat Q Milk = x L, Water = (42 – x) L
= 231/(19 – 5) + 507/(21 + 5) In 2nd mixture,
= 16.5 + 19.5 = 36 hours Milk = 56 × 17/28 = 34 L,
Water = 56 – 34 = 22 L
51. Ans. (B) (x + 34)/(42 – x + 22) = 27/22
Investment of Akshay 22(x + 34) = 27(64 – x)
= 12500 × 2P + 10000 × (20 – 2P) 22x + 748 = 1728 – 27x
= 5000(40 + P) Rs. 49x = 980
Investment of Prashant x = 29 L
= 7500(20 – P) Rs. In 1st mixture
5000(40 + P)/7500(20 – P) = 46/21 Milk = 20 L, Water = 42 – 20 = 22 L
2(40 + P) × 21 = 3(20 – P) × 46 Difference = 22 – 20 = 2 L
7(40 + P) = 23(20 – P)
280 + 7P = 460 – 23P 255. Ans. (D)
30P = 180 Other diagonal of Rhombus
P=6 = 2 × √(132 – 52) = 2 × 12 = 24 cm
Area of Square
252. Ans. (C) = 24 × 10/2 + 24 = 144 cm
Actual Efficiency = 15 Side of Square = √144 = 12 cm
Increased Efficiency = 22
Decreased Efficiency = 13 256. Ans. (A)
Total work = 13 × x = 22 × (x – 18) Total money that Amit had = x Rs.
22x – 13x = 22 × 18 Prerna receives
9x = 22 × 18 = 0.12x + 0.28x × 0.2 = 2728
x = 44 days 0.12x + 0.056x = 2728
Time taken to complete the work 0.176x = 2728
with original efficiency x = 15500 Rs.
= 44 × 13/15 = 38 2/15 days Money given by Amit to Soniya
= 0.28 × 15500 = 4340 Rs.
253. Ans. (C) https://instagra
https://youtu https ://ww
be.com/chan w.fa cebook.
m.com/aashish https://
arorasocial(?)
173
nel/UCYa4_Jr
Orf8R5Kz2uO
com/a ashis utm_medium = t.me/st
ha rorasocial copy_link
tccXQ udified
257. Ans. (C)
CP = 10500 Rs. 263. Ans. (D)
MP = 10500 × 23/15 = 16100 Rs. Speed of Train
1st SP = (16100 – 880) × 0.8 = 12176 = P × 5/18 = 5P/18 m/s
Rs. (5P/18) × 24 = (5P/18 – 5) × 27
2nd SP = 16100 × 0.8 – 880 = 12000 4P/3 = 3P/2 – 27
Rs. 3P/2 – 4P/3 = 27
Difference in Profit P/6 = 27
= 12176 – 12000 = 176 Rs. P = 162

258. Ans. (B) 264. Ans. (A)


Age of D = 4 × 52 + 3 × 42 – 5 × 48 Investment of,
= 208 + 126 – 240 = 94 year A = x Rs., B = y Rs., C = z Rs.
7x = 4y = 5z
259. Ans. (A) Profit Ratio, A : B : C = 20 : 35 : 28
Profit Ratio, P : Q : R Total profit
= (17500 × 4) : (21400 × 5) : (12300 × = 10192 × 83/28 = 30212 Rs.
8) = 350 : 535 : 492
Total Profit 265. Ans. (C)
= 6888 × 1377/492 = 19278 Rs. Apples that S get
= 660 × 8/15 × 0.25 × 3/11 = 24
260. Ans. (D)
P × (1.153 – 1) = 4167 266. Ans. (A)
P × (233 – 203)/203 = 4167 Sumit Purchased laptop at
P × (12167 – 8000)/8000 = 4167 = 1995/[(19/12) × 0.7]= 1800 Rs.
P × 4167/8000 = 4167
P = 8000 Rs. 267. Ans. (C)
SI = 8000 × 5 × 0.185 = 7400 Rs. In 12 L mixture A,
Milk = 7 L, Water = 5 L
261. Ans. (C) In 12 + 48 = 60 L mixture of A & B,
Price of replaced article Milk = 60 × 1/6 = 10 L
= 1751 + 17 × 4 = 1819 Rs. In Mixture B, % of Milk
= (10 – 7) × 100/48 = 6.25%
262. Ans. (B)
Ratio of present age, 268. Ans. (B)
Rohit : Kanika : Monu = 15 : 6 : 35 Speed of Stream = 12x km/h,
Present age of Kanika Speed of Boat in Still water = 17x
= 168 × 6/56 = 18 years km/h https://instagra
https://youtu https ://ww
be.com/chan w.fa cebook.
m.com/aashish https://
arorasocial(?)
174
nel/UCYa4_Jr
Orf8R5Kz2uO
com/a ashis utm_medium = t.me/st
ha rorasocial copy_link
tccXQ udified
348/(17x + 12x) + 35/(17x – 12x) = 19 2A = 28000
348/29x + 35/5x = 19 A = 14000 Rs.
12/x + 7/x = 19
19/x = 19 272. Ans. (B)
x=1 Other Parallel sides of Trapezium
D/S speed = 17x + 12x = 29 × 1 = 29 = 45 × 22/15 = 66 m
km/h Height of Trapezium
U/S speed = 17x – 12x = 5 × 1 = 5 = 45 × 19/15 = 57 m
km/h Cost of ploughing
Time taken by Boat to cover 406 km = 12 × (1/2) × (45 + 66) × 57
in D/S & 62.5 km in U/S = 6 × 111 × 57 = 37962 Rs.
= 406/29 + 62.5/5
= 14 + 12.5 = 26.5 hour 273. Ans. (D)
P had = 220 apples,
269. Ans. (C) Q had = 220/(22/15) = 150 apples,
P = 2200/(1 + 3 × 0.125) R had = 150 × 26/15 = 260 apples
= 2200/1.375 = 1600 Rs. Apple not eaten yet by R
CI after 2 year = 1600 × 1.1252 = 260 × 4/13 = 80
= 1600 × 81/64 = 2025 Rs.
274. Ans. (A)
270. Ans. (D) 42 × 172 = 21 × (P + 18) + 21 × (Q –
Let in Trapezium, 12)
1st parallel side = x cm 2 × 172 = P + 18 + Q – 12
2nd parallel side = x + 3 cm P + Q + 6 = 344
Area = (x + x + 3) × 28/2 = 378 P + Q = 338
2x + 3 = 27
2x = 24 275. Ans. (B)
x = 12 cm 4 year ago, Age of Nirmal = 18x year
2nd parallel side = 12 + 3 = 15 cm Age of Pooja = 23x years
4 year later, (18x + 8)/(23x + 8) = 4/5
271. Ans. (C) 5(18x + 8) = 4(23x + 8)
Investment of Munna 90x + 40 = 92x + 32
= 2 × A + 18000 = 2A + 18000 Rs. 2x = 8
Investment of Munni x=4
= 52000 × 2 – 12000 = 92000 Rs. At present,
Profit Ratio, Munna : Munni Age of Nirmal = 18 × 4 + 4 = 76 year
= (2A + 18000) : 92000 = 1 : 2 Age of Pooja = 23 × 4 + 4 = 96 year
2A + 18000 = 46000 Age of Laado = (76 + 96)/2 = 86 year
https://instagra
https://youtu https ://ww
be.com/chan w.fa cebook.
m.com/aashish https://
arorasocial(?)
175
nel/UCYa4_Jr
Orf8R5Kz2uO
com/a ashis utm_medium = t.me/st
ha rorasocial copy_link
tccXQ udified
At present, = 315/20 = 15.75 days
Age of Ladoo : Age of Pooja
= 86 : 96 = 43 : 48 280. Ans. (D)
Initially, in 75 L mixture,
276. Ans. (C) Water = 75 × 7/15 = 35 L,
Speed of Boat in Still water = x Milk = 75 – 35 = 40 L
km/h Quantity of Water after,
[45/(x – 1.3)]/[90/(x + 1.3)]= 11/9 1st replacement
(x + 1.3)/2(x – 1.3) = 11/9 = 0.8 × 35 + 15 = 43 L
9(x + 1.3) = 22(x – 1.3) 2nd replacement
9x + 11.7 = 22x – 28.6 = 0.8 × 43 + 15 = 34.4 + 15 = 49.4 L
13x = 40.3
x = 3.1 km/h 281. Ans. (C)
Time taken to cover 462 km in D/S New Average of village A = a
= 462/(3.1 + 1.3) = 462/4.4 = 105 hr 2 × 4000 + 2 × 6000 = 2 × a + 2 ×
(6000 + 450)
277. Ans. (A) 4000 + 6000 = a + 6000 + 450
CP of 1 Article = 1 Rs. a = 4000 – 450 = 3550
Loss = 75 × 1 × 7/15 = X × 1
X = 35 282. Ans. (A)
CP is between  55 Rs. – 65 Rs.
278. Ans. (D) Let CP = 10x + y Rs.
Let the sum is = x Rs. SP = (10x + y) + 27 = 10y + x
0.5x × 2 × 0.15 + 0.5x × [(17/15)2 – 9(y – x) = 27
1]= 1578 y–x=3
0.15x + 0.5x × (289 – 225)/225 = Possible CP = 58 Rs.,
1578 Possible SP = 85 Rs.
0.15x + 0.5x × 64/225 = 1578
0.15x + 32x/225 = 1578 283. Ans. (A)
263x/900 = 1578 x year ago, Age of Ram = 4a year
x = 5400 Rs. Age of Piya = 5a years
5a – 4a = 3
279. Ans. (B) a=3
Efficiency of Q = 12 (3x + 3) year later,
Efficiency of P = 17 Age of Ram = 9b year,
Q alone to complete 60% of work in Age of Piya = 10b year
= 0.6 × (15 × 17 + 5 × 12)/12 10b – 9b = 3
= (255 + 60)/20 b=3 https://instagra
https://youtu https ://ww
be.com/chan w.fa cebook.
m.com/aashish https://
arorasocial(?)
176
nel/UCYa4_Jr
Orf8R5Kz2uO
com/a ashis utm_medium = t.me/st
ha rorasocial copy_link
tccXQ udified
Difference of Age of Piya’s age after 288. Ans. (C)
(3x + 3) year & before x year Let the consumption increased by
x + (3x + 3) = 10 × 3 – 5 × 3 x%
4x + 3 = 5 × 3 1.26 = 1.14(1 + x/100)
4x = 15 – 3 1 + x/100 = 21/19
4x = 12 x/100 = 4/19
x=3 x = 10.52%

284. Ans. (D) 289. Ans. (A)


84/(y – 5) = 144/y Side of Square = 2x cm
7x = 12(y – 5) For Rectangle, Length = x cm,
12y – 7y = 60 Breadth = x – 2 cm
5y = 60 (4 × 2x)/2 × (x + x – 2) = 3/1
y = 12 4x = 3 × (2x – 2)
2x = 3x – 3
285. Ans. (A) x=3
12m = 14b Area of Square = (2 × 3)2 = 36 cm2
m = 7b/6 Area of Rectangle
5 × 15 × 14b = d × (3m + 4b) = 3 × (3 – 2) = 3 cm2
1050b = d × (3 × 7b/6 + 4b) Difference = 36 – 3 = 33 cm2
1050 = d × 7.5
d = 140 290. Ans. (C)
In Apartment D, Male : Female
286. Ans. (D) = (4 × 4/3) : (7 × 5/6) = 32 : 35
Profit Ratio, A : B : C : D
= 1/14 : 1/15 : 1/4 : 1/6 291. Ans. (B)
= 30 : 28 : 105 : 70 Investment of Akansha
Difference between profit of A & B = 12 × 24000 Rs.
= 58250 × (70 – 30)/233 Investment of Prateek
= 250 × 40 = 10000 Rs. = 6 × 24000 × 11/6 = 11 × 24000 Rs.
Investment of Chirayu
287. Ans. (C) = x × 42000 Rs.
Y = 880/(1 + 6 × 0.2) = 880/2.2 = 400 Profit Ratio,
Rs. Akansha : Prateek : Chirayu
X = 464/(1 + 3 × 0.15) = 464/1.45 = = (12 × 24000) : (11 × 24000) : (x ×
320 Rs. 42000) = 48 : 44 : 7x
Y – X = 400 – 320 = 80 Rs. 7x/(92 + 7x) = 10500/79500 = 7/53
53x = 92 + 7x
https://instagra
https://youtu https ://ww
be.com/chan w.fa cebook.
m.com/aashish https://
arorasocial(?)
177
nel/UCYa4_Jr
Orf8R5Kz2uO
com/a ashis utm_medium = t.me/st
ha rorasocial copy_link
tccXQ udified
46x = 92 296. Ans. (A)
x=2 1 + 6r = 6
Chirayu invest after r = 5/6
= 12 – 2 = 10 month 1 + t × 5/6 = 11
t × 5/6 = 10
292. Ans. (A) t = 12 years
Monthly Income of Bhumika
= 7200/(0.48 × 0.15) = 100000 Rs. 297. Ans. (C)
P + Q  24 3
293. Ans. (C) } 72 {
For Triangle, R  36 2
Perpendicular = Base = x Efficiency of P = x, Q = 3 – x
Hypotenuse = 1.4x 72 = 8 × (x + 2) + 40 × (3 – x)
Perimeter = x + x + 1.4x = 40.8 72 = 8x + 16 + 120 – 40x
3.4x = 40.8 32x = 64
x = 12 x=2
Area = 1/2 × 12 × 12 = 72 cm2 P Complete the work in
= 72/2 = 36 days
294. Ans. (D)
Speed of, 298. Ans. (B)
Bike P = 288/12 = 24 km/h, Average height of Male students
Bike Q = 24 × 9/6 = 36 km/h, = x cm
Bike R = 24 × 5/6 = 20 km/h 50 × 180 = 30 × 160 + 20 × x
Sum of Distances covered by Bike 2x = 900 – 480
Q & Bike R = 36 × 5 + 20 × 4 2x = 420
= 180 + 80 = 260 km x = 210 cm
295. Ans. (A) 299. Ans. (A)
Present Age of, Let, Distance = D km
Aakash = 8x year, Badal = 5x year D/(6 – 4) – D/(6 + 4) = 24
(8x + 6) + (5x – 3) = 2(8x – 3) D/2 – D/10 = 24
13x + 3 = 16x – 6 2D/5 = 24
3x = 9 D = 60 km
x=3
Present Age of Badal 300. Ans. (A)
= 5 × 3 = 15 year In Class 8th, Girls : Boys
= (9 × 3/8) : (8 × 7/4) = 27 : 112
https://youtu https://instagra
https ://ww
be.com/chan w.fa cebook.
m.com/aashish https://
arorasocial(?)
178
nel/UCYa4_Jr
Orf8R5Kz2uO
com/a ashis utm_medium = t.me/st
ha rorasocial copy_link
tccXQ udified
https://t.mhttps/t.me
e/studified/studified

https://instagram.com/aashisharora
social?utm_medium = copy_link

https://youtube.com/channel
/UCYa4_JrOrf8R5Kz2uOtccXQ

https://www.facebook.
com/aashisharorasocial

https://youtu https://instagra
https ://ww
be.com/chan w.fa cebook.
m.com/aashish https://
arorasocial(?)
179
nel/UCYa4_Jr
Orf8R5Kz2uO
com/a ashis utm_medium = t.me/st
ha rorasocial copy_link
tccXQ udified
(1 – 5) In a college A there are 1320 students, the ratio of Boys and
girls is 6 : 5 respectively. Chocolates, Toffee and Chips are distributed
among them. 15% of boys get only chocolates, 10% of boys get only
Toffee and 20% of boys get chips. 7.5% of boys get only Toffee and
chocolates. 12.5% of boys get only Toffee and chips and 30% of boys
get only chips and chocolates. Remaining boys get all three items.
22% of girls get only chocolate, 18% girls get only toffee and 12% of
girls get only chips. 10% of girls get only chocolates and toffee, 20%
of girls get only toffee and chips and 8% of girls get only chips and
chocolates. Remaining girls get all the three items.
(1 – 5) एक कॉरे ज A भें 1320 छात्र हैं , रडकों औय रडककमों का अनुऩात क्रभि् 6 :
5 है। उनभें चॉकरे ट, टॉफी औय चचप्स ववतरयत ककए जाते हैं 15% रडकों को केवर
चॉकरे ट शभरती है, 10% रडकों को केवर टॉफी शभरती है औय 20% रडकों को चचप्स
शभरती हैं । 7.5% रडकों को शसपष टॉपी औय चॉकरे ट शभरती है। 12.5% ​रडकों को
केवर टॉफी औय चचप्स शभरती हैं औय 30% रडकों को केवर चचप्स औय चॉकरे ट
शभरती हैं । िे र्ष रडकों को तीनों वस्तुएॉ शभरती हैं । 22 पीसदी रडककमों को शसपष
चॉकरे ट, 18 पीसदी रडककमों को शसपष टॉपी औय 12 पीसदी रडककमों को शसपष चचप्स
शभरती हैं । 10% रडककमों को केवर चॉकरे ट औय टॉफी शभरती है, 20% रडककमों को
केवर टॉफी औय चचप्स शभरती है औय 8% रडककमों को केवर चचप्स औय चॉकरे ट
शभरती है। िे र्ष रडककमों को तीनों वस्तुएॉ प्राप्त होती हैं ।

1. How many boys get only one kind of items?


ककतने रडकों को केवर एक ही प्रकाय की वस्तुएॉ शभरती हैं ?
(A) 324 (B) 290 (C) 444
(D) 360 (E) None of these

2. How many girls get only two kind of items?


ककतने रडककमों को के वर दो प्रकाय की वस्तुएॉ शभरती हैं ?
(A) 276 (B) 290 (C) 228
(D) 280 (E) None of these

3. How many students got item of all three kinds?


ककतने ववद्माचथषमों को तीनों प्रकाय की वस्तुएॉ शभरीॊ?
(A) 114 (B) 96 (C) 100
(D) 120 (E) None of these

4. What is the ratio between the number of boys who get only toffee
and that of girls who get only toffee? https://instagra
https://youtu https ://ww
be.com/chan w.fa cebook.
m.com/aashish https://
arorasocial(?)
nel/UCYa4_Jr com/a ashis utm_medium = t.me/st
180 Orf8R5Kz2uO
ha rorasocial copy_link
tccXQ udified
केवर टॉफी प्राप्त कयने वारे रडकों औय केवर टॉफी प्राप्त कयने वारी रडककमों की
सॊख्मा के फीच का अनुऩात ककतना है?
(A) 4 : 5 (B) 2 : 3 (C) 5 : 6
(D) 6 : 7 (E) None of these

5. How many girls got only one kind of item?


ककतनी रडककमों को केवर एक प्रकाय की वस्तु शभरी?
(A) 577 (B) 290 (C) 410
(D) 312 (E) None of these

(6 – 10) There are five classes 8, 9, 10, 11 and 12 in a school and


total number of boys in year 2020 in all classes is 822 and total
number of girls in year 2020 in all classes is 828. The number of boys
in class 9 is 135. The ratio of the number of boys and girls in class 9
is 3 : 5. Number of girls in class 12 is 10 more than the average of
number of boys and number of girls in class 9. The number of boys in
class 11 is 10% less than the number of girls in class 12. The number
of boys in class 10 is 17/4 times the difference between the number of
boys in class 11 and the number of boys in class 9. The ratio of the
number of boys and girls in class 10 is 17 : 13. The number of girls
in class 8 is 34 less than the number of girls in class 12. The ratio of
the number of boys to the number of girls in class 8 is the same as
the ratio of the number of boys to the number of girls in class 10.
(6 – 10) एक ववद्मारम भें ऩाॊच कऺाएॊ 8, 9, 10, 11 औय 12 है औय सबी कऺाओॊ
भें वर्षष 2020 भें रडको की कु र सॊख्मा 822 है औय सबी कऺाओॊ भें वर्षष 2020 भें
रडककमो की कु र सॊख्मा 828 है। कऺा 9 भें रडको की सॊख्मा 135 है। कऺा 9 भें
रडको औय रडककमो की सॊख्मा का अनुऩात 3 : 5 है। कऺा 12 भें रडककमो की सॊख्मा,
कऺा 9 भें रडको की सॊख्मा औय रडककमो की सॊख्मा के औसत से 10 अचधक है। कऺा
11 भें रडको की सॊख्मा, कऺा 12 भें रडककमो की सॊख्मा से 10% कभ है। कऺा 10 भें
रडको की सॊख्मा, कऺा 11 भें रडको की सॊख्मा औय कऺा 9 भें रडको की सॊख्मा के
अॊतय का 17/4 गुना है। कऺा 10 भें रडको औय रडककमो की सॊख्मा का अनुऩात 17 :
13 है। कऺा 8 भें रडककमो की सॊख्मा, कऺा 12 भें रडककमो की सॊख्मा से 34 कभ है।
कऺा 8 भें रडको की सॊख्मा औय रडककमो की सॊख्मा का अनुऩात, कऺा 10 भें रडको
की सॊख्मा औय रडककमो की सॊख्मा के अनुऩात के सभान है।

6. What is the difference between the total number of students in class


8 and 11 in the year 2020?
वर्षष 2020 भें कऺा 8 औय 11 भें ववद्माचथषमों की कुर सॊख्मा के फीच का अॊतय ककतना
है?
(A) 48 (B) 46 (C) 41
(D) 49 (E) None of these

7. The number of students studying in class 9 in the year 2020 is


https://instagra
https://youtu https ://ww
be.com/chan w.fa cebook.
m.com/aashish https://
arorasocial(?)
nel/UCYa4_Jr com/a ashis utm_medium = t.me/st
181 Orf8R5Kz2uO
ha rorasocial copy_link
tccXQ udified
what percent of the number of students studying in class 10 in the
year 2020?
वर्षष 2020 भें कऺा 9 भें ऩढने वारे ववद्माचथषमों की सॊख्मा, वर्षष 2020 भें कऺा 10 भें
ऩढने वारे ववद्माचथषमों की सॊख्मा का ककतने प्रनतित है ?
(A) 139.50% (B) 135.25% (C) 133.33%
(D) 139.66% (E) None of these

8. What is the difference between the total number of boys in class


10, 11 and 12 together and the total number of girls in class 8, 9 and
12 together?
कऺा 10, 11 औय 12 भें रडको की कुर सॊख्मा औय कऺा 8, 9 औय 12 भें रडककमो
की कु र सॊख्मा के फीच का अॊतय ककतना है?
(A) 84 (B) 85 (C) 86
(D) 88 (E) None of these

9. What is the ratio of the number of boys in class 9 to the number of


girls in class 11?
कऺा 9 भें रडको की सॊख्मा औय कऺा 11 भें रडककमो की सॊख्मा का अनुऩात ककतना
है?
(A) 28 : 27 (B) 27 : 28 (C) 29 : 28
(D) 28 : 29 (E) None of these

10. If the number of students studying in class 8 is increased by 25%


and the ratio of boys and girls remains the same, then what will be
the total number of girls in all the classes?
मदद कऺा 8 भें ऩढने वारे ववद्माचथषमों की सॊख्मा भें 25% की वरवि कय दी जाए औय
रडको औय रडककमो का अनुऩात सभान यहे , तो सबी कऺाओॊ भें रडककमो की कुर
सॊख्मा ककतनी होगी?
(A) 848 (B) 852 (C) 842
(D) 867 (E) None of these

(11 – 15) There are three car companies – Hyundai, Toyota and Maruti
Suzuki. The number of cars of all the companies sold in three months
in a particular year are given below.
Hyundai : The ratio between the number of cars sold in May and the
number of cars sold in July is 5 : 4. Number of car sold in June is 55
more than the average of total number of car sold in May and July.
The difference between the number of cars sold in June and July is
70.
Toyota : Ratio between number of car sold in May and June is 3 : 4
and number of car sold in July is 23(1/3) % more than number of car
sold in May. The total number of cars sold in all the three months
together is 428. https://instagra
https://youtu https ://ww
be.com/chan w.fa cebook.
m.com/aashish https://
arorasocial(?)
nel/UCYa4_Jr com/a ashis utm_medium = t.me/st
182 Orf8R5Kz2uO
ha rorasocial copy_link
tccXQ udified
Maruti Suzuki : Number of car sold in May is 40% more than number
of car sold in July. The number of Maruti Suzuki cars sold in June is
four times the difference between the number of Hyundai cars and the
number of Toyota cars sold in June. The total number of Maruti
Suzuki cars sold in all the three months together is 44 less than the
total number of Toyota caRs.
(11 – 15) तीन काय कॊऩननमाॊ – Hyundai, Toyota औय Maruti Suzuki हैं । एक
वविे र्ष वर्षष भें तीन भहीनों भें फे चे गए सबी कॊऩननमों की काय की सॊख्मा नीचे दी गई
जानकायी भें दी गमी है।
Hyundai : भई भें फे ची गई काय की सॊख्मा औय जु राई भें फे ची गई काय की सॊख्मा के
फीच का अनुऩात 5 : 4 है। ज ून भें फे ची गई काय की सॊख्मा भई औय जु राई भें फे ची
गई काय की कु र सॊख्मा के औसत से 55 अचधक है। ज ून औय जु राई भें फे ची गई काय
की सॊख्मा के फीच का अॊतय 70 है।
Toyota : भई औय ज ून भें फे ची गई काय की सॊख्मा के फीच का अनुऩात 3 : 4 है औय
जु राई भें फे ची गई काय की सॊख्मा भई भें फे ची गई काय की सॊख्मा से 23(1/3) %
अचधक है। सबी तीन भहीनों भें एक साथ फे ची गई काय की कु र सॊख्मा 428 है।
Maruti Suzuki : भई भें फे ची गई काय की सॊख्मा जु राई भें फे ची गई काय की सॊख्मा
से 40% अचधक है। ज ून भें फे ची गई Maruti Suzuki काय की सॊख्मा, ज ून भें फे ची गई
Hyundai काय की सॊख्मा औय Toyota काय की सॊख्मा के फीच के अॊतय का चाय गुना
है। सबी तीन भहीनों भें एक साथ फे ची गई Maruti Suzuki काय की कुर सॊख्मा
Toyota काय की कु र सॊख्मा से 44 कभ है।

11. What is the total number of cars of all the companies together
sold in July?
जु राई भें फे ची गई सबी कॊऩननमों की कायों की कु र सॊख्मा ककतनी है?
(A) 348 (B) 358 (C) 368
(D) 378 (E) None of these

12. The total number of cars sold in May by all the companies
together is approximately what percent less/more than the cars sold in
June by all the companies together?
भई भें सबी कॊ ऩननमों को शभराकय फे ची गई काय की कु र सॊख्मा ज ून भें सबी कॊ ऩननमों
की शभराकय फे ची गई काय से रगबग ककतने प्रनतित कभ/ज्मादा है?
(A) 9.78% (B) 11.67% (C) 8.48%
(D) 10.59% (E) None of these

13. If the price of Toyota car is Rs. 8 lakh, then what is the total
earning of Toyota company in all the three months together?
मदद Toyota काय की कीभत 8 राख रुऩमे है, तो तीनों भहीनों भें Toyota कॊऩनी की
कुर कभाई ककतनी है?
(A) 3388 lakh (B) 3248 lakh (C) 3424 lakh
(D) 3348 lakh (E) None of these
https://youtu https://instagra
https ://ww
be.com/chan w.fa cebook.
m.com/aashish https://
arorasocial(?)
nel/UCYa4_Jr com/a ashis utm_medium = t.me/st
183 Orf8R5Kz2uO
ha rorasocial copy_link
tccXQ udified
14. What is the average of the total number of Maruti Suzuki cars sold
in all the three months together?
सबी तीन भहीनों भें एक साथ फे ची गई Maruti Suzuki काय की कु र सॊख्मा का
औसत ककतना है?
(A) 148 (B) 128 (C) 134
(D) 138 (E) None of these

15. If the cost of 1 Hyundai car is Rs. 7 lakh and that of 1 Maruti
Suzuki car is Rs. 4 lakh, then what is the difference between the total
income of Hyundai company and that of Maruti Suzuki company in the
month of May?
मदद 1 Hyundai काय की कीभत 7 राख रुऩमे है औय 1 Maruti Suzuki काय की
कीभत 4 राख रुऩमे है, तो भई भहीने भें Hyundai कॊऩनी औय Maruti Suzuki
कॊ ऩनी की कु र आभदनी के फीच का अॊतय ककतना है?
(A) 448 lakh (B) 434 lakh (C) 424 lakh
(D) 448 lakh (E) None of these

(16 – 20) Some people work in a factory and they have three types of
bikes, namely Hero, TVS and Bajaj. The number of people who have
only Hero bikes is 248. The number of people who have all the bikes
is 3/4 of the number of people who have both Hero and Bajaj bikes.
The number of people who have only TVS bike is 7/4 of Total number
of people who have only Bajaj bikes & who have both Hero and Bajaj
bikes. The number of people who have only Hero bikes is four times
the number of people who have both Bajaj and TVS bikes but do not
have Hero bikes. The total number of people working in the factory is
2056. The total number of people is 68 more than twice the number of
people who have only TVS bikes. The number of people who have
both Hero and Bajaj bikes but not TVS bikes is 18 less than 1/2 of
the people who have only Hero bikes. It is known that every person in
the factory has at least one out of three bikes.
(16 – 20) कु छ रोग एक कायखाने भें काभ कयते हैं औय उनके ऩास Hero, TVS औय
Bajaj नाभ की तीन तयह की फाइक हैं । केवर Hero फाइक यखने वारों की सॊख्मा 248
है। सबी फाइक यखने वारों की सॊख्मा Hero औय Bajaj दोनों फाइक यखने वारों की
सॊख्मा का 3/4 है। ऐसे रोगों की सॊख्मा जजनके ऩास केवर TVS फाइक है, उन रोगों
की कु र सॊख्मा का 7/4 है जजनके ऩास केवर Bajaj फाइक है औय जजनके ऩास Hero
औय Bajaj दोनों फाइक हैं । केवर Hero फाइक यखने वारे रोगों की सॊख्मा उन रोगों
की सॊख्मा से चाय गन ु ा है जजनके ऩास Bajaj औय TVS दोनों फाइक हैं रे ककन Hero
फाइक नहीॊ हैं । कायखाने भें काभ कयने वारे रोगों की कु र सॊख्मा 2056 है। कु र रोगों
की सॊख्मा केवर TVS फाइक यखने वारों की सॊख्मा के दोगन ु े से 68 अचधक है। ऐसे
रोगों की सॊख्मा जजनके ऩास Hero औय Bajaj दोनों फाइक हैं रे ककन TVS फाइक नहीॊ
हैं , उन रोगों की सॊख्मा 1/2 से 18 कभ है जजनके ऩास केवर Hero फाइक हैं । ऻात हो
कक पैक्ट्री भें प्रत्मे क व्मजक्त के ऩास तीन भें से कभ से कभ एक फाइक होती है।
https://instagra
https://youtu https ://ww
be.com/chan w.fa cebook.
m.com/aashish https://
arorasocial(?)
nel/UCYa4_Jr com/a ashis utm_medium = t.me/st
184 Orf8R5Kz2uO
ha rorasocial copy_link
tccXQ udified
16. How many persons have both Hero and Bajaj bikes?
ककतने व्मजक्तमों के ऩास Hero औय Bajaj दोनों तयह की फाइक है?
(A) 318 (B) 106 (C) 418
(D) 424 (E) None of these

17. What is the ratio of the number of persons who have all three
types of bikes to the persons who have both Hero and TVS but not
Bajaj?
वह व्मजक्त जजनके ऩास तीनों तयह की फाइक है औय वह व्मजक्त जजनके ऩास दोनों
Hero औय TVS है रे ककन Bajaj नहीॊ है उनकी सॊख्मा का अनुऩात ककतना है?
(A) 149 : 97 (B) 145 : 95 (C) 159 : 92
(D) 138 : 89 (E) None of these

18. What is the difference between the number of persons who have
only TVS and those who have only Bajaj?
वह व्मजक्त जजनके ऩास केवर TVS है औय जजनके ऩास केवर Bajaj है उनकी सॊख्मा
के फीच का अॊतय ककतना है?
(A) 840 (B) 850 (C) 860
(D) 870 (E) None of these

19. The number of persons who have both Hero and Bajaj is what
percent of the total number of persons who have Bajaj bike?
वे व्मजक्त जजनके ऩास Hero औय Bajaj दोनों है उनकी सॊख्मा, वह व्मजक्त जजनके
ऩास Bajaj फाइक है उनकी कुर सॊख्मा का ककतना प्रनतित है?
(A) 66.60% (B) 62.40% (C) 69.50%
(D) 67.30% (E) None of these

20. How many persons have only two bikes?


ककतने व्मजक्तमों के ऩास केवर दो फाइक हैं ?
(A) 348 (B) 352 (C) 342
(D) 358 (E) None of these

(21 – 25) In an auction, there were watches of three popular inventors


A, B and C. It was found that total600 customers have visited auction.
45% customer bought watches of A, 582 % customers purchased
watches of B and 50% customers bought watches of C. 60 customers
bought watches of all three inventoRs. 30 customers bought watches
of only A and B but not C, 90 customers bought paintings of only B
and C but not A and 140 customers bought watches of A and C but
not B.
(21 – 25) एक नीराभी भें तीन रोकवप्रम आववष्कायकों A, B औय C की घडडमाॉ थीॊ मह
ऩामा गमा कक कुर 600 ग्राहक नीराभी भें आए हैं । 45% ग्राहकों ने A की घडडमाॉ
खयीदी 58 % ग्राहकों ने B की घडडमाॉ खयीदीॊ औय 50% ग्राहकों ने C की घडडमाॉ खयीदीॊ
https://instagra
https://youtu https ://ww
be.com/chan w.fa cebook.
m.com/aashish https://
arorasocial(?)
nel/UCYa4_Jr com/a ashis utm_medium = t.me/st
185 Orf8R5Kz2uO
ha rorasocial copy_link
tccXQ udified
60 ग्राहकों ने तीनों आववष्कायकों की घडडमाॉ खयीदी 30 ग्राहकों ने केवर A औय 8 की
घडडमाॉ खयीदी रे ककन C की नहीॊ, 90 ग्राहकों ने केवर B औय C की नहीॊ फजल्क A की
ऩें दटॊग खयीदी औय 140 ग्राहकों ने A औय C की घडडमाॉ खयीदीॊ रे ककन B की नहीॊ।

21. What is the total number of customers who bought watches of only
C?
केवर C की घडडमाॉ खयीदने वारे ग्राहकों की कुर सॊख्मा ककतनी है?
(A) 30 (B) 10 (C) 8
(D) 20 (E) None of these

22. Find the average number of customers who bought watches of only
A and only C.
केवर A औय केवर C की घडडमाॉ खयीदने वारे ग्राहकों की औसत सॊख्मा ऻात कीजजए।
(A) 15 (B) 20 (C) 28
(D) 25 (E) None of these

23. How many customers bought watches of atleast two inventors?


ककतने ग्राहकों ने कभ से कभ दो आववष्कायकों की घडडमाॉ खयीदीॊ ?
(A) 320 (B) 340 (C) 260
(D) 300 (E) None of these

24. Total customers who bought watches of B is how much percent


more than number of number of customers who bought watches of A
and B but not C?
B की घडडमाॉ खयीदने वारे कु र ग्राहक, A औय B की घडडमाॉ खयीदने वारे रे ककन C की
नहीॊ खयीदने वारे ग्राहकों की सॊख्मा से ककतने प्रनतित अचधक हैं ?
(A) 1660.06% (B) 1066.66% (C) 1060.6%
(D) 1066.06 (E) None of these

25. What is the difference between the number of customers who


bought watches of only type B and only type C?
केवर B प्रकाय औय केवर C प्रकाय की घडडमाॉ खयीदने वारे ग्राहकों की सॊख्मा के फीच
का अॊतय ककतना है?
(A) 160 (B) 30 (C) 140
(D) 120 (E) None of these

(26 – 30) There are 640 people in city A where the gender ratio is 5 :
3 and the percentage of Doctors is 50%. The number of males in city
A is same as that in city B and ratio of number of doctors to
engineers in city B is 7 : 4. Out of all the females,35% are from city
C. Number of females in city B is 60 less than in city C. The total
number of people in city C is 10 more than double the number of
doctors in city A. There are 680 engineers in all 3 cities combined.
https://instagra
https://youtu https ://ww
be.com/chan w.fa cebook.
m.com/aashish https://
arorasocial(?)
nel/UCYa4_Jr com/a ashis utm_medium = t.me/st
186 Orf8R5Kz2uO
ha rorasocial copy_link
tccXQ udified
(26 – 30) िहय A भें 640 रोग हैं जहाॊ शरॊग अनुऩात 5 : 3 है औय डॉक्टयों का
प्रनतित 50% है। िहय A भें ऩुरुर्षों की सॊख्मा िहय B के ऩुरुर्षों की सॊख्मा के सभान है
औय िहय B भें डॉक्टयों की सॊख्मा का इॊज ीननमयों से अनुऩात 7 : 4 है। सबी भदहराओॊ
भें से , 35% िहय C से हैं । िहय B भें भदहराओॊ की सॊख्मा िहय C से 60 कभ है।
िहय C भें कु र रोगों की सॊख्मा िहय A भें डॉक्टयों की सॊख्मा के दोगन ु े से 10 अचधक
है। सबी 3 िहयों भें कु र इॊज ीननमय 680 है।

26. What is the overall ratio of males and females in all cities
combined?
सबी िहयों भें सॊमुक्त रूऩ से ऩुरुर्षों औय भदहराओॊ का कु र अनुऩात ककतना है?
(A) 15 : 31 (B) 31 : 15 (C) 15 : 46
(D) 46 : 15 (E) None of these

27. What is the percentage of doctors in all cities combined?


सबी िहयों भें सॊमुक्त रूऩ से डॉक्टयों का प्रनतित ककतना है?
(A) 63.40% (B) 33.33% (C) 63.04%
(D) 50% (E) None of these

28. The sum of number of females in city A and B is what %


more/less than the sum of number of males in city B and C?
िहय A औय B भें भदहराओॊ की सॊख्मा का मोग िहय B औय C भें ऩुरुर्षों की सॊख्मा के
मोग से ककतना % अचधक/कभ है?
(A) 53.57% (B) 33.33% (C) 57%
(D) 50% (E) None of these

29. What is the average number of engineers from all cities together?
सबी िहयों से शभराकय इॊजीननमयों की औसत सॊख्मा ककतनी है ?
(A) 226.86 (B) 226.66 (C) 225.58
(D) 228.55 (E) None of these

30. What is the difference between the average number of engineers


in city A and C and the total number of doctors in city A?
िहय A औय C भें इॊजीननमयों की औसत सॊख्मा औय िहय A भें डॉक्टयों की कुर सॊख्मा
के फीच का अॊतय ककतना है?
(A) 85 (B) 83 (C) 85
(D) 80 (E) None of these

(31 – 35) There are 300 products manufactured by factory A in which


number of Ice cream and chocolates manufactured in ratio 2 : 3 and
the number of 5 star and Kit - Kat chocolate manufactured in ratio 2 :
7. In factory B, total number of 5 star and Kit - Kat chocolates
manufactured are 50 and 80 respectively while the total number of Ice
cream and chocolates manufactured is in ratio 10 : 13. In factory C
https://instagra
https://youtu https ://ww
be.com/chan w.fa cebook.
m.com/aashish https://
arorasocial(?)
nel/UCYa4_Jr com/a ashis utm_medium = t.me/st
187 Orf8R5Kz2uO
ha rorasocial copy_link
tccXQ udified
the number of Ice cream manufactured is 150 and the number of
chocolates manufactured is 20% more than the number of chocolates
manufactured by factory B whereas the umber of Kit - Kat and 5 star
chocolates manufactured is in the ratio 7 : 5.
(31 – 35) पैक्ट्री A द्वाया ननशभषत 300 उत्ऩाद हैं जजनभें 2 : 3 के अनुऩात भें ननशभषत
आइसक्रीभ औय चॉकरे ट की सॊख्मा औय 2 : 7 के अनुऩात भें ननशभषत 5 स्टाय औय
ककटकैट चॉकरे ट की सॊख्मा है। पैक्ट्री B भें ननशभषत 5 स्टाय औय ककटकैट चॉकरे ट की
कु र सॊख्मा क्रभि् 50 औय 80 है, जफकक ननशभषत आइसक्रीभ औय चॉकरे ट की कु र
सॊख्मा 10 : 13 के अनुऩात भें है। पैक्ट्री C भें ननशभषत आइसक्रीभ की सॊख्मा 150 है
औय ननशभषत चॉकरे ट की सॊख्मा पैक्ट्री B द्वाया ननशभषत चॉकरे ट की सॊख्मा से 20%
अचधक है जफकक ककटकैट औय 5 स्टाय चॉकरे ट की सॊख्मा 7 : 5 के अनुऩात भें है।

31. What is the overall ratio of Kit - Kat and 5 star chocolates
manufactured by all three factories together?
तीनों कायखानों द्वाया एक साथ ननशभषत ककटकैट औय 5 स्टाय चॉकरे ट का अनुऩात
ककतना है?
(A) 311.155 (B) 115.311 (C) 126.125
(D) 254.255 (E) None of these

32. The total number of Ice cream manufactured in all 3 factories is


what % more or less than total number of Kit - Kat chocolates
manufactured in all these 3 factories?
सबी 3 कायखानों भें ननशभषत कु र आइसक्रीभ की सॊख्मा इन सबी 3 कायखानों भें ननशभषत
ककटकैट चॉकरे ट की कुर सॊख्मा से ककतने प्रनतित अचधक मा कभ है?
(A) 18.97% (B) 19.99% (C) 18.65%
(D) 20% (E) None of these

33. Find the average number of chocolates manufactured in factory B


and C together.
पैक्ट्री B औय C भें शभराकय ननशभषत चॉकरे ट की औसत सॊख्मा ऻात कीजजए।
(A) 1123 (B) 143 (C) 156
(D) 165 (E) None of these

34. Find the total products produced in factory C.


पैक्ट्री C भें उत्ऩाददत कुर उत्ऩाद ऻात कीजजए।
(A) 310 (B) 306 (C) 308
(D) 320 (E) None of these

35. What is the difference between the average number of Kit - Kat
chocolates manufactured in factory A and C and the total number of
Ice cream manufactured in factory A?
पैक्ट्री A औय C भें ननशभषत ककटकैट चॉकरे ट की औसत सॊख्मा औय पैक्ट्री A भें ननशभषत
आइसक्रीभ की कु र सॊख्मा के फीच का अॊतय ककतना है? https://instagra
https://youtu https ://ww
be.com/chan w.fa cebook.
m.com/aashish https://
arorasocial(?)
nel/UCYa4_Jr com/a ashis utm_medium = t.me/st
188 Orf8R5Kz2uO
ha rorasocial copy_link
tccXQ udified
(A) 4 : 5 (B) 3 : 8 (C) 7.5
(D) 7.6 (E) None of these

(36 – 40) P, Q, R and S are four students who give exam of English
and Hindi with Maximum marks 100 and get different marks in both
the subjects. P got 75 in English and Q got 64 in Hindi. R got 20%
more marks in English than marks scored by P in English. Ratio of
marks of P in Hindi and S in English is 13 : 11 where as total marks
P got in both the subjects is 140. Marks scored by S in Hindi is
27.27% more than marks scored by S in English. Total marks scored
by all the students in English and Hindi is 280 and 270.
(36 – 40) P, Q, R औय S चाय छात्र हैं जो अचधकतभ 100 अॊक ों के साथ अॊग्रेज ी
औय दहॊदी की ऩयीऺा दे ते हैं औय दोनों ववर्षमों भें अरग – अरग अॊक प्राप्त कयते हैं । P
को अॊग्रेज ी भें 75 औय Q को दहॊदी भें 64 अॊक शभरे । R को अॊग्रेज ी भें P द्वाया अॊग्रेज ी
भें प्राप्त अॊकों से 20% अचधक अॊक प्राप्त हुए। दहॊदी भें P औय अॊग्रेजी भें S के अॊकों
का अनुऩात 13 : 11 है, जफकक दोनों ववर्षमों भें P के कु र अॊक 140 हैं । S द्वाया दहॊदी
भें प्राप्त अॊक , S द्वाया अॊग्रेज ी भें प्राप्त अॊक ों से 27.27% अचधक है। सबी छात्रों द्वाया
अॊग्रेज ी औय दहॊदी भें प्राप्त ककए गए कुर अॊक 280 औय 270 हैं ।

36. What is the average of percentages of all the students?


सबी छात्रों के प्रनतित का औसत क्मा है?
(A) 66.75% (B) 68.75% (C) 60.75%
(D) 78.75% (E) None of these

37. The marks scored by P and Q in English is how much percent


less than number of marks scored by R and S in Hindi?
P औय Q द्वाया अॊग्रेजी भें प्राप्त अॊक , R औय S द्वाया दहॊदी भें प्राप्त अॊकों की सॊख्मा
से ककतने प्रनतित कभ है?
(A) 6.25% (B) 3.25% (C) 4.25%
(D) 5.25% (E) None of these

38. If T got 36.36% more Marks in English than number of marks


scored by S in English and T got 25% less marks in Hindi than
number of marks scored by Q in Hindi than find the percentage T
scored in the examination.
मदद T को अॊग्रेज ी भें S द्वाया प्राप्त अॊक ों की सॊख्मा से अॊग्रेज ी भें 36.36 प्रनतित
अचधक अॊक प्राप्त होते हैं औय दहॊदी भें T को Q द्वाया प्राप्त दहॊदी भें अॊक ों की सॊख्मा
की तुरना भें 25% कभ अॊक प्राप्त होते हैं , तो ऩयीऺा भें T द्वाया प्राप्त अॊकों का
प्रनतित ऻात कीजजए।
(A) 63.5% (B) 62.5% (C) 61.5%
(D) 60.5% (E) None of these

39. Which of the student scored the maximum percentage


https://instagra of marks in
https://youtu https ://ww
be.com/chan w.fa cebook.
m.com/aashish https://
arorasocial(?)
nel/UCYa4_Jr com/a ashis utm_medium = t.me/st
189 Orf8R5Kz2uO
ha rorasocial copy_link
tccXQ udified
the examination?
ककस छात्र ने ऩयीऺा भें अचधकतभ प्रनतित अॊक प्राप्त ककए हैं ?
(A) S (B) P (C) Q
(D) R (E) None of these

40. If P got 7.69% more marks in Math than in Hindi and maximum
mark of Math were also 100 then find the average number of marks
scored by P.
मदद P ने गणणत भें दहॊदी से 7.69% अचधक अॊक प्राप्त ककए औय गणणत के अचधकतभ
अॊक बी 100 थे, तो P द्वाया प्राप्त अॊक ों की औसत सॊख्मा ऻात कीजजए।
(A) 70% (B) 72% (C) 78%
(D) 75% (E) None of these

(41 – 45) Ratio of total number of TV and Fridge sold by company P


is 13 : 9 and total number of items sold is 132. Total number of TV
sold by Company Q is 14.28% less than number of Fridge sold by
Company Q and total number of items sold by Company Q is 11 more
than total number of items sold by Company P. Number of TV and
Fridge sold by Company R is 70 and 80. Number of TV and Fridge
sold by Company S is 24 less and 20 more than number of TV and
Fridge sold by Company R. The ratio of TV and Fridge sold by
Company T is 11 : 9 and total number of items sold by Company T is
14 more than total items sold by Company S.
(41 – 45) कॊऩनी P द्वाया फे चे गए TV औय Fridge की कुर सॊख्मा का अनुऩात 13 :
9 है औय फे ची गई वस्तुओ ॊ की कु र सॊख्मा 132 है। कॊऩनी Q द्वाया फे चे गए TV की
कु र सॊख्मा कॊ ऩनी Q द्वाया फे चे गए Fridge की सॊख्मा से 14.28% कभ है औय कॊऩनी
Q द्वाया फे ची गई कुर आइटभों की सॊख्मा, कॊऩनी P द्वाया फे ची गई वस्तुओ ॊ की कुर
सॊख्मा से 11 अचधक है। कॊऩनी R द्वाया फे चे गए TV औय Fridge की सॊख्मा 70 औय
80 है। कॊऩनी S द्वाया फे चे गए TV औय Fridge की सॊख्मा कॊऩनी R द्वाया फे चे गए
TV औय Fridge की सॊख्मा से 24 कभ औय 20 अचधक है। कॊऩनी T द्वाया फे चे गए
TV औय Fridge का अनुऩात 11 : 9 है औय कॊऩनी T द्वाया फे ची गई वस्तुओ ॊ की कुर
सॊख्मा कॊऩनी S द्वाया फे ची गई कु र वस्तुओ ॊ से 14 अचधक है।

41. What is the average number of Fridge sold by all companies?


सबी कॊऩननमों द्वाया फे चे गए किज की औसत सॊख्मा ककतनी है ?
(A) 74.6 (B) 88.6 (C) 76.6
(D) 66.6 (E) None of these

42. What is the difference between number of TV sold by Company R


and average number of TV sold by all the companies?
कॊऩनी R द्वाया फे चे गए टीवी की सॊख्मा औय सबी कॊऩननमों द्वाया फे चे गए टीवी की
औसत सॊख्मा के फीच का अॊतय ककतना है?
https://youtu https://instagra
https ://ww
be.com/chan w.fa cebook.
m.com/aashish https://
arorasocial(?)
nel/UCYa4_Jr com/a ashis utm_medium = t.me/st
190 Orf8R5Kz2uO
ha rorasocial copy_link
tccXQ udified
(A) 2.4 (B) 0.9 (C) 1.2
(D) 0.4 (E) None of these

43. In which of the Company number of fridge sold is less than the
average number of fridge sold by all the companies?
ककस कॊऩनी भें फे चे गए किज की सॊख्मा सबी कॊऩननमों द्वाया फे चे गए किज की औसत
सॊख्मा से कभ थी?
(A) P and (B) P and R (C) S and Q
(D) R and Q (E) None of these

44. Total number of items sold by Company S is what percent more


than the total number of TV sold by Company Q?
कॊऩनी S द्वाया फे ची गई वस्तुओ ॊ की कुर सॊख्मा, कॊऩनी Q द्वाया फे चे गए टीवी की
कु र सॊख्मा से ककतने प्रनतित अचधक है?
(A) 126.21% (B) 124.11% (C) 121.21%
(D) 122.21% (E) None of these

45. What is the ratio of Total number of fridge sold by Company P, R


and T with the total number of TV sold by Company Q and S?
कॊऩनी P, R औय T द्वाया फे चे गए किज की कुर सॊख्मा का कॊऩनी Q औय S द्वाया
फे चे गए टीवी की कु र सॊख्मा से अनुऩात ककतना है?
(A) 56 : 103 (B) 103 : 56 (C) 45 : 103
(D) 103 : 45 (E) None of these

(46 – 50) 18 people speak Hindi only. 101 people speak at most 1
language. 8 people speak none of the languages. The ratio of the
number of people speaking only Spanish to number of people speaking
only English is 8 : 7. 48 people speak Hindi but not Spanish. 54
people speak both Hindi and English. 114 people speak English. 11%
out of total persons speaking Spanish speak both Hindi and Spanish
but not English.
(46 – 50) 18 रोग केवर दहॊदी फोरते हैं । 101 रोग अचधकतभ 1 बार्षा फोरते हैं । 8
रोग कोई बी बार्षा न फोरते हैं । केवर स्ऩे ननि फोरने वारे रोगों की सॊख्मा का केवर
अॊग्रेज ी फोरने वारों की सॊख्मा से अनुऩात 8 : 7 है। 48 रोग दहॊदी फोरते हैं रे ककन
स्ऩे ननि नहीॊ। 54 रोग दहॊदी औय अॊग्रेजी दोनों फोरते हैं । 114 रोग अॊग्रेजी फोरते हैं ।
स्ऩे ननि फोरने वारे कु र व्मजक्तमों भें से 11% दहॊदी औय स्ऩे ननि दोनों फोरते हैं रे ककन
अॊग्रेज ी नहीॊ फोरते हैं ।

46. How many people speak Hindi?


ककतने रोग दहॊदी फोरते हैं ?
(A) 83 (B) 63 (C) 75
(D) 73 (E) None of these
https://youtu https://instagra
https ://ww
be.com/chan w.fa cebook.
m.com/aashish https://
arorasocial(?)
nel/UCYa4_Jr com/a ashis utm_medium = t.me/st
191 Orf8R5Kz2uO
ha rorasocial copy_link
tccXQ udified
47. What is the ratio of people speaking English only with the people
speaking none of the three languages?
केवर अॊग्रेज ी फोरने वारे रोगों का तीन बार्षाओॊ भें से कोई बी नहीॊ फोरने वारे रोगों
से अनुऩात ककतना है?
(A) 8 : 35 (B) 35 : 8 (C) 45 : 7
(D) 7 : 45 (E) None of these

48. What is the number of people speaking at least two languages?


कभ से कभ दो बार्षाएॊ फोरने वारे रोगों की सॊख्मा क्मा है ?
(A) 76 (B) 100 (C) 108
(D) 93 (E) None of these

49. What is the ratio of the number of people speaking Hindi only to
the number of people speaking Spanish only?
केवर दहॊदी फोरने वारे रोगों की सॊख्मा का केवर स्ऩे ननि फोरने वारे रोगों की सॊख्मा
से अनुऩात ककतना है?
(A) 20 : 9 (B) 10 : 11 (C) 9 : 20
(D) 11 : 10 (E) None of these

50. How many people do not speak Hindi language?


ककतने रोग दहॊदी बार्षा नहीॊ फोरते हैं ?
(A) 120 (B) 108 (C) 1115
(D) 121 (E) None of these

(51 – 55) In a college there are four branches ME, CS, EC and Civil
having different number of boys and girls. The ratio of number of Boys
in ME with the number of Girls in CS is 12 : 7. Number of boys in EC
is 28.56% more than number of Girls in CS. Number of boys in Civil is
68 which is 41⅔% more than the number of girls in Civil. Number of
girls in ME is 8⅓% less than number of girls in Civil. Total number of
boys and girls in all branches is 240 and 180 and number of girls in
CS is 42.
(51 – 55) एक कॉरे ज भें चाय िाखाएॊ ME, CS, EC औय Civil हैं जजनभें रडके औय
रडककमाॊ अरग – अरग सॊख्मा भें हैं । ME भें रडकों की सॊख्मा औय CS भें रडककमों
की सॊख्मा का अनुऩात 12 : 7 है। EC भें रडकों की सॊख्मा CS भें रडककमों की सॊख्मा
से 28.56 प्रनतित अचधक है। Civil भें रडकों की सॊख्मा 68 है जो Civil भें रडककमों
की सॊख्मा से 41⅔% अचधक है। ME भें रडककमों की सॊख्मा Civil भें रडककमों की
सॊख्मा से 8⅓% कभ है। सबी िाखाओॊ भें रडकों औय रडककमों की कुर सॊख्मा 240
औय 180 है औय CS भें रडककमों की सॊख्मा 42 है।

51. Number of boys in ME, EC and CS is how much percent more


than number of girls in EC, CS and Civil?
ME, EC औय CS भें रडकों की सॊख्मा EC, CS औय Civil भें रडककमों की सॊख्मा से
https://instagra
https://youtu https ://ww
be.com/chan w.fa cebook.
m.com/aashish https://
arorasocial(?)
nel/UCYa4_Jr com/a ashis utm_medium = t.me/st
192 Orf8R5Kz2uO
ha rorasocial copy_link
tccXQ udified
ककतने प्रनतित अचधक है?
(A) 23.47% (B) 20.47% (C) 22.47%
(D) 26.47 (E) None of these

52. What is the ratio of total number of student in ME and Civil with
the total number of student in CS and EC?
ME औय शसववर भें छात्रों की कुर सॊख्मा का CS औय EC भें छात्रों की कुर सॊख्मा से
अनुऩात ककतना है?
(A) 47 : 58 (B) 58 : 47 (C) 45 : 47
(D) 47 : 45 (E) None of these

53. If next year number of boys in CS increased by 50% and number


of Girls in CS decreased by 14.28%, number of student in ME
increased by 25% then number of students in ME are what percent
more than the number of students in CS?
मदद अगरे वर्षष CS भें रडकों की सॊख्मा भें 50% की वरवि हुई औय CS भें रडककमों
की सॊख्मा भें 14.28% की कभी आई, ME भें छात्रों की सॊख्मा भें 25% की वरवि हुई,
तो ME भें छात्रों की सॊख्मा CS भें छात्रों की सॊख्मा से ककतने प्रनतित अचधक है ?
(A) 36.09% (B) 35.09% (C) 38.09%
(D) 32.09% (E) None of these

54. In which branch number of girls are approximately 15% less than
number of boys in that branch?
ककस ब्ाॊच भें रडककमों की सॊख्मा उस ब्ाॊच भें रडकों की सॊख्मा से रगबग 15% कभ
है?
(A) ME (B) CS (C) EC
(D) CIVIL (E) None of these

55. Find the difference between number of Girls in CS and ME and


number of boys in EC and Civil.
CS औय ME भें रडककमों की सॊख्मा औय EC औय शसववर भें रडकों की सॊख्मा के
फीच अॊतय ऻात कीजजए।
(A) 33 (B) 34 (C) 35
(D) 36 (E) None of these

(56 – 60) Three type of chocolates A, B and C liked by 2400 students


of a School. 15.5%, 12.5% and 18.75% of total students likes only A,
only B and only C respectively and 10.5% of them like all three
chocolates. The number of students who like A and B but not C is
42% of the person who like only B. the number of student who like A
and C but not B is 80% of the number of student who like B and C
but not A.
(56 – 60) एक स्कू र के 2400 छात्रों द्वाया तीन प्रकाय कीhttps://instagra
चॉकरे ट A, B औय C
https://youtu https ://ww
be.com/chan w.fa cebook.
m.com/aashish https://
arorasocial(?)
nel/UCYa4_Jr com/a ashis utm_medium = t.me/st
193 Orf8R5Kz2uO
ha rorasocial copy_link
tccXQ udified
ऩसॊद की जाती हैं । कु र छात्रों भें से 15.5%, 12.5% औय 18.75% क्रभि् केवर A,
केवर B औय केवर C ऩसॊद कयते हैं औय उनभें से 10.5% तीनों चॉकरे ट ऩसॊद कयते
हैं । A औय B को ऩसॊद कयने वारे रे ककन C को नहीॊ ऩसॊद कयने वारे छात्रों की सॊख्मा
केवर B को ऩसॊद कयने वारे छात्रों की सॊख्मा का 42% है। A औय C को ऩसॊद कयने
वारे रे ककन B को नहीॊ ऩसॊद कयने वारे छात्रों की सॊख्मा, B औय C को ऩसॊद कयने
वारे रे ककन A को नहीॊ ऩसॊद कयने वारे छात्रों की सॊख्मा का 80% है।

56. The number of students who like C is what percentage of the


number of student who like A?
C को ऩसॊद कयने वारे ववद्माचथषमों की सॊख्मा, A को ऩसॊद कयने वारे ववद्माचथषमों की
सॊख्मा का ककतना प्रनतित है?
(A) 138.5% (B) 137% (C) 135.5%
(D) 139% (E) None of these

57. What is the ratio of number of students who like both A and C
with the students who like both A and B?
A औय C दोनों को ऩसॊद कयने वारे छात्रों की सॊख्मा का A औय B दोनों को ऩसॊद
कयने वारे छात्रों से अनुऩात ककतना है?
(A) 189 : 326 (B) 326 : 189 (C) 45 : 189
(D) 71 : 45 (E) None of these

58. How many student like exactly two chocolates?


ककतने ववद्माथी ठीक दो चॉकरे ट ऩसॊद कयते हैं ?
(A) 1076 (B) 1075 (C) 1026
(D) 1085 (E) None of these

59. The number of students who only likes C is what percentage more
or less than the number of students who like only B?
केवर C को ऩसॊद कयने वारे ववद्माचथषमों की सॊख्मा केवर B को ऩसॊद कयने वारे
ववद्माचथषमों की सॊख्मा से ककतने प्रनतित अचधक मा कभ है ?
(A) 30% (B) 40% (C) 50%
(D) 60% (E) None of these

60. What percent of students like exactly one type of chocolate?


ककतने प्रनतित ववद्माथी एक ही प्रकाय की चॉकरे ट ऩसॊद कयते हैं ?
(A) 45.25% (B) 46.75% (C) 38.75%
(D) 35.50% (E) None of these

(61 – 65) Four persons A, B, C and D made different investment in


year 2018 and 2019. The ratio of investment made by D in 2018 and
2019 is 8 : 11. Investment made by A in 2018 is 58⅓% more than
investment made by A in 2019. investment made by B in 2018 is Rs.
7000 which is 53⅓% less than investment madehttps://instagra
by B in 2019.
https://youtu https ://ww
be.com/chan w.fa cebook.
m.com/aashish https://
arorasocial(?)
nel/UCYa4_Jr com/a ashis utm_medium = t.me/st
194 Orf8R5Kz2uO
ha rorasocial copy_link
tccXQ udified
Difference in the investment of D in two years is Rs. 3000. C invest
Rs. 8000 less in 2018 than investment made by A in 2019 and ratio of
investment made by C in 2018 and 2019 is 4 : 3. total investment
made by all persons in 2019 is Rs. 62000.
(61 – 65) चाय व्मजक्तमों A, B, C औय D ने वर्षष 2018 औय 2019 भें अरग – अरग
ननवे ि ककमा। 2018 औय 2019 भें D द्वाया ककए गए ननवे ि का अनुऩात 8 : 11 है।
2018 भें A द्वाया ककमा गमा ननवे ि 2019 भें A द्वाया ककए गए ननवे ि से 58⅓%
अचधक है। 2018 भें B द्वाया ककमा गमा ननवे ि 7000 रुऩमे है जो 2019 भें B द्वाया
ककए गए ननवे ि से 53⅓% कभ है। दो वर्षों भें D के ननवे ि भें अॊतय 3000 रुऩमे है।
C, 2019 भें A द्वाया ककए गए ननवे ि की तुरना भें 2018 भें 8000 रुऩमे कभ ननवे ि
कयता है औय 2018 औय 2019 भें C द्वाया ककए गए ननवे ि का अनुऩात 4 : 3 है।
2019 भें सबी व्मजक्तमों द्वाया ककमा गमा कु र ननवे ि 62000 रुऩमे है।

61. What is the ratio of investment made by C in 2019 to the


investment made by B in 2018?
2019 भें C द्वाया ककए गए ननवे ि का 2018 भें B द्वाया ककए गए ननवे ि से अनुऩात
ककतना है?
(A) 12 : 7 (B) 6 : 5 (C) 7 : 12
(D) 15 : 6 (E) None of these

62. Investment made by A in 2018 is what percent more than


investment made by B in 2019?
2018 भें A द्वाया ककमा गमा ननवे ि 2019 भें B द्वाया ककए गए ननवे ि से ककतना
प्रनतित अचधक है?
(A) 166.67% (B) 153.33% (C) 266.67%
(D) 243.33% (E) None of these

63. If investment made by D in 2018 is increased by 12.5% and


Investment made by B in 2018 is decreased by 14.28% then what is
the difference between there investments?
2018 भें D द्वाया ककए गए ननवे ि भें 12.5% ​क ी वरवि होती है औय 2018 भें B द्वाया
ककए गए ननवे ि भें 14.28% की कभी होती है, तो वहाॊ ननवे ि के फीच क्मा अॊतय है ?
(A) Rs. 5000 (B) Rs. 4000 (C) Rs. 3000
(D) Rs. 2000 (E) None of these

64. What is the difference between investment made by A, B and D in


2018 and the investment made by A, B and C in 2019?
2018 भें A, B औय D द्वाया ककए गए ननवे ि औय 2019 भें A, B औय C द्वाया ककए
गए ननवे ि के फीच ककतना अॊतय है ?
(A) Rs. 2000 (B) Rs. 1800 (C) Rs. 1600
(D) Rs. 2200 (E) None of these\

65. Find the average investment made by all the persons in 2018.
https://instagra
https://youtu https ://ww
be.com/chan w.fa cebook.
m.com/aashish https://
arorasocial(?)
nel/UCYa4_Jr com/a ashis utm_medium = t.me/st
195 Orf8R5Kz2uO
ha rorasocial copy_link
tccXQ udified
2018 भें सबी व्मजक्तमों द्वाया ककमा गमा औसत ननवे ि ऻात कीजजए।
(A) Rs. 17200 (B) Rs. 16500 (C) Rs. 15200
(D) Rs. 13800 (E) None of these

(66 – 70) There is a college in which student study in 9th, 10th, 11th
and 12th class in two different batches P and Q.
P batch : Total students in 11th class is 3 times of number of
students in 9th class which is 60 less than number of students in 12th
class. Total student in 10th class is one – third of the number of
student in 12th class.
Q batch : Number of students in 9th class is same as number of
students in class 11 in batch P. Number of students in 10th and 12th
is 200 and 165. Average of all students in Q batch is 150 which is 50
more than number of student in 11th class.
(66 – 70) एक कॉरे ज है जजसभें छात्र 9वीॊ, 10वीॊ, 11वीॊ औय 12वीॊ कऺा भें दो अरग –
अरग फैचों P औय Q भें ऩढते हैं ।
P फैच : 11वीॊ कऺा भें कु र ववद्माचथषमों की सॊख्मा 9वीॊ कऺा भें ववद्माचथषमों की सॊख्मा
का 3 गुना है जो 12वीॊ कऺा के ववद्माचथषमों की सॊख्मा से 60 कभ है। 10वीॊ कऺा भें
कु र ववद्माथी, 12वीॊ कऺा भें ववद्माचथषमों की सॊख्मा का एक नतहाई है।
Q फैच : 9वीॊ कऺा भें छात्रों की सॊख्मा फैच P भें कऺा 11 के छात्रों की सॊख्मा के
सभान है। 10वीॊ औय 12वीॊ भें छात्रों की सॊख्मा 200 औय 165 है। Q फैच भें सबी
ववद्माचथषमों का औसत 150 है जो 11वीॊ कऺा भें ववद्माचथषमों की सॊख्मा से 50 अचधक
है।

66. Find the ratio of number of student in class 9th and 10th in P
batch to the number of students in class 11th and 12th in Q batch.
P फैच भें कऺा 9वीॊ औय 10वीॊ भें ववद्माचथषमों की सॊख्मा का Q फैच भें कऺा 11वीॊ औय
12वीॊ के ववद्माचथषमों की सॊख्मा से अनुऩात ऻात कीजजए।
(A) 59 : 45 (B) 45 : 59 (C) 53 : 22
(D) 16 : 53 (E) None of these

67. Number of student in 10th class in P batch is what percent less


than number of student in 11th class in Q batch?
P फैच भें 10वीॊ कऺा भें ववद्माचथषमों की सॊख्मा Q फैच भें 11वीॊ कऺा के ववद्माचथषमों
की सॊख्मा से ककतना प्रनतित कभ है?
(A) 35% (B) 30% (C) 40%
(D) 25% (E) None of these

68. If number of students of class 12th in Q batch is increased by


53⅓% and number of students in class 9th in P batch is increased by
80% then find the difference between students in these classes.
मदद Q फैच भें कऺा 12वीॊ के छात्रों की सॊख्मा भें 53⅓% की वरवि की जाती है औय P
फैच भें कऺा 9वीॊ के छात्रों की सॊख्मा भें 80% की ववर ि कीhttps://instagra
जाती है, तो इन कऺाओॊ भें
https://youtu https ://ww
be.com/chan w.fa cebook.
m.com/aashish https://
arorasocial(?)
nel/UCYa4_Jr com/a ashis utm_medium = t.me/st
196 Orf8R5Kz2uO
ha rorasocial copy_link
tccXQ udified
छात्रों के फीच का अॊतय ऻात कीजजए।
(A) 176 (B) 172 (C) 186
(D) 185 (E) None of these

69. Find the average number of students in P batch.


P फैच भें छात्रों की औसत सॊख्मा ऻात कीजजए।
(A) 110 (B) 100 (C) 90
(D) 80 (E) None of these

70. If fees of P batch is 25% more than Q batch of each student and
fees of Q batch of each student is Rs. 1800, then find the fees
received by P batch of 9th class.
मदद P फैच की पीस प्रत्मे क छात्र के Q फैच से 25% अचधक है औय प्रत्मे क छात्र के Q
फैच की पीस रु. 1800 है, तो 9वीॊ कऺा के P फैच द्वाया प्राप्त पीस ऻात कीजजए।
(A) Rs. 102430 (B) Rs. 106540 (C) Rs. 102450
(D) Rs. 101250 (E) None of these

(71 – 75) Number of students in SSC coaching in morning and evening


batch is 1850 and 1650. Number of students in Bank coaching in
morning and evening batch are 150 and 600 less than number of
student in SSC coaching in respective batches. Number of student in
UPSC coaching in morning and evening batch are 2150 and 2200.
Number of students in GATE coaching in morning batch is 145 less
than number of student in UPSC in morning batch whereas number of
student in evening batch of GATE is 26.67% more than number of
student in BANK coaching in evening batch. Number of student in CAT
coaching in morning batch is 85 more than number of students in
GATE coaching in morning batch and number of students in evening
batch of CAT coaching is 18.18% more than number of students in
evening batch of UPSC coaching.
(71 – 75) सुफह औय िाभ के फैच भें SSC कोचच ॊग भें छात्रों की सॊख्मा 1850 औय
1650 है। फैंक कोचच ॊग भें सुफह औय िाभ फैच भें छात्रों की सॊख्मा सॊफॊचधत फैच भें SSC
कोचच ॊग भें छात्रों की सॊख्मा से 150 औय 600 कभ है। UPSC कोचच ॊग भें सुफह औय
िाभ फैच भें छात्रों की सॊख्मा 2150 औय 2200 है। सुफह के फैच भें GATE कोचच ॊग भें
छात्रों की सॊख्मा UPSC भें सुफह के फैच भें छात्रों की सॊख्मा से 145 कभ है जफकक
GATE के िाभ के फैच भें छात्रों की सॊख्मा िाभ के फैच भें फैंक कोचच ॊग भें छात्रों की
सॊख्मा से 26.67% अचधक है। सुफह फैच भें CAT कोचच ॊग भें छात्रों की सॊख्मा सुफह फैच
भें GATE कोचच ॊग भें छात्रों की सॊख्मा से 85 अचधक है औय CAT कोचच ॊग के िाभ फैच
भें छात्रों की सॊख्मा UPSC कोचच ॊग के िाभ फैच भें छात्रों की सॊख्मा से 18.18% अचधक
है।

71. Find the ratio of number of students in morning batch of CAT


coaching with the number of student in morninghttps://instagra
batch of UPSC
https://youtu https ://ww
be.com/chan w.fa cebook.
m.com/aashish https://
arorasocial(?)
nel/UCYa4_Jr com/a ashis utm_medium = t.me/st
197 Orf8R5Kz2uO
ha rorasocial copy_link
tccXQ udified
coaching.
CAT कोचच ॊग के सुफह के फैच भें छात्रों की सॊख्मा का UPSC कोचच ॊग के सुफह के फैच
भें छात्रों की सॊख्मा से अनुऩात ऻात कीजजए।
(A) 209 : 147 (B) 209 : 215 (C) 215 : 209
(D) 147 : 209 (E) None of these

72. Total Number of student in UPSC coaching is what percent more


than total number of students in Bank coaching?
UPSC कोचच ॊग भें छात्रों की कुर सॊख्मा, फैंक कोचच ॊग भें छात्रों की कुर सॊख्मा से
ककतना प्रनतित अचधक है?
(A) 58.18% (B) 57.284% (C) 55.255%
(D) 52% (E) None of these

73. Find the average of number of students in evening batch of SSC,


UPSC and GATE.
SSC, UPSC औय GATE के िाभ के फैच भें छात्रों की सॊख्मा का औसत ऻात
कीजजए।
(A) 1878 1/3 (B) 1456 2/3 (C) 1854 1/3
(D) 1726 2/3 (E) None of these

74. If 40% percent of student preparing for CAT cleared the exam and
37% of the student cleared the SSC exam then find the difference
between the number of student who do not clear the exam in those
coaching.
मदद CAT की तैमायी कयने वारे 40% प्रनतित छात्र ने ऩयीऺा उत्तीणष की है औय 37%
छात्र ने SSC ऩयीऺा उत्तीणष की है, तो उन कोचच ॊग भें ऩयीऺा भें उत्तीणष नहीॊ होने वारे
छात्रों की सॊख्मा के फीच का अॊतय ऻात कीजजए।
(A) 616 (B) 609 (C) 709
(D) 516 (E) None of these

75. Find the average number of student in morning batch.


सुफह के फैच भें छात्रों की औसत सॊख्मा ऻात कीजजए।
(A) 1959 (B) 1854 (C) 1545
(D) 1895 (E) None of these

(76 – 80) Below data is regarding number of Dettol and Lifebuoy soaps
sold by a shop in three months August, September and October. The
ratio of number of Dettol soap to Lifebuoy soap sold in three months
together is 1 : 2 and number of Lifebuoy soap sold is 58.33% of total
Dettol and Lifebuoy soaps sold in October. The number of Lifebuoy
soaps sold in August is twice the number of Dettol soaps sold in
October and number of Dettol soaps sold in August is 180% of the
number of Lifebuoy soaps sold in September. The number of Dettol
https://instagra
https://youtu https ://ww
be.com/chan w.fa cebook.
m.com/aashish https://
arorasocial(?)
nel/UCYa4_Jr com/a ashis utm_medium = t.me/st
198 Orf8R5Kz2uO
ha rorasocial copy_link
tccXQ udified
soaps sold in September is 85 less than number of Dettol soaps sold
in August. The number of Lifebuoy soaps sold in September is half of
number of Dettol soaps sold in October.
(76 – 80) नीचे ददए गए डे टा अगस्त, शसतॊफय औय अक्टूफय तीन भहीनों भें एक दुक ान
द्वाया फे चे गए डे टॉर औय राइपफॉम साफुन की सॊख्मा के फाये भें हैं । तीन भहीनों भें फे चे
गए डे टॉर साफुन का राइपफॉम साफुन से अनुऩात 1 : 2 है औय अक्टूफय भें फे चे गए
राइपफॉम साफुन की सॊख्मा फे चे गए डे टॉर औय राइपफॉम साफुन की कुर सॊख्मा का
58.3% है। अगस्त भें फे चे गए राइपफॉम साफुनों की सॊख्मा अक्टूफय भें फे चे गए डे टॉर
साफुनों की सॊख्मा का दोगुना है औय अगस्त भें फे चे गए डे टॉर साफुनों की सॊख्मा शसतॊफय
भें फे चे गए राइपफॉम साफुनों की सॊख्मा का 180 प्रनतित है। शसतॊफय भें फे चे गए डे टॉर
साफुनों की सॊख्मा अगस्त भें फे चे गए डे टॉर साफुनों की सॊख्मा से 85 कभ है। शसतॊफय भें
फे चे गए राइपफॉम साफुनों की सॊख्मा अक्टूफय भें फे चे गए डे टॉर साफुनों की सॊख्मा से
आधी है।

76. What is the ratio of number of Dettol soaps sold to number of


Lifebuoy soaps sold in August?
अगस्त भें फे चे गए डे टॉर साफुनों की सॊख्मा का फे चे गए राइपफॉम साफुनों की सॊख्मा से
अनुऩात ककतना है?
(A) 15 : 31 (B) 31 : 15 (C) 9 : 20
(D) 46 : 15 (E) None of these

77. The number of Lifebuoy soaps sold in August and September


together is what % of the number of Dettol soaps sold in September
and October together?
अगस्त औय शसतॊफय भें शभराकय फे चे गए राइपफॉम साफुन की सॊख्मा शसतॊफय औय
अक्टूफय भें शभराकय फे चे गए डे टॉर साफुनों की सॊख्मा का ककतना प्रनतित है?
(A) 263.40% (B) 233.33% (C) 263.04%
(D) 238.09% (E) None of these

78. What is the average number of Dettol soaps sold in all three
months together?
तीनों भहीनों भें शभराकय फे चे गए डे टॉर साफुनों की औसत सॊख्मा ककतनी है?
(A) 68 (B) 65 (C) 57
(D) 50 (E) None of these

79. What is the difference between soaps sold in September and


soaps sold in October?
शसतॊफय भें फे चे गए साफुन औय अक्टूफय भें फे चे गए साफुन के फीच क्मा अॊतय है ?
(A) 186 (B) 185 (C) 198
(D) 195 (E) None of these

80. The number of Lifebuoy soaps sold in October is what % more or


less than the number of Dettol soaps sold in August?
https://instagra
https://youtu https ://ww
be.com/chan w.fa cebook.
m.com/aashish https://
arorasocial(?)
nel/UCYa4_Jr com/a ashis utm_medium = t.me/st
199 Orf8R5Kz2uO
ha rorasocial copy_link
tccXQ udified
अक्टूफय भें फे चे गए राइपफॉम साफुन की सॊख्मा अगस्त भें फे चे गए डे टॉर साफुन की
सॊख्मा से ककतने प्रनतित अचधक मा कभ है?
(A) 25.55% (B) 33.33% (C) 55.55%
(D) 50.05% (E) None of these

(81 – 85) There are 400 products manufactured by factory A in which


number of toothpaste and toothbrush manufactured in ratio 5 : 3 and
the number of Colgate and Pepsodent toothbrush manufactured in ratio
2 : 3. In factory B, total number of Colgate and Pepsodent toothbrush
manufactured are 24 and 84 respectively, while the total number of
toothpaste and toothbrush manufactured is in ratio 11 : 12. In factory
C the number of toothpaste manufactured is 150 and the number of
toothbrush manufactured is 50% more than the number of toothbrush
manufactured by factory B whereas the number of Pepsodent and
Colgate toothbrush manufactured is in the ratio 5 : 4.
(81 – 85) पैक्ट्री A द्वाया ननशभषत 400 उत्ऩाद हैं जजसभें 5 : 3 के अनुऩात भें ननशभषत
टूथऩे स्ट औय टूथब्ि की सॊख्मा औय 2 : 3 के अनुऩात भें ननशभषत कोरगेट औय ऩे प्सोडेंट
टूथब्ि की सॊख्मा है। पैक्ट्री B भें ननशभषत कोरगेट औय ऩे प्सोडेंट टूथब्ि की कुर सॊ
ख्मा क्रभि् 24 औय 84 है, जफकक ननशभषत टूथऩे स्ट औय टूथब्ि की कु र सॊख्मा 11 :
12 के अनुऩात भें है। पैक्ट्री C भें ननशभषत टूथऩे स्ट की सॊख्मा 150 है औय ननशभषत
टूथब्ि की सॊख्मा पै क्ट्री B द्वाया ननशभषत टूथब्ि की सॊख्मा से 50% अचधक है जफकक
ननशभषत ऩे प्सोडेंट औय कोरगेट टूथब्ि की सॊख्मा 5 : 4 के अनुऩात भें है।

81. What is the overall ratio of Pepsodent and Colgate toothbrush


manufactured by all three factories together?
तीनों कायखानों द्वाया एक साथ ननशभषत ऩे प्सोडेंट औय कोरगेट टूथब्ि का अनुऩात
ककतना है?
(A) 31 : 15 (B) 11 : 33 (C) 16 : 15
(D) 22 : 13 (E) None of these

82. The total number of toothpaste manufactured in all 3 factories is


what % more or less than total number of Pepsodent toothbrush
manufactured in all these 3 factories?
सबी 3 कायखानों भें ननशभषत टूथऩे स्ट की कु र सॊख्मा इन सबी 3 कायखानों भें ननशभषत
ऩे प्सोडेंट टूथब्ि की कुर सॊख्मा से ककतने प्रनतित अचधक मा कभ है?
(A) 88.79% (B) 89.01% (C) 88.65%
(D) 90% (E) None of these

83. Find the average number of toothbrush manufactured in factory B


and C together.
पैक्ट्री B औय C भें शभराकय ननशभषत टूथब्ि की औसत सॊख्मा ऻात कीजजए।
(A) 123 (B) 135 (C) 156
(D) 165 (E) None of these https://instagra
https://youtu https ://ww
be.com/chan w.fa cebook.
m.com/aashish https://
arorasocial(?)
nel/UCYa4_Jr com/a ashis utm_medium = t.me/st
200 Orf8R5Kz2uO
ha rorasocial copy_link
tccXQ udified
84. Find the total products produced in factory B.
पैक्ट्री B भें उत्ऩाददत कुर उत्ऩाद ऻात कीजजए।
(A) 207 (B) 206 (C) 208
(D) 220 (E) None of these

85. What is the difference between the average number of Pepsodent


toothbrush manufactured in factory A and C and the total number of
toothpaste manufactured in factory A?
पैक्ट्री A औय C भें ननशभषत ऩे प्सोडेंट टूथब्ि की औसत सॊख्मा औय पैक्ट्री A भें ननशभषत
टूथऩे स्ट की कु र सॊख्मा के फीच का अॊतय ककतना है?
(A) 189 (B) 180 (C) 169
(D) 160 (E) None of these

(86 – 90) Amit saves 34% of his income and spend remaining Rs.
19800. Monthly incomes of Bablu and Chandu are in the ratio 6 : 5
respectively. Chandu saves Rs. 4400 more than Amit and spend 40%
less than Amit. Respective ratio of expenditure and savings of Bablu is
2 : 1. Monthly income of Deshmukh is Rs. 5000 less than monthly
income of Chandu and monthly saving is Rs. 400 less than that of
Amit.
(86 – 90) अशभत अऩनी आम का 34% फचाता है औय िे र्ष रु. 19800 खचष कयता है।
फफरू औय चॊद ू की भाशसक आम क्रभि् 6 : 5 के अनुऩात भें है। चॊद ू अशभत से 4400
रुऩमे अचधक फचाता है औय अशभत से 40% कभ खचष कयता है। फफरू के व्मम औय
फचत का क्रभि् अनुऩात 2 : 1 है। दे िभुख की भाशसक आम चॊद ू की भाशसक आम से
5000 रु. कभ है औय भाशसक फचत अशभत से 400 रु. कभ है।

86. What is the difference between monthly income of Bablu and


monthly savings of Chandu and Deshmukh together?
फफरू की भाशसक आम औय चॊद ू औय दे िभुख की भाशसक फचत के फीच का अॊतय
ककतना है?
(A) 7736 (B) 7366 (C) 7367
(D) 7376 (E) None of these

87. What is the average of monthly expenditure of Amit, Bablu and


Chandu?
अशभत, फफरू औय चॊद ू के भाशसक खचष का औसत ककतना है?
(A) 17621.59 (B) 17261.59 (C) 17162.59
(D) 17621.33 (E) None of these

88. What is the difference between monthly savings of Amit and


Bablu?
अशभत औय फफरू की भाशसक फचत भें ककतना अॊतय है?
https://youtu https://instagra
https ://ww
be.com/chan w.fa cebook.
m.com/aashish https://
arorasocial(?)
nel/UCYa4_Jr com/a ashis utm_medium = t.me/st
201 Orf8R5Kz2uO
ha rorasocial copy_link
tccXQ udified
(A) 323 (B) 343 (C) 356
(D) 392 (E) None of these

89. What is the ratio of monthly income of Amit to monthly expenditure


of Bablu and Chandu together?
अशभत की भाशसक आम का फफरू औय चॊद ू के भाशसक खचष से अनुऩात ककतना है?
(A) 3670 : 4313 (B) 3570 : 4311 (C) 3650 : 4133
(D) 3750 : 4133 (E) None of these

90. Amit and Chandu saves approximately what % of their monthly


income?
अशभत औय चॊद ू अऩनी भाशसक आम का रगबग ककतना प्रनतित फचाते हैं ?
(A) 45% (B) 43.90% (C) 46%
(D) 43.75% (E) None of these

(91 – 95) Following data is about the population increase of two towns
A,B in three years 2018, 2019 and 2020. If the present population of
town A is 12825 which increased by 12.5% and 14% from the last 2
yeaRs. The present day population of town A is 125% more than the
population of town B in year 2019 and the population of town B in
year 2020 is increased by 45% with respect to last year and the
present day (2020) population of town B is 5 times of its population
that was in year 2018.
(91 – 95) ननम्नशरणखत डे टा तीन सार 2018, 2019 औय 2020 भें दो िहयों A, B
की जनसॊख्मा ववर ि के फाये भें है। मदद िहय A की वतषभान जनसॊख्मा 12825 है जो
वऩछरे 2 वर्षों से 12.5% ​औय 14% की वरवि हुई है। िहय A की वतषभान जनसॊख्मा वर्षष
2019 भें िहय B की जनसॊख्मा से 125% अचधक है औय वर्षष 2020 भें िहय B की
जनसॊख्मा भें वऩछरे वर्षष औय वतषभान ददन (2020) िहय B की जनसॊख्मा के सॊफॊध भें
45% की वरवि हुई है। िहय B की वतषभान जनसॉख्मा 2018 की जनसॉख्मा का 5 गुना
है।

91. What is the present day population of town B?


िहय B की वतषभान जनसॊख्मा ककतनी है?
(A) 8841 (B) 8265 (C) 8648
(D) 8175 (E) None of these

92. Find the % increase of population of town B from 2018 to 2020.


2018 से 2020 तक िहय B की जनसॊख्मा भें % ववर ि ऻात कीजजए।
(A) 630% (B) 400% (C) 300%
(D) 500% (E) None of these

93. Find the ratio of population of town A in 2019 and town B in year
2018. https://instagra
https://youtu https ://ww
be.com/chan w.fa cebook.
m.com/aashish https://
arorasocial(?)
nel/UCYa4_Jr com/a ashis utm_medium = t.me/st
202 Orf8R5Kz2uO
ha rorasocial copy_link
tccXQ udified
2019 भें िहय A औय वर्षष 2018 भें िहय B की जनसॊख्मा का अनुऩात ऻात कीजजए।
(A) 200 : 229 (B) 29 : 200 (C) 200 : 29
(D) 229 : 200 (E) None of these

94. Total population of town A over 3 years is what % of the total


population of town B over 3 years?
3 वर्षों भें िहय A की कुर जनसॊख्मा, 3 वर्षों भें िहय B की कुर जनसॊख्मा का ककतना
प्रनतित है?
(A) 219.13% (B) 219.99% (C) 215.65%
(D) 219.35% (E) None of these

95. Find the total present population of town A and B together.


िहय A औय B की कुर वतषभान जनसॊख्मा ऻात कीजजए।
(A) 21090 (B) 33040 (C) 15050
(D) 15028 (E) None of these

(96 – 100) Only two tribes X and Y live in an island. The population of
X is 200% more than that of Y. In the island, the ratio of males to
females is 23 : 9 and the population of females in Y tribe is 40% less
than that of males and in tribe X, the ratio of male to female
population is 3 : 1. The total number of male population in Y tribe is
250.
(96 – 100) एक द्वीऩ भें केवर दो जनजानतमाॉ X औय Y यहती हैं । X की जनसॊख्मा Y
की तुरना भें 200% अचधक है। द्वीऩ भें , ऩुरुर्षों का भदहराओॊ से अनुऩात 23 : 9 औय
Y जनजानत भें भदहराओॊ की जनसॊख्मा ऩुरुर्षों की जनसॊख्मा की तुरना भें 40% कभ है
औय X जनजानत भें , ऩुरुर्षों औय भदहरा जनसॊख्मा का अनुऩात 3 : 1 है। Y जनजानत
भें ऩुरुर्ष जनसॊख्मा की कुर सॊख्मा 250 है।

96. What is the ratio of the total population of the island to the total
female population of the Y tribe?
द्वीऩ की कुर जनसॊख्मा का Y जनजानत की कुर भदहरा जनसॊख्मा से अनुऩात ककतना
है?
(A) 32.3% (B) 11 : 31 (C) 1 : 15
(D) 22 : 13 (E) None of these

97. What is the total number of male population in the island?


द्वीऩ भें ऩुरुर्ष जनसॊख्मा की कु र सॊख्मा ककतनी है?
(A) 1500 (B) 1150 (C) 1520
(D) 1510 (E) None of these

98. In the island, the total female population is how much more or less
than that of male population?
द्वीऩ भें कु र भदहरा जनसॊख्मा ऩुरुर्ष जनसॊख्मा से ककतनी https://instagra
अचधक मा कभ है?
https://youtu https ://ww
be.com/chan w.fa cebook.
m.com/aashish https://
arorasocial(?)
nel/UCYa4_Jr com/a ashis utm_medium = t.me/st
203 Orf8R5Kz2uO
ha rorasocial copy_link
tccXQ udified
(A) 710 (B) 620 (C) 700
(D) 640 (E) None of these

99. 13% of the total population of the island is below 18 then find the
number of people who are above 18.
द्वीऩ की कु र जनसॊख्मा का 13% 18 वर्षष से कभ है, तो 18 वर्षष से अचधक आमु के
व्मजक्तमों की सॊख्मा ऻात कीजजए।
(A) 1207 (B) 1026 (C) 1208
(D) 1392 (E) None of these

100. The total number of female population in X tribe is how much %


more or less than female population of Y tribe?
X जनजानत भें भदहरा जनसॊख्मा की कुर सॊख्मा, Y जनजानत की भदहरा जनसॊख्मा से
ककतना% अचधक मा कभ है?
(A) 100% (B) 180% (C) 169%
(D) 160% (E) None of these

(101 – 105) Elections were held in a village in two rounds. 3


candidates (A, B and C) were selected for the elections. In first round
A got 320 votes which is 60% more than the number of votes B got in
the same round. The number of votes C got in 2nd round is 100 more
than number of votes A got in 1st round and the ratio of number of
votes A, B and C got in 2nd round is 5 : 6 : 7 respectively. Same
number of votes casted in both the rounds.
(101 – 105) एक गाॊव भें दो याउॊ ड भें चनु ाव हुए। चनु ाव के शरए 3 उम्भीदवायों (A, B
औय C) का चमन ककमा गमा था। ऩहरे याउॊ ड भें A को 320 वोट शभरे जो कक उसी
याउॊ ड भें B को शभरे वोटों की सॊख्मा से 60% अचधक है। दस ू ये याउॊ ड भें C को शभरे
वोटों की सॊख्मा, ऩहरे याउॊ ड भें A को शभरे वोटों की सॊख्मा से 100 अचधक है औय दस ू ये
याउॊ ड भें A, B औय C को शभरे वोटों की सॊख्मा का अनुऩात क्रभि् 5 : 6 : 7 है।
दोनों याउॊ ड भें सभान वोट ऩडे ।

101.What is the total number of votes casted in both rounds together?


दोनों याउॊ ड भें एक साथ डारे गए वोटों की कुर सॊख्मा ककतनी है?
(A) 2160 (B) 2480 (C) 2520
(D) 2820 (E) None of these

102. Which candidate won the elections?


चन
ु ाव ककस उम्भीदवाय ने जीता?
(A) B (B) C (C) A
(D) CND (E) None of these

103. If the total number of votes casted in both round is 60% of the
total population of the village then find the totalhttps://instagra
number of people who
https://youtu https ://ww
be.com/chan w.fa cebook.
m.com/aashish https://
arorasocial(?)
nel/UCYa4_Jr com/a ashis utm_medium = t.me/st
204 Orf8R5Kz2uO
ha rorasocial copy_link
tccXQ udified
did not cast their votes in the election.
मदद दोनों याउॊ ड भें डारे गए वोटों की कुर सॊख्मा गाॉव की कुर आफादी का 60% है, तो
चुनाव भें वोट नहीॊ डारने वारे रोगों की कु र सॊख्मा ऻात कीजजए।
(A) 1710 (B) 1140 (C) 1440
(D) 1640 (E) None of these

104. What is the average number of votes A and B got in both the
rounds together?
A औय B को दोनों याउॊ ड भें शभराकय शभरे वोटों की औसत सॊख्मा ककतनी है ?
(A) 590 (B) 575 (C) 580
(D) 680 (E) None of these

105. The number of votes that winning candidate got is what % of the
total number of votes that both of the losing candidates got together?
जीतने वारे उम्भीदवाय को शभरे वोटों की सॊख्मा, दोनों हायने वारे उम्भीदवायों को शभरे
वोटों की कु र सॊख्मा का ककतना प्रनतित है?
(A) 82% (B) 84.05% (C) 86%
(D) 83.05% (E) None of these

(106 – 110) A bank launched three schemes A,B and C. Only in


scheme B and C have some people enrolled commonly. The total
number of males and females enrolled in scheme A only are 400 and
160 respectively. The number of males and females enrolled in
scheme B only are 580 and 140 respectively. The number of males
and females enrolled in scheme C only are 640 and 440 respectively.
The number of males and females enrolled in both schemes B and C
are 340 and 260 respectively.
(106 – 110) एक फैंक ने तीन मोजनाएॊ A, B औय C िुरू की। के वर मोजना B औय
C भें कुछ रोगों ने साभान्म रूऩ से नाभाॊककत ककमा है। मोजना A भें नाभाॊककत ऩुरुर्षों
औय भदहराओॊ की कु र सॊख्मा क्रभि् 400 औय 160 है। केवर मोजना B भें नाभाॊककत
ऩुरुर्षों औय भदहराओॊ की सॊख्मा क्रभि् 580 औय 140 है। केवर मोजना C भें
नाभाॊककत ऩुरुर्षों औय भदहराओॊ की सॊख्मा क्रभि् 640 औय 440 है। दोनों मोजनाओॊ B
औय C भें नाभाॊककत ऩुरुर्षों औय भदहराओॊ की सॊख्मा क्रभि् 340 औय 260 है।

106. Find the sum of total number of females enrolled in scheme A


and number of males enrolled in scheme B together.
मोजना A भें नाभाॊककत भदहराओॊ की कु र सॊख्मा औय मोजना B भें नाभाॊककत ऩुरुर्षों की
कुर सॊख्मा का मोग ऻात कीजजए।
(A) 1080 (B) 1280 (C) 1020
(D) 1820 (E) None of these

107. What is the ratio between total number of males enrolled in


scheme A and total number of females enrolledhttps://instagra
in scheme B?
https://youtu https ://ww
be.com/chan w.fa cebook.
m.com/aashish https://
arorasocial(?)
nel/UCYa4_Jr com/a ashis utm_medium = t.me/st
205 Orf8R5Kz2uO
ha rorasocial copy_link
tccXQ udified
मोजना A भें नाभाॊककत ऩुरुर्षों की कु र सॊख्मा औय मोजना B भें नाभाॊककत भदहराओॊ की
कुर सॊख्मा के फीच का अनुऩात ककतना है?
(A) 2 : 3 (B) 3 : 2 (C) 1 : !
(D) CND (E) None of these

108. What is the difference between the total number of males and
females enrolled in scheme C?
मोजना C भें नाभाॊककत ऩुरुर्षों औय भदहराओॊ की कु र सॊख्मा के फीच का अॊतय ककतना
है?
(A) 171 (B) 114 (C) 144
(D) 280 (E) None of these

109. Total number of males enrolled in scheme C is what % of the


total number of people enrolled in scheme A?
मोजना C भें नाभाॊककत ऩुरुर्षों की कुर सॊख्मा, मोजना A भें नाभाॊककत रोगों की कुर
सॊख्मा का ककतना प्रनतित है?
(A) 190% (B) 175% (C) 180%
(D) 160% (E) None of these

110. The total number of people enrolled in scheme C is what % more


or less than the total number of people enrolled in scheme B?
मोजना C भें नाभाॊककत रोगों की कुर सॊख्मा, मोजना B भें नाभाॊककत रोगों की कुर
सॊख्मा से ककतने प्रनतित अचधक मा कभ है?
A) 27.27% (B) 24.05% (C) 26%
(D) 23.05% (E) None of these

(111 – 115) In three offices A, B and C different number of males and


female are there. Total number of employees in A is 3360. the number
of male in C is 25% of the total number of employee in A. ratio of
number of males and females in B is 17 : 15. number of males in A is
46% more than number of females in B and number of females in A is
18.18% more than number of males in A. total number of feamles in
all the offices is 3830.
(111 – 115) तीन कामाषरमों A, B औय C भें ऩुरुर्षों औय भदहराओॊ की अरग – अरग
सॊख्मा है। A भें कभषचारयमों की कुर सॊख्मा 3360 है। C भें ऩुरुर्षों की सॊख्मा A भें
कभषचारयमों की कु र सॊख्मा का 25% है। 8 भें ऩुरुर्षों औय भदहराओॊ की सॊख्मा का
अनुऩात 17 : 15 है। A भें ऩुरुर्षों की सॊख्मा भें भदहराओॊ की सॊख्मा से 462/3 है%
अचधक औय A भें भदहराओॊ की सॊख्मा A भें भदहराओॊ की सॊख्मा A भें ऩुरुर्षों की सॊख्मा
से 18.18% अचधक है। सबी कामाषरमों भें भदहराओॊ की कु र सॊख्मा 3830 है।

111. Find the ratio of number of males in B and C with the number of
female in A.
B औय C भें ऩुरुर्षों की सॊख्मा का A भें भदहराओॊ की सॊख्मा से अनुऩात ऻात कीजजए।
https://instagra
https://youtu https ://ww
be.com/chan w.fa cebook.
m.com/aashish https://
arorasocial(?)
nel/UCYa4_Jr com/a ashis utm_medium = t.me/st
206 Orf8R5Kz2uO
ha rorasocial copy_link
tccXQ udified
(A) 182 : 203 (B) 203 : 182 (C) 215 : 209
(D) 147 : 209 (E) None of these

112. Total number of employees in Office B is what percent more than


number of males in C?
कामाषरम B भें कभषचारयमों की कु र सॊख्मा, C भें ऩुरुर्षों की सॊख्मा से ककतने प्रनतित
अचधक है?
(A) 163(2/3) % (B) 164(2/3) % (C) 165(2/3) %
(D) 166(2/3) % (E) None of these

113. If number of females of office B increase by 53¹% and number of


males of B increases by 28.56% then find the new number of
employees in B.
मदद कामाषरम B की भदहराओॊ की सॊख्मा भें 531/3% की वरवि होती है औय B के ऩुरुर्षों
की सॊख्मा भें 28.56% की ववर ि होती है, तो B भें कभषचारयमों की नई सॊख्मा ऻात
कीजजए।
(A) 3340 (B) 3140 (C) 3240
(D) 3040 (E) None of these

114. If 35% of the males of office A and 55% of the females of office
A are married then find the ratio of the number of unmarried males to
the number of unmarried females in office A.
मदद कामाषरम A के 35% ऩुरुर्ष औय कामाषरम A की 55% भदहराएॊ वववादहत हैं , तो
कामाषरम A भें अवववादहत ऩुरुर्षों की सॊख्मा का अवववादहत भदहराओॊ की सॊख्मा से
अनुऩात ऻात कीजजए।
(A) 9 : 11 (B) 11 : 9 (C) 99 : 169
(D) 169 : 99 (E) None of these

115. Find the difference between total number of males in all offices
and total number of females in all offices.
सबी कामाषरमों भें ऩुरुर्षों की कुर सॊख्मा औय सबी कामाषरमों भें भदहराओॊ की कुर
सॊख्मा के फीच का अॊतय ऻात कीजजए।
(A) 280 (B) 254 (C) 250
(D) 260 (E) None of these

(116 – 120) Two bikes Bullet and R15 were sold by 4 different
showrooms on Diwali. Number of Bullet sold by A is 28.56% more than
number of R15 sold by A. number of Bullet sold by D is 58 1/3% more
than number of R15 sold by B. Ratio of number of R15 sold by B. nd
number of bullet sold by B is 12 : 17. Number of R15 sold by B is
1440 which is 20% more than number of R15 sold by C. Number of
bullet sold by c is 53 1/3% more than number of R15 sold by C.
Number of bikes sold by A and D is 1648 and https://instagra
4840.
https://youtu https ://ww
be.com/chan w.fa cebook.
m.com/aashish https://
arorasocial(?)
nel/UCYa4_Jr com/a ashis utm_medium = t.me/st
207 Orf8R5Kz2uO
ha rorasocial copy_link
tccXQ udified
(116 – 120) ददवारी ऩय दो फाइक Bullet औय R15 को 4 अरग – अरग िोरूभ भें
फे चा गमा. A द्वाया फे ची गई फुरेट की सॊख्मा, A द्वाया फे ची गई R15 की सॊख्मा से
28.56% अचधक है। Dद्वायो फे ची गई फॎरेट की सॊख्मा, B द्वाया फे ची गई R15 की
सॊख्मा से 581/3% अचधक है। B द्वाया फे ची गई R15 को सॊख्मा औय फुरेट की सॊख्मा
का अनुऩात B द्वाया फे चा गमा 12 : 17 है। B द्वाया फे ची गई R15 की सॊख्मा 1440°
है जो C द्वाया फे ची गई R15 की सॊख्मा से 20% अचधक है। द्वाया फे ची गई फुरेट की
सॊख्मा C द्वाया फे ची गई R15 की सॊख्मा से 531/3% अचधक है। A औय D द्वाया फे ची
गई फाइक की सॊख्मा है 1648 औय 48401 है।

116. Number of bullet sold by B is what percent less than number of


R15 sold by D?
B द्वाया फे ची गई फुरेट की सॊख्मा, D द्वाया फे ची गई R15 की सॊख्मा से ककतने
प्रनतित कभ है?
(A) 20.125% (B) 20.3225% (C) 120.3125%
(D) 16.3125% (E) None of these

117. Find the ratio of number of bullet sold by A to the number of R15
sold by B.
A द्वाया फे ची गई फुरेट की सॊख्मा का B द्वाया फे ची गई R15 की सॊख्मा से अनुऩात
ऻात कीजजए।
(A) 103 : 120 (B) 103 : 140 (C) 103 : 180
(D) 103 : 160 (E) None of these

118. If price of Bullet is 2. 5lacs which is 25% more than the price of
R15, then find the difference between revenue generated by selling
R15 and Bullet by D.
मदद फुरेट की कीभत 2.5 राख है जो कक R15 की कीभत से 25% अचधक है, तो R15
औय फुरेट को D द्वाया फे चने से उत्ऩन्न याजस्व के फीच का अॊतय ऻात कीजजए।
(A) 560 Lacs (B) 580 Lacs (C) 540 Lacs
(D) 530 Lacs (E) None of these

119. Find the difference of number of Bullets sold by A and B between


number of R15 sold by C and D.
C औय D द्वाया फे ची गई R15 की सॊख्मा के फीच A औय B द्वाया फे ची गई फुरेट की
सॊख्मा का अॊतय ऻात कीजजए।
(A) 793 (B) 893 (C) 563
(D) 773 (E) None of these

120. Find the average number of bullet sold by all the showrooms.
सबी िोरूभों द्वाया फे ची गई फुरेट की औसत सॊख्मा ऻात कीजजए।
(A) 1772.3125 (B) 1776.25 (C) 1771.5
(D) 1774.5 (E) None of these
https://youtu https://instagra
https ://ww
be.com/chan w.fa cebook.
m.com/aashish https://
arorasocial(?)
nel/UCYa4_Jr com/a ashis utm_medium = t.me/st
208 Orf8R5Kz2uO
ha rorasocial copy_link
tccXQ udified
(121 – 125) Number of employees of 4 companies is 5160. Male and
female in company P is in the ratio 15 : 8. male population of
company P and S is 1600. 41 2/3% of the employees of R are males
and rest 420 employees are females. Total employees in company P
and Sis 3190 and difference is 490 (employee in P> employee in S) .
60% of company Q employees are male.
(121 – 125) 4 कॊऩननमों के कभषचारयमों की सॊख्मा 5160 है। कॊऩनी P भें ऩुरुर्ष औय
भदहरा का अनुऩात 15 : 8 है। कॊऩनी P औय S की ऩुरुर्ष जनसॊख्मा 1600 है। R के
कभषचारयमों भें से 41 2/3% ऩुरुर्ष हैं औय िे र्ष 420 कभषचायी भदहराएॊ हैं । कॊऩनी P औय
S भें कु र कभषचायी 3190 है औय अॊतय 490 है (P भें कभषचायी > 5 भें कभषचायी)।
कॊऩनी Q के 60% कभषचायी ऩुरुर्ष हैं ।

121. Find the ratio of number of Males in Company R to the number


of females in company S.
कॊऩनी R भें ऩुरुर्षों की सॊख्मा का कॊऩनी S भें भदहराओॊ की सॊख्मा से अनुऩात ऻात
कीजजए।
(A) 6 : 19 (B) 6 : 13 (C) 15 : 19
(D) 5 : 17 (E) None of these

122. Number of males in P are how much percent more than the
number of females in P?
P भें ऩुरुर्षों की सॊख्मा, P भें भदहराओॊ की सॊख्मा से ककतने प्रनतित अचधक है?
(A) 85.5% (B) 85% (C) 87.5%
(D) 75% (E) None of these

123. If salary of females is 25% more than that of males which is Rs.
24000 per month, then find the total monthly salary of employees of
company Q.
मदद भदहराओॊ का वे तन ऩुरुर्षों की तुरना भें 25% अचधक है जो की प्रनत भाह 24000
है, तो कॊऩनी Q के कभषचारयमों का कु र प्रनत भाह वे तन ऻात कीजजए।
(A) 3.6 cr. (B) 3.3 cr. (C) 4.4 cr.
(D) 4.6 cr. (E) None of these

124. If number of males in R is increased by 58 – % and females in R


is decreased by 28.56% then find the difference between the number
of males and females in R.
मदद R भें ऩुरुर्षों की सॊख्मा भें 581/3% की वरवि की जाती है औय R भें भदहराओॊ की
सॊख्मा भें 28.56% की कभी की जाती है, तो R भें ऩुरुर्षों औय भदहराओॊ की सॊख्मा के
फीच का अॊतय ऻात कीजजए।
(A) 175 (B) 150 (C) 4 140
(D) 120 (E) None of these

125. Find the average number of females in all https://instagra


the companies.
https://youtu https ://ww
be.com/chan w.fa cebook.
m.com/aashish https://
arorasocial(?)
nel/UCYa4_Jr com/a ashis utm_medium = t.me/st
209 Orf8R5Kz2uO
ha rorasocial copy_link
tccXQ udified
सबी कॊऩननमों भें भदहराओॊ की औसत सॊख्मा ऻात कीजजए।
(A) 627.5 (B) 352 (C) 624.5
(D) 654.5 (E) None of these

(126 – 130) There are four companies producing shirts and t – shirts.
Ratio of number of shirts produced by B and number of t – shirts
produced by D is 5 : 8. and ratio of number of shirts produced by D
to the number of t – shirts produced by A is 11 : 8. Number of shirts
and t – shirts produced by D is 880 and 640. Number of shirts
produced by C is 480 which is 14.28% less than the number of t –
shirts produced by C. Number of shirts produced by B is 25% more
than number of t – shirts produced by B. Total number of shirts
produced by all the companies is 3180.
(126 – 130) िटष औय टी – िटष फनाने वारी चाय कॊऩननमाॊ हैं । B द्वाया उत्ऩाददत िट्षस
की सॊख्मा औय D द्वाया उत्ऩाददत टी – िटों की सॊख्मा का अनुऩात 5 : 8 है औय D
द्वाया उत्ऩाददत िट्षस की सॊख्मा का A द्वाया उत्ऩाददत टी – िटों की सॊख्मा से अनुऩात
11 : 8 है। D द्वाया उत्ऩाददत िटों औय टी – िटों की सॊख्मा 880 औय 640 है। C
द्वाया उत्ऩाददत िट्षस की सॊख्मा 480 है जो C द्वाया उत्ऩाददत टी – िटों की सॊख्मा से
14.28% कभ है। B द्वाया उत्ऩाददत िटषस की सॊख्मा, B द्वाया उत्ऩाददत टी – िटष की
सॊख्मा से 25% अचधक है। सबी कॊऩननमों द्वाया उत्ऩाददत िटों की कुर सॊख्मा 3180
है।

126. Find the ratio of shirts produced by D with the number of t –


shirts produced by C.
D द्वाया उत्ऩाददत िटों का C द्वाया उत्ऩाददत टी – िटों की सॊख्मा से अनुऩात ऻात
कीजजए।
(A) 11 : 4 (B) 11 : 7 (C) 7 : 11
(D) 11 : 9 (E) None of these

127. Number of t – shirts sold by D is how much percent more than


the number of shirts sold by C?
D द्वाया फे ची गई टी – िटों की सॊख्मा, C द्वाया फे ची गई िटों की सॊख्मा से ककतना
प्रनतित अचधक है?
(A) 38% (B) 36% (C) 40.40%
(D) 33.33% (E) None of these

128. If number of shirts produced by E is 46 2/3% more than the


number of shirts produced by A and number of t – shirts produced by
E is 31.25% less than number of T – shirts produced by D, then find
number of shirts are what percent more than number of t – shirts
produced by E?
मदद E द्वाया उत्ऩाददत कभीजों की सॊख्मा A द्वाया उत्ऩाददत कभीजों की सॊख्मा से 46
2/3% अचधक है औय E दवाया उत्ऩाददत टी – िटो की सॊख्https://instagra
मा D दद्वाया उत्ऩाददत
https://youtu https ://ww
be.com/chan w.fa cebook.
m.com/aashish https://
arorasocial(?)
nel/UCYa4_Jr com/a ashis utm_medium = t.me/st
210 Orf8R5Kz2uO
ha rorasocial copy_link
tccXQ udified
टी – िटो की सॊख्मा से 31.25% कभ है, तो िटष की सॊख्मा ऻात कीजजए द्वाया
उत्ऩाददत टी – िटो की सॊख्मा से ककतना प्रनतित अचधक है?
(A) 40% (B) 50% (C) 60%
(D) 70% (E) None of these

129. Find the difference between number of shirts sold by A and B


and number of t – shirts sold by B and C.
A औय B द्वाया फे ची गई िटष की सॊख्मा औय B औय C द्वाया फे ची गई टी – िटष की
सॊख्मा के फीच का अॊतय ऻात कीजजए।
(A) 50 (B) 80 (C) 60
(D) 20 (E) None of these

130. which company produced exactly 16 2/3% more t – shirts than


number of shirts?
ककस कॊऩनी ने िटष की सॊख्मा से ठीक 16 2/3% अचधक की – िटष का उत्ऩादन ककमा?
(A) A (B) B (C) C
(D) D (E) None of these

(131 – 135) The average number of AC and cooler sold by P and S is


2520. The ratio of number of Cooler and AC sold by P is 11 : 9 and
number of total AC and Cooler sold by S is 760 more than total items
sold by P. The number of Cooler sold by Q is 41 2/3% more than
number of AC sold by Q. number of coolers sold by R is 27.27% more
than number of coolers sold by P and number of AC sold by R is
22.22% more than the number of AC sold by P. total items sold by Q
is 1682. total number of coolers sold by all companies is 5161.
(131 – 135) P औय S द्वाया फे चे गए AC औय कूरय की औसत सॊख्मा 2520 है। P
द्वाया फे चे गए कू रय औय AC की सॊख्मा का अनुऩात 11 : 9 है औय S द्वाया फे चे गए
कुर AC औय कूरय की सॊख्मो, P द्वाया फे ची गई कुर वस्तुओ ॊ से 760 अचधक है।
द्वाया फे चे गए कू रय की सॊख्मा Q द्वाया फे चे गए AC की सॊख्मा से 41 2/3% अचधक
है। R द्वाया फे चे गए कूरय की सॊख्मा, Pद्वाया फे चे गए कूरय की सॊख्मा से 27.27%
अचधक है औय R दे वाया फे चे गए AC की सॊख्मा P द्वायो फे चे गए AC की सॊख्मा से
22.22% अचधक है। Q दव ु ाया फे चीॊ गई कुर वस्तुएॉ 1682 हैं । सबी कॊऩननमों द्वाया फे चे
गए कू रय की कु र सॊख्मा 5161 हैं .।

131. Find the ratio of number of Cooler sold by P with the number of
AC sold by S.
P द्वाया फे चे गए कूरय की सॊख्मा का S द्वाया फे चे गए AC की सॊख्मा से अनुऩात
ऻात कीजजए।
(A) 1400 : 1177 (B) 1177 : 1400 (C) 1577 : 1155
(D) 1155 : 1577 (E) None of these

132. Number of AC sold by P is what percent less than number of


https://instagra
https://youtu https ://ww
be.com/chan w.fa cebook.
m.com/aashish https://
arorasocial(?)
nel/UCYa4_Jr com/a ashis utm_medium = t.me/st
211 Orf8R5Kz2uO
ha rorasocial copy_link
tccXQ udified
cooler sold by S?
P द्वाया फे चे गए AC की सॊख्मा, S द्वाया फे चे गए कूरय की सॊख्मा से ककतने प्रनतित
कभ है?
(A) 35.8% (B) 32.4% (C) 44.5%
(D) 25.8% (E) None of these

133. In summer sales of Coolers of R increased by 28.56% and sale


of AC is decreased by 45.45% then find the total items sold by R.
गशभषमों भें R के कूरय की त्रफक्री भें 28.56% की वरवि हुई औय AC की त्रफक्री भें
45.45% की कभी आई, तो R द्वाया फे ची गई कु र वस्तुओ ॊ का ऩता रगाएॊ।
(A) 2768 (B) 2728 (C) 2568
(D) 2858 (E) None of these

134. Find the difference between number of items sold by Q and R.


Q औय R द्वाया फे ची गई वस्तुओ ॊ की सॊख्मा के फीच का अॊतय ऻात कीजजए।
(A) 993 (B) 995 (C) 793
(D) 893 (E) None of these

135. Find the average number of AC sold by all the shopkeepeRs.


सबी दकु ानदायों द्वाया फे चे गए एसी की औसत सॊख्मा ऻात कीजजए।
(A) 1055 (B) 1057 (C) 1059
(D) 1048 (E) None of these

(136 – 140) A survey is conducted in four cities to find out the number
of person like Samsung and l phone. Number of persons who like
Samsung in city A is 73 1/3% more than the number of person like I –
phone in city D. Number of person who like Samsung in city D is 26
2/3% less than the number of persons who like I – phone in city D.
Ratio of number of people who like Samsung to the number of people
who like I – phone in city B is 7 : 8. Total number of people like
Samsung and I – phone in City D is 936 which is 73 1/3% more than
number of persons in city B. Number of people like I – phone in City
C is 456 and total number of persons in all cities who like Samsung
and I – phone is 2008 and 1650.
(136 – 140) सैभसॊग औय आई – पोन ऩसॊद कयने वारे व्मजक्तमों की सॊख्मा का ऩता
रगाने के शरए चाय िहयों भें एक सवे ऺण ककमा जाता है। िहय A भें सैभसॊग ऩसॊद ू
कयने वारे व्मजक्तमों की सॊख्मा, िहय D भें 1 – पोन ऩसॊद ू कयने वारे व्मजक्तमों की
सॊख्मा से 73 1/3% अचधक है। िहय भें सैभसॊग ऩसॊद कयने वारे व्मजक्तमों की सॊख्मा
उन व्मजक्तमों की सॊख्मा से 26 2/3% कभ है, जो िहय D भें आई – पोन ऩसॊद कयते
हैं । सैभसॊग ऩसॊद कयने वारों की सॊख्मा का िहय B भें 1 – फोन ऩसॊद कयने वारों की
सॊख्मा से अनुऩात 7 : 8 है। िहय भें सैभसॊग औय आई – फोन ऩसॊद कयने वारे रोगों
की कुर सॊख्मा 936 है जो की िहय B भें व्मजक्तमों की सॊख्मा से 73 1/3% अचधक है।
िहय भें 1 – फोन ऩसॊद कयने वारे रोगों की सॊख्मा 456 हैhttps://instagra
औय सबी िहयों भें
https://youtu https ://ww
be.com/chan w.fa cebook.
m.com/aashish https://
arorasocial(?)
nel/UCYa4_Jr com/a ashis utm_medium = t.me/st
212 Orf8R5Kz2uO
ha rorasocial copy_link
tccXQ udified
व्मजक्तमों की कु र सॊख्मा जो सैभसॊग औय आई – फोन को ऩसॊद कयते हैं , वे 2008 औय
1650 हैं ।

136. Find the ratio of number of people like I – phone in city D with
the number of people like Samsung in city B.
िहय D भें I – फोन ऩसॊद कयने वारे व्मजक्तमों की सॊख्मा का िहय B भें सैभसॊग ऩसॊद
कयने वारे व्मजक्तमों की सॊख्मा से अनुऩात ऻात कीजजए।
(A) 15 : 7 (B) 8 : 15 (C) 12 : 17
(D) 17 : 12 (E) None of these

137. Number of people like Samsung in city A are what percent more
than the number of people like I – phone in city B?
िहय A भें सैभसॊग ऩसॊद कयने वारे रोगों की सॊख्मा, िहय B भें I – पोन ऩसॊद कयने
वारे रोगों की सॊख्मा से ककतने प्रनतित अचधक है ?
(A) 235% (B) 230% (C) 240%
(D) 225% (E) None of these

138. Due to increase in price of I – phone 25% of the people who like
I – phone in city C moved to Samsung the find the percentage of
people who like Samsung now in city C.
I – पोन की कीभत भें वरवि के कायण िहय C भें I – पोन ऩसॊद कयने वारे 25% रोग
सैभसॊग भें चरे गए, िहय C भें अफ सैभसॊग को ऩसॊद कयने वारे रोगों का प्रनतित
ऻात कीजजए।
(A) 51.1% (B) 64.7% (C) 61.1%
(D) 47.5% (E) None of these

139. if in city F number of people like I – phone are 41 2/3% more


than the number of people like I phone in city B and number of
people like Samsung in city F is 28.56% more than the number of
people like Samsung in city B, then find the number of people in city
F.
मदद िहय F भें 1 – फोन ऩसॊद कयने वारे रोगों की सॊख्मा िहय B भें 1 – फोन ऩसॊद
कयने वारे रोगों की सॊख्मा से 41 2/3% अचधक है औय िहय F भें सैभसॊग ऩसॊद कयने
वारे रोगों की सॊख्मा िहय B भें सैभसॊग रोगों की सॊख्मा से 28.56% अचधक है, तो
िहय F भें रोगों की सॊख्मा ऻात कीजजए।
(A) 710 (B) 750 (C) 732
(D) 722 (E) None of these

140. Find the average number of people like I – phone in all the cities.
सबी िहयों भें I – phone ऩसॊद कयने वारे रोगों की औसत सॊख्मा ऻात कीजजए।
(A) 412.5 (B) 432.5 (C) 465.25
(D) 400.75 (E) None of these
https://youtu https://instagra
https ://ww
be.com/chan w.fa cebook.
m.com/aashish https://
arorasocial(?)
nel/UCYa4_Jr com/a ashis utm_medium = t.me/st
213 Orf8R5Kz2uO
ha rorasocial copy_link
tccXQ udified
(141 – 145) Anushka has decided to start a new company and she
wants to buy some furniture – table, chair, AC. She also wants to buy
a few things which will include monitor, CPU, keyboard and mouse
and the cost of each table is 1.5 times the cost of a mouse and the
cost of chair is 3/10 times of the cost of a table. The cost of an AC is
13 times the cost of a chair. The cost of a monitor is 260% more than
the cost of a chair. A CPU costs Rs. 1500 more than a keyboard and
the keyboard costs 1.5 times a mouse. The cost of a mouse is Rs.
2000.
(141 – 145) अनुष्का ने एक नई कॊ ऩनी िुरू कयने का पै सरा ककमा है औय वह कु छ
पनीचय – टे फर, कुसी, एसी खयीदना चाहती है। वह कुछ चीजें बी खयीदना चाहती है
जजसभें भॉननटय, सीऩीमू, कीफोडष औय भाउस िाशभर हैं औय प्रत्मे क टे फर की रागत एक
भाउस की रागत का 1.5 गुना है औय कुसी की रागत एक टे फर की रागत का 3/10
गुना है। एक एसी की कीभत एक कु सी की कीभत का 13 गुना है। एक भॉननटय की
कीभत एक कुसी की कीभत से 260% अचधक है। एक सीऩीमू की कीभत एक कीफोडष से
रु. 1500 अचधक औय कीफोडष की कीभत एक भाउस से 1.5 गुना अचधक है। एक भाउस
की कीभत रु. 2000 है।

141. What will be the cost of 2 monitors and 3 CPU’s together?


2 भॉननटय औय 3 CPU की एकसाथ कीभत क्मा होगी?
(A) Rs. 19980 (B) Rs. 19890 (C) Rs. 18890
(D) Rs. 18980 (E) None of these

142. What is the ratio of the cost of a keyboard to the total cost of a
chair and table together?
एक कीफोडष की रागत का एक कु सी औय टे फर की कु र रागत से अनुऩात ककतना है?
(A) 10 : 13 (B) 11 : 10 (C) 14 : 25
(D) CND (E) None of these

143. 142. What is the ratio of the cost of a keyboard to the total cost
of a chair and table together?
एक कीफोडष की रागत का एक कु सी औय टे फर की कु र रागत से अनुऩात ककतना है?
(A) Rs. 17570 (B) Rs. 17750 (C) 1 Rs. 18510
(D) Rs. 17850 (E) None of these

144. The total price of a chair and keyboard together is what % of the
cost of a mouse and a CPU together?
एक कुसी औय कीफोडष की कुर कीभत एक भाउस औय एक सीऩीमू की कुर कीभत का
ककतना प्रनतित है?
(A) 59% (B) 57 (C) 60%
(D) 69% (E) None of these

145. What is the total cost of the whole items bought by Anushka?
https://instagra
https://youtu https ://ww
be.com/chan w.fa cebook.
m.com/aashish https://
arorasocial(?)
nel/UCYa4_Jr com/a ashis utm_medium = t.me/st
214 Orf8R5Kz2uO
ha rorasocial copy_link
tccXQ udified
अनुष्का द्वाया खयीदी गई सॊऩूणष वस्तुओ ॊ की कु र रागत ककतनी है?
(A) Rs. 27440 (B) Rs. 24700 (C) Rs. 27400
(D) Rs. 28340 (E) None of these

(146 – 150) On Diwali, Ayush wanted to decorate his house and also
wanted to buy some electronic items for his new home. He bought
lamps, lights, Oven, AC and Refrigerator. He bought a lamp for Rs.
5000. The oven was marked at a price 120% more than the price of
lamp but due to Diwali he got a special discount of 12%. The marked
price of AC is 35% more than the price at which he bought oven and
he got a special discount of 25% on it. He bought the refrigerator for
Rs. 14000 and the price of lights is 10% of the marked price of oven.
(146 – 150) दीवारी ऩय आमुर्ष अऩने घय को सजाना चाहता था औय अऩने नए घय के
शरए कु छ इरे क्ट्रॉननक साभान बी खयीदना चाहता था। उन्होंने रैंऩ , राइट, ओवन, AC
औय किज खयीदा। उसने 5000 रुऩमे भें एक रैंऩ खयीदा। ओवन को रैंऩ की कीभत से
120% अचधक भूल्म ऩय अॊककत ककमा गमा था रे ककन ददवारी के कायण उसे 12% की
वविे र्ष छू ट शभरी। AC का अॊककत भूल्म उस भूल्म से 35% अचधक है जजस ऩय उसने
ओवन खयीदा औय उसे उस ऩय 25% की वविे र्ष छूट शभरी। उसने ये किजये टय 14000
रुऩमे भें खयीदा। औय राइट्स की कीभत ओवन के अॊककत भूल्म का 10% है।

146. What is the price at which Ayush bought the AC?


आमुर्ष ने ककस कीभत ऩय AC खयीदा?
(A) 9160 (B) 9480 (C) 9801
(D) 9801 (E) None of these

147. What is the ratio of marked price of AC and the price at which
he bought the oven?
AC के अॊककत भल्
ू म औय उस भल्ू म का अनुऩात क्मा है जजस ऩय उसने ओवन खयीदा?
(A) 20 : 27 (B) 27 : 20 (C) 35 : 17
(D) 81 : 80 (E) None of these

148. How much Ayush spent on lights?


आमुर्ष ने राइट्स ऩय ककतना खचष ककमा?
(A) 1100 (B) 1140 (C) 1440
(D) 1640 (E) None of these

149. If due to discount, he bought the refrigerator for Rs. 12500 then
find the discount %.
मदद छू ट के कायण, उसने ये किजये टय 12500 रुऩमे भें खयीदा। तो छू ट% ऻात कीजजए।
(A) 10.33% (B) 10.69% (C) 10.54%
(D) 10.71% (E) None of these

150. What is the total cost of the whole items bought by Ayush?
https://instagra
https://youtu https ://ww
be.com/chan w.fa cebook.
m.com/aashish https://
arorasocial(?)
nel/UCYa4_Jr com/a ashis utm_medium = t.me/st
215 Orf8R5Kz2uO
ha rorasocial copy_link
tccXQ udified
आमुर्ष द्वाया खयीदी गई सॊऩूणष वस्तुओ ॊ की कु र रागत ककतनी है
(A) Rs. 39158 (B) Rs. 39185 (C) Rs. 39581
(D) Rs. 39518 (E) None of these

https://youtu https://instagra
https ://ww
be.com/chan w.fa cebook.
m.com/aashish https://
arorasocial(?)
nel/UCYa4_Jr com/a ashis utm_medium = t.me/st
216 Orf8R5Kz2uO
ha rorasocial copy_link
tccXQ udified
(1 – 5)

1) Ans. (A)
Boys who get only 1 type of item = 108 + 72 + 144 = 324

2) Ans. (C)
Girls who get only 2 type of item = 60 + 120 + 48 = 228

3) Ans. (D)
Students who get all three types of item = 36 + 60 = 96

4) Ans. (B)
Boys who get only Toffee : Girls who get only Toffee = 72 : 108 = 2 : 3

5) Ans. (D)
Girls who get only 1 type of item = 132 + 108 + 72 = 312

(6 – 10)
Class  8th 9th 10th 11th 12th Total
Boys 204 135 153 171 159 822
Girls 156 225 117 140 190 828
Total 360 360 270 311 349 1650

6) Ans. (D)
Students in class 8th – Students in class 11th = 360 – 311 = 49

7) Ans. (C)
% of number of Students in class 9 with Students in class 10
= 360 × 100/270 = 133.33% https://instagra
https://youtu https ://ww
be.com/chan w.fa cebook.
m.com/aashish https://
arorasocial(?)
nel/UCYa4_Jr com/a ashis utm_medium = t.me/st
217 Orf8R5Kz2uO
ha rorasocial copy_link
tccXQ udified
8) Ans. (D)
Boys in class 10th, 11th & 12th = 153 + 171 + 159 = 483
Girls in Class 8th, 9th & 12th = 156 + 225 + 190 = 571
Difference = 571 – 483 = 88

9) Ans. (B)
Boys in class 9th : Girls in class 11th = 135 : 140 = 27 : 28

10) Ans. (D)


New Total of Girls in all classes = 828 + 0.25 × 156 = 828 + 39 = 867

(11 – 15)
Month  May June July Total
Hyundai 150 190 120 460
Toyota 120 160 148 428
Maruti Suzuki 154 120 110 384
Total 424 470 378 1272

11) Ans. (D)


Total Cars sold in July = 378

12) Ans. (A)


Total cars sold in May are less than total cars sold in June by
= (470 – 424) × 100/470 = 9.78%

13) Ans. (C)


Total earning of Toyota cars = 8 × 428 = 3424 lakh

14) Ans. (B)


Average number of Maruti Suzuki cars sold = 384/3 = 128

15) Ans. (B)


In May, Earning of Hyundai cars – Earning of Maruti Suzuki cars
= 7 × 150 – 4 × 154 = 1050 – 616 = 434 lakh

https://youtu https://instagra
https ://ww
be.com/chan w.fa cebook.
m.com/aashish https://
arorasocial(?)
218
nel/UCYa4_Jr
Orf8R5Kz2uO
com/a ashis utm_medium = t.me/st
ha rorasocial copy_link
tccXQ udified
(16 – 20)

16) Ans. (D)


People who have both Hero & Bajaj Bikes = 106 + 318 = 424

17) Ans. (C)


Who have all Bikes : Who have both Hero & TVS but not Bajaj
= 318 : 184 = 159 : 92

18) Ans. (B)


Who have only TVS – Who have only Bajaj = 994 – 144 = 850

19) Ans. (D)


% of people who have both Hero & Bajaj bikes with people who have Bajaj
bikes = 424 × 100/630 = 67.3%

20) Ans. (B)


Person who have only 2 bikes = 184 + 106 + 62 = 352

(21 – 25)

https://youtu https://instagra
https ://ww
be.com/chan w.fa cebook.
m.com/aashish https://
arorasocial(?)
nel/UCYa4_Jr com/a ashis utm_medium = t.me/st
219 Orf8R5Kz2uO
ha rorasocial copy_link
tccXQ udified
21) Ans. (B)
Who bought watches of only C = 10

22) Ans. (D)


Average of customers who bought watches of only A & C = (40 + 10)/2 = 25

23) Ans. (A)


Customers who bought watches of at least 2 inventors
= 30 + 90 + 140 + 60 = 320

24) Ans. (B)


Customers who bought watches of B, are more than of customers who
bought watches of A & B but not C by = (350 – 30) × 100/30 = 1066.66%

25) Ans. (A)


Customers who bought watches of only B – only C = 170 – 10 = 160

(26 – 30)
Male Female Total Doctors Engineers
City A 400 240 640 320 320
City B 400 150 550 350 200
City C 440 210 650 490 160
Total 1240 600 1840 1160 680

26) Ans. (B)


Males : Females = 1240 : 600 = 31 : 15

27) Ans. (C)


% of Doctors in all cities = 1160 × 100/1840 = 63.04%

28) Ans. (A)


Females in City A & B = 240 + 150 = 390
Males in city B & C = 400 + 440 = 840
Female in city A & B are less than Males in city B & C by
= (840 – 390) × 100/840 = 53.57%

https://youtu https://instagra
https ://ww
be.com/chan w.fa cebook.
m.com/aashish https://
arorasocial(?)
220
nel/UCYa4_Jr
Orf8R5Kz2uO
com/a ashis utm_medium = t.me/st
ha rorasocial copy_link
tccXQ udified
29) Ans. (B)
Average Numbers of Engineers in all cities together = 680/3 = 226.66

30) Ans. (D)


Average Numbers of Engineers in City A & C = (320 + 160)/2 = 480/3 = 240
Doctors in City A – Average Engineers in City A & C = 320 – 240 = 80

(31 – 35)

Chocolates
Product  Ice Cream Total
Kit - Kat 5 Star
A 120 140 40 300
B 100 80 50 230
C 150 91 65 306
Total 370 311 155 836

31) Ans. (A)


Total Kit - Kat Chocolates produced : Total 5 Star Chocolates produced
= 311 : 155

32) Ans. (A)


Ice Creams produced are more than Kit - Kat Chocolates produced by
= (370 – 311) × 100/311 = 18.97%

33) Ans. (B)


Average Chocolates produced in, Factory B & Factory C
= [(80 + 50) + (91 + 65)]/2 = (130 + 156)/2 = 143

34) Ans. (B)


Total product produced in Factory C = 306

35) Ans. (A)


Average Kit - Kat Chocolates produced in Factory A & C – Ice Creams
produced in Factory A = 120 – (140 + 91)/2 = 120 – 115.5 = 4.5

https://youtu https://instagra
https ://ww
be.com/chan w.fa cebook.
m.com/aashish https://
arorasocial(?)
nel/UCYa4_Jr com/a ashis utm_medium = t.me/st
221 Orf8R5Kz2uO
ha rorasocial copy_link
tccXQ udified
(36 – 40)
Student  P Q R S Total
English 75 60 90 55 280
Hindi 65 64 71 70 270
Total 140 124 161 125 550
% 70 62 80.5 62.5

36) Ans. (B)


Average of % of All students = (70 + 62 + 80.5 + 62.5)/4 = 275/4 = 68.75%

37) Ans. (C)


Marks scored by P & Q in English = 75 + 60 = 135
Marks scored by R & S in Hindi = 71 + 70 = 141
Marks scored by P & Q in English is less than Marks scored by R & S in
Hindi by = (141 – 135) × 100/141 = 4.25%

38) Ans. (C)


Total Marks of T = 55 × 15/11 + 64 × 0.75 = 75 + 48 = 123
% of T = 123 × 100/200 = 61.5%

39) Ans. (D)


Maximum % is scored by = R

40) Ans. (A)


Marks of P in Math = 65 × 14/13 = 70, Total marks = 140 + 70 = 210
% of P = 210 × 100/300 = 70%

(41 – 45)
Company  P Q R S T Total
TV 78 66 70 46 88 348
Fridge 54 77 80 100 72 383
Total 132 143 150 146 160 731

41) Ans. (C)


Average Fridge sold by All companies = 383/5 = 76.6
https://youtu https://instagra
https ://ww
be.com/chan w.fa cebook.
m.com/aashish https://
arorasocial(?)
nel/UCYa4_Jr com/a ashis utm_medium = t.me/st
222 Orf8R5Kz2uO
ha rorasocial copy_link
tccXQ udified
42) Ans. (D)
Average TV sold by All companies = 348/5 = 69.6
TV sold by Company R – Average TV sold by all companies = 70 – 69.6 = 0.4

43) Ans. (A)


Average Fridge sold = 76.6
Fridge sold more than Average by Company = P & T

44) Ans. (C)


Total items sold by Company S is more than Total TV sold by Company Q
by = (146 – 66) × 100/66 = 121.21%

45) Ans. (B)


Fridge sold by Company (P, R & T) : TV sold by Company (Q & S)
= (54 + 80 + 72) : (66 + 46) = 206 : 112 = 103 : 56

(46 – 50)

46) Ans. (A)


People who speak Hindi = 83

47) Ans. (B)


People who speak only English : People who speak none of the three
language = 35 : 8

48) Ans. (B)


People who speak at least 2 language = 30 + 35 + 11 + 24 = 100

https://youtu https://instagra
https ://ww
be.com/chan w.fa cebook.
m.com/aashish https://
arorasocial(?)
nel/UCYa4_Jr com/a ashis utm_medium = t.me/st
223 Orf8R5Kz2uO
ha rorasocial copy_link
tccXQ udified
49) Ans. (C)
People who speak only Hindi : People who speak only Spanish
= 18 : 40 = 9 : 20

50) Ans. (B)


People who do not speak Hindi language = 35 + 25 + 40 + 8 = 108

(51 – 55)
Branch  ME CS EC Civil Total
Boys 72 46 54 68 240
Girls 44 42 46 48 180
Total 116 88 100 116 420

51) Ans. (D)


Boys in (ME, EC & CS) = 72 + 54 + 46 = 172,
Girls in (EC + CS + Civil) = 46 + 42 + 48 = 136
Boys in ME, EC & CS are more than Girls in EC, CS & Civil by
= (172 – 136) × 100/136 = 26.47%

52) Ans. (B)


Students in (ME + Civil) : Students (CS + EC)
= (116 + 116) : (88 + 100) = 232 : 188 = 58 : 47

53) Ans. (C)


Next year Students in CS = 46 × 1.5 + 42 × 6/7 = 69 + 36 = 105
Students in ME = 1.25 × 116 = 145
Students in ME are more than Students in CS by
= (145 – 105) × 100/105 = 38.09%

54) Ans. (C)


EC

55) Ans. (D)


Boys in EC & Civil – Girls in CS & ME = (54 + 68) – (44 + 42) = 122 – 86 = 36

https://youtu https://instagra
https ://ww
be.com/chan w.fa cebook.
m.com/aashish https://
arorasocial(?)
nel/UCYa4_Jr com/a ashis utm_medium = t.me/st
224 Orf8R5Kz2uO
ha rorasocial copy_link
tccXQ udified
(56 – 60)

56) Ans. (D)


% of Students who like C with Students who like A
= 1602 × 100/1150 = 139.3%

57) Ans. (B)


Who like A & C : Who like A & B
= (400 + 252) : (126 + 252) = 652 : 378 = 326 : 189

58) Ans. (C)


Students who like exactly 2 chocolates = 126 + 500 + 400 = 1026

59) Ans. (C)


Students who like C are more than Students who like B by
= (450 – 300) × 100/300 = 50%

60) Ans. (B)


Students who like only 1 chocolate = 15.5 + 12.5 + 18.75 = 46.75%

(61 – 65)
Person  A B C D Total
2018 38000 7000 16000 8000 69000
2019 24000 15000 12000 11000 62000
Total 62000 22000 28000 19000 131000

62) Ans. (A)


Investment of C in 2019 : Investment https
https://youtu in 2018 =https://instagra
of B://ww 12000 : 7000 = 12 : 7
be.com/chan w.fa cebook.
m.com/aashish https://
arorasocial(?)
nel/UCYa4_Jr
Orf8R5Kz2uO
com/a ashis utm_medium = t.me/st
225 tccXQ
ha rorasocial copy_link
udified
62) Ans. (B)
Investment of A in 2018 is more than Investment of B in 2019
= (38000 – 15000) × 100/15000 = 153.33%

63) Ans. (C)


In 2018, Investment of D – Investment of B
= 8000 × 1.125 – 7000 × 6/7 = 9000 – 6000 = 3000 Rs.

64) Ans. (A)


Investment of A, B & D in 2018 – Investment of A, B & C in 2019
= (38000 + 7000 + 8000) – (24000 + 15000 + 12000) = 53000 – 51000 = 2000

65) Ans. (D)


Average investment by All the person in 2018 = 69000/5 = 13800

(66 – 70)
Class  9th 10th 11th 12th Total
Batch P 45 35 135 105 320
Batch Q 135 200 100 165 600
Total 180 235 235 270 920

66) Ans. (D)


In Batch P (9th + 10th) : Batch Q (11th + 12th) = (45 + 35) : (100 + 165) = 80 :
265 = 16 : 53

67) Ans. (A)


% of Students in 10th class of batch P with Students in 11th class of batch Q
= 35 × 100/100 = 35%

68) Ans. (B)


Students in 12th class of batch Q – Students in 9th class of batch P
= 165 × 23/15 – 45 × 1.8 = 253 – 81 = 172

69) Ans. (D)


Average students in batch P = 320/4 = 80

https://youtu https://instagra
https ://ww
be.com/chan w.fa cebook.
m.com/aashish https://
arorasocial(?)
nel/UCYa4_Jr com/a ashis utm_medium = t.me/st
226 Orf8R5Kz2uO
ha rorasocial copy_link
tccXQ udified
70) Ans. (D)
Fees received by Batch P of 9th class = 45 × 1.25 × 1800 = 101250 Rs.

(71 – 75)
Coaching  SSC Bank UPSC GATE CAT Total
Morning 1850 1700 2150 2005 2090 9795
Evening 1650 1050 2200 1330 2600 8830
Total 3500 2750 4350 3335 4690 18625

71) Ans. (B)


Student in morning batch in CAT coaching : Students in morning batch in
UPSC coaching = 2090 : 2150 = 209 : 215

72) Ans. (A)


Total students in UPSC coaching are more than Total students in Bank
coaching by = (4350 – 2750) × 100/2750 = 58.18%

73) Ans. (D)


Average students in SSC, UPSC & GATE coaching in Evening batch
= (1650 + 2200 + 1330)/3 = 5180/3 = 1726 2/3

74) Ans. (B)


Failed students in, CAT exam – SSC exam
= 4690 × 0.6 – 3500 × 0.63 = 2814 – 2205 = 609

75) Ans. (A)


Average students in Morning Batch = 9795/5 = 1959

(76 – 80)
Month  August September October Total
Dettol 90 5 100 195
Lifebuoy 200 50 140 390
Total 290 55 240 585

76) Ans. (C)


In August, Dettol soaps sold : Lifebuoy soaps soldhttps://instagra
= 90 : 200 = 9 : 20
https://youtu https ://ww
be.com/chan w.fa cebook.
m.com/aashish https://
arorasocial(?)
nel/UCYa4_Jr com/a ashis utm_medium = t.me/st
227 Orf8R5Kz2uO
ha rorasocial copy_link
tccXQ udified
77) Ans. (D)
Lifebuoy soaps sold in (August + September) = 200 + 50 = 250
Dettol soaps sold in (September + October) = 5 + 100 = 105
% of Lifebuoy sold in August & September with Dettol soaps sold in
September & October = 250 × 100/105 = 238.09%

78) Ans. (B)


Average Dettol soaps sold in Three months = 195/3 = 65

79) Ans. (B)


Soaps sold in October – Soaps sold in September = 240 – 55 = 185

80) Ans. (C)


Lifebuoy soaps sold in October are more than Dettol soaps sold in August
by = (140 – 90) × 100/90 = 55.55%

(81 – 85)

Toothbrush
Product Toothpaste Total
Colgate Pepsodent
A 250 60 90 400
B 99 24 84 207
C 150 72 90 312
Total 499 156 264 919

81) Ans. (D)


Pepsodent toothbrush : Colgate toothbrush = 264 : 156 = 22 : 13

82) Ans. (B)


Toothpaste manufacture in all factories are more than Pepsodent
toothbrush manufactured in all factories by
= (499 – 264) × 100/264 = 89.01%

83) Ans. (B)


Average toothbrush in, Factory B & Factory C
= [(24 + 84) + (72 + 90)]/2 = (108 + 162)/2 = 135
https://youtu https://instagra
https ://ww
be.com/chan w.fa cebook.
m.com/aashish https://
arorasocial(?)
nel/UCYa4_Jr com/a ashis utm_medium = t.me/st
228 Orf8R5Kz2uO
ha rorasocial copy_link
tccXQ udified
84) Ans. (A)
Total product produced in Factory B = 207

85) Ans. (D)


Average Pepsodent toothbrush manufactured in Factory A & C –
Toothpaste manufactured in Factory A = 250 – (90 + 90)/2 = 250 – 90 = 160

(86 – 90)
Person  Amit Bablu Chandu Deshmukh Total
Income 30000 31776 26480 21480 109736
Expenditure 19800 21184 11880 11680 64544
Saving 10200 10592 14600 9800 45192

86) Ans. (D)


Income of Bablu – Saving of Chandu & Deshmukh
= 31776 – (14600 + 9800) = 7376

87) Ans. (D)


Average of Expenditure of Amit, Bablu & Chandu
= (19800 + 21184 + 11880)/3 = 17621.33

88) Ans. (D)


Saving of Bablu – Saving of Amit = 10592 – 10200 = 392

89) Ans. (D)


Income of Amit : Expenditure (Bablu + Chandu)
= 30000 : (21184 + 11880) = 3750 : 4133

90) Ans. (B)


Income of Amit & Chandu = 30000 + 26480 = 56480
Saving of Amit & Chandu = 10200 + 14600 = 24800
% of Saving of Amit & Chandu = 24800 × 100/56480 = 43.90%

https://youtu https://instagra
https ://ww
be.com/chan w.fa cebook.
m.com/aashish https://
arorasocial(?)
nel/UCYa4_Jr com/a ashis utm_medium = t.me/st
229 Orf8R5Kz2uO
ha rorasocial copy_link
tccXQ udified
(91 – 95)
Year  2018 2019 2020 Total
A 10000 11400 12825 34225
B 1653 5700 8265 15618
Total 11653 17100 21090 49843

91) Ans. (B)


Present population of Town B = 8265

92) Ans. (B)


Increase of population of Town B from 2018 to 2020
= (8265 – 1653) × 100/1653 = 400%

93) Ans. (C)


Population of Town A in 2019 : Population of Town B in 2018
= 11400 : 1653 = 200 : 29

94) Ans. (A)


% of population of Town A with Population of Town B in all three year
= 34225 × 100/15618 = 219.13%

95) Ans. (A)


At Present, Total population of Town A & Town B = 21090

(96 – 100)
Tribe  X Y Total
Male 900 250 1150
Female 300 150 450
Total 1200 400 1600

96) Ans. (A)


Total Population of Island : Female Population of Tribe Y
= 1600 : 150 = 32 : 3

97) Ans. (B)


Total Male population in the Island = 1150 https://instagra
https://youtu https ://ww
be.com/chan w.fa cebook.
m.com/aashish https://
arorasocial(?)
nel/UCYa4_Jr com/a ashis utm_medium = t.me/st
230 Orf8R5Kz2uO
ha rorasocial copy_link
tccXQ udified
98) Ans. (C)
Female population is less than Male population by = 1150 – 450 = 700

99) Ans. (D)


People above age 18 = 0.87 × 1600 = 1392

100) Ans. (A)


Female population in Tribe X is more than female population in Tribe Y by
= (300 – 150) × 100/150 = 100%

(101 – 105)
Candidate  A B C Total
Round 1 320 200 560 1080
Round 2 300 360 420 1080
Total 620 560 980 2160

101) Ans. (A)


Total votes casted in both rounds = 2160

102) Ans. (B)


Candidate, who won the election = C

103) Ans. (C)


Number of people who don’t cast their votes = 2160 × 40/60 = 1440

104) Ans. (A)


Average of Total votes of A & B in both round = (620 + 560)/2 = 590

105) Ans. (D)


% of votes of winning candidate with total votes of both losing candidates
= 980 × 100/(620 + 560) = 83.05%

https://youtu https://instagra
https ://ww
be.com/chan w.fa cebook.
m.com/aashish https://
arorasocial(?)
nel/UCYa4_Jr com/a ashis utm_medium = t.me/st
231 Orf8R5Kz2uO
ha rorasocial copy_link
tccXQ udified
(106 – 110)
Scheme  A Only B Only C B & C both Total
Males 400 580 640 340 1960
Females 160 140 440 260 1000
Total 560 720 1080 600 2960

106) Ans. (A)


Females enrolled in Scheme A + Males enrolled in Scheme B
= 160 + (580 + 340) = 1080

107) Ans. (C)


Males enrolled in Scheme A : Females enrolled in Scheme B
= 400 : (140 + 260) = 400 : 400 = 1 : 1

108) Ans. (D)


In Scheme C, Males – Females = (640 + 340) – (440 + 260) = 980 – 700 = 280

109) Ans. (B)


% of Males enrolled in scheme C with Total people enrolled in scheme A
= 980 × 100/560 = 175%

110) Ans. (A)


Total people enrolled in scheme B = 720 + 600 = 1320
Total people enrolled in scheme C = 1080 + 600 = 1680
Total people enrolled in scheme C is more than Total people enrolled in
scheme B by = (1680 – 1320) × 100/1320 = 27.27%

(111 – 115)
Office  A B C Total
Male 1540 1190 840 3570
Female 1820 1050 960 3830
Total 3360 2240 1800 7400

111) Ans. (B)


Males in B & C : Females in A = (1190 + 840) : 1820 = 2030 : 1820 = 203 : 182
https://youtu https://instagra
https ://ww
be.com/chan w.fa cebook.
m.com/aashish https://
arorasocial(?)
nel/UCYa4_Jr com/a ashis utm_medium = t.me/st
232 Orf8R5Kz2uO
ha rorasocial copy_link
tccXQ udified
112) Ans. (D)
Total Employees in Office B are more than Male Employees in Office C by
= (2240 – 840) × 100/840 = 166(2/3) %

113) Ans. (B)


New Number of Employees in Office B
= 1050 × 23/15 + 1190 × 9/7 = 1610 + 1530 = 3140

114) Ans. (B)


In Office A, Unmarried Males : Unmarried Females
= (1540 × 0.65) : (1820 × 0.45) = (11 × 13) : (13 × 9) = 11 : 9

115) Ans. (D)


Total Female Employees – Total Male Employees = 3830 – 3570 = 260

(116 – 120)
Showroom  A B C D Total

Bullet 927 2040 1840 2280 7087

R15 721 1440 1200 2560 5921

Total 1648 3480 3040 4840 13008

116) Ans. (C)


Bullet sold by B is less than R15 sold by D by
= (2560 – 2040) × 100/2560 = 20.3125%

117) Ans. (D)


Bullet sold by A : R15 sold by B = 927 : 1440 = 103 : 160

118) Ans. (B)


By Showroom D
Revenue generated by Bullets = 2280 × 2.5 = 5700 Lacs
Revenue generated by R15 = 2560 × 2.5/1.25 = 5120 Lacs
Difference = 5700 – 5120 = 580 Lacs

119) Ans. (A)


Bullets sold by A & B = 927 + 2040 = 2967 https://instagra
https://youtu https ://ww
be.com/chan w.fa cebook.
m.com/aashish https://
arorasocial(?)
nel/UCYa4_Jr com/a ashis utm_medium = t.me/st
233 Orf8R5Kz2uO
ha rorasocial copy_link
tccXQ udified
R15 sold by C & D = 1200 + 2560 = 3760
Difference = 3760 – 2967 = 793

120) Ans. (C)


Average Bullets sold by all Showroom = 7087/4 = 1771.75

(121 – 125)
Company  P Q R S Total
Male 1200 750 300 400 2650
Female 640 500 420 950 2510
Total 1840 1250 720 1350 5160

121) Ans. (A)


Males in Company R : Females in Company S = 300 : 950 = 6 : 19

122) Ans. (C)


In Company A, Males are more than Females by
= (1200 – 640) × 100/640 = 87.5%

123) Ans. (B)


Total monthly salary of employees of Company Q = 750 × 24000 + 500 ×
24000 × 1.25 = 18000000 + 15000000 = 33000000 = 3.3 Cr.

124) Ans. (A)


In Company R, Males = 300 × 19/12 = 475, Females = 420 × 5/7 = 300
Difference = 475 – 300 = 175

125) Ans. (A)


Average number of females in all companies = 2510/4 = 627.5

(126 – 130)

Company  A B C D Total
Shirt 420 400 480 880 2180
T – Shirt 640 320 560 640 2160
Total 1060 720 1040 1520 4340
https://youtu https://instagra
https ://ww
be.com/chan w.fa cebook.
m.com/aashish https://
arorasocial(?)
nel/UCYa4_Jr com/a ashis utm_medium = t.me/st
234 Orf8R5Kz2uO
ha rorasocial copy_link
tccXQ udified
126) Ans. (B)
Shirts produced by D : T – shirts produced by C = 880 : 560 = 11 : 7

127) Ans. (D)


T – shirts sold by D are more than Shirts sold by C by
= (640 – 480) × 100/480 = 33.33%

128) Ans. (A)


Shirt produced by E = 420 × 22/15 = 616
T – shirt produced by E = 640 × 0.6875 = 440
Shirts sold are more than T – shirts by = (616 – 440) × 100/440 = 40%

129) Ans. (C)


Shirts sold by A & B = 420 + 400 = 820
T – shirts sold by B & C = 320 + 560 = 880
Difference = 880 – 820 = 60

130) Ans. (C)


T – shirt produced exactly 16(2/3) % more than shirts by Company = C

(131 – 135)
Company  P Q R S Total
AC 963 696 1177 1400 4236
Coolers 1177 986 1498 1500 5161
Total 2140 1682 2675 2900 9397

131) Ans. (B)


Cooler sold by P : AC sold by S = 1177 : 1400

132) Ans. (A)


AC sold by P are less than Coolers sold by S
= (1500 – 963) × 100/1500 = 35.8%

133) Ans. (C)


Total items sold in Summer by Company R
= 1498 × 9/7 + 1177 × 6/11 = 1926 + 642 = 2568
https://youtu https://instagra
https ://ww
be.com/chan w.fa cebook.
m.com/aashish https://
arorasocial(?)
nel/UCYa4_Jr com/a ashis utm_medium = t.me/st
235 Orf8R5Kz2uO
ha rorasocial copy_link
tccXQ udified
134) Ans. (A)
Item sold by R – Item sold by Q = 2675 – 1682 = 993

135) Ans. (C)


Average of AC sold by all the shopkeepers = 4236/4 = 1059

(136 – 140)
City  A B C D Total
Samsung 936 252 424 396 2008
I – Phone 366 288 456 540 1650
Total 1302 540 880 936 3658

136) Ans. (A)


People like I – phone in city D : People like Samsung in city B
= 540 : 252 = 15 : 7

137) Ans. (D)


People like Samsung in city A are more than People like I – phone in city B
by = (936 – 288) × 100/288 = 225%

138) Ans. (C)


In city C, New number of people who like Samsung
= 424 + 0.25 × 456 = 424 + 114 = 538
% of people like Samsung = 538 × 100/880 = 61.1%

139) Ans. (C)


Number of people in city F = 288 × 17/12 + 252 × 9/7 = 408 + 324 = 732

140) Ans. (A)


Average of people like I – phone in all cities = 1650/4 = 412.5

(141 – 145)
Cost of,
Mouse = 2000 Rs., Table = 1.5 × 2000 = 3000 Rs.
Chair = 3000 × 3/10 = 900 Rs., AC = 13 × 900 = 11700 Rs.
Monitor = 3.6 × 900 = 3240 Rs., Key-Board = 1.5 × 2000 = 3000 Rs.
CPU = 3000 + 1500 = 4500 Rs. https://instagra
https://youtu https ://ww
be.com/chan w.fa cebook.
m.com/aashish https://
arorasocial(?)
nel/UCYa4_Jr com/a ashis utm_medium = t.me/st
236 Orf8R5Kz2uO
ha rorasocial copy_link
tccXQ udified
141) Ans. (A)
Cost of, 2 Monitor + 3 CPU = 2 × 3240 + 3 × 4500 = 6480 + 13500 = 19980 Rs.

142) Ans. (A)


Cost of Key – Board : Cost of (Chair + Table)
= 3000 : (900 + 3000) = 3000 : 3900 = 10 : 13

143) Ans. (C)


Cost of Table & AC = 3000 × 1.1 + 11700 × 1.3 = 3300 + 15210 = 18510 Rs.

144) Ans. (C)


Cost of Chair & Key – Board = 900 + 3000 = 3900 Rs.
Cost of Mouse & CPU = 2000 + 4500 = 6500 Rs.
% of cost of Char & Key – Board with cost of Mouse & CPU
= 3900 × 100/6500 = 60 Rs.

145) Ans. (D)


Total Cost of All Items
= 2000 + 3000 + 900 + 11700 + 3240 + 3000 + 4500 = 28340 Rs.

(146 – 150)
CP for, Lamp = 5000 Rs. Oven = 5000 × 2.2 × 0.88 = 9680 Rs.
AC = 9680 × 1.35 × 0.75 = 9801 Rs. Refrigerator = 14000 Rs.
Light = 5000 × 2.2 × 0.1 = 1100 Rs.
146) Ans. (C)
Ayush Bought AC at = 9801 Rs.

147) Ans. (D)


CP of AC : CP of Oven = 9801 : 9680 = 81 : 80

148) Ans. (A)


Money Spent on Lights = 1100 Rs.

149) Ans. (D)


Discount on AC = (14000 – 12500) × 100/14000 = 10.71%

150) Ans. (C)


Total cost of items purchased = 5000 + 9680 + 9801https://instagra
+ 14000 + 1100 = 39581
https://youtu https ://ww
be.com/chan w.fa cebook.
m.com/aashish https://
arorasocial(?)
nel/UCYa4_Jr com/a ashis utm_medium = t.me/st
237 Orf8R5Kz2uO
ha rorasocial copy_link
tccXQ udified
https://t.mhttps/t.me
e/studified/studified

https://instagram.com/aashisharora
social?utm_medium = copy_link

https://youtube.com/channel
/UCYa4_JrOrf8R5Kz2uOtccXQ

https://www.facebook.
com/aashisharorasocial

https://youtu https://instagra
https ://ww
be.com/chan w.fa cebook.
m.com/aashish https://
arorasocial(?)
nel/UCYa4_Jr com/a ashis utm_medium = t.me/st
238 Orf8R5Kz2uO
tccXQ
ha rorasocial copy_link
udified
(1 – 5) The given pie chart shows the actual number of total television
purchased by store from five different companies.
Total television purchased from each company = Number of (Semi smart +
smart) Television purchased. The given table shows the ratio of number of
semi smart and smart TV, respectively purchased by a store from five
companies.
ददमा गमा ऩाई चाटष ऩाॊच अरग – अरग कॊ ऩननमों से स्टोय द्वाया खयीदे गए कु र
टे रीववजन की वास्तववक सॊख्मा को दिाषता है।
प्रत्मे क कॊ ऩनी से खयीदा गमा कु र टे रीववजन = खयीदे गए (अधष स्भाटष + स्भाटष)
टे रीववजन की सॊख्मा।
दी गई ताशरका ऩाॊच कॊ ऩननमों से एक स्टोय द्वाया खयीदे गए से भी स्भाटष औय स्भाटष टीवी
की सॊख्मा का अनुऩात दिाषती है।

120 84
105
75
130

Sony LG MI Samsung Panasonic

Company Semi Smart TV : Smart TV


Sony 9:5
LG 7:8
MI 6:7
Samsung 2:3
Panasonic 5:3
https : https : https : https :
//youtube.co //www.face //instagram.c
m/channel/U book.com/a om/aashishar //t.me/
CYa4_JrOrf8R orasocial?ut
239 5Kz2uOtccXQ
a s hisharora
m_medium = studifie
s ocial
copy_link d
1. Find the average number of Smart TV purchased from Samsung and
Panasonic.
सैभसॊग औय ऩैनासोननक से खयीदे गए स्भाटष टीवी की औसत सॊख्मा ऻात कीजजए।
(A) 30 (B) 35 (C) 40
(D) 45 (E) None of these

2. The number of Semi smart TV purchased from MI is how much percent


more/less than the number of smart TV purchased from Sony?
एभआई से खयीदे गए से भी स्भाटष टीवी की सॊख्मा सोनी से खयीदे गए स्भाटष टीवी की सॊख्मा से
ककतने प्रनतित अचधक/कभ है?
(A) 200% (B) 150% (C) 100%
(D) 50% (E) None of these

3. Find the ratio of number of Semi smart TV purchased from LG and


Smart TV purchased from MI?
एरजी से खयीदे गए से भी स्भाटष टीवी औय एभआई से खयीदे गए स्भाटष टीवी की सॊख्मा का
अनुऩात ऻात कीजजमे ?
(A) 10 : 7 (B) 3 : 4 (C) 5 : 3
(D) 7 : 10 (E) None of these

4. The Number Semi Smart TV purchased from Sony,Samsung and


Panasonic together is approximately what percent of total TV purchased
from LG.
सोनी, सैभसॊग औय ऩैनासोननक से खयीदे गए से भी स्भाटष टीवी की सॊख्मा एरजी से खयीदे गए
कु र टीवी का रगबग ककतना प्रनतित है?
(A) 50% (B) 151% (C) 149%
(D) 48% (E) None of these

5. The cost price of a smart TV of Sony is Rs. 21500. Find the total price
paid for smart TV of Sony.
सोनी के एक स्भाटष टीवी की रागत भल्ू म 21500 रुऩमे है। सोनी के स्भाटष टीवी के शरए
बुगतान की गई कु र कीभत ऻात कीजजए।
(A) Rs. 645000 (B) Rs. 925000 (C) Rs. 116100
(D) Rs. 205100 (E) None of these

(6 – 10) The given radar chart shows the average of the number of Laser
printers and Inkjet printers sold and number of (Laser printers – Inkjet
printer) sold, by five different sellers.
ददमा गमा याडाय चाटष ऩाॊच अरग – अरग ववक्रे ताओॊ द्वाया फे चे गए रे ज य वप्रॊटय औय इॊक जे ट
वप्रॊटय की सॊख्मा औय फे चे गए (रे जय वप्रॊटय – इॊकजे ट वप्रॊटय) की सॊख्मा का औसत ददखाता है।

https : https : https : https :


//youtube.co //www.face //instagram.c
m/channel/U book.com/a om/aashishar //t.me/
CYa4_JrOrf8R orasocial?ut
240 5Kz2uOtccXQ
a s hisharora
m_medium = studifie
s ocial
copy_link d
6. The number of Laser printers sold by Seller A and E, together is how
much percent more/less than the total printers sold by seller B?
ववक्रे ता A औय E द्वाया फे चे गए रे जय वप्रॊटयों की सॊख्मा, ववक्रे ता B द्वाया फे चे गए कु र वप्रॊटयों
से ककतने प्रनतित अचधक/कभ है?
(A) 14 2/7% (B) 16 2/3% (C) 66 2/3%
(D) 33 1/3% (E) None of these

7. Find the average number of Inkject printers sold by C and E.


C औय E द्वाया फे चे गए इॊक जे क्ट वप्रॊटयों की औसत सॊख्मा ऻात कीजजए।
(A) 400 (B) 200 (C) 300
(D) 500 (E) None of these

8. Seller B sold each Laser Printer and each Inkject printer for Rs. 1100
and Rs. 950 respectively. Find the difference between the amounts
received by selling 30% of Laser printers and 44% of Inkject printers by
Seller B.
ववक्रे ता B ने प्रत्मे क रे जय वप्रॊटय औय प्रत्मे क इॊकजे क्ट वप्रॊटय को क्रभि् 1100 रुऩमे औय 950
रुऩमे भें फे चा। ववक्रे ता B द्वाया 30% रे ज य वप्रॊटय औय 44% इॊक जे क्ट वप्रॊटय फे चकय प्राप्त याशि
के फीच अॊतय ऻात कीजजए।
(A) Rs. 264000 (B) Rs. 314000 (C) Rs. 184800
(D) Rs. 211400 (E) None of these

9. The ratio of number of Inkject printers sold by seller A and another


seller F is 15 : 6, respectively. IF total printers (Laser + Inkject) sold by F is
45% less than number of laser printer sold by seller 'D', then find the
number of Laser printershttps :sold by seller
httpsF.
: https : https :
//youtube.co //www.face //instagram.c
m/channel/U book.com/a om/aashishar //t.me/
CYa4_JrOrf8R orasocial?ut
241 5Kz2uOtccXQ
a s hisharora
m_medium = studifie
s ocial
copy_link d
ववक्रे ता A औय अन्म ववक्रे ता F द्वाया फे चे गए इॊक जे क्ट वप्रॊटयों की सॊख्मा का अनुऩात क्रभि्
15 : 6 है। मदद F द्वाया फे चे गए कु र वप्रॊटय (रे ज य + इॊक जे क्ट) ववक्रे ता 'D' द्वाया फे चे गए
रे जय वप्रॊटय की सॊख्मा से 45% कभ है, तो ववक्रे ता F द्वाया फे चे गए रे जय वप्रॊटय की सॊख्मा
ऻात कीजजए।
(A) 240 (B) 150 (C) 319
(D) 213 (E) None of these

10. Find 22.22% of number of Inkject printers sold by seller D.


ववक्रे ता D द्वाया फे चे गए इॊकजे क्ट वप्रॊटयों की सॊख्मा का 22.22% ऻात कीजजए।
(A) 80 (B) 110 (C) 120
(D) 240 (E) None of these

(11 – 15) Table given below shows percentage of waiter working in five
different hotels out of total employees and line graph shows the
difference between chef and waiter working in respective hotels.
नीचे दी गई ताशरका कु र कभषचारयमों भें से ऩाॊच अरग – अरग होटरों भें काभ कयने वारे वे टय
का प्रनतित ददखाती है औय राइन ग्राप सॊफॊचधत होटरों भें काभ कयने वारे िे प औय वे टय के
फीच के अॊतय को दिाषता है।

https : https : https : https :


//youtube.co //www.face //instagram.c
m/channel/U book.com/a om/aashishar //t.me/
CYa4_JrOrf8R orasocial?ut
242 5Kz2uOtccXQ
a s hisharora
m_medium = studifie
s ocial
copy_link d
Hotel Percentage of waiter in the
Hotel
Sunshine 48%
Diamond 35%
Lotus 40%
Star 25%
Sunflower 55%
11. Total number of waiter in hotel Lotus is what percent more/less than
the total number of chef working in hotel Sunshine?
होटर रोटस भें वे टय की कु र सॊख्मा होटर सनिाइन भें कामषय त िे प की कु र सॊख्मा से ककतने
प्रनतित अचधक/कभ है?
(A) 32% (B) 54.6% (C) 48.9%
(D) 69.2% (E) None of these

12. What is the average of the number of waiter working in hotel Diamond
and Star together?
होटर डामभॊड औय स्टाय भें शभराकय काभ कयने वारे वे टय की सॊख्मा का औसत ककतना है?
(A) 800 (B) 720 (C) 450
(D) 500 (E) None of these

13. Find the difference between number of waiter working in hotel Lotus
and Sunflower together, and number of chef working in hotel Diamond.
होटर रोटस औय सनफ्रावय भें शभराकय काभ कयने वारे वे टय की सॊख्मा औय होटर डामभॊड
भें काभ कयने वारे िे प की सॊख्मा के फीच का अॊतय ऻात कीजजए।
(A) 1820 (B) 1840 (C) 1825
(D) 1850 (E) None of these

14. If the number of chef and waiter in a sixth hotel, hotel Moon is 40%
and 20% of the number of chef and waiter working in hotel Lotus,
respectively. Then, find the total number of employees working in hotel
Moon.
मदद छठे होटर, होटर भून भें िे प औय वे टय की सॊख्मा होटर रोटस भें काभ कयने वारे िे प
औय वे टय की सॊख्मा का क्रभि् 40% औय 20% है। तो, होटर भून भें काभ कयने वारे
कभषचारयमों की कु र सॊख्मा ऻात कीजजए।
(A) 360 (B) 640 (C) 580
(D) 790 https (E)
: None ofhttps
these: https : https :
//youtube.co //www.face //instagram.c
m/channel/U book.com/a om/aashishar //t.me/
CYa4_JrOrf8R orasocial?ut
243 5Kz2uOtccXQ
a s hisharora
m_medium = studifie
s ocial
copy_link d
15. What is the ratio of the number of waiter working in hotel Sunshine to
the number of chef working in hotel Sunflower?
होटर सनिाइन भें काभ कयने वारे वे टय की सॊख्मा का होटर सनफ्रावय भें काभ कयने वारे
िे प की सॊख्मा से अनुऩात ककतना है?
(A) 4 : 5 (B) 7 : 9 (C) 8 : 9
(D) 6 : 7 (E) None of these

(16 – 20) Directions : The following bar graph shows the percentage of
families in five different societies that own petrol vehicles. A family owns
either petrol vehicle or diesel vehicle. The pie chart shows the percentage
distribution of the number of families in five different societies.
Total families = 24000.
ननदे ि : ननम्नशरणखत फाय ग्राप ऩाॊच अरग – अरग सभाजों के ऩरयवायों का प्रनतित दिाषता है
जजनके ऩास ऩे ट्रोर वाहन हैं । एक ऩरयवाय के ऩास ऩे ट्रोर वाहन मा डीजर वाहन है। ऩाई चाटष
ऩाॊच अरग – अरग सभाजों भें ऩरयवायों की सॊख्मा का प्रनतित ववतयण दिाषता है।
कु र ऩरयवाय = 24000।

https : https : https : https :


//youtube.co //www.face //instagram.c
m/channel/U book.com/a om/aashishar //t.me/
CYa4_JrOrf8R orasocial?ut
244 5Kz2uOtccXQ
a s hisharora
m_medium = studifie
s ocial
copy_link d
16. What is the respective ratio of the number of families that do not own
petrol vehicle in society D to the number of families that do not own
diesel vehicle in society E?
सभाज D भें ऩे ट्रोर वाहन न यखने वारे ऩरयवायों की सॊख्मा औय सभाज E भें डीजर वाहन न
यखने वारे ऩरयवायों की सॊख्मा का सॊफॊचधत अनुऩात ककतना है?
(A) 4 : 11 (B) 11 : 4 (C) 11 : 8
(D) 8 : 11 (E) None of these

17. The number of families that do not own petrol vehicles in society B is
what percent of the total number of families in society E?
सभाज B भें उन ऩरयवायों की सॊख्मा जजनके ऩास ऩे ट्रोर वाहन नहीॊ है, सभाज E भें ऩरयवायों की
कु र सॊख्मा का ककतना प्रनतित है?
(A) 55.5% (B) 51.5% (C) 52.5%
(D) 50.5% (E) None of these

18. In society B there are only two types vehicles (sedan or hatchback). If
33.33% of the petrol vehicles are sedan while 25% of the diesel vehicles
are sedan, then find the total number of hatchback vehicles in society B.
सोसाइटी B भें के वर दो प्रकाय के वाहन (से डान मा हैचफैक) हैं । मदद 33.33% ऩे ट्रोर वाहन
से डान हैं जफकक 25% डीजर वाहन से डान हैं , तो सोसाइटी B भें हैचफैक वाहनों की कु र सॊख्मा
ऻात कीजजए।
(A) 3188 (B) 3321 (C) 2312
(D) 3132 (E) None of these
https : https : https : https :
//youtube.co //www.face //instagram.c
m/channel/U book.com/a om/aashishar //t.me/
CYa4_JrOrf8R orasocial?ut
245 5Kz2uOtccXQ
a s hisharora
m_medium = studifie
s ocial
copy_link d
19. The total number of families in society that has maximum number of
diesel vehicles are how much more of less than the total number of
families in society C?
सभाज, जजनके ऩास डीजर वाहनों की अचधकतभ सॊख्मा है, भें ऩरयवायों की कु र सॊख्मा सभाज
C भें ऩरयवायों की कु र सॊख्मा से ककतनी अचधक मा कभ है?
(A) 720 less (B) 720 more (C) 12.5% less
(D) Both A and C (E) None of these

20. Find the average number of families that have diesel vehicles in all the
societies excluding society C.
सभाज C को छोडकय सबी सोसामदटमों भें ऐसे ऩरयवायों की औसत सॊख्मा ऻात कीजजए
जजनके ऩास डीजर वाहन हैं ।
(A) 2310 (B) 2210 (C) 2110
(D) 2410 (E) None of these

(21 – 25) The graph below shows the percentage distribution of the total
number of branches of SBI Bank in 5 different districts, and the ratio of
the number of branches with less than 8 crore balance sheets to the
number of branches with more than 8 crore balance sheets. Note : The
total number of branches with exactly 8 crore balance sheets is zero.
नीचे ददमा गमा ग्राप 5 अरग – अरग जजरों भें SBI फैंक की ब्ाॊच की कु र सॊख्मा के प्रनतित
ववतयण को दिाषता है, औय 8 कयोड से कभ फैरेंसिीट वारी ब्ाॊच की सॊख्मा का 8 कयोड से
अचधक फैरेंसिीट वारी ब्ाॊच की सॊख्मा के अनुऩात को दिाषता है। नोट : ठीक 8 कयोड
फैरेंसिीट वारी ब्ाॊच की कु र सॊख्मा िून्म है।

https : https : https : https :


//youtube.co //www.face //instagram.c
m/channel/U book.com/a om/aashishar //t.me/
CYa4_JrOrf8R orasocial?ut
246 5Kz2uOtccXQ
a s hisharora
m_medium = studifie
s ocial
copy_link d
District The number of branches with less than 8 crore :
s The number of branches with more than 8 crore

Sikar 15 : 8
Bikaner 3:5
Jaipur 12 : 11
Ajmer 8:9
Kota 4:3

21. What is the average number of branches in Bikaner and Ajmer with
more than 8 crore balance sheets?
फीकाने य औय अजभे य भें 8 कयोड से अचधक फैरेंस िीट वारी ब्ाॊचों की औसत सॊख्मा
क्मा है?
(A) 154 (B) 151 (C) 164
(D) 161 (E) None of these

22. What is the difference between Sikar and Jaipur branches with less
than 8 crore balance sheet?
सीकय औय जमऩुय भें 8 कयोड से कभ फैरेंसिीट वारी ब्ाॊचों के फीच क्मा अॊतय है?
(A) 52 (B) 58 (C) 54
(D) 50 (E) None of these

23. What is the ratio of the number of branches in Kota with less than 8
crore balance sheets to the number of branches in Sikar having more than
8 crore balance sheets?
कोटा भें 8 कयोड से कभ फैरेंसिीट वारी ब्ाॊचों की सॊख्मा औय सीकय भें 8 कयोड से अचधक
फैरेंसिीट वारी ब्ाॊचों की सॊख्मा का अनुऩात क्मा है?
(A) 23 : 12 (B) 16 : 17 (C) 17 : 16
(D) 25 : 13 (E) None of these

24. The number of branches in Ajmer with balance sheet less than 8 crore
is what percent more/less than the number of branches in Jaipur having
balance sheet more than 8 crore?
https : https : https : https :
//youtube.co //www.face //instagram.c
m/channel/U book.com/a om/aashishar //t.me/
CYa4_JrOrf8R orasocial?ut
247 5Kz2uOtccXQ
a s hisharora
m_medium = studifie
s ocial
copy_link d
अजभे य भें 8 कयोड से कभ फैरेंसिीट वारी ब्ाॊचों की सॊख्मा जमऩुय भें 8 कयोड से अचधक
फैरेंसिीट वारी ब्ाॊचों की सॊख्मा से ककतने प्रनतित अचधक/कभ है?
(A) 33.66% (B) 32.36% (C) 39.39%
(D) 35.69% (E) None of these

25. If the branches of Bikaner, Jaipur and Sikar have 25% , 75% and 50% new
employees respectively, then find the average number of total old
employees in these districts.
मदद फीकाने य , जमऩुय औय सीकय की ब्ाॊचों भें क्रभि् 25% , 75% औय 50% नमे कभषचायी है, तो
इन जजरों भें कु र ऩुय ाने कभषचारयमों की औसत सॊख्मा ऻात कयें ।
(A) 99 (B) 92 (C) 97
(D) 94 (E) None of these

(26. – 30) The radar graph given below shows the total number of students
(boys + girls) at five different coaching centers and the number of boys in
the coaching centers..
नीचे ददए गए यडाय ग्राप ऩाॊच अरग – अरग कोचच ॊग सेंटय ऩय ववद्माचथषमो (रडके + रडककमो)
की कुर सॊख्मा औय कोचच ॊग सेंटय भें रडकों की सॊख्मा को दिाषता है।

https : https : https : https :


//youtube.co //www.face //instagram.c
m/channel/U book.com/a om/aashishar //t.me/
CYa4_JrOrf8R orasocial?ut
248 5Kz2uOtccXQ
a s hisharora
m_medium = studifie
s ocial
copy_link d
26. What is the ratio of the total number of girls in center A and center B
together to the total number of students in center F?
सेंटय A औय सेंटय B भें रडककमो कु र सॊख्मा औय सेंटय F भें ववद्माचथषमों की कु र सॊख्मा का
अनुऩात क्मा है?
(A) 57 : 58 (B) 49 : 72 (C) 57 : 71
(D) 69 : 71 (E) None of these

27. The fee of each boy is Rs. 1800 and that of each girl is Rs. 1500, then
find the total income of center E.
प्रत्मे क रडके की पीस रु. 1800 औय प्रत्मे क रडकी की पीस रु. 1500 हैं तो सेंटय E
की कु र आभदनी ऻात कीजजए।
(A) 345400 (B) 365900 (C) 385200
(D) 375800 (E) None of these

28. The number of students in center C is what percent of the total


number of girls in center D, center E and center F?
सेंटय C भें ववद्माचथषमो की सॊख्मा, सेंटय D, सेंटय E ओय सेंटय F भें रडककमो की कु र सॊख्मा
का ककतने प्रनतित है?
(A) 65% (B) 75.5% (C) 65.5%
(D) 77.5% (E) None of these

29. The number of boys in center A is what percent more/less than the
number of girls in center D?
सेंटय A भें रडको की सॊख्मा सेंटय D भें रडककमो की सॊख्मा से ककतने प्रनतित अचधक/कभ है?
(A) 63.15% (B) 65.25% (C) 67.50%
(D) 48% (E) None of these

30. What is the difference between the number of boys in center B and
center C and the number of girls in center E and center F?
सेंटय B औय सेंटय C भें रडको की सॊख्मा औय सेंटय E औय सेंटय F भें रडककमो की सॊख्मा भें
क्मा अॊतय है?
(A) 22 (B) 32 (C) 42
(D) 52 (E) None of these

(30 – 35) The horizontal bar graph shows total three types of shirt sold by
a store on five months. It also shows the percentage of full sleeve and
sleeveless shirt and number of half sleeve shirt sold by store sold on five
different months. [Note : Only three types of shirt are sold by the store.]
ऺैनतज फाय ग्राप ऩाॊच भहीनों भें एक स्टोय द्वाया फे ची गई कु र तीन प्रकाय की िटष
ददखाता है। मह ऩाॊच अरग – अरग भहीनों भें फे ची गई ऩूय ी आस्तीन औय त्रफना
आस्तीन की िटष का प्रनतित औय स्टोय द्वाया फे ची गई आधी फाॊह की िटष की सॊख्मा
को बी दिाषता है। [नोट : स्टोय द्वाया के वर तीन प्रकाय की िटष फे ची जाती है।]
https : https : https : https :
//youtube.co //www.face //instagram.c
m/channel/U book.com/a om/aashishar //t.me/
CYa4_JrOrf8R orasocial?ut
249 5Kz2uOtccXQ
a s hisharora
m_medium = studifie
s ocial
copy_link d
31. Total number of full sleeve shirt sold by store on January and
February together are what percent less/more than the number of half
sleeve sold by store on January and March?
जनवयी औय पयवयी को एकसाथ स्टोय द्वाया फे ची गई ऩूय ी आस्तीन की िटष की कु र सॊख्मा
जनवयी औय भाचष को स्टोय द्वाया फे ची गई आधी आस्तीन की सॊख्मा से ककतने प्रनतित
कभ/अचधक है?
(A) 37.2% (B) 81.52% (C) 22.66%
(D) 18.47% (E) None of these

32. Find the Difference between total number of Full sleeve shirt sold by
store on January, February and April and total number of half sleeve shirt
sold by store on March and May.
जनवयी, पयवयी औय अप्रैर भें स्टोय द्वाया फे ची गई ऩूय ी आस्तीन की िटष की कु र सॊख्मा औय
भाचष औय भई भें स्टोय द्वाया फे ची गई आधी आस्तीन की िटष की कु र सॊख्मा के फीच अॊतय
ऻात कीजजए।
(A) 576 (B) 329 (C) 440
(D) 620 (E) None of these

33. What is the sum of average number of sleeveless shirt sold by store
on January, April and May and average number of half sleeve shirt sold by
store on Feb and March?
जनवयी, अप्रैर औय भई भें स्टोय द्वाया फे ची गई त्रफना आस्तीन की िटष की औसत सॊख्मा औय
पयवयी औय भाचष भें स्टोय द्वाया फे ची गई आधी आस्तीन की िटष की औसत सॊख्मा का मोग
ककतना है?
(A) 1600 (B) 1548 (C) 1460
(D) 1305 (E) None of these
https : https : https : https :
//youtube.co //www.face //instagram.c
m/channel/U book.com/a om/aashishar //t.me/
CYa4_JrOrf8R orasocial?ut
250 5Kz2uOtccXQ
a s hisharora
m_medium = studifie
s ocial
copy_link d
34. If total number of half sleeve sold by store on June is 40% more than
that sold on April and total full sleeve sold on June is 55% more than that
sold on February. Find the total number of half sleeve and full sleeve sold
by store on June.
मदद ज ून को स्टोय द्वाया फे ची गई आधी आस्तीन की कु र सॊख्मा अप्रैर भें फे ची गई आधी
आस्तीन की सॊख्मा से 40% अचधक है औय ज ून को फे ची गई कु र आधी आस्तीन पयवयी भें
फे ची गई कु र आस्तीन से 55% अचधक है। ज न ू को स्टोय द्वाया फे ची गई आधी आस्तीन औय
ऩूय ी आस्तीन की कु र सॊख्मा ऻात कीजजए।
(A) 1677 (B) 1982 (C) 1465
(D) 1465 (E) None of these

35. What is the ratio of total number of shirt sold on January to total
number of shirt sold on May?
जनवयी भें फे ची गई िटष की कु र सॊख्मा का भई भें फे ची गई िटष की कु र सॊख्मा से
अनुऩात ककतना है?
(A) 8 : 5 (B) 7 : 3 (C) 9 : 2
(D) 3 : 4 (E) None of these

(36 – 40) The bar graph shows the percentage of person who play three
different types of games in five different cities.
[Note : One person plays only one type of game.]
फाय ग्राप ऩाॊच अरग – अरग िहयों भें तीन अरग – अरग प्रकाय के खेर खेरने वारे व्मजक्त
का प्रनतित दिाषता है।
[नोट : एक व्मजक्त केवर एक ही प्रकाय का खेर खेरता है।]

36. If the difference between number of person who play cricket to person
who play tennis in Jaipur is 400 while the average of number of person
https : https : https : https :
//youtube.co //www.face //instagram.c
m/channel/U book.com/a om/aashishar //t.me/
CYa4_JrOrf8R orasocial?ut
251 5Kz2uOtccXQ
a s hisharora
m_medium = studifie
s ocial
copy_link d
who play football and cricket in Surat is 2000 then find the ratio of total
number of person in Jaipur to Surat.
मदद जमऩुय भें टे ननस खेरने वारे व्मजक्त से कक्रकेट खेरने वारे व्मजक्तमों की सॊख्मा का अॊतय
400 है जफकक सूय त भें पु टफॉर औय कक्रके ट खेरने वारे व्मजक्तमों की औसत सॊख्मा 2000 है,
तो जमऩुय भें व्मजक्तमों की कु र सॊख्मा का सूय त से अनुऩात ऻात कीजजए।
(A) 3 : 7 (B) 2 : 9 (C) 4 : 5
(D) 10 : 9 (E) None of these

37. Find the total number of person in Mumbai who play football. If the
difference between number of person who play football and tennis and
number of person who play cricket and football is 300.
भुॊफई भें पु टफॉर खेरने वारे व्मजक्तमों की कु र सॊख्मा ऻात कीजजए। मदद पु टफॉर औय
टे ननस खेरने वारे व्मजक्तमों की सॊख्मा औय कक्रके ट औय पु टफॉर खेरने वारे व्मजक्तमों
की सॊख्मा के फीच का अॊतय 300 है।
(A) 1800 (B) 1500 (C) 3000
(D) 2700 (E) None of these

38. Find the total number of person in Ranchi. If the ratio of number of
person in Ranchi to number of person in Kolkata is 3 : 2 and total number
of person in Ranchi and Kolkata who play tennis is 1700.
याॊची भें व्मजक्तमों की कु र सॊख्मा ऻात कीजजए। मदद याॊची भें व्मजक्तमों की सॊख्मा का
कोरकाता भें व्मजक्तमों की सॊख्मा से अनुऩात 3 : 2 है औय याॊची औय कोरकाता भें टे ननस
खेरने वारे व्मजक्तमों की कु र सॊख्मा 1700 है।
(A) 3000 (B) 4000 (C) 6000
(D) 9000 (E) None of these

39. Find the total number of persons in Kolkata is what percent of total
person in Mumbai. If the difference between number of person who play
tennis and cricket in Kolkata is 400 and the difference between number of
person who play football and tennis in Mumbai is 900.
कोरकाता भें व्मजक्तमों की कु र सॊख्मा ऻात कीजजए, भुॊफई भें कु र व्मजक्त का ककतना प्रनतित
है। मदद कोरकाता भें टे ननस औय कक्रके ट खेरने वारे व्मजक्तमों की सॊख्मा का अॊतय 400 है औय
भुॊफई भें पु टफॉर औय टे ननस खेरने वारे व्मजक्तमों की सॊख्मा का अॊतय 900 है।
(A) 46.6% (B) 33.3% (C) 25.9%
(D) 55.8% (E) None of these

40. Find the total number of person in Jaipur. If total number of person in
Jaipur is 1000 less than the total number of person in Surat. The
difference between number of person who play cricket and football in
Surat is 2000.
जमऩुय भें व्मजक्तमों की कु र सॊख्मा ऻात कीजजए। मदद जमऩुय भें व्मजक्तमों की कु र सॊख्मा
सूय त भें व्मजक्तमों की कु र सॊख्मा से 1000 कभ है। सूय त भें कक्रके ट औय पु टफॉर खेरने वारे
व्मजक्तमों की सॊख्मा के फीचhttps
का :अॊतय 2000 हैhttps
। : https : https :
//youtube.co //www.face //instagram.c
m/channel/U book.com/a om/aashishar //t.me/
CYa4_JrOrf8R orasocial?ut
252 5Kz2uOtccXQ
a s hisharora
m_medium = studifie
s ocial
copy_link d
(A) 3000 (B) 5000 (C) 4000
(D) 6000 (E) None of these

(41 – 45) Following bar graph shows the percentage distribution of the
number of each types of products manufactured in a factory. The line
graph shows the percentage of each type of products sold.
Total numbers of products of all 5 types manufactured = 5600
ननम्नशरणखत फाय ग्राप एक कायखाने भें ननशभषत प्रत्मे क प्रकाय के उत्ऩादों की सॊख्मा का
प्रनतित ववतयण दिाषता है। राइन ग्राफ फे चे गए प्रत्मे क प्रकाय के उत्ऩादों का प्रनतित
दिाषता है।
ननशभषत सबी 5 प्रकाय के उत्ऩादों की कु र सॊख्मा = 5600

https : https : https : https :


//youtube.co //www.face //instagram.c
m/channel/U book.com/a om/aashishar //t.me/
CYa4_JrOrf8R orasocial?ut
253 5Kz2uOtccXQ
a s hisharora
m_medium = studifie
s ocial
copy_link d
41. What is the ratio of the number of products manufactured of type P to
the number of products sold of type M?
प्रकाय P के ननशभषत उत्ऩादों की सॊख्मा औय प्रकाय M के फे चे गए उत्ऩादों की सॊख्मा का अनुऩात
ककतना है?
(A) 20 : 23 (B) 20 : 3 (C) 20 : 17
(D) 3 : 17 (E) None of these

42. If the number of products manufactured of type Q is 66.66% more than


number of products manufactured of type M and ratio of products unsold
of type P and Q is 12 : 15, then find the number of products sold of type
Q.
मदद प्रकाय Q के ननशभषत उत्ऩादों की सॊख्मा M प्रकाय के ननशभषत उत्ऩादों की सॊख्मा से
66.66% अचधक है औय प्रकाय P औय Q के त्रफना त्रफके उत्ऩादों का अनुऩात 12 : 15
है, तो Q प्रकाय के फे चे गए उत्ऩादों की सॊख्मा ऻात कीजजए।
(A) 455 (B) 555 (C) 445
(D) 405 (E) None of these

43. The sum of number of type L and type O products sold is how much
less than sum of number of type M and type N products unsold?
फे चे गए प्रकाय L औय प्रकाय O उत्ऩादों की सॊख्मा का मोग, प्रकाय M औय प्रकाय N के त्रफना
फीके उत्ऩादों की सॊख्मा के मोग से ककतना कभ है?
(A) 340 (B) 325 (C) 320
(D) 308 (E) None of these

44. The number of type P product unsold is what percent of the number of
type N product sold?
फे चे गए प्रकाय P उत्ऩाद की सॊख्मा, फे चे गए प्रकाय N उत्ऩाद की सॊख्मा का ककतना प्रनतित
है?
(A) 35% (B) 45% (C) 25%
(D) 15% (E) None of these

45. What is the average of total number of products unsold of all 5 types
together.
सबी 5 प्रकाय के एक साथ त्रफना त्रफके उत्ऩादों की कु र सॊख्मा का औसत क्मा है?
(A) 761.8 (B) 716.8 (C) 781.8
(D) 711.8 (E) None of these

(46 – 50) Following bar graph shows the expenditure and profit of
company Q through 5 years. All figures are in lakhs.
Profit = Revenue – Expenditure
ननम्नशरणखत फाय ग्राप 5 वर्षों भें कॊ ऩनी Q के व्मम औय राब को दिाषता है। सबी आॊक डे राख
भें हैं ।
राब = याजस्व – व्मम https : https : https : https :
//youtube.co //www.face //instagram.c
m/channel/U book.com/a om/aashishar //t.me/
CYa4_JrOrf8R orasocial?ut
254 5Kz2uOtccXQ
a s hisharora
m_medium = studifie
s ocial
copy_link d
46. In which year the increase in expenditure is maximum with respect to
previous year?
वऩछरे वर्षष की तुरना भें ककस वर्षष व्मम भें वरवि अचधकतभ है?
(A) 2019 (B) 2021 (C) 2020
(D) 2017 (E) None of these

47. Find the ratio of the revenue in 2019 to that in 2021?


2019 के याजस्व का 2021 के याजस्व से अनुऩात ऻात कीजजए।
(A) 4 : 7 (B) 7 : 4 (C) 21 : 19
(D) 19 : 21 (E) None of these

48. The revenue in 2018 is what % of the sum of profit and expenditure in
year 2020?
2018 भें याजस्व, वर्षष 2020 भें राब औय व्मम के मोग का ककतना% है?
(A) 600/7% (B) 129.41% (C) 33.33%
(D) 50% (E) None of these

49. Find the average revenue of all 5 years together.


सबी 5 वर्षों का एक साथ औसत याजस्व ऻात कीजजए।
(A) 94 (B) 98 (C) 84
(D) 88 (E) None of these

50. In which year, the profit % is maximum?


ककस वर्षष, राब% अचधकतभ है?
(A) 2018 (B) 2020 (C) 2021
(D) 2017 (E) None of these

https : https : https : https :


//youtube.co //www.face //instagram.c
m/channel/U book.com/a om/aashishar //t.me/
CYa4_JrOrf8R orasocial?ut
255 5Kz2uOtccXQ
a s hisharora
m_medium = studifie
s ocial
copy_link d
(51 – 55) Following pie chart shows the percentage distribution of runs
scored by 5 different players in an ODI. The line graph shows the number
of sixes hit by respective player. Total runs scored = 360.
ननम्नशरणखत ऩाई चाटष एक ओडीआई भें 5 अरग – अरग णखराडडमों द्वाया फनाए गए यनों के
प्रनतित ववतयण को दिाषता है। राइन ग्राप सॊफॊचधत णखराडी द्वाया रगाए गए छक्कों की
सॊख्मा को दिाषता है। फनाए गए कु र यन = 360.

51. If the total runs of KL Rahul obtained by only fours and sixes then find
the total number of fours he hit?
मदद के एर याहुर के कु र यन के वर चौकों औय छक्कों से प्राप्त हुए, तो उसके द्वाया
रगाए गए चौकों की कु र सॊख्मा ऻात कीजजए।
(A) 12 (B) 15 (C) 18
(D) 10 (E) None of these
https : https : https : https :
//youtube.co //www.face //instagram.c
m/channel/U book.com/a om/aashishar //t.me/
CYa4_JrOrf8R orasocial?ut
256 5Kz2uOtccXQ
a s hisharora
m_medium = studifie
s ocial
copy_link d
52. Find the average runs of all 5 cricketers scored by hitting sixes only?
केवर छक्के भायकय फनाए गए सबी 5 कक्रकेटयों के औसत यन ऻात कीजजए।
(A) 28 (B) 28.8 (C) 27.2
(D) 28.2 (E) None of these

53. Total runs scored by Virat is how much percent more/less than total runs
scored by Dhoni?
ववयाट द्वाया फनाए गए कुर यन, धोनी द्वाया फनाए गए कुर यनों से ककतने प्रनतित
अचधक/कभ हैं ?
(A) 25% (B) 33.33% (C) 15%
(D) 50% (E) None of these

54. How much % more or less is the runs scored by Rohit in sixes than the
total runs scored by Dhawan?
योदहत द्वाया छक्कों भें फनाए गए यन, धवन द्वाया फनाए गए कु र यनों से ककतने प्रनतित अचधक
मा कभ है?
(A) 77.77% (B) 75.45% (C) 77.95%
(D) 76.42% (E) None of these

55. If Dhawan hit 2 fours and Dhoni hit 5 fours then find the difference
between the runs scored without hitting boundaries by Dhawan and Dhoni?
मदद धवन ने 2 चौके औय धोनी ने 5 चौके रगाए, तो धवन औय धोनी द्वाया त्रफना फाउॊ री रगाए
फनाए गए यनों के फीच अॊतय ऻात कीजजए।
(A) 12 (B) 22 (C) 32
(D) 42 (E) None of these

(56 – 60) Table shows the number of students whose age in different ranges
from four different classes. Range of Age
ताशरका उन छात्रों की सॊख्मा दिाषती है जजनकी आमु चाय अरग – अरग कऺाओॊ से
अरग – अरग श्ेणणमों भें है।

Range of age Class 9 Class 10 Class 11 class12

<10 8 0 0 0

≥ 10 and <14 15 10 8 0

≥14 and <16 0 20 18 40

≥16 and <18 0 0 24 25

https : https : https : https :


//youtube.co //www.face //instagram.c
m/channel/U book.com/a om/aashishar //t.me/
CYa4_JrOrf8R orasocial?ut
257 5Kz2uOtccXQ
a s hisharora
m_medium = studifie
s ocial
copy_link d
56. Find that total number of students from all the four classes together
whose age is less than or equal to 16 is what % of total number of
students in class 12?
ऻात कीजजए कक सबी चाय कऺाओॊ के कु र शभराकय 16 से कभ आमु मा फयाफय वारे
ववद्माचथषमों की सॊख्मा, कऺा 12 भें कु र ववद्माचथषमों की सॊख्मा का ककतना प्रनतित है?
(A) 225% (B) 133.33% (C) 150.45%
(D) 183.07% (E) None of these

57. Total number of students in class 11 is what % more than total number
of students in class 9?
कऺा 11 भें छात्रों की कु र सॊख्मा कऺा 9 भें छात्रों की कु र सॊख्मा से ककतना% अचधक
है?
(A) 17.39% (B) 117.39% (C) 19.39%
(D) 150% (E) None of these

58. Find the average age of the students whose age in the range (≥14 and
<16) from all classes together if sum of their ages is 1716.
सबी कऺाओॊ से एक साथ उन छात्रों की औसत आमु ऻात कीजजए जजनकी आमु (≥14 औय
<16) के श्ेणी भें है, मदद उनकी आमु का मोग 1716 है।
(A) 22 (B) 22.5 (C) 24
(D) 24.5 (E) None of these

59. Find ratio of total number of students in all four class together whose
age in the range (<10) to the total number of students on Class 10.
सबी चाय कऺाओॊ भें कु र छात्रों की सॊख्मा जजनकी आमु (<10) की श्ेणी भें है औय कऺा 10 भें
छात्रों की कु र सॊख्मा का अनुऩात ऻात कयें ।
(A) 4 : 11 (B) 26 : 29 (C) 15 : 4
(D) 4 : 15 (E) None of these

60. What is the total number of students in all four classes together?
सबी चाय कऺाओॊ भें शभराकय ववद्माचथषमों की कु र सॊख्मा ककतनी है?
(A) 146 (B) 168 (C) 188
(D) 148 (E) None of these

(61 – 65) Following pie chart shows the percentage distribution of workers
in 5 different factories and the table represents the ratio of male and
female workers in these 5 factories. Total number of workers = 400
ननम्नशरणखत ऩाई चाटष 5 ववशबन्न कायखानों भें श्शभकों के प्रनतित ववतयण को दिाषता है औय
ताशरका इन 5 कायखानों भें ऩुरुर्ष औय भदहरा श्शभकों के अनुऩात को दिाषती है।
श्शभकों की कु र सॊख्मा = 400

https : https : https : https :


//youtube.co //www.face //instagram.c
m/channel/U book.com/a om/aashishar //t.me/
CYa4_JrOrf8R orasocial?ut
258 5Kz2uOtccXQ
a s hisharora
m_medium = studifie
s ocial
copy_link d
Factory Men : Women

A 8:5

B 3:4

C 3:2

D 4:1

E 1:2

61. What is the ratio of sum of number of male workers in factory B and C
to the sum of number of female workers in the same 2 factories?
पैक्ट्री B औय C भें ऩुरुर्ष श्शभकों की सॊख्मा का सभान 2 कायखानों भें भदहरा श्शभकों की
सॊख्मा के मोग से अनुऩात ककतना है?
(A) 1 : 1 (B) 3 : 2 (C) 2 : 5
(D) 1 : 4 (E) None of these
https : https : https : https :
//youtube.co //www.face //instagram.c
m/channel/U book.com/a om/aashishar //t.me/
CYa4_JrOrf8R orasocial?ut
259 5Kz2uOtccXQ
a s hisharora
m_medium = studifie
s ocial
copy_link d
62. Find the average number of female workers in all these 5 factories
together.
इन सबी 5 कायखानों भें शभराकय भदहरा श्शभकों की औसत सॊख्मा ऻात कीजजए।
(A) 31.5 (B) 31.2 (C) 31.8
(D) 31.6 (E) None of these

63. What is the ratio of the difference between the number of male
workers and female workers in factory D and factory A?
पैक्ट्री D औय पैक्ट्री A भें ऩुरुर्ष श्शभकों औय भदहरा श्शभकों की सॊख्मा के फीच के अॊतय का
अनुऩात ककतना है?
(A) 2 : 7 (B) 7 : 2 (C) 5 : 9
(D) 8 : 5 (E) None of these

64. The sum of the number of male workers in factory A and E is what
percent more or less than the sum of the number of female workers in
factory B and C?
कायखाने A औय E भें ऩुरुर्ष श्शभकों की सॊख्मा का मोग, कायखाने B औय C भें भदहरा श्शभकों
की सॊख्मा के मोग से ककतना प्रनतित अचधक मा कभ है?
(A) 75% (B) 56% (C) 58%
(D) 70% (E) None of these

65. The number of female workers in factory E is what % of the total


number of workers in factory B?
पै क्ट्री E भें भदहरा काभगायों की सॊख्मा, पै क्ट्री B भें काभ कयने वारों की कु र सॊख्मा का
ककतना प्रनतित है?
(A) 50% (B) 74% (C) 66.66%
(D) 71.42% (E) None of these

(66 – 70). Table shows the ratio of income of 5 farmers for 3 respective
months and the bar graph shows the difference between the income of
January and March for these 5 respective farmers.
ताशरका 3 भहीनों के शरए 5 ककसानों की आम का अनुऩात ददखाती है औय फाय ग्राप इन्ही 5
ककसानों के शरए जनवयी औय भाचष की आम के फीच अॊतय को दिाषता है।

Farmers January : February : March

A 4:6:9
B 2:8:7
C 1:4:9
D 3:5:7
E 6:4:9
https : https : https : https :
//youtube.co //www.face //instagram.c
m/channel/U book.com/a om/aashishar //t.me/
CYa4_JrOrf8R orasocial?ut
260 5Kz2uOtccXQ
a s hisharora
m_medium = studifie
s ocial
copy_link d
66. What is the average income of all the farmers together for February
month?
पयवयी भहीने भें सबी ककसानों की एकसाथ औसत आम ककतनी है?
(A) 2558 (B) 2670 (C) 2885
(D) 2760 (E) None of these

67. Total incomes of farmers B and C in all 3 months together is what % of


the total incomes of farmers D and E in all 3 months together?
सबी 3 भहीनों भें ककसान B औय C की कु र आम, सबी 3 भहीनों भें ककसान D औय E की कु र
आम का ककतना प्रनतित है?
(A) 70.45 % (B) 25% (C) 70.58%
(D) 33.33% (E) None of these

68. Total incomes of farmers A, B and C together in month January is what


percent more/less than the total income of these 3 farmers together in
month March?
जनवयी भाह भें ककसानों A, B औय C की कु र आम, भाचष भाह भें इन 3 ककसानों की कु र आम
से ककतने प्रनतित अचधक/कभ है?
(A) 50% (B) 70% (C) 70.47%
(D) 33.33% (E) None of these

69. Total income of E in February is decreased by what % over his income


in month January?
पयवयी भहीने भें E की कु र आम भें जनवयी भहीने की कु र आम से ककतने प्रनतित कभी हुई
है? https : https : https : https :
//youtube.co //www.face //instagram.c
m/channel/U book.com/a om/aashishar //t.me/
CYa4_JrOrf8R orasocial?ut
261 5Kz2uOtccXQ
a s hisharora
m_medium = studifie
s ocial
copy_link d
(A) 50% (B) 25% (C) 66.66%
(D) 33.33% (E) None of these

70. What is the ratio of total income of farmers A, D and E together in


month March to the total income of these 3 farmers together in month
January?
भाचष भहीने भें ककसानों A, D औय E की कु र आम का जनवयी भहीने भें इन 3 ककसानों की कु र
आम से अनुऩात ककतना है?
(A) 66.35 (B) 66 : 38 (C) 95 : 56
(D) 35 : 66 (E) None of these

(71 – 75) The pie chart shows the degree distribution of total number of
Aspirants of SSC and Bank in 5 different coaching and the line chart
represents the percentage by which SSC aspirants are more than Banking
Aspirants. Total Number of Aspirants = 10800.
ऩाई चाटष 5 अरग – अरग कोचच ॊग भें एसएससी औय फैंक के उम्भीदवायों की कु र सॊख्मा का
डडग्री ववतयण ददखाता है औय राइन चाटष उस प्रनतित का प्रनतननचधत्व कयता है जजसके द्वाया
एसएससी उम्भीदवाय फैंककॊ ग उम्भीदवायों से अचधक हैं । उम्भीदवायों की कु र सॊख्मा = 10800।

https : https : https : https :


//youtube.co //www.face //instagram.c
m/channel/U book.com/a om/aashishar //t.me/
CYa4_JrOrf8R orasocial?ut
262 5Kz2uOtccXQ
a s hisharora
m_medium = studifie
s ocial
copy_link d
71. Find the ratio of Number of Banking Aspirants from A, B and C with the
number of SSC Aspirants in all the coaching together?
A, B औय C से फैंककॊग उम्भीदवायों की सॊख्मा का सबी कोचच ॊग भें SSC उम्भीदवायों की सॊख्मा के
साथ अनुऩात ऻात कीजजए।
(A) 6067 : 2475 (B) 2475 : 6067 (C) 2475 : 6867
(D) 6867 : 2475 (E) None of these

72. If coaching F has 41(2/3) % more SSC aspirants than number of SSC
aspirants in coaching A and coaching F has 20% less banking Aspirants
than number of Banking aspirants of Coaching C, then find total number of
aspirants in coaching F?
मदद कोचच ॊग F भें कोचच ॊग A भें SSC उम्भीदवायों की सॊख्मा से 41(2/3) % अचधक SSC
उम्भीदवाय हैं औय कोचच ॊग F भें कोचच ॊग C के फैंककॊ ग उम्भीदवायों की सॊख्मा से 20% कभ
फैंककॊग उम्भीदवाय हैं , तो कोचच ॊग F भें उम्भीदवायों की कुर सॊख्मा ऻात कयें ?
(A) 1514 (B) 1785 (C) 1815
(D) 1458 (E) None of these

73. Number of SSC aspirants in coaching E is how much percent of number


of banking aspirants in coaching D?
कोचच ॊग E भें SSC उम्भीदवायों की सॊख्मा कोचच ॊग D भें फैंककॊग उम्भीदवायों की सॊख्मा का ककतना
प्रनतित है?
(A) 87.36% (B) 78.36% (C) 87.66%
(D) 77.36% (E) None of these

74. If the number of Banking aspirants of coaching B are increased by


13.33% and SSC aspirants decreased by 12.5% then total number of
students in coaching B is how much percent of number of students of
coaching D?
मदद कोचच ॊग B के फैंककॊग उम्भीदवायों की सॊख्मा भें 13.33% की वरवि की जाती है औय SSC
उम्भीदवायों की सॊख्मा भें 12.5% की कभी होती है, तो कोचच ॊग B भें छात्रों की कु र सॊख्मा कोचच ॊग
D के छात्रों की सॊख्मा का ककतना प्रनतित है?
(A) 84 2/3% (B) 77 2/3% (C) 75 2/3%
(D) 74 2/3% (E) None of these

75. Find the difference of number of SSC aspirants in coaching C, D and E


and number of banking aspirants in coaching A, B and C?
कोचच ॊग C, D औय E भें एसएससी उम्भीदवायों की सॊख्मा औय कोचच ॊग A, B औय C भें फैंककॊ ग
उम्भीदवायों की सॊख्मा का अॊतय ऻात कीजजए।
(A) 1492 (B) 1568 (C) 1522
(D) 1652 (E) None of these

https : https : https : https :


//youtube.co //www.face //instagram.c
m/channel/U book.com/a om/aashishar //t.me/
CYa4_JrOrf8R orasocial?ut
263 5Kz2uOtccXQ
a s hisharora
m_medium = studifie
s ocial
copy_link d
(76 – 80) The table shows the 5 families consisting of Father, mother and
children. It also shows father’s age, mother’s age and average age of
families.
Average Age of Family = (Father’s Age + Mother’s Age + Sum of Children
Age) /3
ताशरका भें 5 ऩरयवायों को ददखामा गमा है जजनभें वऩता, भाता औय फच्चे िाशभर हैं । मह वऩता
की आमु, भाता की आमु औय ऩरयवायों की औसत आमु को बी दिाषता है।
ऩरयवाय की औसत आमु = (वऩता की आमु + भाता की आमु + फच्चों की आमु का मोग) /3

Families Average age Father’s age Mother’s age

A 45 52 48

B 52 56 54

C 38 47 38

D 47 41 39

E 40 45 35

76. If Family A has 2 children and Family B has 3 children, then by what
percent average age of children of family A is more/less than average age
of children of Family B?
मदद ऩरयवाय A के 2 फच्चे हैं औय ऩरयवाय B के 3 फच्चे हैं , तो ऩरयवाय A के फच्चों की औसत
आमु, ऩरयवाय B के फच्चों की औसत आमु से ककतने प्रनतित अचधक/कभ है?
(A) 14% (B) 16% (C) 18%
(D) 20% (E) None of these

77. Find the average age of all the children if each family has 2 children?
मदद प्रत्मे क ऩरयवाय भें 2 फच्चे हैं , तो सबी फच्चों की औसत आमु ऻात कीजजए।
(A) 22.1 years (B) 20.1 years (C) 21.1 years
(D) 23.1 years (E) None of these

78. If Family B has two children, one boy and one girl if the age of boy is
9.09% more than age of the girl then find the difference between their
ages?
मदद ऩरयवाय B के दो फच्चे हैं , एक रडका औय एक रडकी, मदद रडके की आमु रडकी की आमु
से 9.09% अचधक है, तो उनकी आमु के फीच का अॊतय ऻात कीजजए।
https : https : https : https :
//youtube.co //www.face //instagram.c
m/channel/U book.com/a om/aashishar //t.me/
CYa4_JrOrf8R orasocial?ut
264 5Kz2uOtccXQ
a s hisharora
m_medium = studifie
s ocial
copy_link d
(A) 2.5 years (B) 3 years (C) 4 years
(D) 2 years (E) None of these

79. Find the ratio of sum of ages of children of Family A,B and C with the
sum of age of Fathers of family B,C,D and E?
ऩरयवाय A, B औय C के फच्चों की आमु के मोग का ऩरयवाय B, C, D औय E के वऩता
की आमु के मोग से अनुऩात ऻात कीजजए।
(A) 189 : 110 (B) 110 : 189 (C) 278 : 196
(D) 196 : 278 (E) None of these

80. Find the average age of all the mothers from all the families?
सबी ऩरयवायों की सबी भाताओॊ की औसत आमु ऻात कीजजए।
(A) 44.8 years (B) 42.8 years (C) 45.8 years
(D) 46.8 years (E) None of these

(81 – 85). The bar graph show the percentage by which number of I – pad
sold is more than the number of I – phone sold and difference between
number of I – pad and I – phone sold by 5 different shops.
फाय ग्राप उस प्रनतित को, जजसके द्वाया फे चे गए I – pad की सॊख्मा फे ची गई I – phone की
सॊख्मा से अचधक है औय 5 अरग – अरग दुक ानों द्वाया फे चे गए I – pad औय I – phone की
सॊख्मा के फीच के अॊतय को दिाषता है ।

https : https : https : https :


//youtube.co //www.face //instagram.c
m/channel/U book.com/a om/aashishar //t.me/
CYa4_JrOrf8R orasocial?ut
265 5Kz2uOtccXQ
a s hisharora
m_medium = studifie
s ocial
copy_link d
81. Total number of I – phone sold by Shop P is what percent of the average
number of I – pad sold by all the 5 Shops?
दक
ु ान P द्वाया फे चे गए I – phone की कुर सॊख्मा सबी 5 दक
ु ानों द्वाया फे चे गए I – pad की
औसत सॊख्मा का ककतना प्रनतित है?
(A) 122.9% (B) 102.9% (C) 112.9%
(D)132.9% (E) None of these

82. Total number of I – pad sold by Shop P, Q and R is how much more than
total number of I – phone sold by Shop R, S and T?
दुक ान P, Q औय R द्वाया फे चे गए I – pad की कु र सॊख्मा, दुक ान R, S औय T द्वाया फे चे
गए I – phone की कु र सॊख्मा से ककतनी अचधक है?
(A) 44.24% (B) 64.25% (C) 84.26%
(D) 104.27% (E) None of these

83. What is the ratio of total number of I – pad and I – phone sold by Shop T
to the total number of I – pad and I – phone sold by Shop R?
दुक ान T द्वाया फे चे गए I – pad औय I – phone की कु र सॊख्मा का दुक ान R द्वाया फे चे गए I
– pad औय I – phone की कुर सॊख्मा से अनुऩात ककतना है?
(A) 15 : 11 (B) 11 : 15 (C) 14 : 11
(D) 11 : 14 (E) None of these

84. What is the difference between average number of I – pad sold by Shop
P, Q and T and average of total number of I – phone sold by all the shops?
दुक ान P, Q औय T द्वाया फे चे गए I – pad की औसत सॊख्मा औय सबी दुक ानों द्वाया फे चे गए I
– phone की कुर सॊख्मा के औसत के फीच का अॊतय ककतना है?
(A) 26.6 (B) 16.6 (C) 46.6
(D) 36.6 (E) None of these

85. Total number of I – phone sold by Shop T is how much percent less than
total number of I – pad sold by Shop Q?
दक
ु ान T द्वाया फे चे गए I – phone की कु र सॊख्मा, दक
ु ान Q द्वाया फे चे गए I – pad की कु र
सॊख्मा से ककतना प्रनतित कभ है?
(A) 25% (B) 24% (C) 34%
(D) 44% (E) None of these

(86 – 90) Table shows the number of bikes manufactured and percentage of
it sold by 5 companies.
ताशरका ननशभषत फाइक की सॊख्मा औय 5 कॊऩननमों द्वाया फे ची गई फाइक का प्रनतित दिाषता है।

https : https : https : https :


//youtube.co //www.face //instagram.c
m/channel/U book.com/a om/aashishar //t.me/
CYa4_JrOrf8R orasocial?ut
266 5Kz2uOtccXQ
a s hisharora
m_medium = studifie
s ocial
copy_link d
Company Number of bikes manufactured %age of bikes sold

P 17400 65%

Q 13500 –––

R ––– 85%

S 18400 –––

T ––– 40%

U ––– 70%

V 19200 80%

86. If the number of bikes manufactured by T and U together is 1.5 times


the number of bikes manufactured by P and the ratio of number of bikes
manufactured by T and U is 15 : 14 then what is the difference between
unsold bikes by T and U?
मदद T औय U द्वाया ननशभषत फाइकों की सॊख्मा, P द्वाया ननशभषत फाइक की सॊख्मा का 1.5 गुना
है औय T औय U द्वाया ननशभषत फाइक की सॊख्मा का अनुऩात 15 : 14 है, तो T औय U द्वाया
त्रफना त्रफकी फाइक के फीच का अॊतय क्मा है?
(A) 4320 (B) 3510 (C) 3420
(D) 4410 (E) None of these

87. If the ratio of number of bikes produced by company R, Q and T is 58 :


45 : 37 and 30% of bikes are unsold by company Q, then what is the
average number of bikes sold by Company R, Q and T?
मदद कॊऩनी R, Q औय T द्वाया उत्ऩाददत फाइक की सॊख्मा का अनुऩात 58 : 45 : 37 है औय
कॊ ऩनी Q द्वाया 30% फाइक नहीॊ त्रफकी हैं , तो कॊ ऩनी R, Q औय T द्वाया फे ची गई फाइक की
औसत सॊख्मा क्मा है?
(A) 9560 (B) 9760 (C) 9460
(D) 9960 (E) None of these

88. If 60% of bikes manufactured is sold by company Q and the number of


bikes manufactured by company U is 16500. the number of bikes sold
company P and Q together is what percent more or less than the number
of bikes sold by company
https : U and V together?
https : https : https :
//youtube.co //www.face //instagram.c
m/channel/U book.com/a om/aashishar //t.me/
CYa4_JrOrf8R orasocial?ut
267 5Kz2uOtccXQ
a s hisharora
m_medium = studifie
s ocial
copy_link d
मदद ननशभषत फाइक का 60% कॊ ऩनी Q द्वाया फे चा जाता है औय कॊ ऩनी U द्वाया ननशभषत फाइक
की सॊख्मा 16500 है। कॊ ऩनी P औय Q को शभराकय फे ची गई फाइक की सॊख्मा कॊ ऩनी U औय
V द्वाया फे ची गई फाइक की सॊख्मा से ककतना प्रनतित अचधक मा कभ है?
(A) 30.87% (B) 28.87% (C) 37.87%
(D) 27.87% (E) None of these

89. What is the ratio of number of bikes sold by company P to the number
of bikes sold by company Q, if 70% of manufactured bikes is sold by
company Q?
कॊ ऩनी P द्वाया फे ची गई फाइक की सॊख्मा का कॊ ऩनी Q द्वाया फे ची गई फाइक की
सॊख्मा से अनुऩात ककतना है, मदद कॊ ऩनी Q द्वाया ननशभषत फाइक का 70% फे चती है?
(A) 945 : 1131 (B) 1131 : 945 (C) 1544 : 925
(D) 925 : 1544 (E) None of these

90. If number of bikes manufactured by R and T is 30% and 45% more


than that of company S and V, then what is the number of unsold bikes by
company R and T together?
मदद R औय T द्वाया ननशभषत फाइकों की सॊख्मा कॊऩनी S औय V की तुरना भें 30% औय 45%
अचधक है, तो कॊ ऩनी R औय T द्वाया त्रफना त्रफकी फाइकों की सॊख्मा ककतनी है?
(A) 25292 (B) 20292 (C) 20452
(D) 23520 (E) None of these

(91 – 95) Pie chart shows the percentage distribution of income of 5


employee in a office out of total income 2 Lakhs and bar graph shows the
expenditure of those employees.
ऩाई चाटष एक कामाषरम भें कु र 2 राख आम भें से 5 कभषचारयमों की आम का प्रनतित ववतयण
दिाषता है औय फाय ग्राप उन कभषचारयमों के व्मम को दिाषता है।

https : https : https : https :


//youtube.co //www.face //instagram.c
m/channel/U book.com/a om/aashishar //t.me/
CYa4_JrOrf8R orasocial?ut
268 5Kz2uOtccXQ
a s hisharora
m_medium = studifie
s ocial
copy_link d
91. Which employee saves the most amount and how much out of his
income?
कौन सा कभषचायी सफसे अचधक याशि फचाता है औय ककतनी याशि फचाता है?
(A) R, 16000 (B) Q, 8000 (C) P,16000
(D) S, 7200 (E) None of these

92. What is the difference between average of savings of P, Q and R and


the average saving of S and T?
P, Q औय R की औसत फचत औय S औय T की औसत फचत के फीच ककतना अॊतय है?
(A) 733.33 (B) 833.66 (C) 866.66
(D) 766.66 (E) None of these

93. If ‘A’ is the value of ratio of expenditure of R and T with saving of Q


and S then find the approximate value of ‘A’?
मदद ‘A' एक साथ R औय T के व्मम का एक साथ Q औय S की फचत का अनुऩात है तो ‘A'
का अनुभाननत भान ऻात कयें ?
(A) 2.61 (B) 3.51 (C) 2.45
(D) 2.88 (E) None of these

94. Total expenditure of P and R is how much percent more than the
Savings of Q and T?
P औय R का कु र व्मम, Q औय T की फचत से ककतना प्रनतित अचधक है?
(A) 150% (B) 120% (C) 140%
(D) 100% (E) None of these

https : https : https : https :


//youtube.co //www.face //instagram.c
m/channel/U book.com/a om/aashishar //t.me/
CYa4_JrOrf8R orasocial?ut
269 5Kz2uOtccXQ
a s hisharora
m_medium = studifie
s ocial
copy_link d
95. If another employee A saves 30% more than savings of R and spends
Rs. 12500 more than P then monthly income of R is how much percent of
monthly income of A?
मदद एक अन्म कभषचायी A, R की फचत से 30% अचधक फचाता है औय P से 12500 रु. अचधक
खचष कयता है, तो R की भाशसक आम, A की भाशसक आम का ककतना प्रनतित है?
(A) 58.9% (B) 56.9% (C) 48.9%
(D) 68.9% (E) None of these

(96 – 100) Table shows the number of employees, percentage of number


males more than number of females and percentage of number of vacant
post in those companies.
ताशरका कभषचारयमों की सॊख्मा, भदहराओॊ की सॊख्मा से अचधक ऩुरुर्षों का प्रनतित औय उन
कॊ ऩननमों भें रयक्त ऩदों की सॊख्मा का प्रनतित दिाषती है।
A. 60%
Number of total %age male more than %age of
B. 50%
Company
employee working females vacant post

C. 55%
Infosys 160 28.56% 20%

D. 25%TCS 90 14.28% 10%

E. None
IBM
of ese 210 33⅓% 30%

Wipro 290 41⅔% 42%

Genpact 190 37.5% 5%

96. Total number of vacant seat in Genpact are what percent of total
number of vacant post in Infosys?
जे नऩैक्ट भें रयक्त सीटों की कु र सॊख्मा, इॊप ोशसस भें रयक्त ऩदों की कु र सॊख्मा का ककतना
प्रनतित है?
(A) 60% (B) 50% (C) 55%
(D) 25% (E) None of these

97. Find the average of total number of males in all the companies?
सबी कॊ ऩननमों भें ऩुरुर्षों की कु र सॊख्मा का औसत ऻात कीजजए।
(A) 108.6 (B) 105.2 (C) 109.4
(D) 107.6 (E) None of these

https : https : https : https :


//youtube.co //www.face //instagram.c
m/channel/U book.com/a om/aashishar //t.me/
CYa4_JrOrf8R orasocial?ut
270 5Kz2uOtccXQ
a s hisharora
m_medium = studifie
s ocial
copy_link d
98. Number of Females in TCS, Genpact and Infosys are how much
percent more/less than number of Male in Wipro and IBM?
TCS, जे नऩैक्ट औय इॊपोशसस भें भदहराओॊ की सॊख्मा ववप्रो औय आईफीएभ भें ऩुरुर्षों की सॊख्मा
से ककतने प्रनतित अचधक/कभ है?
(A) 33.79% (B) 35.78% (C) 32.14%
(D) 34.32% (E) None of these

99. Find the ratio of total number of employee working in IBM and
Genpact with the total number of females in all the companies?
आईफीएभ औय जे नऩैक्ट भें कामषय त कभषचारयमों की कु र सॊख्मा का सबी कॊ ऩननमों भें
भदहराओॊ की कु र सॊख्मा से अनुऩात ऻात कीजजमे ?
(A) 201 : 200 (B) 200 : 201 (C) 200 : 301
(D) 301 : 200 (E) None of these

100. If Sandisk has 40% more post than total post in Wipro and all post
are full, then find total number of females working in Sandisk, if males are
33.33% more than number of females?
मदद सैंडडस्क के ऩास ववप्रो भें कु र ऩद से 40% अचधक ऩद हैं औय सबी ऩद बये हुए हैं , तो
सैंडडस्क भें काभ कयने वारी भदहराओॊ की कु र सॊख्मा ऻात कीजजए, मदद ऩुरुर्ष भदहराओॊ की
सॊख्मा से 33.33% अचधक हैं ?
(A) 400 (B) 300 (C) 350
(D) 250 (E) None of these

(101 – 105) Bar graph shows number of books purchased by 5


shopkeepers of two type i.e. Story and Sci – Fi and line chart show the
percentage of books sold by them.
फाय ग्राप 5 दुक ानदायों द्वाया खयीदी गई दो प्रकाय की ऩुस्तकों की सॊख्मा को दिाषता है अथाषत
कहानी औय ववऻान – कथा औय राइन चाटष उनके द्वाया फे ची गई ऩुस्तकों का प्रनतित दिाषता
है।

https : https : https : https :


//youtube.co //www.face //instagram.c
m/channel/U book.com/a om/aashishar //t.me/
CYa4_JrOrf8R orasocial?ut
271 5Kz2uOtccXQ
a s hisharora
m_medium = studifie
s ocial
copy_link d
Story Sci-Fi
12000
10000 9600 9400
8400 8500 8000 8200
7200 8000
8000 7000 6400
6000
4000
2000
0
P Q R S T
101. Find the ratio of number of story books sold by P, Q and R with the
number of Sci – fi books sold by S and T?
P, Q औय R द्वाया फे ची गई कहानी की ऩुस्तकों की सॊख्मा का S औय T द्वाया फे ची गई
ववऻान – कथा ऩुस्तकों की सॊख्मा से अनुऩात ऻात कीजजए।
(A) 2061 : 1310 (B) 1310 : 2261 (C) 2261 : 1310
(D) 1310 : 2061 (E) None of these

102. Find the difference between average number of Story books sold by
all the shopkeepers and average number of Sci – Fi books sold by all the
shopkeepers together?
सबी दुक ानदायों द्वाया फे ची गई कहानी की ककताफों की औसत सॊख्मा औय सबी
दुक ानदायों द्वाया फे ची गई ववऻान – कथा ऩुस्तकों की औसत सॊख्मा के फीच का अॊतय
ऻात कीजजए।
(A) 87.4 (B) 4050.6 (C) 580
(D) 492.6 (E) None of these

103. If shopkeeper A purchased 26⅔% more story books than number of


story books sold by P and A purchased 43.75% more Sci – Fi books than
number of Sci – Fi books sold by S then find the average number of
books purchased by A?
मदद दुक ानदाय A ने P द्वाया फे ची गई कहानी की ऩुस्तकों की सॊख्मा से 26⅔% अचधक कहानी
की ककताफें खयीदीॊ औय दकू ानदाय A ने S द्वाया फे ची गई ववऻान – कथा की ऩुस्तकों की सॊख्मा
से 43.75% अचधक ववऻान – कथा ऩुस्तकें खयीदीॊ, तो A द्वाया खयीदी गई ऩुस्तकों की औसत
सॊख्मा ऻात कयें ? https : https : https : https :
//youtube.co //www.face //instagram.c
m/channel/U book.com/a om/aashishar //t.me/
CYa4_JrOrf8R orasocial?ut
272 5Kz2uOtccXQ
a s hisharora
m_medium = studifie
s ocial
copy_link d
(A) 8467 (B) 8458 (C) 8869
(D) 8268 (E) None of these

104. If number of Sci – fi books sold by S increased by 58⅓% and number


of story books sold by S is decreased by 6.25% then find the difference
between Story books and Sci – Fi books sold by S now?
मदद S द्वाया फे ची गई ववऻान – कथा ऩुस्तकों की सॊख्मा भें 58⅓% की वरवि होती है
औय S द्वाया फे ची गई कहानी की ऩुस्तकों की सॊख्मा भें 6.25% की कभी होती है, तो
S द्वाया अफ फे ची गई कहानी की ऩुस्तकों औय ववऻान – कथा ऩुस्तकों के फीच का
अॊतय ऻात कीजजए।
(A) 4523 (B) 5203 (C) 4056
(D) 4587 (E) None of these

105. Find by how much percent number of books sold by P are more than
number of books sold by T?
ऻात कीजजए कक P द्वाया फे ची गई ऩुस्तकों की सॊख्मा T द्वाया फे ची गई ऩुस्तकों की सॊख्मा से
ककतने प्रनतित अचधक है?
(A) 16.53% (B) 12.94% (C) 18.52%
(D) 38.9% (E) None of these

(106 – 110) Pie chart shows the total marks out of 300 in three subjects
Maths, English and Science each has maximum marks 100 and bar graph
shows the ratio of marks in these subjects.
ऩाई चाटष तीन ववर्षमों भें 300 भें से कु र अॊक ददखाता है जो की गणणत, अॊग्रेज ी औय ववऻान है,
प्रत्मे क के अचधकतभ अॊक 100 हैं औय फाय ग्राप इन ववर्षमों भें अॊकों के अनुऩात को दिाषता है।

https : https : https : https :


//youtube.co //www.face //instagram.c
m/channel/U book.com/a om/aashishar //t.me/
CYa4_JrOrf8R orasocial?ut
273 5Kz2uOtccXQ
a s hisharora
m_medium = studifie
s ocial
copy_link d
106. Find the ratio of average marks in Science scored by Shashi, Meenal
and Krati with the Average of marks scored by all students in English?
िशि, भीनर औय कर नत द्वाया ववऻान भें प्राप्त औसत अॊक ों का अॊग्रेज ी भें सबी छात्रों द्वाया
प्राप्त अॊकों के औसत से अनुऩात ऻात कीजजमे ?
(A) 311 : 325 (B) 325 : 311 (C) 285 : 287
(D) 301 : 200 (E) None of these

107. If the passing marks are 60% of total marks then what percent of
student failed in at least one subject?
मदद उत्तीणष अॊक कु र अॊक ों के 60% हैं , तो ककतने प्रनतित छात्र कभ से कभ एक
ववर्षम भें अनुत्तीणष हुए?
(A) 40% (B) 20% (C) 50%
(D) 60% (E) None of these

108. find the difference between average number scored by all the
students in Maths and average number scored by all students in Science?
गणणत भें सबी छात्रों द्वाया प्राप्त औसत सॊख्मा औय ववऻान भें सबी छात्रों द्वाया प्राप्त औसत
सॊख्मा के फीच का अॊतय ऻात कीजजए।
(A) 8 (B) 7 (C) 6
(D) 5 (E) None of these

109. If the marks scored by Soniya in Maths is 41⅔% less than the marks
scored by Krati in Maths and marks scored by Soniya in English is 20
more than marks scored in Maths by her, if marks scored by her in
Science is average of marks scored by her in Maths and English then find
the percentage of marks she got in the examination?
मदद सोननमा द्वाया गणणत भें प्राप्त अॊक कर नत द्वाया गणणत भें प्राप्त अॊक ों से 41⅔% कभ है
औय सोननमा द्वाया अॊग्रेजी भें प्राप्त अॊक उसके द्वाया गणणत भें प्राप्त अॊकों से 20 अचधक हैं ,
मदद ववऻान भें उसके द्वाया प्राप्त अॊक , उसके द्वाया गणणत औय अॊग्रेज ी भें प्राप्त अॊक ों का
औसत है, तो ऩयीऺा भें प्राप्तhttps
अॊक : ों का प्रनतितhttps
ऻात: कीजजए। https : https :
//youtube.co //www.face //instagram.c
m/channel/U book.com/a om/aashishar //t.me/
CYa4_JrOrf8R orasocial?ut
274 5Kz2uOtccXQ
a s hisharora
m_medium = studifie
s ocial
copy_link d
(A) 79% (B) 59% (C) 49%
(D) 69% (E) None of these

110. Marks scored by Mahima in English is what percent more than marks
scored by Sameeksha in Maths?
भदहभा द्वाया अॊग्रेज ी भें प्राप्त अॊक, सभीऺा द्वाया गणणत भें प्राप्त अॊक ों से ककतने
प्रनतित अचधक है?
(A) 6% (B) 8% (C) 9%
(D) 10% (E) None of these

(111 – 115) Bar graph shows the number of people like Swimming and
Basketball in a city in 5 different years. It also shows percentage of
persons who like football.
Total number of person = number of people like (Swimming + Basketball +
Football)
फाय ग्राप 5 अरग – अरग वर्षों भें एक िहय भें तैय ाकी औय फास्केटफॉर ऩसॊद कयने वारे रोगों
की सॊख्मा को दिाषता है। मह उन रोगों का प्रनतित बी दिाषता है जो पु टफॉर ऩसॊद कयते हैं ।
व्मजक्तमों की कु र सॊख्मा = ऩसॊद कयने वारे रोगों की सॊख्मा (तैय ाकी फास्केटफॉर फु टफॉर)

111. Find the ratio of number of person who like Football with the number
of person who like Swimming in all the years?
पु टफॉर ऩसॊद कयने वारे व्मजक्तमों की सॊख्मा का सबी वर्षों भें तैय ाकी ऩसॊद कयने वारे
व्मजक्तमों की सॊख्मा से अनुऩात ऻात कीजजमे ?
(A) 1452 : 1651 (B) 1841 : 1826 (C) 1826 : 1841
(D) 1651 : 1452 (E) None of these

112. The number of person who like basketball in 2012 is what percent
more than the number of person who likes swimming in 2013?
2012 भें फास्के टफॉर ऩसॊद कयने वारे व्मजक्तमों की सॊख्मा, 2013 भें तैय ाकी ऩसॊद कयने वारे
व्मजक्तमों की सॊख्मा से ककतना
https प्रनतित
: अचधकhttps
है? : https : https :
//youtube.co //www.face //instagram.c
m/channel/U book.com/a om/aashishar //t.me/
CYa4_JrOrf8R orasocial?ut
275 5Kz2uOtccXQ
a s hisharora
m_medium = studifie
s ocial
copy_link d
(A) 58.7% (B) 68.7% (C) 88.7%
(D) 78.7% (E) None of these

113. If number of person who like Basketball in year 2011 reduced by


46⅔% and number of people who like Swimming increases by 20% and
football player remains unchanged then find the total number of players in
2011?
मदद वर्षष 2011 भें फास्के टफॉर ऩसॊद कयने वारे व्मजक्तमों की सॊख्मा 46⅔% कभ हो
जाती है औय तैय ाकी ऩसॊद कयने वारों की सॊख्मा 20% फढ जाती है औय पु टफॉर
णखराडी अऩरयवनतषत यहता है, तो 2011 भें णखराडडमों की कु र सॊख्मा ऻात कीजजए।
(A) 625 (B) 575 (C) 500
(D) 610 (E) None of these

114. During a tournament in 2012 all person are playing, 20% of Swimming
players, 12/137 of the Basketball players and 7/13 of the Football players
got injured then how much percent of total players left uninjured?
2012 भें एक टूनाषभेंट के दौयान सबी व्मजक्तखेर यहे हैं , 20% तैय ाकी णखराडी, 12/137
फास्केटफॉर णखराडी औय 7/13 पु टफॉर णखराडी घामर हो गए, तो कु र णखराडडमों का ककतना
प्रनतित िे र्ष ठीक फचे है?
(A) 80% (B) 78.5% (C) 76.4%
(D) 90% (E) None of these

115. What is the difference between number of person who like Basketball
and Football in 2014 and number of person who like Swimming and
Basketball in 2010?
2014 भें फास्के टफॉर औय पु टफॉर ऩसॊद कयने वारे व्मजक्तमों की सॊख्मा औय 2010 भें तैय ाकी
औय फास्केटफॉर ऩसॊद कयने वारे व्मजक्तमों की सॊख्मा के फीच का अॊतय ककतना है?
(A) 380 (B) 360 (C) 340
(D) 320 (E) None of these

(116 – 120) Pie charts shows percentage distribution of the distance


travelled in by 5 Boats. Table shows the speed of those Boats in upstream
and downstream. Total distance is 2400 km.
ऩाई चाटष 5 नावों द्वाया तम की गई दयू ी का प्रनतित ववतयण दिाषता है। ताशरका उन नावों की
अऩस्ट्रीभ औय डाउनस्ट्रीभ भें गनत को ददखाता है। कु र दुय ी = 2400 ककभी।

https : https : https : https :


//youtube.co //www.face //instagram.c
m/channel/U book.com/a om/aashishar //t.me/
CYa4_JrOrf8R orasocial?ut
276 5Kz2uOtccXQ
a s hisharora
m_medium = studifie
s ocial
copy_link d
Downstream
Boat name Upstream speed
speed
A 24 8
B 32 12
C 18 6
D 22 4
E 19 13

116. Find the approximate average time taken by all the Boats in still
water?
िाॊत जर भें सबी नावों द्वाया शरमा गमा अनुभाननत औसत सभम ऻात कीजजए।
(A) 37 hr (B) 39 hr (C) 30 hr
(D) 43 hr (E) None of these

https : https : https : https :


//youtube.co //www.face //instagram.c
m/channel/U book.com/a om/aashishar //t.me/
CYa4_JrOrf8R orasocial?ut
277 5Kz2uOtccXQ
a s hisharora
m_medium = studifie
s ocial
copy_link d
117. What is the ratio of time taken by Boat A in still water and time taken
by Boat D in Downstream?
नाव A द्वाया िाॊत जर भें शरए गए सभम औय नाव D द्वाया अनुप्रवाह भें शरए गए सभम का
अनुऩात ककतना है?
(A) 18 : 25 (B) 25 : 18 (C) 20 : 33
(D) 33 : 20 (E) None of these

118. If the speed of Boat C in still water is increased by 41⅔% and the
speed of Boat C in upstream is decreased by 16⅔% , then find the time
taken by Boat C in covering 3596 km Downstream?
मदद िाॊत जर भें नाव C की गनत भें 41⅔% की ववर ि की जाती है औय धाया के प्रनतकू र नाव C
की गनत भें 16⅔% की कभी की जाती है, तो नाव C द्वाया डाउनस्ट्रीभ 3596 ककभी की दयू ी
तम कयने भें शरमा गमा सभम ऻात कीजजए।
(A) 124 hrs (B) 134 hrs (C) 128 hrs
(D) 126 hrs (E) None of these

119. By how much percent time taken by boat C in still water is less than
the time taken by Boat A in Downstream?
नाव C द्वाया िाॊत जर भें शरमा गमा सभम, नाव A द्वाया डाउनस्ट्रीभ भें शरए गए सभम से
ककतना प्रनतित कभ है?
(A) 16.66% (B) 33.33% (C) 27.27%
(D) 36.36% (E) None of these

120. If the time taken by Boat B in upstream is equal to the time taken by
Boat F in downstream. If Boat F covers 1240 km in that time and speed of
boat in still water is 58⅓% more than the speed of stream then find the
time taken by Boat F in covering 1652 km upstream?
मदद नाव B द्वाया धाया के प्रनतकू र शरमा गमा सभम नाव F द्वाया अनुप्रवाह भें शरए गए
सभम के फयाफय है। मदद नाव F उस सभम भें 1240 ककभी की दयू ी तम कयती है औय िाॊत ऩानी
भें नाव की गनत धाया की गनत से 58⅓% अचधक है, तो नाव F द्वाया 1652 ककभी धाया के
ववरुि की दयू ी तम कयने भें शरमा गमा सभम ऻात कयें ?
(A) 252 (B) 236 (C) 242
(D) 258 (E) None of these

(121 – 125) One pie chart show the total number of flats (3BHK + 2BHK)
sold in 5 different societies and other show the number of 2BHK flats sold
in those societies. Total flats sold = 1200 and 2 BHK Flats sold = 800.
एक ऩाई चाटष 5 ववशबन्न सोसाइदटमों भें फे चे गए फ्रैटों की कु र सॊख्मा (3BHK + 2BHK)
ददखाता है औय अन्म उन सोसाइदटमों भें फे चे गए 2BHK फ्रैटों की सॊख्मा को दिाषता है। फे चे
गए कु र फ्रैट = 1200 औय 2 BHK फे चे गए फ्रैट = 800।

https : https : https : https :


//youtube.co //www.face //instagram.c
m/channel/U book.com/a om/aashishar //t.me/
CYa4_JrOrf8R orasocial?ut
278 5Kz2uOtccXQ
a s hisharora
m_medium = studifie
s ocial
copy_link d
121. Number of 3 BHK flats sold by C are what percent more than number
of 3 BHK flat sold by E?
C द्वाया फे चे गए 3 BHK फ्रैटों की सॊख्मा, E द्वाया फे चे गए 3 BHK फ्रैटों की सॊख्मा से
ककतने प्रनतित अचधक है?
(A) 165% (B) 145% (C) 185%
(D) 175% (E) None of these

122. If number of 3BHK flats sold by D increased by 58⅓% and 2 BHK flats
sold by D decreased by 41⅔% then find the difference between number of
3 BHK flats and 2BHK flats sold by D?
मदद D द्वाया फे चे गए 3 BHK फ्रैटों की सॊख्मा भें 58⅓% की वरवि हुई औय D द्वाया
फे चे गए 2 BHK फ्रैटों की सॊख्मा भें 41⅔% की कभी आई, तो D द्वाया फे चे गए 3
BHK फ्रैटों औय 2 BHK फ्रैटों की सॊख्मा के फीच का अॊतय ऻात कीजजए।
(A) 87 (B) 85 (C) 88
(D) 79 https : https
(E) None of these: https : https :
//youtube.co //www.face //instagram.c
m/channel/U book.com/a om/aashishar //t.me/
CYa4_JrOrf8R orasocial?ut
279 5Kz2uOtccXQ
a s hisharora
m_medium = studifie
s ocial
copy_link d
123. If the price of 3BHK flats is 12.5 Lakhs and price of 2 BHK flats is 5.8
Lakhs then find the amount received by C society?
मदद 3 BHK फ्रैटों की कीभत 12.5 राख है औय 2 BHK फ्रैटों की कीभत 5.8 राख है तो C
सोसाइटी को ककतनी याशि प्राप्त हुई?
(A) 2560 lakhs (B) 2840 lakhs (C) 2240 lakhs
(D) 2260 lakhs (E) None of these

124. What is the difference between number of 3 BHK flats sold by D and
E together and number of 2 BHK flats sold by A and C?
D औय E द्वाया शभराकय फे चे गए 3 BHK फ्रैटों की सॊख्मा औय A औय C द्वाया फे चे
गए 2 BHK फ्रैटों की सॊख्मा के फीच का अॊतय ककतना है?
(A) 248 (B) 225 (C) 236
(D) 260 (E) None of these

125. Find the ratio of 3 BHK flat sold A, B and C societies with the number
of 2 BHK Flats sold by C, D and E societies?
A, B औय C सोसामदटमों भें फे चे गए 3 BHK फ्रैटों का C, D औय E सोसाइदटमों द्वाया फे चे
गए 2 BHK फ्रैटों की सॊख्मा से अनुऩात ऻात कीजजए।
(A) 48 : 131 (B) 65 : 126 (C) 55 : 141
(D) 49 : 131 (E) None of these

(126 – 130) The bar graph shows the number of students applied in 5
different colleges while the line chart shows that the number of selected
students is what percent more than the number of unselected students.
फाय ग्राप 5 अरग – अरग कॉरे जों भें आवे दन कयने वारे छात्रों की सॊख्मा को दिाषता है जफकक
राइन चाटष से ऩता चरता है कक चमननत छात्रों की सॊख्मा अचमननत छात्रों की सॊख्मा से ककतने
प्रनतित अचधक है।

https : https : https : https :


//youtube.co //www.face //instagram.c
m/channel/U book.com/a om/aashishar //t.me/
CYa4_JrOrf8R orasocial?ut
280 5Kz2uOtccXQ
a s hisharora
m_medium = studifie
s ocial
copy_link d
126. What is the ratio of number of student selected in NIET to the number
of student unselected in MNIT?
NIET भें चमननत छात्रों की सॊख्मा का MNIT भें न चुने गए छात्रों की सॊख्मा से अनुऩात ककतना
है?
(A) 7 : 6 (B) 6 : 7 (C) 4 : 3
(D) 3 : 4 (E) None of these

127. Number of selected student in MITRC is what percent more than


number of unselected students in LIET?
MITRC भें चमननत छात्रों की सॊख्मा LIET भें नहीॊ चुने गए छात्रों की सॊख्मा से ककतना
प्रनतित अचधक है?
(A) 11% (B) 7% (C) 8%
(D) 12% (E) None of these

128. If there is an increase of 73⅓% in number of student applied in


MITRC and decrease of 77.77% in number of selected students then what
is the number of students unselected?
मदद MITRC भें आवे दन कयने वारे छात्रों की सॊख्मा भें 73⅓% की वरवि होती है औय चमननत
छात्रों की सॊख्मा भें 77.77% की कभी होती है, तो चमननत नहीॊ होने वारे छात्रों की सॊख्मा क्मा
है?
(A) 22600 (B) 21600 (C) 19800
(D) 21500 (E) None of these

129.Find the difference between number of students applied in MNIT, IET


and NIET and number of students unselected in MITRC and LIET?
MNIT, IET औय NIET भें आवे दन कयने वारे छात्रों की सॊख्मा औय MITRC औय LIET भें
चमननत नहीॊ होने वारे छात्रोंhttps
की :सॊख्मा के फीचhttps
का अॊ: तय ऻात कीजजए।
https : https :
//youtube.co //www.face //instagram.c
m/channel/U book.com/a om/aashishar //t.me/
CYa4_JrOrf8R orasocial?ut
281 5Kz2uOtccXQ
a s hisharora
m_medium = studifie
s ocial
copy_link d
(A) 32500 (B) 32300 (C) 36400
(D) 45200 (E) None of these

130. Find the average number of student selected in all the colleges
together?
सबी कॉरे जों भें चमननत छात्रों की औसत सॊख्मा ऻात कीजजमे ?
(A) 9260 (B) 8300 (C) 8350
(D) 9250 (E) None of these

(131 – 135) Pie chart show the % age of number of employees in 5 different
companies and line chart shows the % age of number of males more the
number of females in those companies. Total Number of Employees =
18000
ऩाई चाटष 5 अरग – अरग कॊ ऩननमों भें कभषचारयमों की सॊख्मा का% आमु ददखाता है औय
राइन चाटष उन कॊ ऩननमों भें ऩुरुर्षों की सॊख्मा का प्रनतित भदहराओॊ की सॊख्मा से अचधक
ददखाता है। कभषचारयमों की कु र सॊख्मा = 18000

https : https : https : https :


//youtube.co //www.face //instagram.c
m/channel/U book.com/a om/aashishar //t.me/
CYa4_JrOrf8R orasocial?ut
282 5Kz2uOtccXQ
a s hisharora
m_medium = studifie
s ocial
copy_link d
131. Find the ratio of number of females in D to the number of males in C?
D भें भदहराओॊ की सॊख्मा का C भें ऩुरुर्षों की सॊख्मा से अनुऩात ऻात कीजजमे ?
(A) 7 : 12 (B) 10 : 9 (C) 9 : 10
(D) 12 : 7 (E) None of these

132. Number of males in A and E is what percent more than number of


females in B and D?
A औय E भें ऩुरुर्षों की सॊख्मा B औय D भें भदहराओॊ की सॊख्मा से ककतने प्रनतित
अचधक है?
(A) 8% (B) 6% (C) 5%
(D) 7% (E) None of these

133. If number of males in D increased by 26⅔% and number of females


increased by 16 ⅔% then by how much percent number of males in D is
more than number of females?
मदद D भें ऩुरुर्षों की सॊख्मा भें 26⅔% की वरवि हुई औय भदहराओॊ की सॊख्मा भें 16 ⅔% की
ववर ि हुई, तो D भें ऩुरुर्षों की सॊख्मा भदहराओॊ की सॊख्मा से ककतने प्रनतित अचधक है?
(A) 62.85% (B) 60.78% (C) 57.5%
(D) 64.25% (E) None of these

134. Find the difference between number of males in B, C and D and


number of females in A, D and E?
B, C औय D भें ऩुरुर्षों की सॊख्मा औय A, D औय E भें भदहराओॊ की सॊख्मा के फीच का अॊतय
ऻात कीजजए।
(A) 1900 (B) 1805 (C) 1800
(D) 1750 (E) None of these

135. If income of each female is 14.28% less than that of a male and
annual package of a male is 14.7 lacs per annum then find the monthly
income of each female?
मदद प्रत्मे क भदहरा की आम ऩुरुर्ष की आम से 14.28% कभ है औय ऩुरुर्ष का वावर्षषक ऩैके ज
14.7 राख प्रनत वर्षष है तो प्रत्मे क भदहरा की भाशसक आम ऻात कीजजमे ?
(A) 1.02 lacs (B) 1.03 lacs (C) 1.04 lacs
(D) 1.05 lacs (E) None of these

(136 – 140) Bar graph shows the Sum and difference of the CP two items A
and B (where CP of A > CP of B)
फाय ग्राप CP दो वस्तुओ ॊ A औय B का मोग औय अॊतय ददखाता है (जहाॊ A का CP > B का
CP)

https : https : https : https :


//youtube.co //www.face //instagram.c
m/channel/U book.com/a om/aashishar //t.me/
CYa4_JrOrf8R orasocial?ut
283 5Kz2uOtccXQ
a s hisharora
m_medium = studifie
s ocial
copy_link d
136. CP of B in 2015 is what percent more than CP of B in 2014?
2015 भें B का CP, 2014 भें B के CP से ककतना प्रनतित अचधक है?
(A) 12% (B) 11.9% (C) 12.8%
(D) 10% (E) None of these

137. If in 2011 a shopkeeper marks price of item A up by 25% and sold the
product for Rs. 264 then find the discount percentage given by the
shopkeeper?
मदद 2011 भें एक दुक ानदाय वस्तु A की कीभत भें 25% की वरवि कयता है औय वस्तु
को 264 रु. भें फे च ददमा तो दक
ु ानदाय द्वाया ददमा गमा छू ट प्रनतित ऻात कीजजमे?
(A) 13.44% (B) 12.44% (C) 11.44%
(D) 14.44% (E) None of these

138. Find the average CP of B in all the years?


सबी वर्षों भें B का औसत क्रम भल्
ू म ऻात कीजजए।
(A) 124.7 (B) 134.8 (C) 127.6
(D) 126.5 (E) None of these

139. To gain 12% on product A in 2014 the shopkeeper sold the product at
what price?
2014 भें उत्ऩाद A ऩय 12% राब प्राप्त कयने के शरए दुक ानदाय ने उत्ऩाद को ककस कीभत ऩय
फे चा?
(A) Rs. 253.12 (B) Rs. 254.24 (C) Rs. 255.36
(D) Rs. 256.48 (E) None of these

140. Find the difference of SP of item A and B in 2012 if A is sold at 29


7/17% profit and B at 20% profit?
2012 भें वस्तु A औय B के SP का अॊतय ऻात कीजजए मदद A को 29 7/17% राब ऩय औय B
को 20% राब ऩय फे चा जाता है?
https : https : https : https :
//youtube.co //www.face //instagram.c
m/channel/U book.com/a om/aashishar //t.me/
CYa4_JrOrf8R orasocial?ut
284 5Kz2uOtccXQ
a s hisharora
m_medium = studifie
s ocial
copy_link d
(A) Rs. 152.5 (B) Rs. 178.4 (C) Rs. 185.4
(D) Rs. 148.4 (E) None of these

(141 – 145) Table shows the population, ratio of number of male to number
of females and literacy rate percent of 5 states.
ताशरका 5 याज्मों की जनसॊख्मा, ऩुरुर्षों की सॊख्मा का भदहराओॊ की सॊख्मा से अनुऩात औय
साऺयता दय प्रनतित को दिाषती है।

Male : Literacy
States Population
Female rate%
Andhra Pradesh 920000 12 : 11 32%

Bihar 520000 7:6 45%

Chandigarh 480000 5:3 38%

Delhi 720000 7:5 40%

Rajasthan 850000 9:8 48%

141. Find the ratio of the number of males in Rajasthan to the number of
illiterate percent in Delhi?
याजस्थान भें ऩुरुर्षों की सॊख्मा का ददल्री भें ननयऺय प्रनतित की सॊख्मा से अनुऩात ऻात
कीजजमे ?
(A) 261 : 310 (B) 310 : 261 (C) 225 : 216
(D) 216 : 225 (E) None of these

142. Find the average number of literate persons in all the states?
सबी याज्मों भें साऺय व्मजक्तमों की औसत सॊख्मा ऻात कीजजए।
(A) 281360 (B) 254350 (C) 281370
(D) 254360 (E) None of these

143. If the literacy rate of Bihar is increased by 13.33% and the literacy
rate of Rajasthan increased by 58 1/3% then find the difference between
the illiterate person in these states?
मदद त्रफहाय की साऺयता दय भें 13.33% औय याजस्थान की साऺयता दय भें 58 1/3% की वरवि
की जाती है, तो इन याज्मों भें ननयऺय व्मजक्तमों के फीच अॊतय ऻात कीजजए।
(A) 56800 (B) 54800 (C) 50800
(D) 60800 (E) None of these
https : https : https : https :
//youtube.co //www.face //instagram.c
m/channel/U book.com/a om/aashishar //t.me/
CYa4_JrOrf8R orasocial?ut
285 5Kz2uOtccXQ
a s hisharora
m_medium = studifie
s ocial
copy_link d
144. Number of males in Delhi is what percent more than number of males
in Bihar?
ददल्री भें ऩुरुर्षों की सॊख्मा त्रफहाय भें ऩुरुर्षों की सॊख्मा से ककतने प्रनतित अचधक है?
(A) 45% (B) 50% (C) 40%
(D) 44% (E) None of these

145. If Government is providing Rs. 500 to illiterate person for their


education then find the total amount spend by Delhi, Chandigarh and
Andhra Pradesh Government?
मदद सयकाय ननयऺय व्मजक्त को उनकी शिऺा के शरए 500 रु प्रदान कय यही है, तो
ददल्री, चॊडीगढ औय आॊध्र प्रदे ि सयकाय द्वाया खचष की गई कु र याशि ऻात कीजजए।
(A) 767600000 (B) 548400000 (C) 867600000
(D) 677600000 (E) None of these

(146 – 150) Bar graph shows the percentage of sale of mobiles and
laptops by 5 different shops.
Total mobiles sold = 36000, Total laptops sold = 18500
फाय ग्राप 5 ववशबन्न दक
ु ानों द्वाया भोफाइर औय रैऩटॉऩ की त्रफक्री का प्रनतित दिाषता है।
फे चे गए कु र भोफाइर = 36000, फे चे गए कु र रैऩटॉऩ = 18500

146. Find the difference between number of mobiles sold by C and


number of Laptops sold by E?
C द्वाया फे चे गए भोफाइरों की सॊख्मा औय E द्वाया फे चे गए रैऩटॉऩ की सॊख्मा के फीच का
अॊतय ऻात कीजजए।
(A) 7830 (B) 7460 (C) 7640
(D) 7800 (E) None of these
https : https : https : https :
//youtube.co //www.face //instagram.c
m/channel/U book.com/a om/aashishar //t.me/
CYa4_JrOrf8R orasocial?ut
286 5Kz2uOtccXQ
a s hisharora
m_medium = studifie
s ocial
copy_link d
147. If number of mobiles sold next year by B is 13.33% more than this
year and number of laptops sold by B is 25% more than this year then find
the difference between number of laptops and number of mobiles sold by
B next year?
मदद अगरे वर्षष B द्वाया फे चे गए भोफाइरों की सॊख्मा इस वर्षष की तुरना भें 13.33% अचधक है
औय B द्वाया फे चे गए रैऩटॉऩ की सॊख्मा इस वर्षष की तुरना भें 25% अचधक है, तो अगरे वर्षष
B द्वाया फे चे गए रैऩटॉऩ की सॊख्मा औय B द्वाया फे चे गए भोफाइरों की सॊख्मा के फीच का
अॊतय ऻात कीजजए।
(A) 4856 (B) 7848 (C) 5868
(D) 4868 (E) None of these

148. Find the ratio of number of laptops sold by B to the number of


mobiles sold by A?
B द्वाया फे चे गए रैऩटॉऩ की सॊख्मा का A द्वाया फे चे गए भोफाइरों की सॊख्मा से
अनुऩात ऻात कीजजमे ?
(A) 37 : 81 (B) 37 : 42 (C) 47 : 81
(D) 44 : 37 (E) None of these

149. Number of mobiles sold by D is what percent more than number of


laptops sold by B?
D द्वाया फे चे गए भोफाइरों की सॊख्मा, B द्वाया फे चे गए रैऩटॉऩ की सॊख्मा से ककतने प्रनतित
अचधक है?
(A) 92.5% (B) 94.5% (C) 95.5%
(D) 96.5% (E) None of these

150. Which shop sell number of laptops more than number of mobiles?
कौन सी दुक ान भोफाइर की सॊख्मा से अचधक रैऩटॉऩ फे चती है?
(A) 50% (B) 151% (C) 149%
(D) 48% (E) None of these

(151 – 155) Following table shows the ratio of number of employees of 5


different companies in 2 years and the bar chart shows the difference
between the number of employees in given 2 years.
ननम्नशरणखत ताशरका 2 वर्षों भें 5 ववशबन्न कॊऩननमों के कभषचारयमों की सॊख्मा के अनुऩात को
दिाषती है औय फाय चाटष ददए गए 2 वर्षों भें कभषचारयमों की सॊख्मा के फीच के अॊतय को दिाषता
है।

https : https : https : https :


//youtube.co //www.face //instagram.c
m/channel/U book.com/a om/aashishar //t.me/
CYa4_JrOrf8R orasocial?ut
287 5Kz2uOtccXQ
a s hisharora
m_medium = studifie
s ocial
copy_link d
Company 2019 : 2020

A 4:7
B 3:8
C 10 : 9
D 12 : 17
E 11 : 7
151. If the ratio of male and female employees in company D in year 2019
is 7 : 5 and it increases in the ratio 7 : 9 and 5 : 8 then what is the
difference between the number of females in 2 years.
मदद वर्षष 2019 भें कॊ ऩनी D भें ऩुरुर्ष औय भदहरा कभषचारयमों का अनुऩात 7 : 5 है औय मह 7 :
9 औय 5 : 8 के अनुऩात भें फढता है, तो 2 वर्षों भें भदहराओॊ की सॊख्मा के फीच का अॊतय
ककतना है?
(A) 120 (B) 210 (C) 180
(D) 104 (E) None of these

152. What is the average number of employees in the year 2020?


वर्षष 2020 भें कभषचारयमों की औसत सॊख्मा क्मा है?
(A) 586 (B) 58.6 (C) 736
(D) 73.6 (E) None of these

153. If the salary of each employee in company A in year 2019 is Rs. 3000
and it increases to Rs. 3500 in year 2020 then find the difference between
total salary given by company A in these 2 years.
मदद वर्षष 2019 भें कॊ ऩनी A भें प्रत्मे क कभषचायी का वे तन रु. 3000 औय मह फढकय वर्षष 2020
भें 3500 रुऩमे हो जाता है तो कॊऩनी A द्वाया इन 2 वर्षों भें ददए गए कु र वे तन के फीच का
अॊतय ऻात कीजजए।
(A) 3,28,500 (B) 32,85,000 (C) 3,15,000
(D) 31,50,000 (E) None of these

154. What is the ratio of number of employees of company C in year 2020


to the number of employees of company B in year 2019?
वर्षष 2020 भें कॊ ऩनी C के कभषचारयमों की सॊख्मा का वर्षष 2019 भें कॊ ऩनी B के कभषचारयमों की
सॊख्मा से अनुऩात ककतना है?
(A) 8 : 3 (B) 3 : 8 (C) 4 : 7
(D) 7 : 4 https (E) None ofhttps
these
: : https : https :
//youtube.co //www.face //instagram.c
m/channel/U book.com/a om/aashishar //t.me/
CYa4_JrOrf8R orasocial?ut
288 5Kz2uOtccXQ
a s hisharora
m_medium = studifie
s ocial
copy_link d
155. The number of employees of company A and E together in 2019 is
what % less or more than total number of employees of company B and D
together in 2020?
2019 भें कॊ ऩनी A औय E के कभषचारयमों की सॊख्मा, 2020 भें कॊ ऩनी B औय D के कभषचारयमों
की कु र सॊख्मा से ककतने प्रनतित कभ मा अचधक है?
(A) 13.14% (B) 29.64% (C) 38.55%
(D) 46.59% (E) None of these

(156 – 160) Chart shows the amount (in thousand) invested by three
people in buying 4 different crypto-currency.
चाटष तीन रोगों द्वाया 4 अरग – अरग कक्रप्टो – भुद्रा खयीदने भें ननवे ि की गई याशि
(हजाय भें ) को दिाषता है।

156. What is the average investment (in thousand) by Amit in all Crypto-
currency together?
सबी कक्रप्टो – भुद्रा भें अशभत द्वाया ककमा गमा औसत ननवे ि (हजाय भें ) ककतना है?
(A) 220 (B) 133 (C) 240
(D) 180 (E) None of these

157. What is the ratio of investment by Pankaj in Dogecoin and Etherum to


that of investment by Amit in Etherum and Bitcoin?
ऩॊकज द्वाया डॉगकोइन औय एथेय भ भें ननवे ि का अशभत द्वाया एथेय भ औय त्रफटकॉइन भें
ननवे ि से अनुऩात ककतना है?
(A) 35 : 33 (B) 38 : 33 (C) 33 : 38
(D) 33 : 35 (E) None of these

158. The amount invested by Riya in Etherum is what percent of the total
investment of Pankaj?
रयमा द्वाया एथेय भ भें ननवे िhttps
की :गई याशि ऩॊकज के :कु र ननवे ि का
https httpsककतना
: प्रनतित है? :
https
//youtube.co //www.face //instagram.c
m/channel/U book.com/a om/aashishar //t.me/
CYa4_JrOrf8R orasocial?ut
289 5Kz2uOtccXQ
a s hisharora
m_medium = studifie
s ocial
copy_link d
(A) 22% (B) 22.5% (C) 20.42%
(D) 24.52% (E) None of these

159. If the price of one Dogecoin is Rs. 150 and the price of one Etherum
coin is Rs. 250 then what is the total number of these two coins bought by
Pankaj?
मदद एक डॉगकॉइन की कीभत रु. 150 औय एक ईथयभ कॉइन की कीभत रु. 250 तो
ऩॊक ज द्वाया खयीदे गए इन दो भुद्रा की कु र सॊख्मा ककतनी है?
(A) 1920 (B) 1960 (C) 1460
(D) 1985 (E) None of these

160. Find the total amount invested by all three people in all 4 coins.
तीनों रोगों द्वाया सबी 4 कॉइन भें ननवे ि की गई कु र याशि ऻात कीजजए।
(A) 1460000 (B) 1680000 (C) 2670000
(D) 1480000 (E) None of these

(161 – 165) Following line chart shows the average number of students in
5 different schools and the bar chart shows the ratio distribution of male
and female students in these schools
ननदे ि : ननम्नशरणखत चाटष का ध्मानऩूवषक ऩदढए औय नीचे ददए गए प्रश्नों के उत्तय दीजजए।
ननम्नशरणखत राइन चाटष 5 अरग – अरग स्कू रों भें छात्रों की औसत सॊख्मा ददखाता है औय
फाय चाटष इन स्कू रों भें ऩुरुर्ष औय भदहरा छात्रों के अनुऩात ववतयण को दिाषता है।

https : https : https : https :


//youtube.co //www.face //instagram.c
m/channel/U book.com/a om/aashishar //t.me/
CYa4_JrOrf8R orasocial?ut
290 5Kz2uOtccXQ
a s hisharora
m_medium = studifie
s ocial
copy_link d
161. What is the total number of male students from all 5 schools
together?
सबी 5 स्कू रों से शभराकय ऩुरुर्ष छात्रों की कु र सॊख्मा ककतनी है?
(A) 1785 (B) 1783 (C) 1795
(D) 1894 (E) None of these

162. Find the average number of female students in all 5 schools together.
सबी 5 स्कू रों भें शभराकय छात्राओॊ की औसत सॊख्मा ऻात कीजजए।
(A) 311.5 (B) 321.2 (C) 331.4
(D) 323.4 (E) None of these

163. What is the ratio of the difference between the number of male and
female students in school D to that of school A?
स्कू र D औय स्कू र A भें ऩुरुर्ष औय भदहरा छात्रों की सॊख्मा के फीच अॊतय का अनुऩात ककतना
है?
(A) 29 : 117 (B) 117 : 29 (C) 59 : 171
(D) 59 : 117 (E) None of these

164. The sum of the number of male students in school A and E is what
percent more or less than the sum of the number of female students in
school B and C?
स्कू र A औय E भें ऩुरुर्ष छात्रों की सॊख्मा का मोग, स्कू र B औय C भें भदहरा छात्रों की सॊख्मा
के मोग से ककतना प्रनतित अचधक मा कभ है?
(A) 7.5% (B) 2.19% (C) 5.8%
(D) 2.14% (E) None of these

165. The number of female students in school E is what % of the total


number of students in school B?
स्कू र E भें भदहरा छात्रों की सॊख्मा, स्कू र B भें छात्रों की कु र सॊख्मा का ककतना प्रनतित है?
(A) 34.5% (B) 34.56% (C) 34.66%
(D) 71.42% https : https
(E) None of these : https : https :
//youtube.co //www.face //instagram.c
m/channel/U book.com/a om/aashishar //t.me/
CYa4_JrOrf8R orasocial?ut
291 5Kz2uOtccXQ
a s hisharora
m_medium = studifie
s ocial
copy_link d
(166 – 170) Pie chart shows the % of sold coolers by 4 different
shopkeepers and the bar chart shows the number of unsold coolers by
these 4 shopkeepers.
ऩाई चाटष 4 अरग – अरग दुक ानदायों द्वाया फे चे गए कू रय का% ददखाता है औय फाय चाटष इन
4 दुक ानदायों द्वाया त्रफना त्रफके कू रय की सॊख्मा ददखाता है।

https : https : https : https :


//youtube.co //www.face //instagram.c
m/channel/U book.com/a om/aashishar //t.me/
CYa4_JrOrf8R orasocial?ut
292 5Kz2uOtccXQ
a s hisharora
m_medium = studifie
s ocial
copy_link d
166. What is the average number of sold coolers by all 4 shopkeepers?
सबी 4 दुक ानदायों द्वाया फे चे गए कू रय की औसत सॊख्मा ककतनी है?
(A) 435 (B) 42.5 (C) 43.5
(D) 425 (E) None of these

167. If shopkeeper A sold 30% more coolers than the number of unsold
coolers by shopkeeper S and the ratio of sold and unsold coolers for
shopkeeper A is 13 : 17, then find the difference between total number of
coolers of P and A?
मदद दुक ानदाय A ने दुक ानदाय S द्वाया फे चे गए कू रय की सॊख्मा से 30% अचधक कू रय
फे चे औय दुक ानदाय S के शरए फे चे गए औय न त्रफके कू रय का अनुऩात 13 : 17 है, तो
P औय A के कु र कू रय की सॊख्मा के फीच का अॊतय ऻात कीजजए।
(A) 120 (B) 68 (C) 24
(D) 140 (E) None of these

168. The number of unsold coolers by shopkeeper R is what % more or


less than total number of sold coolers by shopkeeper P and Q together?
दकु ानदाय R द्वाया त्रफना फीके कू रयों की सॊख्मा, दक
ु ानदाय P औय Q द्वाया शभराकय फे चे गए
कू रयों की कु र सॊख्मा से ककतने प्रनतित अचधक मा कभ है?
(A) 10.50% (B) 12.82% (C) 10.47%
(D) 33.33% (E) None of these

169. What is the ratio of difference between sold coolers by shopkeeper R


and S to the total number of unsold coolers by Q and R together?
दुक ानदाय R औय S द्वाया फे चे गए कू रय औय Q औय R द्वाया त्रफना त्रफके कू रय की कु र
सॊख्मा के फीच अॊतय का अनुऩात ककतना है?
(A) 4.43% (B) 43.4% (C) 45.4%
(D) 4.45% (E) None of these

170. The total number of sold coolers by all 4 shopkeepers together is


what % of total number of unsold coolers by all 4 shopkeepers?
सबी 4 दुक ानदायों द्वाया फे चे गए कू रय की कु र सॊख्मा सबी 4 दुक ानदायों द्वाया त्रफना त्रफके
कू रय की कु र सॊख्मा का ककतना प्रनतित है?
(A) 66.% (B) 60% (C) 56%
(D) 58% (E) None of these

(171 – 175) In a school, elections were held to choose the school monitor
and 2 students were selected as candidates. Following table shows the
ratio of votes got by 2 students in 5 sections (i.e A, B, C, D and E) and the
bar chart represents the average number of votes casted from all 5
sections.
एक स्कू र भें , स्कू र भॉननटय चुनने के शरए चुनाव हुए औय 2 छात्रों को उम्भीदवाय के रूऩ भें
चुना गमा। ननम्नशरणखत ताशरका https : 5 वगों (अथाष त A,: B, C, D औय
https : भें 2 छात्रों https
httpsE) द्वाया :प्राप्त
//youtube.co //www.face //instagram.c
m/channel/U book.com/a om/aashishar //t.me/
CYa4_JrOrf8R orasocial?ut
293 5Kz2uOtccXQ
a s hisharora
m_medium = studifie
s ocial
copy_link d
वोटों के अनुऩात को दिाषती है औय फाय चाटष सबी 5 वगों से डारे गए वोटों की औसत सॊख्मा को
दिाषता है।

Class 1ststudent : 2nd student

A 3:5
B 10 : 11
C 13 : 15
D 9 : 11
E 5:4

171. Find the total number of students in all 5 sections together who gave
votes.
सबी 5 वगों भें एक साथ वोट दे ने वारे छात्रों की कु र सॊख्मा ऻात कीजजए ।
(A) 420 (B) 426 (C) 428
(D) 440 (E) None of these
https : https : https : https :
//youtube.co //www.face //instagram.c
m/channel/U book.com/a om/aashishar //t.me/
CYa4_JrOrf8R orasocial?ut
294 5Kz2uOtccXQ
a s hisharora
m_medium = studifie
s ocial
copy_link d
172. Find the average number of votes 1st student got from section B, C
and E together.
ऩहरे छात्र को से क्िन B, C औय E से शभराकय शभरे वोटों की औसत सॊख्मा ऻात कीजजए।
(A) 44 (B) 33 (C) 35
(D) 38 (E) None of these

173. Which student selected as the school monitor?


स्कू र भॉननटय के रूऩ भें ककस छात्र का चमन ककमा गमा?
(A) 1st (B) 2nd (C) Cannot determine
(D) Draw (E) None of these

174. The sum of the number of votes 1st student got is what % of the total
students in class B and D together?
ऩहरे छात्र को शभरे वोटों की सॊख्मा का मोग कऺा B औय D भें शभराकय कु र छात्रों का ककतना
प्रनतित है?
(A) 175% (B) 120.73% (C) 125.45%
(D) 124.46% (E) None of these

175. Find the ratio of number of votes casted by section C and E together
to number of votes by which 2nd students won the elections.
से क्िन C औय E द्वाया शभराकय डारे गए वोटों की सॊख्मा का चन
ु ाव भें दस
ू ये छात्र का ऩहरे
छात्र को हयाने वारे वोटों की सॊख्मा से अनुऩात ऻात कीजजए।
(A) 4 : 23 (B) 23 : 4 (C) 29 : 7
(D) 7 : 29 (E) None of these

(175 – 176) The given bar chart shows the % difference of selling of 2
articles P and Q and total % of selling of these 2 articles by 5
shopkeepers. (Every shopkeeper has equal number of articles in stock)
ददमा गमा फाय चाटष 2 वस्तुओ ॊ P औय Q की त्रफक्री का% अॊतय औय 5 दुक ानदायों द्वाया इन 2
वस्तुओ ॊ की त्रफक्री का कु र% दिाषता है। (प्रत्मे क दुक ानदाय के ऩास स्टॉक भें सभान सॊख्मा भें
वस्तुएॉ हैं )

https : https : https : https :


//youtube.co //www.face //instagram.c
m/channel/U book.com/a om/aashishar //t.me/
CYa4_JrOrf8R orasocial?ut
295 5Kz2uOtccXQ
a s hisharora
m_medium = studifie
s ocial
copy_link d
176. If the number of article P sold by shopkeeper N is 108 then find the
total number of article Q sold by J, K and L together.
मदद दकु ानदाय N द्वाया फे ची गई वस्तु P की सॊख्मा 108 है, तो J, K औय L द्वाया फे ची गई
वस्तु Q की कु र सॊख्मा ऻात कीजजए।
(A) 255 (B) 254 (C) 114
(D) 176 (E) None of these

177. What is the ratio of total number of article P sold by L, M and N to the
difference between total articles sold by J and K?
L, M औय N द्वाया फे ची गई वस्तु P की कु र सॊख्मा का J औय K द्वाया फे ची गई कु र
वस्तुओ ॊ के अॊतय से अनुऩात ककतना है?
(A) 113 : 36 (B) 113 : 107 (C) 113 : 35
(D) 107 : 113 (E) None of these

178. If the total number of articles (P and Q) in every shop is 1200 then
find the number of articles that remained unsold by all 5 shopkeepers
together.
मदद प्रत्मे क दक
ु ान भें वस्तुओ ॊ (P औय Q) की कु र सॊख्मा 1200 है, तो सबी 5 दक
ु ानदायों
द्वाया एक साथ न त्रफकी हुई वस्तुओ ॊ की सॊख्मा ऻात कीजजए।
(A) 2616 (B) 2814 (C) 2641
(D) 2145 (E) None of these

179. Total sale by shopkeeper L is what % of the total sale of N?


दुक ानदाय L द्वाया कु र त्रफक्री, N की कु र त्रफक्री का ककतना प्रनतित है?
(A) 70.39% (B) 70.37% (C) 70.31%
(D) 70.30% (E) None of these

180. The total number of articles P sold by all 5 shopkeepers is what %


more or less than the total number of articles Q sold by all these 5
shopkeepers?
सबी 5 दक ु ानदायों द्वाया फे ची गई वस्तुओ ॊ P की कु र सॊख्मा, इन सबी 5 दक
ु ानदायों द्वाया
फे ची गई वस्तुओ ॊ की कु र सॊख्मा से ककतने प्रनतित अचधक मा कभ है?
(A) 113.33% (B) 152.04% (C) 123.34%
(D) 112.69% (E) None of these

(181 – 185) Following bar chart shows the total number of bikes sold of
three types (1st, 2nd and 3rd) by 5 different showrooms. And the line
graph shows the % of 1st and 2nd type of bikes sold by these
showrooms.
ननम्नशरणखत फाय चाटष 5 ववशबन्न िोरूभों द्वाया तीन प्रकाय (प्रथभ, द्ववतीम औय तरतीम) की
फे ची गई फाइकों की कु र सॊख्मा को दिाषता है। औय राइन ग्राप इन िोरूभों द्वाया फे ची गई
ऩहरी औय दस ू यी प्रकाय की फाइक का% ददखाता है।
https : https : https : https :
//youtube.co //www.face //instagram.c
m/channel/U book.com/a om/aashishar //t.me/
CYa4_JrOrf8R orasocial?ut
296 5Kz2uOtccXQ
a s hisharora
m_medium = studifie
s ocial
copy_link d
181. The number of bikes of type 3rd sold by showroom E is what % more
or less than total number of bikes sold by showroom B?
िोरूभ E द्वाया फे ची गई प्रकाय 3 की फाइक की सॊख्मा, िोरूभ B द्वाया फे ची गई फाइक की
कु र सॊख्मा से ककतना% अचधक मा कभ है?
(A) 75.95% (B) 75.5% (C) 75.88%
(D) 75.58% (E) None of these

https : https : https : https :


//youtube.co //www.face //instagram.c
m/channel/U book.com/a om/aashishar //t.me/
CYa4_JrOrf8R orasocial?ut
297 5Kz2uOtccXQ
a s hisharora
m_medium = studifie
s ocial
copy_link d
182. What is the ratio of number of bikes of type 3rd sold by showroom B
and C together and the number of bikes of type 2nd sold by showroom C?
िोरूभ B औय C द्वाया एकसाथ फे ची गई टाइऩ 3 की फाइक की सॊख्मा औय िोरूभ C द्वाया
फे ची गई टाइऩ 2 की फाइक की सॊख्मा का अनुऩात ककतना है?
(A) 1291 : 292 (B) 1237 : 299 (C) 1219 : 299
(D) 1129 : 299 (E) None of these

183. Find average number of bikes of type 3rd sold by all showroom
together.
सबी िोरूभ द्वाया एक साथ फे ची गई प्रकाय 3 की फाइक की औसत सॊख्मा ऻात
कीजजए।
(A) 2509 (B) 2504 (C) 2405
(D) 2506 (E) None of these

184. What is the difference between the number of bikes sold of type 2nd
by showroom C and type 1st by showroom A and E together?
िोरूभ C द्वाया टाइऩ 2 की औय िोरूभ A औय E द्वाया शभराकय टाइऩ 1 की फे ची गई फाइक
की सॊख्मा के फीच का अॊतय ककतना है?
(A) 5868 (B) 5604 (C) 5688
(D) 5608 (E) None of these

185. The number of bikes of type 1st sold by B, C and D together is what
% of the number of bikes of type 3rd sold by A, B and C together?
B, C औय D द्वाया शभराकय फे ची गई टाइऩ 1 की फाइक्स की सॊख्मा, A, B औय C द्वाया
शभराकय फे ची गई टाइऩ 3 की फाइक्स की सॊख्मा का ककतना प्रनतित है?
(A) 107.75% (B) 107.25% (C) 107.73%
(D) 107.54% (E) None of these

(186 – 190) The table shows the ratio of marks obtained by 5 students in
Math and Science and the line chart shows the difference between marks
scored in Science and Math by these 5 students,
ताशरका गणणत औय ववऻान भें 5 छात्रों द्वाया प्राप्त अॊक ों के अनुऩात को दिाषती है औय ये खा
चाटष इन 5 छात्रों द्वाया ववऻान औय गणणत भें प्राप्त अॊक ों के फीच के अॊतय को दिाषता है।
Student Math : SCIENCE

P 9 :2

Q 7 :4

R 11 : 6

S 13 : 19

T 12 : 17
https : https : https : https :
//youtube.co //www.face //instagram.c
m/channel/U book.com/a om/aashishar //t.me/
CYa4_JrOrf8R orasocial?ut
298 5Kz2uOtccXQ
a s hisharora
m_medium = studifie
s ocial
copy_link d
186. What is the average marks scored in Math by all students together?
सबी छात्रों द्वाया एक साथ गणणत भें प्राप्त ककए गए औसत अॊक क्मा हैं ?
(A) 1041.6 (B) 1401.9 (C) 1401.6
(D) 1440.6 (E) None of these

187. What is the ratio of total marks obtained by S and the difference
between marks scored by R and P in Math.
S द्वाया प्राप्त कु र अॊक ों औय R औय P द्वाया गणणत भें प्राप्त अॊक ों के फीच के अॊतय
से अनुऩात ककतना है?
(A) 2144 : 247 (B) 247 : 2144 (C) 2471 : 2144
(D) 2144 : 2471 (E) None of these

188. The marks obtained by Q in Math is what % more or less than marks
obtained by T in science?
गणणत भें Q द्वाया प्राप्त अॊक ववऻान भें T द्वाया प्राप्त अॊकों से ककतने प्रनतित अचधक मा
कभ है?
(A) 22.05% (B) 14.83% (C) 24.83%
(D) 24.5% (E) None of these

189. Total marks obtained in science by all 5 students together is :


सबी 5 छात्रों द्वाया ववऻान भें प्राप्त कु र अॊक है :
(A) 6155 (B) 6451 (C) 6154
(D) 6415 (E) None of these

190. Find the average marks scored by all 5 students together in science.
ववऻान भें सबी 5 छात्रों द्वाया एक साथ प्राप्त ककए गए औसत अॊक ऻात कीजजए।
(A) 1283 (B) 1289 (C) 1238
(D) 1298 (E) None of these
https : https : https : https :
//youtube.co //www.face //instagram.c
m/channel/U book.com/a om/aashishar //t.me/
CYa4_JrOrf8R orasocial?ut
299 5Kz2uOtccXQ
a s hisharora
m_medium = studifie
s ocial
copy_link d
(191 – 195) Following pie chart shows the % distribution of number of
candidates who applied for 4 different exams conducted by SSC. It is
given that 4500 students applied for CGL exam.
ननम्नशरणखत ऩाई चाटष SSC द्वाया आमोजजत 4 ववशबन्न ऩयीऺाओॊ के शरए आवे दन कयने
वारे उम्भीदवायों की सॊख्मा का% ववतयण दिाषता है। फतामा गमा है कक 4500 छात्रों ने CGL
ऩयीऺा के शरए आवे दन ककमा था।

191. Find the total number of students who applied for all of the exams
together?
सबी ऩयीऺाओॊ के शरए एक साथ आवे दन कयने वारे छात्रों की कु र सॊख्मा ऻात कीजजए।
(A) 26000 (B) 24000 (C) 30000
(D) 28000 (E) None of these

192. If the ratio of boys and girls who applied for CHSL exam is 11 : 10 and
the for GD exam is 9 : 16 then find the difference between the number of
girls who applied for these two exams.
मदद CHSL ऩयीऺा के शरए आवे दन कयने वारे रडके औय रडककमों का अनुऩात 11 : 10 है
औय GD ऩयीऺा के शरए 9 : 16 है, तो इन दोनों ऩयीऺाओॊ के शरए आवे दन कयने वारी
रडककमों की सॊख्मा के फीच का अॊतय ऻात कीजजए।
(A) 250 (B) 200 (C) 450
(D) 300 (E) None of these

193. The number of candidates who applied for CGL exam is 80% more
than students who applied for banking exam, then the number of students
who applied for banking exam is what % of the number of students who
applied for CHSL and https
GD : exam together?
https : https : https :
//youtube.co //www.face //instagram.c
m/channel/U book.com/a om/aashishar //t.me/
CYa4_JrOrf8R orasocial?ut
300 5Kz2uOtccXQ
a s hisharora
m_medium = studifie
s ocial
copy_link d
CGL ऩयीऺा के शरए आवे दन कयने वारे उम्भीदवायों की सॊख्मा फैंककॊ ग ऩयीऺा के शरए आवे दन
कयने वारे छात्रों की तुरना भें 80% अचधक है, तो फैंककॊ ग ऩयीऺा के शरए आवे दन कयने वारे
छात्रों की सॊख्मा CHSL औय GD ऩयीऺा के शरए एक साथ आवे दन कयने वारे छात्रों की सॊख्मा
का ककतना प्रनतित है?
(A) 13.88% (B) 13.99% (C) 19.78%
(D) 19.66% (E) None of these

194. The average number of candidates who applied for MTS and CHSL
together is what % more or less than the average number of candidates
who applied for CGL and GD exam together?
MTS औय CHSL के शरए एकसाथ आवे दन कयने वारे उम्भीदवायों की औसत सॊख्मा CGL
औय GD ऩयीऺा के शरए एकसाथ आवे दन कयने वारे उम्भीदवायों की औसत सॊख्मा से ककतने
प्रनतित अचधक मा कभ है?
(A) 75% (B) 28% (C) 50%
(D) 25% (E) None of these

195. Find the ratio of sum of candidates who applied for MTS and GD and
difference of candidates who applied for CHSL and MTS.
MTS औय GD के शरए आवे दन कयने वारे उम्भीदवायों के मोग औय CHSL औय MTS के
शरए आवे दन कयने वारे उम्भीदवायों के अॊतय का अनुऩात ऻात कीजजए।
(A) 8 : 3 (B) 3 : 8 (C) 1 : 5
(D) 5 : 1 (E) None of these

(196 – 200). A company sold 34000 bikes to 5 showrooms in the year 2020.
The pie chart shows the % distribution of number of bikes sold by the
company to the showrooms in 2020 and the bar graph shows the % of
bikes sold by these 5 showrooms in the next 2 years.
एक कॊ ऩनी ने सार 2020 भें 5 िोरूभ को 34000 फाइक फे चीॊ। ऩाई चाटष 2020 भें कॊ ऩनी द्वाया
िोरूभ भें फे ची गई फाइक की सॊख्मा का% ववतयण ददखाता है औय फाय ग्राप अगरे 2 वर्षों भें
इन 5 िोरूभ द्वाया फे ची गई फाइक का% ददखाता है।

https : https : https : https :


//youtube.co //www.face //instagram.c
m/channel/U book.com/a om/aashishar //t.me/
CYa4_JrOrf8R orasocial?ut
301 5Kz2uOtccXQ
a s hisharora
m_medium = studifie
s ocial
copy_link d
196. What is the average number of bikes sold by showrooms B and C
together in 2 years?
2 वर्षों भें िोरूभ B औय C द्वाया शभराकय फे ची गई फाइकों की औसत सॊख्मा ककतनी है?
(A) 3434 (B) 3706 (C) 4343
(D) 3760 (E) None of these

197. The number of unsold bikes of showroom E in both years is what %


of the number of bikes sold by showroom D in 2021?
दोनों वर्षों भें िोरूभ E की त्रफना त्रफकी फाइक की सॊख्मा, 2021 भें िोरूभ D द्वाया फे ची गई
फाइक की सॊख्मा का ककतना प्रनतित है?
(A) 165.30% (B) 168.30% (C) 164.30%
(D) 162.30% (E) None of these

198. What is the ratio of the sum of the number of bikes sold by
showroom B in the year 2020 and number of bikes sold by showroom D in
2021 to the difference between the total number of bikes purchased by
showroom B and C?
वर्षष 2020 भें िोरूभ B द्वाया फे ची गई फाइक की सॊख्मा औय 2021 भें िोरूभ D द्वाया फे ची
गई फाइक की सॊख्मा के मोग का िोरूभ B औय C द्वाया खयीदी गई फाइक की कु र सॊख्मा के
फीच के अॊतय से अनुऩात ककतना है?
(A) 289 : 935 (B) 298 : 935 (C) 298 : 253
(D) 953 : 298 (E) None of these

199. Which showroom sold the maximum % of bikes in both years?


दोनों वर्षों भें ककस िोरूभ ने अचधकतभ % फाइक फे चीॊ?
(A) B (B) C (C) D
(D) E (E) None of these
https : https : https : https :
//youtube.co //www.face //instagram.c
m/channel/U book.com/a om/aashishar //t.me/
CYa4_JrOrf8R orasocial?ut
302 5Kz2uOtccXQ
a s hisharora
m_medium = studifie
s ocial
copy_link d
200. Find the number of bikes remained unsold by all 5 showrooms in 2
years?
2 वर्षों भें सबी 5 िोरूभ द्वाया त्रफना त्रफके फाइकों की सॊख्मा ऻात कीजजए।
(A) 16554 (B) 14665 (C) 18772
(D) 14348 (E) None of these

(201 – 205) The two pie charts represents the % of students appeared in
and examination conducted on 5 different days and the % of students who
passed from these appeared students on respective days.
दो ऩाई चाटष 5 अरग – अरग ददनों भें आमोजजत ऩयीऺा भें उऩजस्थत होने वारे छात्रों के % औय
इनभें से उत्तीणष होने वारे छात्रों के प्रनतित को प्रदशिषत कयता है।

https : https : https : https :


//youtube.co //www.face //instagram.c
m/channel/U book.com/a om/aashishar //t.me/
CYa4_JrOrf8R orasocial?ut
303 5Kz2uOtccXQ
a s hisharora
m_medium = studifie
s ocial
copy_link d
201. The number of students failed on Thursday is what percent of the
number of students who failed on Friday?
गरु
ु वाय को अनुत्तीणष छात्रों की सॊख्मा िुक्र वाय को अनुत्तीणष होने वारे छात्रों की सॊख्मा का
ककतना प्रनतित है?
(A) 49.77% (B) 59.77% (C) 49.98%
(D) 59.98% (E) None of these

202. If the number of students who failed on Wednesday is 1620 and the
number of students appeared on Tuesday is 25% more than students who
appeared on Wednesday then find the number of students who passed on
Tuesday.
मदद फुधवाय को अनुत्तीणष होने वारे छात्रों की सॊख्मा 1620 है औय भॊगरवाय को
उऩजस्थत होने वारे छात्रों की सॊख्मा फुधवाय को उऩजस्थत होने वारे छात्रों की सॊख्मा से
25% अचधक है, तो भॊगरवाय को उत्तीणष होने वारे छात्रों की सॊख्मा ऻात कीजजए।
(A) 850 (B) 875 (C) 975
(D) 950 (E) None of these

203. The difference between number of students who failed and passed
on Monday is what % of the number of students who appeared on
Tuesday?
सोभवाय को अनुत्तीणष औय उत्तीणष होने वारे छात्रों की सॊख्मा का अॊतय भॊगरवाय को उऩजस्थत
होने वारे छात्रों की सॊख्मा का ककतना प्रनतित है?
(A) 2773% (B) 2769% (C) 2785%
(D) 2786% (E) None of these

204. If the number of students appeared on Friday is 19900 and the


number of students who passed on Tuesday is 1140 then find the
difference between the number of students who failed on Friday and
Tuesday?
मदद िुक्र वाय को उऩजस्थत होने वारे छात्रों की सॊख्मा 19900 है औय भॊगरवाय को उत्तीणष होने
वारे छात्रों की सॊख्मा 1140 है, तो िुक्र वाय औय भॊगरवाय को अनुत्तीणष होने वारे छात्रों की
सॊख्मा के फीच का अॊतय ऻात कीजजमे ?
(A) 8099 (B) 8000 (C) 8009
(D) 8090 (E) None of these

205. Find the ratio of number of students who passed on Monday and
Friday together to the number of students who failed on Monday and
Thursday together.
सोभवाय औय िुक्र वाय को शभराकय उत्तीणष हुए ववद्माचथषमों की सॊख्मा का सोभवाय औय गुरुवाय
को शभराकय अनुत्तीणष होने वारे ववद्माचथषमों की सॊख्मा से अनुऩात ऻात कीजजए।
(A) 800 : 807 (B) 450 : 401 (C) 401 : 450
(D) 807 : 800 (E) None of these
https : https : https : https :
//youtube.co //www.face //instagram.c
m/channel/U book.com/a om/aashishar //t.me/
CYa4_JrOrf8R orasocial?ut
304 5Kz2uOtccXQ
a s hisharora
m_medium = studifie
s ocial
copy_link d
(206 – 210) Bar chart show the total % of marks scored by four students in
Hindi and English and the % difference of marks in these subjects.
(Note : – the maximum marks of both subjects is same for all the
students)
फाय चाटष चाय छात्रों द्वाया दहॊदी औय अॊग्रेजी भें प्राप्त अॊकों का कु र% औय इन ववर्षमों भें अॊकों
के अॊतय को दिाषता है।
(नोट : – सबी छात्रों के शरए दोनों ववर्षमों के अचधकतभ अॊक सभान हैं )

206. The marks scored by A in Hindi is what % of the marks scored by C in


English?
A द्वाया दहॊदी भें प्राप्त अॊक, अॊग्रेजी भें C द्वाया प्राप्त अॊकों का ककतना प्रनतित है?
(A) 43.99% (B) 71.40% (C) 43.85%
(D) 71% (E) None of these

207. If maximum marks for Hindi is 100 and for English is 150 then find the
total marks scored by A, B and D in Hindi and English together?
मदद दहॊदी के शरए अचधकतभ अॊक 100 है औय अॊग्रेजी के शरए 150 है तो दहॊदी औय
अॊग्रेज ी भें A, B औय D द्वाया प्राप्त कु र अॊक ऻात कीजजमे ?
(A) 154.5 (B) 150.5 (C) 156.5
(D) 150.4 (E) None of these

208. The total marks scored by B is what % more or less than the marks
scored C?
B द्वाया प्राप्त कु र अॊक , C द्वाया प्राप्त अॊ5क से ककतने प्रनतित अचधक मा कभ है?
(A) 42% (B) 43 (C) 44%
(D) 41% (E) None of these

209.What is the ratio of total marks scored by B and C together to the total
marks scored by A, B and D together?
B औय C द्वाया शभराकय प्राप्त ककए गए कु र अॊक ों का A, B औय D द्वाया शभराकय प्राप्त
ककए गए कु र अॊकों से अनुऩhttps
ात ककतना
: है? https : https : https :
//youtube.co //www.face //instagram.c
m/channel/U book.com/a om/aashishar //t.me/
CYa4_JrOrf8R orasocial?ut
305 5Kz2uOtccXQ
a s hisharora
m_medium = studifie
s ocial
copy_link d
(A) 12 : 17 (B) 17 : 12 (C) 13 : 14
(D) 14 : 13 (E) None of these

210. Find the average % of marks scored in English by all the students
together?
सबी छात्रों द्वाया एक साथ अॊग्रेज ी भें प्राप्त अॊक ों का औसत % ऻात कीजजमे ?
(A) 54 (B) 65 (C) 72
(D) 48 (E) None of these

(211 – 215) The pie chart represent the % of Phones and LEDs’ sold by 5
showrooms and the bar chart represent the ratio of number Phones sold
to number of LEDs’ sold.
ऩाई चाटष 5 िोरूभों द्वाया फे चे गए पोन औय एरईडी के % को दिाषता है औय फाय चाटष फे चे गए
पोन की सॊख्मा औय फे चे गए एरईडी की सॊख्मा के अनुऩात को दिाषता है।

https : https : https : https :


//youtube.co //www.face //instagram.c
m/channel/U book.com/a om/aashishar //t.me/
CYa4_JrOrf8R orasocial?ut
306 5Kz2uOtccXQ
a s hisharora
m_medium = studifie
s ocial
copy_link d
211. What is the ratio of mobile phones sold by shopkeeper C to number
of LED’s sold by shopkeeper A?
दक
ु ानदाय C द्वाया फे चे गए भोफाइर पोन का दकु ानदाय A द्वाया फे चे गए LED की सॊख्मा से
अनुऩात ककतना है?
(A) 15 : 43 (B) 43 : 15 (C) 15 : 28
(D) 28 : 15 (E) None of these

212. Find the ratio of total mobiles sold by shopkeeper A, C and E ogether
to total number of LED’s sold by shopkeeper B, C and D together.
दुक ानदाय A, C औय E द्वाया शभराकय फे चे गए कु र भोफाइरों का दुक ानदाय B, C औय
D द्वाया शभराकय फे चे गए एरईडी की कु र सॊख्मा से अनुऩात ऻात कीजजए।
(A) 126 : 125 (B) 125 : 126 (C) 121 : 126
(D) 120 : 121 (E) None of these

213. If every shopkeeper has equal number of items in their shops then
the number of mobile phones sold by shopkeeper B is what % of total
number of mobile phones and LED’s sold by shopkeeper E?
मदद प्रत्मे क दक
ु ानदाय की दक
ु ान भें सभान सॊख्मा भें वस्तुएॉ हैं , तो दक
ु ानदाय B द्वाया फे चे गए
भोफाइर पोन की सॊख्मा, दुक ानदाय E द्वाया फे चे गए भोफाइर पोन औय LED की कु र सॊख्मा
का ककतना प्रनतित है?
(A) 47.73% (B) 44.44% (C) 27.85%
(D) 47.86% (E) None of these

214. If shopkeeper C has total 2000 items ( Mobile phones,LED and


Washing machine) in his shop and this is 25% more than total number of
items of shopkeeper B( mobile phones, LED and washing machine) then
find the total number of LED’s sold and washing machines sold by both
these shopkeepers.
मदद दुक ानदाय C की दुक ान भें कु र 2000 वस्तुएॉ (भोफाइर पोन, एरईडी औय वाशिग ॊ भिीन)

हैं औय मह दुक ानदाय B की कु र वस्तुओ ॊ (भोफाइर पोन, एरईडी औय वाशिग भिीन) की
सॊख्मा से 25% अचधक है, तो इन दोनों दक ॊ
ु ानदायों दवया फे चीॊ गमी कु र LED औय वाशिग
भिीन की कु र सॊख्मा ऻात कीजजमे ।
(A) 2300 (B) 1904 (C) 2380
(D) 1840 (E) None of these

215. If shopkeeper A has sold 63 mobile phones then find the total number
of mobiles and LED sold by shopkeeper A?
मदद दुक ानदाय A ने 63 भोफाइर पोन फे चे हैं , तो दुक ानदाय A द्वाया फे चे गए भोफाइर औय
LED की कु र सॊख्मा ऻात कीजजए।
(A) 114 (B) 125 (C) 117
(D) 129 (E) None of these

https : https : https : https :


//youtube.co //www.face //instagram.c
m/channel/U book.com/a om/aashishar //t.me/
CYa4_JrOrf8R orasocial?ut
307 5Kz2uOtccXQ
a s hisharora
m_medium = studifie
s ocial
copy_link d
(216 – 220) Bar chart shows the average number of people present in a
cinema hall in different months and the line chart shows the ratio of male
female and children present in these respective months.
फाय चाटष ववशबन्न भहीनों भें एक शसने भा हॉर भें उऩजस्थत रोगों की औसत सॊख्मा को दिाषता
है औय ये खा चाटष इन सॊफॊचधत भहीनों भें भौज ूद ऩुरुर्ष भदहरा औय फच्चों के अनुऩात को दिाषता
है।

216. The total number of males present in February is what % of total


number of females and children present in the same month.
पयवयी भें उऩजस्थत ऩुरुर्षों की कु र सॊख्मा सभान भहीने भें उऩजस्थत भदहराओॊ औय फच्चों की
कु र सॊख्मा का ककतना प्रनतित है?
(A) 44.44% (B) 71.40% (C) 43.85%
(D) 71% (E) None of these
https : https : https : https :
//youtube.co //www.face //instagram.c
m/channel/U book.com/a om/aashishar //t.me/
CYa4_JrOrf8R orasocial?ut
308 5Kz2uOtccXQ
a s hisharora
m_medium = studifie
s ocial
copy_link d
217. If the piece of per ticket in February is Rs. 100 and it increases by
60% in March, then find the increase % in total revenue.
मदद पयवयी भें प्रनत दटकट का भूल्म रु. 100 है औय भाचष भें इसभें 60% की वरवि होती है, तो
कु र याजस्व भें ववर ि% ऻात कीजजए।
(A) 254% (B) 106% (C) 206%
(D) 178% (E) None of these

218. If in January, all the children went to cinema hall with their Parents
then how many male and female went individually?
मदद जनवयी भें , सबी फच्चे अऩने भाता – वऩता के साथ शसने भा हॉर गए, तो ककतने
ऩुरुर्ष औय भदहराएॊ अरग – अरग गए?
(A) 420 (B) 430 (C) 440
(D) 480 (E) None of these

219. Total number of females in all the months together is how much more
or less than total number of males in all months together.
सबी भहीनों भें शभराकय भदहराओॊ की कु र सॊख्मा, सबी भहीनों भें शभराकय ऩुरुर्षों की कु र
सॊख्मा से फहुत अचधक मा कभ है।
(A) 365 (B) 270 (C) 340
(D) 180 (E) None of these

220. What is the average number of children present in Jan, March and
May?
जनवयी, भाचष औय भई भें उऩजस्थत फच्चों की औसत सॊख्मा क्मा है?
(A) 540 (B) 650 (C) 750
(D) 480 (E) None of these

(221 – 225) Pie chart shows the number of students in 5 classes and bar
graph shows the number of student in 3 different sections that are A and
B, and remaining are students of C.
ऩाई चाटष 5 कऺाओॊ भें छात्रों की सॊख्मा ददखाता है औय फाय ग्राप 3 अरग – अरग वगों भें
छात्रों की सॊख्मा ददखाता है जो A औय B हैं , औय िे र्ष C के छात्र हैं ।

https : https : https : https :


//youtube.co //www.face //instagram.c
m/channel/U book.com/a om/aashishar //t.me/
CYa4_JrOrf8R orasocial?ut
309 5Kz2uOtccXQ
a s hisharora
m_medium = studifie
s ocial
copy_link d
221. The number of students of class 9 in section C are what percent less
than the number of students of class 8 in section C?
से क्िन C भें कऺा 9 के छात्रों की सॊख्मा, से क्िन C भें कऺा 8 के छात्रों की सॊख्मा से ककतने
प्रनतित कभ है?
(A) 28% (B) 24% (C) 25%
(D) 30% (E) None of these

222. Find the ratio of number of students in section B of Class 10th, 11th and
12th with the number of students in section A of Class 8th, 9th and 10th?
कऺा 10वीॊ, 11वीॊ औय 12वीॊ के से क्िन B भें ववद्माचथषमों की सॊख्मा का कऺा 8वीॊ,
9वीॊ औय 10वीॊ के से क्िन A भें ववद्माचथषमों की सॊख्मा से अनुऩात ऻात कीजजए।
(A) 1667 : 1170 (B) 1170 : 1667 (C) 1542 : 1710
(D) 1710 : 1542 (E) None of these

223 If total number of students passed in class 10th in the examination is


33.33% and percentage of student pass in section A and B is 16 2/3% and
66 2/3% then find the number of students passed in section C?
मदद ऩयीऺा भें कऺा 10वीॊ भें उत्तीणष छात्रों की कु र सॊख्मा 33.33% है औय से क्िन A औय B
भें छात्रों का प्रनतित 16 2/3% औय 66 2/3% है, तो से क्िन C भें उत्तीणष छात्रों की सॊख्मा
ऻात कीजजए।
(A) 143 (B) 183 (C) 154
(D) 153 (E) None of these

224. Find the average number of students in section C in all classes?


सबी कऺाओॊ भें से क्िन C भें छात्रों की औसत सॊख्मा ऻात कीजजए।
(A) 1754.4 (B) 1829.4 (C) 1522.4
(D) 1751.4 (E) None of these
https : https : https : https :
//youtube.co //www.face //instagram.c
m/channel/U book.com/a om/aashishar //t.me/
CYa4_JrOrf8R orasocial?ut
310 5Kz2uOtccXQ
a s hisharora
m_medium = studifie
s ocial
copy_link d
225. Find the difference between number of total students in section A
and number of total students in section B in all classes?
से क्िन A भें कु र छात्रों की सॊख्मा औय सबी कऺाओॊ भें से क्िन B भें कु र छात्रों की सॊख्मा के
फीच का अॊतय ऻात कीजजए।
(A) 3813 (B) 5484 (C) 2175
(D) 6776 (E) None of these

(226 – 230) Line chart shows the percentage of number of mobile phones
sold by three companies in 5 different years. Number of total mobile
phones sold is different every year.
राइन चाटष 5 अरग – अरग वर्षों भें तीन कॊ ऩननमों द्वाया फे चे गए भोफाइर पोन की सॊख्मा का
प्रनतित दिाषता है। फे चे गए कु र भोफाइर पोन की सॊख्मा हय सार अरग होती है

226. Number of mobiles sold in all the years is 15200 then find the average
number of Apple mobiles sold in all years?
सबी वर्षों भें फे चे गए भोफाइरों की सॊख्मा 15200 है, तो सबी वर्षों भें फे चे गए Apple भोफाइरों
की औसत सॊख्मा ऻात कीजजए।
(A) 5867.6 (B) 4587.6 (C) 6201.7
(D) 4578.6 (E) None of these

227. If number of mobiles sold in 2012 and 2015 is 19200 and 18500 then
find the difference between total number of Samsung mobiles and total
number of blackberry mobiles sold in 2012 and 2015?
मदद 2012 औय 2015 भें फे चे गए भोफाइरों की सॊख्मा 19200 औय 18500 है, तो 2012 औय
2015 भें फे चे गए सैभसॊग भोफाइरों की कु र सॊख्मा औय ब्रैक फे य ी भोफाइरों की कु र सॊख्मा के
फीच का अॊतय ऻात कीजजए।
(A) 2618 (B) 2541 (C) 3548
(D) 2488 (E) None of these
https : https : https : https :
//youtube.co //www.face //instagram.c
m/channel/U book.com/a om/aashishar //t.me/
CYa4_JrOrf8R orasocial?ut
311 5Kz2uOtccXQ
a s hisharora
m_medium = studifie
s ocial
copy_link d
228. If number of mobiles sold in 2011 is 4750 which is 26 2/3% more than
number of mobiles sold in 2013, then find the ratio of number of mobiles
phones sold by Samsung in 2011 with the number of mobiles sold by
Apple In 2013?
मदद 2011 भें वे चे गए भोफाइरों की सॊख्मा 4750 है जो 2013 भें फे चे गए भोफाइरों की सॊख्मा
से 26 2/3% अचधक है, तो 2011 भें से भसॊग द्वाया फे चे गए भोफाइर पोनों की सॊख्मा का
2013 भें एप्ऩर द्वाया फे चे गए भोफाइरों की सॊख्मा से अनुऩात ऻात कीजजए।
(A) 37 : 41 (B) 37 : 43 (C) 27 : 19
(D) 19 : 27 (E) None of these

229. Number of mobiles sold by Samsung in 2014 is how much percent


less than total mobiles sold by Apple and blackberry in 2014?
2014 भें सैभसॊग द्वाया फे चे गए भोफाइरों की सॊख्मा 2014 भें एप्ऩर औय ब्रैक फे य ी द्वाया फे चे
गए कु र भोफाइरों से ककतने प्रनतित कभ है?
(A) 56.14% (B) 57.14% (C) 55.5%
(D) 46.5% (E) None of these

230. Find the number of total mobiles sold by Apple in 2012 if Samsung
sold 4712 mobiles in 2012?
2012 भें Apple द्वाया फे चे गए कु र भोफाइरों की सॊख्मा ऻात कीजजए मदद सैभसॊग ने 2012
भें 4712 भोफाइर फे च?

(A) 3572 (B) 3472 (C) 3672
(D) 3482 (E) None of these

(231 – 235) Pie chart shows percentage of total crackers sold by 5 shops
and bar graph shows the ratio of type of crackers sold by these shops.
Total crackers sold = 36000.
ऩाई चाटष 5 दक ु ानों द्वाया फे चे गए कु र ऩटाखों का प्रनतित दिाषता है औय फाय ग्राप इन दक
ु ानों
द्वाया फे चे गए ऩटाखों के प्रकाय का अनुऩात दिाषता है। फे चे गए कु र ऩटाखे = 36000

https : https : https : https :


//youtube.co //www.face //instagram.c
m/channel/U book.com/a om/aashishar //t.me/
CYa4_JrOrf8R orasocial?ut
312 5Kz2uOtccXQ
a s hisharora
m_medium = studifie
s ocial
copy_link d
231. Find the ratio of number of Anars sold by shop B, C and E with the
number of sparklers sold by A, C and D?
दकु ान B, C औय E द्वाया फे चे गए अनायों की सॊख्मा का A, C औय D द्वाया फे चे गए
पु रझडडमों की सॊख्मा से अनुऩात ऻात कीजजमे ?
(A) 110 : 197 (B) 111 : 196 (C) 112 : 195
(D) 113 : 194 (E) None of these

232. The number of Sparkles sold by B is what percent more than the
number of rockets sold by C?
B द्वाया फे चे गए पु रझडडमों की सॊख्मा, C द्वाया फे चे गए यॉकेटों की सॊख्मा से ककतने प्रनतित
अचधक है?
(A) 90% (B) 100% (C) 110%
(D) 120% (E) None of these

233. If the cost of an Anar, a Sparkles and a Rocket is Rs. 5, Rs. 10 and Rs.
15, find the revenue of shop A and shop B?
मदद एक अनाय, एक स्ऩाकष ल्स औय एक यॉके ट की कीभत 5 रु., 10 रु. औय 15 रु. है, तो दुक ान
A औय दुक ान B का याजस्व ऻात कीजजए।
(A) Rs. 143454 (B) Rs. 183454 (C) Rs. 154545
(D) Rs. 179775 (E) None of these

234. If there is an increase of 46 2/3% in sales of Rockets sold by Shop E


and decrease of 412% in sales of Sparkders and total number of Anars
sold by E remains same then find the total number of crackers sold by
shop E?
मदद दक ु ान द्वाया फे चे गए यॉके ट की त्रफक्री भें 46 2/3% की ववर ि होती है औय स्ऩाकष रय की
त्रफक्री भें 412/3% की कभी होती है औय दद्वाया फे चे गए अनायों की कु र सॊख्मा सभान यहती है,
तो दुक ान द्वाया फे चे गए ऩटाखेhttps की
: कु र सॊख्माhttps
ऻात: कीजजए। https : https :
//youtube.co //www.face //instagram.c
m/channel/U book.com/a om/aashishar //t.me/
CYa4_JrOrf8R orasocial?ut
313 5Kz2uOtccXQ
a s hisharora
m_medium = studifie
s ocial
copy_link d
(A) 7666 (B) 6336 (C) 4556
(D) 6896 (E) None of these

235. Find the average number of Rockets sold by all the shop together?
सबी दुक ानों द्वाया फे चे गए यॉके टों की औसत सॊख्मा ऻात कीजजए।
(A) 3816 (B) 2412 (C) 2452
(D) 2464 (E) None of these

(236 – 240) Table shows the number of chairs sold by 5 different


showrooms in 3 months.
ताशरका 3 भहीनों भें 5 अरग – अरग िोरूभ भें फे ची गई कु शसषमों की सॊख्मा ददखाती है।

Ratio of number Number of Difference between


of chairs sold in chairs sold number of chairs
Showrooms
January : in sold in January and
February February March

A 17 : 13 962 92

B 11 : 19 988 – 142

C 12 : 17 1156 – 198

D 5:8 984 55

E 14 : 11 1375 450

236. Number of chairs sold by C in January is what percent less than the
number of chairs sold by E in March?
C द्वाया जनवयी भें फे ची गई कु शसषमों की सॊख्मा, भाचष भें E द्वाया फे ची गई कु शसषमों की सॊख्मा
से ककतना प्रनतित कभ है?
(A) 36.9% (B) 33.3% (C) 37.2%
(D) 38.4% (E) None of these

237. If 53 1/32% of chairs sold by D in January are broken and 28.56% of


chair sold by D in March are also broken then find what percent of chairs
remains good?
मदद जनवयी भें द्वाया फे ची गई 53 1/3% कु शसषमों टूट जाती है औय भाचष भें D द्वाया 28.56
प्रनतित कु शसषमों बी टूट जाती है, ऻात कीजजए कक ककतने प्रनतित कु नतषमाॊ ठीक यहती है?

https : https : https : https :


//youtube.co //www.face //instagram.c
m/channel/U book.com/a om/aashishar //t.me/
CYa4_JrOrf8R orasocial?ut
314 5Kz2uOtccXQ
a s hisharora
m_medium = studifie
s ocial
copy_link d
(A) 55.6% (B) 52.5% (C) 56.4%
(D) 58.4% (E) None of these

238. Find the difference between chairs sold by A, B and C in January and
the number of chairs sold by D and E in March?
जनवयी भें A, B औय C द्वाया फे ची गई कु शसषमों औय भाचष भें D औय E द्वाया फे ची गई
कु शसषमों की सॊख्मा के फीच का अॊतय ऻात कीजजए।
(A) 786 (B) 856 (C) 896
(D) 776 (E) None of these

239. If cost price of chairs in February is 37.5% more than cost price of
chair in January which is Rs. 1856, then find the total cost price of chairs
sold by C in February?
मदद पयवयी भें कु शसषमों का क्रम भूल्म जनवयी भें कु सी के क्रम भूल्म से 37.5% अचधक है जो
कक 1856 रु. है, तो पयवयी भें C द्वाया फे ची गई कु शसषमों का कु र रागत भूल्म ऻात कीजजए।
(A) Rs. 2950110 (B) Rs. 2950118 (C) Rs. 2950112
(D) Rs. 2950114 (E) None of these

240. Find the average number of chairs sold by all shops in February?
पयवयी भें सबी दुक ानों द्वाया फे ची गई कु शसषमों की औसत सॊख्मा ऻात कीजजमे ?
(A) 1423 (B) 1723 (C) 1523
(D) 1093 (E) None of these

(241 – 245) Bar graph shows the sum of boys and girls, difference
between boys and girls and difference between number of boys and
teachers in 5 different schools.
फाय ग्राप 5 अरग – अरग स्कू रों भें रडकों औय रडककमों का मोग, रडकों औय रडककमों के
फीच अॊतय औय रडकों औय शिऺकों की सॊख्मा के फीच का अॊतय ददखाता है।

https : https : https : https :


//youtube.co //www.face //instagram.c
m/channel/U book.com/a om/aashishar //t.me/
CYa4_JrOrf8R orasocial?ut
315 5Kz2uOtccXQ
a s hisharora
m_medium = studifie
s ocial
copy_link d
241. Number of teachers in E is what percent of number of Boys in C?
E भें शिऺकों की सॊख्मा, C भें रडकों की सॊख्मा का ककतना प्रनतित है?
(A) 25% (B) 20% (C) 30%
(D) 35% (E) None of these

242. Sum of number of Girls in school B and C is what percent less than
the number of Boys in school A and E?
स्कू र B औय C भें रडककमों की सॊख्मा का मोग स्कू र A औय E भें रडकों की सॊख्मा से ककतना
प्रनतित कभ है?
(A) 33.33% (B) 25% (C) 20.75%
(D) 30% (E) None of these

243. f number of boys in school C is increased by 46 2/3% and number of


girls increased by 25% in C, then find the ratio of number of boys and
girls in school C to the teachers in school C?
मदद स्कू र भें रडकों की सॊख्मा भें 46 2/3% की ववर ि होती है औय भें रडककमों की सॊख्मा भें
25% की वरवि होती है, तो स्कू र भें रडकों औय रडककमों की सॊख्मा का स्कू र C भें शिऺकों की
सॊख्मा से अनुऩात ऻात कीजजए।
(A) 64 : 7 (B) 62 : 5 (C) 64 : 5
(D) 62 : 9 (E) None of these

244. Find the average number of teachers in all schools?


सबी स्कू रों भें शिऺकों की औसत सॊख्मा ऻात कीजजमे ?
(A) 74 (B) 78 (C) 80
(D) 84 (E) None of these

245. Find the difference between number of Boys in C, D and E and


number of girls in A, B and C?
C, D औय E भें रडकों की सॊख्मा औय A, B औय C भें रडककमों की सॊख्मा के फीच का अॊतय
ऻात कीजजए।
(A) 300 (B) 370 (C) 350
(D) 330 (E) None of these

(246 – 250) Pie chart shows total number of electric items sold by 5 shops
that is TV and AC. Line chart shows the % age of TV’s sold more than AC’s
sold. Total items = 5000
ऩाई चाटष 5 दुक ानों द्वाया फे चे गए त्रफजरी के साभानों मानी टीवी औय एसी की कु र सॊख्मा को
दिाषता है। राइन चाटष त्रफकने वारे टीवी की एसी की त्रफक्री से अचधक प्रनतित ददखाता है। कु र
आइटभ = 5000

https : https : https : https :


//youtube.co //www.face //instagram.c
m/channel/U book.com/a om/aashishar //t.me/
CYa4_JrOrf8R orasocial?ut
316 5Kz2uOtccXQ
a s hisharora
m_medium = studifie
s ocial
copy_link d
246. Find the ratio of number of TV sold by B with the number of TV sold
by C?
B द्वाया फे चे गए टीवी की सॊख्मा का C द्वाया फे चे गए टीवी की सॊख्मा से अनुऩात ऻात
कीजजमे ?
(A) 21 : 26 (B) 21 : 25 (C) 25 : 27
(D) 25 : 28 (E) None of these

247. The number of TV sold by B is what percent more than number of AC


sold by D?
B द्वाया फे चे गए टीवी की सॊख्मा, D द्वाया फे चे गए AC की सॊख्मा से ककतने प्रनतित अचधक
है?
(A) 33.25% (B) 32.25% (C) 31.25%
(D) 34.25% (E) None of these
https : https : https : https :
//youtube.co //www.face //instagram.c
m/channel/U book.com/a om/aashishar //t.me/
CYa4_JrOrf8R orasocial?ut
317 5Kz2uOtccXQ
a s hisharora
m_medium = studifie
s ocial
copy_link d
248. If number of AC's sold by E is increased by 41 2/3% and number of
TV's sold by E is decreased by 20% then by what percent number of AC's
sold will be more than number of TV's sold by E?
मदद E द्वाया फे चे गए. AC की सॊख्मा भें 41 2/3% की वरवि की जाती है औय द्वाया फे चे जाने
वारे टीवी की सॊख्मा भें 20% की कभी की जाती है, तो द्वाया फे चे गए AC की सॊख्मा द्वाया
फे चे गए टीवी की सॊख्मा से ककतने प्रनतित अचधक होगी?
(A) 20% (B) 25% (C) 30%
(D) 15% (E) None of these

249. Find the difference between number of TV’s sold by A, D and E to the
number of AC’s sold by B, C and E?
A, D औय E द्वाया फे चे गए टीवी की सॊख्मा औय B, C औय E द्वाया फे चे गए AC की सॊख्मा
के फीच का अॊतय ऻात कीजजए।
(A) 475 (B) 500 (C) 525
(D) 550 (E) None of these

250. Find average number of TV sold by all the shops?


सबी दकु ानों द्वाया फे चे गए टीवी की औसत सॊख्मा ऻात कीजजए।
(A) 595 (B) 655 (C) 685
(D) 605 (E) None of these

(251 – 255) Table shows the number of students appeared in PB – I(pre


board I) and PB – II(pre board II) and number of students passed in 5
different schools.
ताशरका ऩाॊच अरग – अरग स्कू रों भें PB – I (प्री फोडष I) औय PB – II (प्री फोडष II) भें
उऩजस्थत छात्रों की सॊख्मा औय उत्तीणष छात्रों की सॊख्मा ददखाती है।
Average number of %age of %age of
Number of students in
School students appear in PB – I students passed students passed
PB – I : PB – II
and PB – II in PB – I in PB – II

P 750 8:7 42% 47%


Q 550 6:5 55% 29%
R 630 4:5 40% 37%
S 840 5:7 35% 40%
T 720 11 : 13 25% 60%

251. Find the ratio of number of students passed in school P in PB – I with


the number of students passed in school R in PB – II?
PB – I भें स्कू र P भें उत्तीणष छात्रों की सॊख्मा का PB – II भें स्कू र R भें उत्तीणष छात्रों की
सॊख्मा से अनुऩात ऻात कीजजए।
(A) 223 : 154 (B) 154 : 223 (C) 336 : 259
(D) 154 : 223 (E) None of these
https : https : https : https :
//youtube.co //www.face //instagram.c
m/channel/U book.com/a om/aashishar //t.me/
CYa4_JrOrf8R orasocial?ut
318 5Kz2uOtccXQ
a s hisharora
m_medium = studifie
s ocial
copy_link d
252. Number of student who passed in PB – I of school Q is what percent
more than the number of students passed in PB – II of school Q?
स्कू र Q के PB – I भें उत्तीणष होने वारे छात्रों की सॊख्मा, स्कू र Q के PB – II भें उत्तीणष छात्रों
की सॊख्मा से ककतना प्रनतित अचधक है?
(A) 127 : 5% (B) 128.9% (C) 145.5%
(D) 136.2% (E) None of these

253. If number of student passed PB – I of school T is increased by 73


1/3% and number of students passed PB – II is decreased by 25% , then
find the difference between number of students passed in PB – I and PB –
II?
मदद स्कू र के PB – 1 भें उत्तीणष छात्रों की सॊख्मा भें 73 1/3% की वरवि होती है औय
PB – 11 भें उत्तीणष होने वारे छात्रों की सॊख्मा भें 25% की कभी होती है, तो PB – 1
औय PB – II भें उत्तीणष छात्रों की सॊख्मा के फीच का अॊतय ऻात कीजजए।
(A) 62 (B) 58 (C) 55
(D) 65 (E) None of these

254. Find the average number of students appeared in PB – II?


PB – II भें उऩजस्थत होने वारे छात्रों की औसत सॊख्मा ऻात कीजजए।
(A) 748 (B) 732 (C) 784
(D) 746 (E) None of these

255. Find the difference between number of students passed in PB – I of


school P, R and S and the number of students passed in the PB – II of the
schools P, Q, R and S?
स्कू र P, R औय S के PB – I भें उत्तीणष छात्रों की सॊख्मा औय P, Q, R औय S स्कू र के PB –
II भें उत्तीणष छात्रों की सॊख्मा के फीच का अॊतय ऻात कीजजए।
(A) 320 (B) 340 (C) 360
(D) 300 (E) None of these

(256 – 260) Pie chart shows the percentage of total automobiles sold on
Diwali and line graph shows the number of percentage of bikes sold more
than cars. Total automobiles = 25000
ऩाई चाटष ददवारी ऩय फे चे गए कु र ऑटोभोफाइर का प्रनतित दिाषता है औय राइन ग्राप कायों
की तुरना भें अचधक त्रफकने वारी फाइक के प्रनतित को दिाषता है।कु र ऑटोभोफाइर = 25000

https : https : https : https :


//youtube.co //www.face //instagram.c
m/channel/U book.com/a om/aashishar //t.me/
CYa4_JrOrf8R orasocial?ut
319 5Kz2uOtccXQ
a s hisharora
m_medium = studifie
s ocial
copy_link d
256. Number of bikes sold by S is what percent more than the number of
cars sold by Q?
S द्वाया फे ची गई फाइकों की सॊख्मा Q द्वाया फे ची गई कायों की सॊख्मा से ककतने प्रनतित
अचधक है?
(A) 242% (B) 240% (C) 220%
(D) 260% (E) None of these

257. Find the difference between number of cars sold by P and Q and
number of bikes sold by R and T?
P औय Q द्वाया फे ची गई कायों की सॊख्मा औय R औय T द्वाया फे ची गई फाइक की सॊख्मा के
फीच का अॊतय ऻात कीजजए।
(A) 2965 (B) 2875 (C) 2950
(D) 2855 (E) None of these

258. If number of bikes sold by U is 26(2/3) % more than the number of


cars sold by R & number of cars sold by U is 22.22% more than the
number of bikes sold by Q, then find the number of vehicles sold by U.
मदद U द्वाया फे ची गई फाइकों की सॊख्मा R द्वाया फे ची गई कायों की सॊख्मा से 26(2/3) %
अचधक है औय U द्वाया फे ची गई कायों की सॊख्मा Q द्वाया फे ची गई फाइक की सॊख्मा से
22.22% अचधक है, तो U द्वाया फे चे गए वाहनों की सॊख्मा ऻात कीजजए।
(A) 5255 (B) 5125 (C) 5362
(D) 5455 (E) None of these

259. Find the average number of cars sold by all the shops together?
सबी दुक ानों द्वाया एक साथ फे ची गई कायों की औसत सॊख्मा ऻात कीजजमे ?
(A) 1975 (B) 1960 (C) 1925
(D) 1090 (E) None of these

https : https : https : https :


//youtube.co //www.face //instagram.c
m/channel/U book.com/a om/aashishar //t.me/
CYa4_JrOrf8R orasocial?ut
320 5Kz2uOtccXQ
a s hisharora
m_medium = studifie
s ocial
copy_link d
260. If 20% of the bikes sold and 30% of the cars sold by P are electric,
then find the number of non – electric vehicles sold by P?
मदद P द्वाया फे ची गई 20% फाइक औय P द्वाया फे ची गई 30% कायें इरे जक्ट्रक हैं , तो P द्वाया
फे चे गए गैय – इरे जक्ट्रक वाहनों की सॊख्मा ऻात कीजजए।
(A) 3555 (B) 3795 (C) 3825
(D) 3625 (E) None of these

(261 – 265) Pie chart 1 shows percentage of toto number of candles and
Diyas sold and pie chart 2 shows percentage of Diya sold by five
shopkeeper during Diwali. Total Diyas sold = 5000 and number of candles
sold by A is 50 more than number of Diya sold by A.
ऩाई चाटष 1 फे ची गई भोभफजत्तमों औय दीमों की कु र सॊख्मा का प्रनतित ददखाता है औय ऩाई
चाटष 2 ददवारी के दौयान ऩाॊच दक
ु ानदायों द्वाया फे चे गए दीमों का प्रनतित दिाषता है। फे चे गए
कु र दीमे = 5000 औय A द्वाया फे ची गई भोभफजत्तमों की सॊख्मा, A द्वाया फे चे गए दीमों की
सॊख्मा से 50 अचधक है।

https : https : https : https :


//youtube.co //www.face //instagram.c
m/channel/U book.com/a om/aashishar //t.me/
CYa4_JrOrf8R orasocial?ut
321 5Kz2uOtccXQ
a s hisharora
m_medium = studifie
s ocial
copy_link d
261. Number of Diya sold by shopkeeper C is what percent less than
number of candles sold by E?
दुक ानदाय C द्वाया फे चे गए दीमों की सॊख्मा E द्वाया फे ची गई भोभफजत्तमों की सॊख्मा से
ककतने प्रनतित कभ है?
(A) 60% (B) 50% (C) 40%
(D) 30% (E) None of these

262. Which shopkeeper does not sell any Candles?


कौन सा दुक ानदाय कोई भोभफत्ती नहीॊ फे चता है?
(A) A (B) B (C) C
(D) D (E) None of these

263. If 37.5% of Diya get broken and 46 2/3% of the candles get broken by
shopkeeper E, then find the percentage of Diya and candles remains
good?
मदद दीमों का 37.5% टूट जाता है औय 46 2/3% भोभफजत्तमाॊ दुक ानदाय द्वाया टूट जाती हैं ,
तो दीमों औय भोभफजत्तमों का प्रनतित ठीक यहता है?
(A) 57% (B) 47% (C) 49%
(D) 55% (E) None of these

264. If the price of a Diya is Rs. 2.5 and price of a candle is 7.5. then find
the revenue of shopkeeper A?
मदद एक दीमे की कीभत 2.5 रु. है औय एक भोभफत्ती की कीभत 7.5 रु. है। तो दुक ानदाय A
का याजस्व ऻात कीजजए।
(A) Rs. 8615 (B) Rs. 8635 (C) Rs. 8625
(D) Rs. 8645 (E) None of these

265. Find the average number of candles sold by all the shopkeepers?
सबी दुक ानदायों द्वाया फे ची गई भोभफजत्तमों की औसत सॊख्मा ऻात कीजजमे ?
(A) 1000 (B) 1200 (C) 1500
(D) 840 (E) None of these

(266 – 270) Bar graph show the ratio of CP to MP of five different items
and Line graph shows the percentage of discount given on them.
फाय ग्राप ऩाॊच अरग – अरग वस्तुओ ॊ के CP औय MP के अनुऩात को ददखाता है औय राइन
ग्राप उन ऩय दी गई छू ट का प्रनतित ददखाता है।

https : https : https : https :


//youtube.co //www.face //instagram.c
m/channel/U book.com/a om/aashishar //t.me/
CYa4_JrOrf8R orasocial?ut
322 5Kz2uOtccXQ
a s hisharora
m_medium = studifie
s ocial
copy_link d
266. If the CP of R and T is Rs. 2166 and Rs. 2354 then find the ratio of SP
of R with the SP of T.
मदद R औय T का CP 2166 रु. औय 2354 रु. है, तो R के SP का T के SP से अनुऩात ऻात
कीजजए।
(A) 1251 : 1444 (B) 1444 : 1391 (C) 1444 : 1251
(D) 1391 : 1251 (E) None of these

267. If CP of Q and R is Rs. 784 and Rs. 792 then find the difference
between the discount given by them.
मदद Q औय R का CP 784 रु. औय 792 रु. है, तो उनके द्वाया दी गई छू ट के फीच का अॊतय
ऻात कीजजए।
(A) Rs. 164 (B) Rs. 154 (C) Rs. 174
(D) Rs. 144 (E) None of these

268. If the discount percent on item P is changes to 46 2/3% then find the
new profit percent of P?
मदद वस्तु P ऩय छू ट प्रनतित 46 2/3% भें फदर जाता है, तो P का नमा राब प्रनतित ऻात
कीजजए।
(A) 20% (B) 25% (C) 5%
(D) No profit (E) None of these

269. If CP of item P and item S is Rs. 512 and Rs. 528 then how much
percent SP of P is more than SP of S?
मदद वस्तु P औय वस्तु S का CP 512 रु. औय 528 रु. है, तो P का SP, S के SP से ककतना
प्रनतित अचधक है?
(A) 44.44.% (B) 19.19% (C) 11.11%
(D) 57.57% (E) None of these
https : https : https : https :
//youtube.co //www.face //instagram.c
m/channel/U book.com/a om/aashishar //t.me/
CYa4_JrOrf8R orasocial?ut
323 5Kz2uOtccXQ
a s hisharora
m_medium = studifie
s ocial
copy_link d
270. Which shopkeeper sold the item at no profit no loss?
ककस दुक ानदाय ने वस्तु को त्रफना राब हानन ऩय फे चा?
(A) P and R (B) Q (C) T
(D) Q and S (E) None of these

(271 – 275) Pie chart shows the percentage of students appeared in bank
exam and line chart shows the percentage of number of student selected.
Total students appeared = 36000.
ननम्नशरणखत फाय चाटष का ध्मानऩूवषक अध्ममन कयें औय नीचे ददए गए प्रश्नों के उत्तय दें।
ऩाई चाटष फैंक ऩयीऺा भें उऩजस्थत छात्रों का प्रनतित दिाषता है औय राइन चाटष चमननत छात्रों
की सॊख्मा का प्रनतित दिाषता है। कु र उऩजस्थत छात्र = 36000।

https : https : https : https :


//youtube.co //www.face //instagram.c
m/channel/U book.com/a om/aashishar //t.me/
CYa4_JrOrf8R orasocial?ut
324 5Kz2uOtccXQ
a s hisharora
m_medium = studifie
s ocial
copy_link d
271. Number of students selected in 2015 is what percent of number of
student not selected in 2019?
2015 भें चमननत छात्रों की सॊख्मा 2019 भें नहीॊ चन
ु े गए छात्रों की सॊख्मा का ककतना प्रनतित
है?
(A) 7.12% (B) 9.67% (C) 9.10%
(D) 12.7% (E) None of these

272. In years 2020 the number of students appeared is 53 1/3% more than
the number of students appeared in 2017 and number of students
selected is 28.56% less than number of student not selected, find the
number of students selected in the exam?
वर्षष 2020 भें उऩजस्थत होने वारे छात्रों की सॊख्मा 2017 भें उऩजस्थत छात्रों की सॊख्मा
से 53 1/3% अचधक है औय चमननत छात्रों की सॊख्मा चमननत नहीॊ होने वारे छात्रों की
सॊख्मा से 28.56% कभ है, ऩयीऺा भें चमननत छात्रों की सॊख्मा ऻात कीजजमे ?
(A) 1450 (B) 4560 (C) 830
(D) 4830 (E) None of these

273. Find the ratio of number of students not selected in 2016 with the
number of students not selected in 2017?
2016 भें न चुने गए ववद्माचथषमों की सॊख्मा का 2017 भें न चुने गए ववद्माचथषमों की सॊख्मा से
अनुऩात ऻात कीजजए।
(A) 5 : 6 (B) 25 : 42 (C) 20 : 21
(D) 21 : 20 (E) None of these

274. Find the difference between the number of students selected in 2017
and 2015 and the number of students selected in 2016, 2018 and 2019?
2017 औय 2015 भें चमननत छात्रों की सॊख्मा औय 2016, 2018 औय 2019 भें चमननत छात्रों
की सॊख्मा के फीच का अॊतय ऻात कीजजए।
(A) 2950 (B) 2805 (C) 2292
(D) 2582 (E) None of these

275. Find the average number of students not selected in the given 5
years?
ददए गए 5 वर्षों भें नहीॊ चुने गए छात्रों की औसत सॊख्मा ऻात कीजजए।
(A) 5642.4 (B) 5012.4 (C) 5006.4
(D) 5642.4 (E) None of these

(276 – 280) Tabular data shows the different number of people like 4
different sports in 5 different colonies.
सायणीफि डे टा 5 अरग – अरग कॉरोननमों भें 4 अरग – अरग खेरों को ऩसॊद कयने वारे
रोगों की अरग – अरग सॊख्मा ददखाता है।

https : https : https : https :


//youtube.co //www.face //instagram.c
m/channel/U book.com/a om/aashishar //t.me/
CYa4_JrOrf8R orasocial?ut
325 5Kz2uOtccXQ
a s hisharora
m_medium = studifie
s ocial
copy_link d
Number of Total number of people Ratio of number of Number of
Colonies people like like Basketball and people like Swimming people like
swimming Football and cricket football

A 95 154 5:7 74

B 87 204 3:4 92

C 49 184 7:9 86

D 72 158 4:3 104

E 51 160 3:5 52

276. Find the ratio of number of people like swimming in colony A, D and
E with the number of people like Football in the colony B, C and E?
कॉरोनी A, D औय E भें तैय ाकी ऩसॊद कयने वारे रोगों की सॊख्मा का कॉरोनी B, C औय E भें
पु टफॉर ऩसॊद कयने वारे रोगों की सॊख्मा से अनुऩात ऻात कीजजए।
(A) 132 : 105 (B) 105 : 132 (C) 109 : 115
(D) 112 : 131 (E) None of these

277. Number of people who play sports in colony A are what percent more
than the number of people play sports in colony E?
कॉरोनी A भें खेर खेरने वारे रोगों की सॊख्मा कॉरोनी E भें खेर खेरने वारे रोगों की सॊख्मा
से ककतने प्रनतित अचधक है?
(A) 29.05% (B) 22.05% (C) 21.05%
(D) 24.05% (E) None of these

278.Find the average number of basketball players in all the colonies?


सबी कॉरोननमों भें फास्के टफॉर णखराडडमों की औसत सॊख्मा ऻात कीजजए।
(A) 94.7 (B) 94.8 (C) 97.6
(D) 90.7 (E) None of these

279. In a cricket tournament 28.56% of colony A, 25% of colony B, 33.33%


of colony C, 16.66% of colony D and 20% of colony E peoples who like
cricket participate, find the number of people who participated in the
tournament?
एक कक्रकेट टूनाषभेंट भें कॉरोनी A के 28.56% , कॉरोनी B के 25% , कॉरोनी C के 33.33% ,
कॉरोनी D के 16.66% औय कॉरोनी E के 20% रोग जो कक्रके ट ऩसॊद कयते हैं भें बाग रे ते हैं ,
टूनाषभेंट भें बाग रे ने वारे रोगों की सॊख्मा ऻात कीजजए।
(A) 114 (B) 115 (C) 116
(D) 117 (E) None of these
https : https : https : https :
//youtube.co //www.face //instagram.c
m/channel/U book.com/a om/aashishar //t.me/
CYa4_JrOrf8R orasocial?ut
326 5Kz2uOtccXQ
a s hisharora
m_medium = studifie
s ocial
copy_link d
280. Find the average number of players who like cricket in all the
colonies?
सबी कॉरोननमों भें कक्रकेट ऩसॊद कयने वारे णखराडडमों की औसत सॊख्मा ऻात कीजजए।
(A) 94.7 (B) 90.2 (C) 97.6
(D) 90.4 (E) None of these

(281 – 285) The pie chart represents the % increase of sale of crackers in 5
cities in year 2020 as compared to 2019 and the bar chart shows the %
increase of sale of crackers in these 5 cities in year 2021 as compared to
2020. ( The number of crackers sold in 2019 is same for every city)
ऩाई चाटष 2019 की तुरना भें वर्षष 2020 भें 5 िहयों भें ऩटाखों की त्रफक्री भें % वरवि का
प्रनतननचधत्व कयता है औय फाय चाटष 2020 की तुरना भें वर्षष 2021 भें इन 5 िहयों भें
ऩटाखों की त्रफक्री भें % ववर ि दिाषता है। (2019 भें त्रफकने वारे ऩटाखों की सॊख्मा हय
िहय के शरए सभान है)

281. What is the ratio of number of crackers sold by city B in 2021 to


number of crackers sold by city C and D together in 2020?
2021 भें िहय B द्वाया फे चे गए ऩटाखों की सॊख्मा का 2020 भें िहय C औय D द्वाया शभराकय
फे चे गए ऩटाखों की सॊख्मा से अनुऩात ककतना है?
(A) 85 : 70 (B) 29 : 56 (C) 97 : 25
(D) 38 : 53 https : https
(E) None of these : https : https :
//youtube.co //www.face //instagram.c
m/channel/U book.com/a om/aashishar //t.me/
CYa4_JrOrf8R orasocial?ut
327 5Kz2uOtccXQ
a s hisharora
m_medium = studifie
s ocial
copy_link d
282. What is the overall increase in number of crackers sold in city E from
year 2019 to 2021?
वर्षष 2019 से 2021 तक िहय E भें फे चे गए ऩटाखों की सॊख्मा भें कुर वरवि ककतनी है?
(A) 105% (B) 102% (C) 107%
(D) 119% (E) None of these

283. What is the average difference between number of crackers sold by all
cities together in year 2021 and 2020?
वर्षष 2021 औय 2020 भें सबी िहयों द्वाया एक साथ फे चे गए ऩटाखों की सॊख्मा के फीच
औसत अॊतय ककतना है?
(A) 49.6 (B) 49.9 (C) 46.9
(D) 46.6 (E) None of these

284. The number of crackers sold in city D in year 2021 is what % of the
number of crackers sold in year B and C in year 2020?
वर्षष 2021 भें िहय D भें फे चे गए ऩटाखों की सॊख्मा वर्षष 2020 भें वर्षष B औय C भें फे चे गए ऩटाखों
की सॊख्मा का ककतना प्रनतित है?
(A) 81.99% (B) 81.25% (C) 81.09%
(D) 81.90% (E) None of these

285. What is the ratio of number of crackers sold in city A and C together in
year 2020 to the average number of crackers sold by all cities together in
2021.
वर्षष 2020 भें िहय A औय C भें शभराकय फे चे गए ऩटाखों की सॊख्मा का 2021 भें सबी िहयों
द्वाया एक साथ फे चे गए ऩटाखों की औसत सॊख्मा से अनुऩात ककतना है?
(A) 800 : 807 (B) 450 : 401 (C) 401 : 450
(D) 610 : 451 (E) None of these

(286 – 290) Bar chart shows the difference between the number of diwali
sweets (X and Y) sold and the average number of X and Y sweet sold by 5
shopkeepers.
फाय चाटष 5 दक
ु ानदायों द्वाया फे ची गई ददवारी शभठाई (X औय Y) की सॊख्मा के फीच का अॊतय औय
X औय Y शभठाई की औसत सॊख्मा को दिाषता है।

https : https : https : https :


//youtube.co //www.face //instagram.c
m/channel/U book.com/a om/aashishar //t.me/
CYa4_JrOrf8R orasocial?ut
328 5Kz2uOtccXQ
a s hisharora
m_medium = studifie
s ocial
copy_link d
286. The total number of sweet Y sold by shopkeeper B is what % of the
total number of sweet sold by shopkeeper E?
दकु ानदाय B द्वाया फे ची गई शभठाई Y की कु र सॊख्मा, दक
ु ानदाय E द्वाया फे ची गई शभठाई की
कु र सॊख्मा का ककतना प्रनतित है?
(A) 16.10% (B) 81.63% (C) 40.60%
(D) 61.70% (E) None of these

287. If the price of sweet X by shopkeeper C is Rs. 330 per box and price
of sweet Y by shopkeeper B is Rs. 240 per box then find the difference
between the profits of these two shopkeepers.
मदद दुक ानदाय C द्वाया शभठाई X की कीभत रु. 330 प्रनत फॉक्स औय दुक ानदाय B
द्वाया शभठाई Y की कीभत रु 240 प्रनत डडब्फा है तो इन दोनों दुक ानदायों के राब के
फीच का अॊतय ऻात कीजजए।
(A) 204340 (B) 200340 (C) 200430
(D) 204650 (E) None of these

288. Find the average number of sweet Y sold by shopkeeper B, C and D


together.
दुक ानदाय B, C औय D द्वाया शभराकय फे ची गई शभठाई Y की औसत सॊख्मा ऻात कीजजए।
(A) 201.33 (B) 295.45 (C) 279.33
(D) 259.54 (E) None of these

289. The total number of sweet boxes sold by shopkeeper B is what %


more or less than number of sweet boxes of Y sold by shopkeeper A and
E together?
दकु ानदाय B द्वाया फे चे गए शभठाई के डब्फे की कु र सॊख्मा, Y शभठाई के दक
ु ानदाय A औय E
द्वाया शभराकय फे चे गए डब्फो की सॊख्मा से ककतने प्रनतित अचधक मा कभ है?
(A) 67.67% (B) 76.66% (C) 74.74%
(D) 76.95% (E) None of these

290. What is the ratio of number of sweet X sold by shopkeeper A and B


together to the number of sweet Y sold by shopkeeper C and E together?
दुक ानदाय A औय B द्वाया शभराकय फे ची गई शभठाई X की सॊख्मा का दुक ानदाय C औय E
द्वाया शभराकय फे ची गई शभठाई Y की सॊख्मा से अनुऩात ककतना है?
(A) 954 : 651 (B) 927 : 671% (C) 927 : 672
(D) 498 : 356 (E) None of these

(291 – 295) Following line graph shows the number of students belonging
to four houses viz. Blue, Pink, Red and Orange house in four consecutive
years.
ननम्नशरणखत ये खा ग्राप रगाताय चाय वर्षों भें चाय सदनों अथाषत नीरा, गर ु ाफी, रार औय
नायॊ गी सदनों से सॊफॊचधत एक स्कू र के छात्रों की सॊख्मा को दिाषता है।
https : https : https : https :
//youtube.co //www.face //instagram.c
m/channel/U book.com/a om/aashishar //t.me/
CYa4_JrOrf8R orasocial?ut
329 5Kz2uOtccXQ
a s hisharora
m_medium = studifie
s ocial
copy_link d
300
240 260 260 222 242 240
220
200
200
200 180 180
140 148 150 148 140
100

0
2018 2019 2020 2021
Blue Pink Red Orange

291. What is the average number of students of Blue house in four years?
चाय वर्षों भें ब्रू हाउस के छात्रों की औसत सॊख्मा क्मा है?
(A) 150 (B) 205 (C) 175
(D) 197 (E) None of these

292. What is the ratio of number of students in Pink house in 2019 to that
of Orange house in 2021?
2019 भें गुराफी हाउस भें छात्रों की सॊख्मा का 2021 भें नायॊ गी हाउस के छात्रों से
अनुऩात ककतना है?
(A) 3 : 7 (B) 4 : 3 (C) 9 : 7
(D) 9 : 5 (E) None of these

293. How much % less are total students in 2018 in the school when
compared to that in 2021?
स्कू र भें 2018 भें कु र छात्रों की सॊख्मा 2021 की तुरना भें ककतने % कभ है?
(A) 3.7% (B) 2.9% (C) 3.9%
(D) 2.7% (E) None of these

294. In which year the increase in the total number of students was the
highest when compared to the previous year?
वऩछरे वर्षष की तुरना भें ककस वर्षष छात्रों की कु र सॊख्मा भें ववर ि सफसे अचधक थी?
(A) 2019 (B) 2020 (C) 2021
(D) 2022 (E) None of these

295. What is the total number of students of Blue and Orange house in all
the years together
सबी वर्षों भें शभराकय ब्रू औय ऑयें ज हाउस के छात्रों की कु र सॊख्मा ककतनी है?
(A) 1268 (B) 1250 (C) 1800
(D) 1670 (E) None of these
https : https : https : https :
//youtube.co //www.face //instagram.c
m/channel/U book.com/a om/aashishar //t.me/
CYa4_JrOrf8R orasocial?ut
330 5Kz2uOtccXQ
a s hisharora
m_medium = studifie
s ocial
copy_link d
(296 – 300) The following pie chart represents the number of students who
voted for a particular actress as % distribution of number of students
surveyed.
ननम्नशरणखत ऩाई चाटष सवे ऺण ककए गए छात्रों की सॊख्मा के % ववतयण के रूऩ भें ककसी वविे र्ष
अशबने त्री के शरए भतदान कयने वारे छात्रों की सॊख्मा का प्रनतननचधत्व कयता है।

296. The total number of students who voted for Alia is what % of the total
number of students who voted for Katrina and Deepika together?
आशरमा को वोट दे ने वारे छात्रों की कु र सॊख्मा कै टयीना औय दीवऩका को शभराकय वोट
कयने वारे छात्रों की कु र सॊख्मा का ककतना प्रनतित है?
(A) 13.99% (B) 11.40% (C) 13.85%
(D) 14.92% (E) None of these

297. What is the ratio of number of students who voted for Katrina and
Priyanka together to the number of students who voted for Alia and
Shradha together?
कै टयीना औय वप्रमॊक ा को शभराकय वोट कयने वारे छात्रों की सॊख्मा का आशरमा औय श्िा को
शभराकय वोट कयने वारे छात्रों की सॊख्मा से अनुऩात ककतना है?
(A) 12 : 38 (B) 20 : 51 (C) 51 : 20
(D) 20 : 37 (E) None of these

298. If 20% of the students who voted for Katrina changed their mood and
now they want to vote for Priyanka then the remaining votes for Katrina
is:
मदद कटयीना को वोट दे ने वारे 20% छात्रों ने अऩना भूड फदर शरमा औय अफ वे वप्रमॊका को
वोट दे ना चाहते हैं तो कै टयीना के शरए िे र्ष वोट है :
(A) 456 (B) 436 (C) 440
(D) 415 (E) None of these

https : https : https : https :


//youtube.co //www.face //instagram.c
m/channel/U book.com/a om/aashishar //t.me/
CYa4_JrOrf8R orasocial?ut
331 5Kz2uOtccXQ
a s hisharora
m_medium = studifie
s ocial
copy_link d
299. What is the difference between average number of students who
voted for Priyanka and Shradha together and who voted for Katrina and
Alia together?
वप्रमॊक ा औय श्िा को शभराकय वोट कयने वारे छात्रों औय कै टयीना औय आशरमा को शभराकय
वोट कयने वारे छात्रों की औसत सॊख्मा के फीच का अॊतय ककतना है?
(A) 188.3 (B) 187.5 (C) 188.5
(D) 192.5 (E) None of these

300. The number of students who voted for Katrina is what % more or less
than number of students who voted for Shraddha?
कै टयीना को वोट दे ने वारे छात्रों की सॊख्मा श्िा को वोट दे ने वारे छात्रों की सॊख्मा से
ककतने प्रनतित अचधक मा कभ है?
(A) 254% (B) 265% (C) 72%
(D) 280% (E) None of these

https : https : https : https :


//youtube.co //www.face //instagram.c
m/channel/U book.com/a om/aashishar //t.me/
CYa4_JrOrf8R orasocial?ut
332 5Kz2uOtccXQ
a s hisharora
m_medium = studifie
s ocial
copy_link d
(1 – 5)
Company  Sony LG MI Samsung Panasonic Total
Semi Smart 54 49 60 30 75 268
Smart 30 56 70 45 45 246
Total 84 105 130 75 120 514

1) Ans. (D)
Average of Smart TV purchased from Samsung & Panasonic
= (45 + 45)/2 = 45

2) Ans. (C)
Semi Smart TV purchased from MI is more than Smart TV purchased from
Sony by = (60 − 30) × 100/30 = 100%

3) Ans. (D)
Semi Smart TV purchased from LG : Smart TV purchased from MI
= 49 : 70 = 7 : 10

4) Ans. (B)
Semi Smart TV purchased from Sony, Samsung & Panasonic
= 54 + 30 + 75 = 159
% of Semi Smart TV purchased from Sony, Samsung & Panasonic with
total TV purchased from LG = 159 × 100/105 = 151%

5) Ans. (A)
Total price paid for Sony Smart TV = 21500 × 30 = 645000 Rs

(6 – 10)
Seller  A B C D E Total
Laser 425 750 1275 660 625 3735
Inkjet 375 150 525 540 475 2065
Total 800 900 180 1200 1100 5800

6) Ans. (B)
Laser printer sold by Seller A & E = 425 + 625 = 1050
https :
Laser printer sold by Seller
https :
A & E are//www.face
https :
more than Total
https :
printer sold by
//youtube.co //instagram.c
m/channel/U book.com/a om/aashishar //t.me/
CYa4_JrOrf8R orasocial?ut
333 5Kz2uOtccXQ
a s hisharora
m_medium = studifie
s ocial
copy_link d
Seller B = (1050 − 900) × 100/900 = 16(2/3) %

7) Ans. (D)
Average of Inkjet printer sold by Seller C & E = (525 + 475)/2 = 500

8) Ans. (C)
By Seller B, Money received from selling 30% laser printer
= 1100 × 0.3 × 750 = 247500 Rs.
Money received from selling 44% Inkjet Printer
= 950 × 0.44 × 150 = 62700 Rs.
Difference = 247500 – 62700 = 184800 Rs

9) Ans. (D)
Inkjet Printer sold by Seller F = 375 × 6/15 = 150
Total Printer sold by Seller F = 0.55 × 660 = 363
Laser Printer sold by Seller F = 363 – 150 = 213

10) Ans. (C)


22.22% of Inkjet Printer sold by Seller D = 540 × 2/9 = 120

(11 – 15)
Hotel  Sunshine Diamond Lotus Star Sunflower Total
Chef 2600 2275 1200 1125 2700 9900
Waiter 2400 1225 800 375 3300 8100
Total 5000 3500 2000 1500 6000 18000

11) Ans. (D)


Waiter in Hotel Lotus are less than Chef in Hotel Sunshine
= (2600 – 800) × 100/2600 = 69.2%

12) Ans. (A)


Average number of Waiter in Hotel Diamond & Hotel Star
= (1225 + 375) /2 = 800

13) Ans. (C)


Waiter in Hotel Lotus & Sunflower – Chef in Hotel Diamond
= (800 + 3300) – 2275 =https
1825: https : https : https :
//youtube.co //www.face //instagram.c
m/channel/U book.com/a om/aashishar //t.me/
CYa4_JrOrf8R orasocial?ut
334 5Kz2uOtccXQ
a s hisharora
m_medium = studifie
s ocial
copy_link d
14) Ans. (B)
Total Employees in Hotel Moon = 1200 × 0.4 + 800 × 0.2 = 480 + 160 = 640

15) Ans. (C)


Waiter in Hotel Sunshine : Chef in Hotel Sunflower = 2400 : 2700 = 8 : 9

(16 – 20)
Society  A B C D E Total
Petrol vehicle 1890 2430 2688 3080 1600 11688
Diesel vehicle 3150 1890 3072 2200 2000 12312
Total 5040 4320 5760 5280 3600 24000

16) Ans. (C)


Families who do not own Petrol vehicle in Society D : Families who do not
own Diesel vehicle in Society E = 2200 : 1600 = 11 : 8

17) Ans. (C)


% of Families who do not own Petrol Vehicle in Society B with Total
families in Society E = 1890 × 100/3600 = 52.5%

18) Ans. (D)


Hatchback vehicle in Society B
= 2430 × 2/3 + 1890 × 0.80 = 1620 + 1512 = 3132

19) Ans. (D)


Society A has maximum Diesel vehicle than
Families in Society A are less than Families in Society D
= 5760 – 5040 = 720
% = (5760 – 5040) × 100/5760 = 12.5%

20) Ans. (A)


Average Families who have Diesel vehicle excluding Society C
= (12312 – 3072) /4 = 2310

https : https : https : https :


//youtube.co //www.face //instagram.c
m/channel/U book.com/a om/aashishar //t.me/
CYa4_JrOrf8R orasocial?ut
335 5Kz2uOtccXQ
a s hisharora
m_medium = studifie
s ocial
copy_link d
(21 – 25)
Branch  Sikar Bikaner Jaipur Ajmer Kota Total
< 8 Crore 90 69 144 184 92 579
> 8 Crore 48 115 132 207 69 571
Total 138 184 276 391 161 1150

21) Ans. (D)


Average number of branches in Bikaner & Ajmer with more than 8 crore
balance sheet = (115 + 207)/2 = 161

22) Ans. (C)


Difference between Sikar & Jaipur branches with less than 8 crore balance
sheet = 144 – 90 = 54

23) Ans. (A)


Branches with less than 8 crore balance sheet in Kota : Branches with
more than 8 core balance sheet in Sikar = 92 : 48 = 23 : 12

24) Ans. (C)


The number of branches in Ajmer with balance sheet less than 8 crore is
more than the number of branches in Jaipur having balance sheet more
than 8 crore = (184 – 132) × 100/132 = 39.39%

25) Ans. (B)


Average of Old employees in Bikaner, Jaipur & Sikar Branches
= (184 × 0.75 + 276 × 0.25 + 138 × 0.5)/3 = (138 + 69 + 69)/3 = 276/3 = 92

(26 – 30)
Center  A B C D E F Total
Boys 124 136 140 132 144 128 804
Girls 144 52 108 76 84 160 624
Total 268 188 248 208 228 288 1428

26) Ans. (B)


Girls (Center A + Center B) : Total student (Center F)
= (144 + 52) : 288 = 196https :
: 288 = 49 : 72https : https : https :
//youtube.co //www.face //instagram.c
m/channel/U book.com/a om/aashishar //t.me/
CYa4_JrOrf8R orasocial?ut
336 5Kz2uOtccXQ
a s hisharora
m_medium = studifie
s ocial
copy_link d
27) Ans. (C)
Income of Center E = 1800 × 144 + 1500 × 84
= 259200 + 126000 = 385200 Rs.

28) Ans. (D)


Girls (Center D + Center E + Center F) = 76 + 84 + 160 = 320
% of Students in Center C with Girls in Center D, E & F
= 248 × 100/320 = 77.5%

29) Ans. (A)


Boys in Center A are more than that of Girls in Center D by
= (124 – 76) × 100/76 = 63.15%

30) Ans. (B)


Boys (Center B + Center C) = 136 + 140 = 276
Girls (Center E + Center F) = 84 + 160 = 244
Difference = 276 – 244 = 32

(31 – 35)
Month  January February March April May Total
Sleeveless 240 450 560 700 800 2750
Half Sleeve 1200 810 640 600 1120 4370
Full Sleeve 960 540 400 700 1280 3880
Total 2400 1800 1600 2000 3200 11000

31) Ans. (D)


Full Sleeve shirt sold in January & February = 960 + 540 = 1500
Half Sleeve shirt sold in January & March = 1200 + 640 = 1840
Full sleeve shirt sold January & February is less than Half Sleeve shirt
sold in January & March by = (1840 – 1500) × 100/1840 = 18.47%

32) Ans. (C)


Full Sleeve shirt sold in January, February & April = 960 + 540 + 700 = 2200
Half Sleeve shirt sold in March & May = 640 + 1120 = 1760
Difference = 2200 – 1760 = 440

https : https : https : https :


//youtube.co //www.face //instagram.c
m/channel/U book.com/a om/aashishar //t.me/
CYa4_JrOrf8R orasocial?ut
337 5Kz2uOtccXQ
a s hisharora
m_medium = studifie
s ocial
copy_link d
33) Ans. (D)
Average of Sleeveless shirt sold in January, April & May
= (240 + 700 + 800) /3 = 580
Average of Half Sleeve shirt sold in February & March
= (810 + 640) /2 = 1450/2 = 725
Sum = 580 + 725 = 1305

34) Ans. (A)


In June, Half Sleeve shirt + Full Sleeve shirt
= 1.4 × 600 + 1.55 × 540 = 840 + 837 = 1677

35) Ans. (D)


Total Shirts sold in, January : May = 2400 : 3200 = 3 : 4

(36 – 40)
36) Ans. (C)
Total Person, In Jaipur : Surat
= 400/(0.3 – 0.2) : (2 × 2000)/(0.6 + 0.2) = 4000 : 5000 = 4 : 5

37) Ans. (D)


Person who play Football in Mumbai
= 0.45 × 300/[(0.45 + 0.3) – (0.25 + 0.45)] = 0.45 × 300/0.05 = 2700

38) Ans. (A)


Total Tennis player in Ranchi & Kolkata = 3x × 0.3 + 2x × 0.4 = 1700
0.9x + 0.8x = 1700
1.7x = 1700
x = 1000, Total person in Ranchi = 3 × 1000 = 3000

39) Ans. (B)


Total Person in Kolkata = 400/(0.4 – 0.2) = 2000
Total Person in Mumbai = 900/(0.45 – 0.3) = 6000
% of person in Kolkata with person in Mumbai = 2000 × 100/6000 = 33.33%

40) Ans. (C)


Total Person in Surat = 2000/(0.6 – 0.2) = 5000
Total Person in Jaipur = 5000 – 1000 = 4000
https : https : https : https :
//youtube.co //www.face //instagram.c
m/channel/U book.com/a om/aashishar //t.me/
CYa4_JrOrf8R orasocial?ut
338 5Kz2uOtccXQ
a s hisharora
m_medium = studifie
s ocial
copy_link d
(41 – 45)
Product  L M N O P Total
Total 560 840 1400 1960 840 5600
Sold 168 126 560 1078 84 2016
Unsold 392 714 840 882 756 3584

41) Ans. (B)


Manufactured product P : Sold product M = 840 : 126 = 20 : 3

42) Ans. (A)


Sold product Q = 840 × 5/3 – 756 × 15/12 = 1400 – 945 = 455

43) Ans. (D)


Unsold product (M + N) – Sold product (L + O)
= (714 + 840) – (168 + 1078) = 1554 – 1246 = 308

44) Ans. (D)


% of sold product P with sold product N = 84 × 100/560 = 15%

45) Ans. (B)


Average unsold product of All 5 type together = 3584/5 = 716.8

(46 – 50)
Year  2017 2018 2019 2020 2021 Total
Expenditure 70 50 35 45 60 260
Profit 40 60 25 40 45 210
Revenue 110 110 60 85 105 470

46) Ans. (B)


Increase in expenditure is maximum with respect to previous year in 2021
= (60 – 45) × 100/60 = 33.33%

47) Ans. (A)


Revenue in 2019 : Revenue in 2021 = 60 : 105 = 4 : 7

https : https : https : https :


//youtube.co //www.face //instagram.c
m/channel/U book.com/a om/aashishar //t.me/
CYa4_JrOrf8R orasocial?ut
339 5Kz2uOtccXQ
a s hisharora
m_medium = studifie
s ocial
copy_link d
48) Ans. (B)
% of Revenue in 2018, with Sum of profit & Expenditure in 2020
= 110 × 100/85 = 129.41%

49) Ans. (A)


Average revenue is All 5 year = 470/5 = 94

50) Ans. (A)


P% is maximum in the year = 2018
60 × 100/50 = 120%

(51 – 55)

Player  Rohit Dhawan Virat KL Rahul Dhoni Total


Run scored 36 54 90 120 60 360
By 6’s 2 × 6 = 12 4 × 6 = 24 3 × 6 = 18 10 × 6 = 60 5 × 6 = 30 144
Other 24 30 72 60 30 216

51) Ans. (B)


Number of 4 hit by KL Rahul = 60/4 = 15

52) Ans. (B)


Average run scored by hitting 6 only = 144/5 = 28.8

53) Ans. (D)


Run scored by Virat are more than that of Dhoni by
= (90 – 30) × 100/60 = 50%

54) Ans. (A)


Run scored by Rohit by hitting 6 are less than total run scored by Dhawan by = (54 – 12) ×
100/54 = 77.77%

55) Ans. (A)


Run scored by without boundary by
Dhawan = 54 – 24 – 2 × 4 = 54 – 32 = 22
Dhoni = 60 – 30 – 5 × 4 = 60 – 50 = 10
Difference = 22 – 10 = 12
https : https : https : https :
//youtube.co //www.face //instagram.c
m/channel/U book.com/a om/aashishar //t.me/
CYa4_JrOrf8R orasocial?ut
340 5Kz2uOtccXQ
a s hisharora
m_medium = studifie
s ocial
copy_link d
(56 – 60)
56) Ans. (D)
Student whose age is less than or equal to 16
= 8 + 15 + 10 + 8 + 20 + 18 + 40 = 119
Total students in 12th = 40 + 25 = 65
% of Student less than equal to 16 year age with total students in class
12th = 119 × 100/65 = 183.07%

57) Ans. (B)


Students in class 11th = 8 + 18 + 24 = 50, Student in class 9th = 8 + 15 = 23
Students in 11th class are more than by Students in 9th class by
= (50 – 23) × 100/23 = 117.39%

58) Ans. (A)


Average age of Students between (≥ 14 & <16) age group = 1716/78 = 22

59) Ans. (D)


Students in age group (<10) : Students in 10th class = 8 : 30 = 4 : 15

60) Ans. (B)


Total students in all 4 classes = 23 + 30 + 50 + 65 = 168

(61 – 65)
Factory  A B C D E Total
Male 64 24 24 112 20 244
Female 40 32 16 28 40 156
Total 104 56 40 140 60 400

61) Ans. (A)


In Factory B & C, Male : Female = (24 + 24) : (32 + 16) = 48 : 48 = 1 : 1

62) Ans. (B)


Average female workers in all 5 factories = 156/5 = 31.2

63) Ans. (B)


In Factory D (Male – Female) : In Factory A (Male – Female)
= (112 – 28) : (64 – 40) https
= 84: : 24 = 7 : 2https : https : https :
//youtube.co //www.face //instagram.c
m/channel/U book.com/a om/aashishar //t.me/
CYa4_JrOrf8R orasocial?ut
341 5Kz2uOtccXQ
a s hisharora
m_medium = studifie
s ocial
copy_link d
64) Ans. (A)
Male workers in Factory A & E = 64 + 20 = 84, Female workers in Factory B
& C = 32 + 16 = 48
Male workers in Factory A & E are more than Female workers in Factory B
& C by = (84 – 48) × 100/48 = 75%

65) Ans. (D)


% of Female workers in Factory E with Total workers in Factory B
= 40 × 100/56 = 71.42%

(66 – 70)
Farmers  A B C D E Total
January 1 6 00 1 2 00 300 1 8 00 3 6 00 8 5 00
February 2 4 00 4 8 00 1 2 00 3 0 00 2 4 00 1 3 800
March 3 6 00 4 2 00 2 7 00 4 2 00 5 4 00 2 0 100
Total 7 6 00 1 0 200 4 2 00 9 0 00 1 1 400 4 2 400

66) Ans. (D)


Average income of all farmers in February = 13800/5 = 2760

67) Ans. (C)


Total income of Farmer B & C = 10200 + 4200 = 14400
Total income of Farmer D & E = 9000 + 11400 = 20400
% of income of B & C with income of D & E = 14400 × 100/20400 = 70.58%

68) Ans. (C)


Total income of Farmers A, B & C in
January = 1600 + 1200 + 300 = 3100, March = 3600 + 4200 + 2700 = 10500
Income of 3 farmers in January is less than that in March by
= (10500 – 3100) × 100/10500 = 70.47%

69) Ans. (D)


Income of E in February decreased by income in January
= (3600 – 2400) × 100/3600 = 33.33%

70) Ans. (A)


https :
For Farmer A, D & E, Income https
in March :
: Income in January
https : https :
//youtube.co //www.face //instagram.c
m/channel/U book.com/a om/aashishar //t.me/
CYa4_JrOrf8R orasocial?ut
342 5Kz2uOtccXQ
a s hisharora
m_medium = studifie
s ocial
copy_link d
= (3600 + 4200 + 5400) : (1600 + 1800 + 3600) = 13200 : 7000 = 66 : 35

(71 – 75)
Coaching  A B C D E Total
SSC 900 1200 1125 1750 1092 6067
Bank 750 1050 675 1250 1008 4733
Total 1650 2250 1800 3000 2100 10800

71) Ans. (B)


Banking Aspirants in Coaching (A + B + C) : SSC Aspirants in All
Coaching = (750 + 1050 + 675) : 6067 = 2475 : 6067

72) Ans. (C)


Total Aspirants in Coaching F = 900 × 17/12+ 675 × 0.8 = 1275 + 540 = 1815

73) Ans. (A)


% of SSC Aspirants in Coaching E with Banking Aspirants in Coaching D
= 1092 × 100/1250 = 87.36%

74) Ans. (D)


New number of Students in Coaching B
= 1050 × 17/15 + 1200 × 0.875 = 1190 + 1050 = 2240
Now, % Students in Coaching B with Students in Coaching D
= 2240 × 100/3000 = 74(2/3) %

75) Ans. (A)


SSC Aspirants in Coaching C, D & E = 1125 + 1750 + 1092 = 3967
Banking Aspirants in Coaching A, B & C = 750 + 1050 + 675 = 2475
Difference = 3967 – 2475 = 1492

(76 – 80)
Family  A B C D E Total
Sum of Age of
35 46 29 61 40 211
Children

https : https : https : https :


//youtube.co //www.face //instagram.c
m/channel/U book.com/a om/aashishar //t.me/
CYa4_JrOrf8R orasocial?ut
343 5Kz2uOtccXQ
a s hisharora
m_medium = studifie
s ocial
copy_link d
76) Ans. (A)
Average Age of Children in Family A = 35/2 = 17.5 year
Average Age of Children in Family B = 46/3 = 15.33 year
Average age of Children in Family A is more than that of Children in
Family B by = (17.5 – 15.33) × 100/15.33 = 14.13% = 14%

77) Ans. (C)


Average age of All the Children = 211/10 = 21.1 year

78) Ans. (D)


In the family B, Age of Girl = 11x year, Age of Boys = 12x year
11x + 12x = 46
23x = 46
Difference between Age of boys & girl = x = 2 year

79) Ans. (B)


Age of Children in Family (A + B + C) : Age of Father in Family (B + C + D +
E) = (35 + 46 + 29) : (56 + 47 + 41 + 45) = 110 : 189

80) Ans. (B)


Average age of Mothers in All families
= (48 + 54 + 38 + 39 + 35)/5 = 214/5 = 42.8 years

(81 – 85)
Company  P Q R S T Total
I – pad 126 50 63 78 49 366
I – phone 90 35 35 54 28 242
Total 216 85 98 132 77 608

81) Ans. (A)


Average of I – pad sold = 366/5 = 73.2
% of I – phone sold by Shop P with Average of I – pad sold by all shops
= 90 × 100/73.2 = 122.9%

82) Ans. (D)


I – pad sold by Shop P, Q & R = 126 + 50 + 63 = 239
I – phone sold by Shop
https
R,: S & T = 35https
+ 54: + 28 = 117
https : https :
//youtube.co //www.face //instagram.c
m/channel/U book.com/a om/aashishar //t.me/
CYa4_JrOrf8R orasocial?ut
344 5Kz2uOtccXQ
a s hisharora
m_medium = studifie
s ocial
copy_link d
I – pad sold are more than I – phone sold by
= (239 – 117) × 100/117 = 104.27%

83) Ans. (D)


I – pad & I – phone sold by Shop T : I – pad & I – phone sold by Shop R
= 77 : 98 = 11 : 14

84) Ans. (A)


Average of I – pad sold by Shop P, Q & T = (126 + 50 + 49)/3 = 225/3 = 75
Average of I – phone sold by all companies = 242/5 = 48.4
Difference = 75 – 48.4 = 26.6

85) Ans. (D)


I – phone sold by Shop T is less than I – pad sold by Shop Q by
= (50 – 28) × 100/50 = 44%

(86 – 90)
86) Ans. (A)
Bikes manufactured by T & U = 1.5 × 17400 = 26100
Bikes manufactured, by T = 26100 × 15/6 = 13500, by U = 26100 – 13500 =
12600
Unsold Bikes of T – Unsold Bikes of U = 13500 × 0.6 – 12600 × 0.3 = 8100 –
3780 = 4320

https : https : https : https :


//youtube.co //www.face //instagram.c
m/channel/U book.com/a om/aashishar //t.me/
CYa4_JrOrf8R orasocial?ut
345 5Kz2uOtccXQ
a s hisharora
m_medium = studifie
s ocial
copy_link d
87) Ans. (A)
Bikes manufactured by, R = 13500 × 58/45= 17400. T = 13500 × 37/45 =
11100
Average of Bikes sold by Q, R & T
= (13500 × 0.7 + 17400 × 0.85 + 11100 × 0.4)/3
= (9450 + 14790 + 4440)/3 = 28680/3 = 9560

88) Ans. (D)


Bikes sold by P & Q = 17400 × 0.65 + 13500 × 0.6 = 11310 + 8100 = 19410
Bikes sold by U & V = 16500 × 0.7 + 19200 × 0.8 = 11550 + 15360 = 26910
Bikes sold by P & Q are less than Bikes sold by U & V by
= (36910 – 19410) × 100/26910 = 27.87%

89) Ans. (B)


Bikes sold by P : Bikes sold by Q
= (17400 × 0.65) : (13500 × 0.7) = (29 × 13) : (45 × 7) = 377 : 315 = 1131 : 945

90) Ans. (B)


Unsold Bikes by R & T
= 1.3 × 18400 × 0.15 + 1.45 × 19200 × 0.6 = 3588 + 16704 = 20292

(91 – 95)
Employee  P Q R S T Total
Income 50000 38000 32000 44000 36000 200000
Expenditure 34000 28000 26000 36000 21000 145000
Saving 16000 10000 6000 8000 15000 55000

91) Ans. (C)


Employee, who save maximum amount = P, 16000 Rs.

92) Ans. (B)


Average saving of S & T – Average Saving of P, Q & R
= (80000 + 15000)/2 – (16000 + 10000 + 6000)/3
= 23000/2 – 32000/3 = 11500 – 10666.66 = 833.33

93) Ans. (A)


Expenditure (R + T) : Saving
https : (Q + S) =https
(26: + 21) : (10 + :8) = 47 : 18https
https :
= 2.61
//youtube.co //www.face //instagram.c
m/channel/U book.com/a om/aashishar //t.me/
CYa4_JrOrf8R orasocial?ut
346 5Kz2uOtccXQ
a s hisharora
m_medium = studifie
s ocial
copy_link d
94) Ans. (C)
Expenditure of P & R = 34000 + 26000 = 60000 Rs.
Saving of Q & T = 10000 + 15000 = 25000 Rs
Expenditure of P & R is more than Saving of Q & T by
= (60 – 25) × 100/25 = 140%

95) Ans. (A)


Monthly income of A = 1.3 × 6000 + (34000 + 12500) = 7800 + 46500 = 54300
% of Monthly income of R with Monthly income of A
= 32000 × 100/54300 = 58.9%

(96 – 100)
Company  Infosys TCS IBM Wipro Genpact Total
Male 90 48 120 170 110 538
Female 70 42 90 120 80 402
Vacant 40 10 90 210 10 360
Total 200 100 300 500 200 1300

96) Ans. (D)


% of vacant seats in Genpact with vacant seats in Infosys
= 10 × 100/40 = 25%

97) Ans. (D)


Average number of males in all companies = 538/5 = 107.6

98) Ans. (A)


Females in TCS, Genpact & Infosys = 42 + 80 + 70 = 192
Males in IBM & Wipro = 120 + 170 = 290
Females in TCS, Genpact & Infosys are less than Males in IBM & Wipro by
= (290 – 192) × 100/290 = 33.79%

99) Ans. (B)


Total employees in (IBM + Genpact) : Female employee in All companies
= (210 + 190) : 402 = 400 : 402 = 200 : 201

100) Ans. (B)


Total posts in Sandiskhttps
= 1.4: × 500 = 700
https : https : https :
//youtube.co //www.face //instagram.c
m/channel/U book.com/a om/aashishar //t.me/
CYa4_JrOrf8R orasocial?ut
347 5Kz2uOtccXQ
a s hisharora
m_medium = studifie
s ocial
copy_link d
Female employees in Sandisk = 700/(1 + 4/3) = 700 × 3/7 = 300

(101 – 105)
Story Books Sci – Fi Books
Shopkeeper
Purchased Sold Unsold Purchased Sold Unsold
P 8400 7140 1260 7000 5460 1540
Q 9600 7104 2496 8500 6035 2465
R 9400 8366 1034 8000 5200 2800
S 6400 5888 512 7200 6048 1152
T 8000 3760 4240 8200 7052 1148
Total 41800 32258 9542 38900 29795 9105

101) Ans. (C)


Story books sold by P, Q & R : Sci – fi books sold by S & T
= (7140 + 7104 + 8366) : (6048 + 7052) = 22610 : 13100 = 2261 : 1310

102) Ans. (B)


Average Story book sold – Average Sci – Fi books sold
= 32258/5 – 29795/5 = 6451.6 – 5959 = 492.6

103) Ans. (C)


Average of Story books & Sci – Fi books purchased by Shopkeeper A
= 1/2 × (7140 × 19/15 + 6048 × 1.4375) = (9044 + 8694)/2 = 8869

104) Ans. (C)


By Shopkeeper S, Story books sold = 5888 × 0.9375 = 5520
Sci – Fi books sold = 6048 × 19/12 = 9576
Difference = 9576 – 5520 = 4056

105) Ans. (A)


Books sold by P = 7140 + 5460 = 12600
Books sold by T = 3760 + 7052 = 10812
Books sold by P are more than that sold by T
= (12600 – 10812) × 100/10812 = 16.53%

https : https : https : https :


//youtube.co //www.face //instagram.c
m/channel/U book.com/a om/aashishar //t.me/
CYa4_JrOrf8R orasocial?ut
348 5Kz2uOtccXQ
a s hisharora
m_medium = studifie
s ocial
copy_link d
(106 – 110)
Student  Shashi Krati Meenal Sameeksha Mahima Total
Math 75 84 54 75 90 378
English 60 48 48 50 81 287
Science 75 48 48 100 72 343
Total 210 180 150 225 243 1008

106) Ans. (C)


Average marks of Shashi, Minal & Krati in Science : Average of Marks in
English = 7(5 + 48 + 48)/3 : 287/5 = 57 : 57.4 = 285 : 287

107) Ans. (D)


Passing Marks in Each subject = 100 × 0.6 = 60
Students who failed in at least 1 subject = 3 (Krati, Meenal & Sameeksha)
% of these students = 3 × 100/5 = 60%

108) Ans. (B)


Average marks scored in Math – Average marks scored in Science
= (378 – 343)/2 = 35/5 = 7

109) Ans. (B)


Marks of Sonia in, Math = 84 × 7/17 = 49, English = 49 + 20 = 69
Science = (49 + 69)/2 = 59
Total = 49 + 69 + 59 = 177, % of Marks = 177 × 100/300 = 59

110) Ans. (B)


Marks score by Mahima in English are more than Marks scored by
Sameeksha in Math by = (81 – 75) × 100/75 = 8%

(111 – 115)
Year  2010 2011 2012 2013 2014 Total
Swimming 250 250 485 406 450 1841
Basketball 100 300 685 580 258 1923
Football 350 150 390 464 472 1826
Total 700 700 1560 1450 1180 5590
https : https : https : https :
//youtube.co //www.face //instagram.c
m/channel/U book.com/a om/aashishar //t.me/
CYa4_JrOrf8R orasocial?ut
349 5Kz2uOtccXQ
a s hisharora
m_medium = studifie
s ocial
copy_link d
111) Ans. (C)
Person who like Football : Person who like Swimming = 1826 : 1841

112) Ans. (B)


Basketball player in 2012 are more than Swimming player in 2013 by
= (685 – 406) × 100/406 = 68.7%

113) Ans. (B)


New number of Player in 2011
= 300 × 8/15 + 250 × 1.2 + 150 = 160 + 300 + 150 = 610

114) Ans. (C)


Uninjured players in 2012
= 485 × 0.8 + 685 × 125/137 + 390 × 6/13 = 388 + 625 + 180 = 1193
% of Uninjured players = 1193 × 100/1560 = 76.4%

115) Ans. (A)


2014 (Basketball + Football) – 2010 (Swimming + Basketball)
= (258 + 472) – (250 + 100) = 730 – 350 = 380

(116 – 120)
Boat  A B C D E
Distance (km) 720 480 240 600 360
Boat Speed (km/h) 16 22 12 13 16
Stream Speed (km/h) 8 10 6 9 3

116) Ans. (C)


Average time taken by all Boats in Still water
= 1/5 × (720/16 + 480/22 + 240/12 + 600/13 + 360/16) = 155.47/5 = 30 hr

117) Ans. (D)


Time taken by, Boat A in Still Water : Boat D Downstream
= 720/16 : 600/22 = 45 ; 300/11 = 33 : 20

118) Ans. (A)


Speed of Boat C in Still Water = 12 × 17/12 = 17 km/h
Upstream speed = 6 ×https
5/6: = 5 km/h https : https : https :
//youtube.co //www.face //instagram.c
m/channel/U book.com/a om/aashishar //t.me/
CYa4_JrOrf8R orasocial?ut
350 5Kz2uOtccXQ
a s hisharora
m_medium = studifie
s ocial
copy_link d
Speed of Stream = 17 – 5 = 12 km/h,
Downstream Speed = 17 + 12 = 29 km/h
Time taken by Boat C to cover 3496 km in Downstream = 3596/29 = 124 hr

119) Ans. (B)


Time taken by, Boat C in Still water = 240/12 = 20 hr
Boat A in Downstream = 720/24 = 30 hr
Time taken by Boat C in still water is less than time taken by Boat in
Downstream by = (20 - 20) × 100/30= 33.33%

120) Ans. (B)


Speed of Boat F in Downstream = 1240 × 12/480 = 31 km/h
Speed of Stream = 12x km/h, Speed of Boat F in still water = 19x km/h
19x + 12x = 31
31x = 31
x=1
Upstream speed of Boat F = 19 × 1 – 12 × 1 = 7 km/h
Time taken by Boat F to cover 1652 km in Upstream = 1652/7 = 236 hr

(121 – 125)
Society  A B C D E Total
2BHK 176 120 200 144 160 800
3BHK 64 108 88 108 32 400
Total 240 228 288 252 192 1200

121) Ans. (D)


3BHK flats sold by C are more than 3BHK flats sold by E by
= (88 – 32) × 100/32 = 175%

122) Ans. (A)


In Society D, 3BHK flats – 2BHK flats
= 108 × 19/12 – 144 × 7/12 = 171 – 84 = 87

123) Ans. (D)


Amount received by Society C
= 200 × 5.8 + 88 × 12.5 = 1160 + 1100 = 2260 Lakhs
https : https : https : https :
//youtube.co //www.face //instagram.c
m/channel/U book.com/a om/aashishar //t.me/
CYa4_JrOrf8R orasocial?ut
351 5Kz2uOtccXQ
a s hisharora
m_medium = studifie
s ocial
copy_link d
124) Ans. (C)
3BHK flats sold by D & E = 108 + 32 = 140
2BHK flats sold by A & C = 176 + 200 = 376
Difference = 376 – 140 = 236

125) Ans. (B)


3BHK flats (A + B + C) : 2BHK flats (C + D + E)
= (64 + 108 + 88) : (200 + 144 + 160) = 260 : 504 = 65 : 126

(126 – 130)

College  IET MNIT MITRC NIET LIET Total


Selected 9200 9600 8100 8400 11000 46300
Unselected 6000 7200 5400 4800 7500 30900
Applied 15200 16800 13500 13200 18500 77200

126) Ans. (A)


Selected students in NIET : Not selected students in MNIT
= 8400 : 7200 = 7 : 6

127) Ans. (C)


Selected students in MITRC are more than Unselected students in LIET by
= (8100 – 7500) × 100/7500 = 8%

128) Ans. (B)


Unselected student in MITRC
= 13500 × 36/15 – 8100 × 2/9 = 23400 – 1800 = 21600

129) Ans. (B)


Students applied in MNIT, IET & NIET – Unselected students in MITRC &
LIET = (16800 + 15200 + 13200) – (5400 + 7500) = 45200 – 12900 = 32300

130) Ans. (A)


Average of Selected students in All colleges = 46300/5 = 9260

https : https : https : https :


//youtube.co //www.face //instagram.c
m/channel/U book.com/a om/aashishar //t.me/
CYa4_JrOrf8R orasocial?ut
352 5Kz2uOtccXQ
a s hisharora
m_medium = studifie
s ocial
copy_link d
(131 – 135)
Company  A B C D E Total
Male 1800 2160 1620 2700 1980 10260
Female 1440 1800 1260 1800 1440 7740
Total 3240 3960 2880 4500 3420 18000

131) Ans. (B)


Female in D : Male in C = 1800 : 1620 = 10 : 9

132) Ans. (C)


Male in A & E = 1800 + 1980 = 3780, Females in B & D = 1800 + 1800 = 3600
Males in A & E are more than Female in B & D by
= (3780 – 3600) × 100/3600 = 5%

133) Ans. (A)


In Company D, Males = 2700 × 19/15 = 3420, Female = 1800 × 7/6 = 2100
Males are more than Females by = (3420 – 2100) × 100/2100 = 62.85%

134) Ans. (C)


Males in B, C & D – Females in A, D & E
= (2160 + 1620 + 2700) – (1440 + 1800 + 1440) = 6480 – 4680 = 1800

135) Ans. (D)


Monthly income of Female = 14.7 × 6/7 × 1/12 = 1.05 Lakhs

(136 – 140)
Year  2011 2012 2013 2014 2015 Total
CP of A 244 289 311 228 278 1350
CP of B 124 163 173 84 94 638
Total 368 452 484 312 372 1988

136) Ans. (B)


CP of B in 2015 is more than CP of B in 2014 by
= (94 – 84) × 100/84 = 11.9%

https : https : https : https :


//youtube.co //www.face //instagram.c
m/channel/U book.com/a om/aashishar //t.me/
CYa4_JrOrf8R orasocial?ut
353 5Kz2uOtccXQ
a s hisharora
m_medium = studifie
s ocial
copy_link d
137) Ans. (A)
In 2011, MP of A = 1.25 × 244 = 305 Rs
Discount % = (305 – 264) × 100/305 = 13.44%

138) Ans. (C)


Average of B in all year = 638/5 = 127.6

139) Ans. (C)


In 2014, SP of product A = 1.12 × 228 = 255.36 Rs

140) Ans. (B)


In 2012, SP of A – SP of B
= 289 × 22/17 – 163 × 1.2 = 374 – 195.6 = 178.4 Rs.

(141 – 145)
State Male Female Total Literate Illiterate
Andhra Pradesh 480000 440000 920000 294400 625600
Bihar 280000 240000 520000 234000 286000
Chandigarh 300000 180000 480000 182400 297600
Delhi 420000 300000 720000 288000 432000
Rajasthan 450000 400000 850000 408000 442000
Total 1930000 1560000 3490000 1406800 2083200

141) Ans. (C)


Males in Rajasthan : Illiterate in Delhi = 450000 : 432000 = 225 : 216

142) Ans. (A)


Average number of Literate people in all the state = 1406800/5 = 281360

143) Ans. (C)


New Literacy Rate in Bihar = 45 × 17/15 = 51%
New Literacy Rate in Rajasthan = 48 × 19/12 = 76%
Illiterate in Bihar = 520000 × 0.49 = 254800
Illiterate in Rajasthan = 850000 × 0.24 = 204000
Difference = 254800 – 204000 = 50800
https : https : https : https :
//youtube.co //www.face //instagram.c
m/channel/U book.com/a om/aashishar //t.me/
CYa4_JrOrf8R orasocial?ut
354 5Kz2uOtccXQ
a s hisharora
m_medium = studifie
s ocial
copy_link d
144) Ans. (B)
Males in Delhi are more than Males in Bihar by = (42 - 28) × 100/28 = 50%

145) Ans. (D)


Total amount spend by Delhi, Chandigarh & Andhra Pradesh government
= 500 × (432000 + 297600 + 625600) = 500 × 1355200 = 677600000

(146 – 150)
Shop  A B C D E Total
Sold Mobile 6480 7560 11160 5760 5040 36000
Sold Laptop 2220 2960 4070 5920 3330 18500
Total 8700 10520 15230 11680 8370 54500

146) Ans. (A)


Mobiles sold by C – Laptops sold by E = 11160 – 3330 = 7830

147) Ans. (D)


Next year by Shop B, Mobile sold – Laptop sold
= 7560 × 17/15 – 2960 × 1.25 = 8568 – 3700 = 4868

148) Ans. (A)


Laptop sold by B : Mobile sold by A = 2960 : 6480 = 37 : 81

149) Ans. (B)


Mobile sold by D are more than Laptop sold by B
= (5760 - 2960) × 100/2960 = 94.5%

150) Ans. (D)


Laptop sold more than mobile by = Shop D

(151 – 155)
Company  A B C D E Total
2019 360 270 800 840 660 2930
2020 630 720 720 1190 420 3680
Total 990 990 1520 2030 1080 6610
https : https : https : https :
//youtube.co //www.face //instagram.c
m/channel/U book.com/a om/aashishar //t.me/
CYa4_JrOrf8R orasocial?ut
355 5Kz2uOtccXQ
a s hisharora
m_medium = studifie
s ocial
copy_link d
151) Ans. (B)
In 2019, Male : Female = 7 : 5, In 2020, Male : Female = 9 : 8
Females in 2019 = 840 × 5/12 = 350, Females in 2020 = 1190 × 8/17 = 560
Difference between females in 2 year = 560 – 350 = 210

152) Ans. (B)


Average Number of employees in year 2020 = 3680/5 = 736

153) Ans. (B)


Total Salary given by Company A in 2 year
= 360 × 3000 + 630 × 3500 = 1080000 + 2205000 = 32,85,000 Rs.

154) Ans. (A)


Employees in Company C in 2020 : Employees in Company B in 2019
= 720 : 270 = 8 : 3

155) Ans. (D)


Employee in A & E in 2019 = 360 + 660 = 1020
Employee in B & D in 2020 = 720 + 1190 = 1910
Employee in Company A & E in 2019 are less than Employee in Company
B & D in 2020 = (1910 - 1020) × 100/1910 = 46.59%

(156 – 160)
156) Ans. (A)
Average investment by Amit in all Crypto-currency
= (364 + 166 + 138 + 212)/4 = 880/4 = 220 Thousand

157) Ans. (D)


Investment of Pankaj in Dogecoin & Etherum : Investment of Amit in
Etherum & Bitcoin = (225 + 105) : (138 + 212) = 330 : 350 = 33 : 35

158) Ans. (C)


Total investment of Pankaj = 225 + 325 + 105 + 285 = 940
% of Investment of Riya in Etherum with Total investment of Pankaj
= (192 × 100)/940 = 20.42%

159) Ans. (A)


httpsin
Coins bought by Pankaj : Dogecoinhttps :
& Etherum https : https :
//youtube.co //www.face //instagram.c
m/channel/U book.com/a om/aashishar //t.me/
CYa4_JrOrf8R orasocial?ut
356 5Kz2uOtccXQ
a s hisharora
m_medium = studifie
s ocial
copy_link d
= 225000/150 + 105000/250 = 1500 + 420 = 1920

160) Ans. (C)


Total Investment made by All three in all 4 coins
= (225 + 200 + 364) + (325 + 170 + 166) + (105 + 192 + 138) + (285 + 288 +
212) = 789 + 661 + 435 + 785 = 2670 Thousand = 2670000

(161 – 165)
School  A B C D E Total
Male 442 315 440 236 350 1783
Female 208 495 280 354 280 1617
Total 650 810 720 590 630 3400

161) Ans. (B)


Total male students in 5 Schools = 1783

162) Ans. (D)


Average female students in 5 school = 1617/5 = 323.4

163) Ans. (D)


In School D (Female – Male) : In School A (Male – Female)
= (354 – 236) : (442 – 208) = 118 : 234 = 59 : 117

164) Ans. (B)


Male students in School A & E = 442 + 350 = 792,
Female Students in School B & C = 495 + 280 = 775
Male students in School A & E are more than Female students in School B
& C by = (792 - 775) × 100/775 = 2.19%

165) Ans. (B)


% of Female students in School E with Total students in School B
= 280 × 100/810 = 34.56%

https : https : https : https :


//youtube.co //www.face //instagram.c
m/channel/U book.com/a om/aashishar //t.me/
CYa4_JrOrf8R orasocial?ut
357 5Kz2uOtccXQ
a s hisharora
m_medium = studifie
s ocial
copy_link d
(166 – 170)
Shopkeeper  P Q R S Total
Sold 18 60 40 56 174
Unsold 78 84 88 40 290
Total 96 144 128 96 464

166) Ans. (C)


Average Number of coolers sold by all 4 shopkeepers = 174/4 = 43.5

167) Ans. (C)


Coolers sold by Shopkeeper A = 1.3 × 40 = 52
Total coolers of Shopkeeper A = 52 × 30/13 = 120
Total Coolers of Shopkeeper A – Total Coolers of Shopkeeper P
= 120 – 96 = 24

168) Ans. (B)


Total Coolers sold by Shopkeeper P & Q = 18 + 60 = 78
Unsold coolers of Shopkeeper R is more than Total coolers sold by
Shopkeeper P & Q by = (88 - 78) × 100/78 = 12.82%

169) Ans. (A)


Sold Coolers by (S – R) : Unsold Coolers by (Q + R)
= (56 – 40) : (84 + 88) = 16 : 172 = 4 : 43

170) Ans. (B)


% of Sold coolers with Unsold coolers of Shopkeepers
= 174 × 100/290 = 60%

(171 – 175)

Class = > A B C D E Total


1st get 30 40 52 36 40 198
2nd get 50 44 60 44 32 230
Total 80 84 112 80 72 428

https : https : https : https :


//youtube.co //www.face //instagram.c
m/channel/U book.com/a om/aashishar //t.me/
CYa4_JrOrf8R orasocial?ut
358 5Kz2uOtccXQ
a s hisharora
m_medium = studifie
s ocial
copy_link d
171) Ans. (C)
Total Student who voted = 428

172) Ans. (A)


Average votes of 1st student from section B, C & E = (40 + 52 + 40)/3 = 44

173) Ans. (B)


Student selected as Class Monitor = 2nd

174) Ans. (B)


Total Student in Class B & D together = 84 + 80 = 164
% of Votes of 1st student with Total students of class B & D together
= (198 × 100)/164 = 120.73%

175) Ans. (B)


Total voted casted by section C & E together : Votes by which 2nd student
won the election = (112 + 72) : (230 – 198) = 184 : 32 = 23 : 4

(176 – 180)
Shopkeeper  J K L M N
% of P 24 55 28 49 36
% of Q 12 16 10 34 18

176) Ans. (C)


Total number of article Q sold by J, K & L
= (12 + 16 + 10) × 108/36 = 38 × 3 = 114

177) Ans. (C)


Article P sold by L, M & N : Total article sold by J & K
= (28 + 49 + 36) : (71 – 36) = 113 : 35

178) Ans. (A)


Unsold article by All shops
= 1200 × (0.64 + 0.29 + 0.62 + 0.17 + 0.46) = 1200 × 2.18 = 2616

179) Ans. (B)


% of sale of L with sale of: N = (38 × 100)/54
https https : = 70.37%
https : https :
//youtube.co //www.face //instagram.c
m/channel/U book.com/a om/aashishar //t.me/
CYa4_JrOrf8R orasocial?ut
359 5Kz2uOtccXQ
a s hisharora
m_medium = studifie
s ocial
copy_link d
180) Ans. (A)
Article P sold = 24 + 55 + 28 + 49 + 36 = 192
Article Q sold = 12 + 16 + 10 + 34 + 18 = 90
Article P sold are more than Article Q sold by
= (192 - 90) × 100/90 = 113.33%

(181 – 185)
Showroom  A B C D E Total
1st type 2100 5520 1288 1800 4704 15412
2nd type 1750 920 1196 950 2842 7658
3rd type 3150 2760 2116 2250 2254 12530
Total 7000 9200 4600 5000 9800 35600

181) Ans. (B)


Type 3 bikes sold by E are less than total bikes sold by B by
= (9200 – 2254) × 100/9200 = 75.5%

182) Ans. (C)


Type 3 bikes sold by B & C : Type 2 bikes sold by C
= (2760 + 2116) : 1196 = 4876 : 1196 = 1219 : 299

183) Ans. (D)


Average of type 3 bikes sold by all showroom = 12530/5 = 2506

184) Ans. (D)


Type 1 bikes sold by A & E – Type 2 bikes sold by C
= (2100 + 4704) – 1196 = 6804 – 1196 = 5608

185) Ans. (B)


Type 1 bikes sold by B, C & D together = 5520 + 1288 + 1800 = 8608
Type 3 bikes sold by A, B & C together = 3150 + 2760 + 2116 = 8026
% of Type 1 bikes sold by B, C & D with Type 3 bikes sold by A, B & C
= 8608 × 100/8026 = 107.25%

https : https : https : https :


//youtube.co //www.face //instagram.c
m/channel/U book.com/a om/aashishar //t.me/
CYa4_JrOrf8R orasocial?ut
360 5Kz2uOtccXQ
a s hisharora
m_medium = studifie
s ocial
copy_link d
(186 – 190)
Student  P Q R S T Total
Math 837 1694 1331 1742 1404 7008
Science 186 968 726 2546 1989 6415
Total 1023 2662 2057 4288 3393 13423

186) Ans. (C)


Average Marks in Math by All student = 7008/5 = 1401.6

187) Ans. (A)


Marks obtained by S : Marks in Math by ( R – P)
= 4288 : (1331 – 837) = 4288 : 494 = 2144 : 247

188) Ans. (B)


Marks of Q in Math is less than Marks of T in Science by
= (1989 – 1694) × 100/1989 = 14.83%

189) Ans. (D)


Total Marks obtain by All students in Science = 6415

190) Ans. (A)


Average marks obtain by all student in Science = 6415/5 = 1283

(191 – 195)

Exam  MTS CHSL CGL GD Total


Number of Students 7500 10500 4500 7500 30000

191) Ans. (C)


Total Students who applied for all the exam = 30000

192) Ans. (B)


Girls who applied for CHSL = 10500 × 10/21 = 5000
Girls who applied for GD = 7500 × 16/25 = 4800
Difference = 5000 – 4800 = 200

https : https : https : https :


//youtube.co //www.face //instagram.c
m/channel/U book.com/a om/aashishar //t.me/
CYa4_JrOrf8R orasocial?ut
361 5Kz2uOtccXQ
a s hisharora
m_medium = studifie
s ocial
copy_link d
193) Ans. (A)
Students who applied for Banking = = 2500
Total Student who applied for CHSL & GD = 10500 + 7500 = 18000
% of students who applied for Banking with Total students applied for
CHSL & GD = 2500 × 100/18000 = 13.88%

194) Ans. (C)


Students who applied for MTS & CHSL = 7500 + 10500 = 18000,
Students who applied for CGL & GD = 4500 + 7500 = 12000,
Students who applied for MTS & CHSL are more than Students who
applied for CGL & GD = (18000 – 12000) × 100/12000 = 50%

195) Ans. (D)


(MTS + GD) : (CHSL – MTS) = (7500 + 7500) : (10500 – 7500)
= 15000 : 3000 = 5 : 1

(196 – 200)
Company  A B C D E Total
Total 8840 3400 10880 4760 6120 34000
2020 2210 646 2176 1904 1836 8772
2021 3094 1326 3264 1666 1530 10880
Unsold 3536 1428 5440 1190 2754 14348

196) Ans. (B)


Bike sold by B = 646 + 1326 = 1972, Bike sold by C = 2176 + 3264 = 5440
Average Bike sold by B & C in 2 year = (1972 + 5440)/2 = 3706

197) Ans. (A)


% of Unsold bike of E with Bike sold by D in 2021
= 2754 × 100/21666 = 165.30%

198) Ans. (A)


Bike sold (by B in 2020 + by D in 2021) : Bike purchased by (by C – by B)
= (646 + 1666) : (10880 – 3400) = 2312 : 7480 = 17 : 55 = 289 : 935

199) Ans. (C)


https :
Maximum Bikes sold by =D https : https : https :
//youtube.co //www.face //instagram.c
m/channel/U book.com/a om/aashishar //t.me/
CYa4_JrOrf8R orasocial?ut
362 5Kz2uOtccXQ
a s hisharora
m_medium = studifie
s ocial
copy_link d
200) Ans. (D)
Unsold bikes of All Showroom in 2 year = 14348

(201 – 205)
201) Ans. (A)
% of Student failed on Thursday with students failed on Friday
= (44 × 56) × 100/(99 × 50) = 49.77%

202) Ans. (D)


Number of Students who passed on Tuesday
= 1620/0.81 × 1.25 × 0.38 = 2000 × 0.475 = 950

203) Ans. (B)


% of difference between students who failed & passed in Monday with
students who appeared on Tuesday = 45 × (70 – 30) × 100/65 = 2769%

204) Ans. (D)


Failed students on Friday – Failed students on Tuesday
= 19900 × 0.5 – 1140 × 62/38 = 9950 – 1860 = 8090

205) Ans. (B)


Students passed on Monday & Friday : Students failed on Monday &
Thursday = (45 × 30 + 99 × 50) : (45 × 70 + 44 × 56)
= (1350 + 4950) : (3150 + 2464) = 6300 : 5614 = 450 : 401

(206 – 210)
Student  A B C D
English 65 40 57 30
Hindi 25 4 19 10

206) Ans. (C)


% of marks scored by A in Hindi with marks scored by C in English
= (25 × 100)/57 = 43.85%

207) Ans. (B)


100 × (0.25 + 0.04 + 0.10) + 150 × (0.65 + 0.4 + 0.3)
= 100 × 0.39 + 150 × 0.77
https=: 39 + 115.5 https :
= 154.5 https : https :
//youtube.co //www.face //instagram.c
m/channel/U book.com/a om/aashishar //t.me/
CYa4_JrOrf8R orasocial?ut
363 5Kz2uOtccXQ
a s hisharora
m_medium = studifie
s ocial
copy_link d
208) Ans. (A)
Marks scored by B is less than marks scored by C by
= (76 - 44) × 100/76 = 42%

209) Ans. (A)


Total marks of (B + C) : Total marks of (A + B + D)
= (44 + 76) : (90 + 40 + 40) = 120 : 170 = 12 : 17

210) Ans. (D)


Average % scored by All students in English
= (65 + 40 + 57 + 30)/4 = 192/4 = 48%

(211 – 215)
Showroom  A B C D E
Mobiles 35% 40% 56% 44% 30%
LED’s 30% 56% 32% 32% 60%
Total 65% 96% 88% 76% 90%

211) Ans. (D)


Mobiles sold by C : LED’s sold by A = 56 : 30 = 28 : 15

212) Ans. (C)


Mobiles sold by (A, C & E) : LED’s sold by (B, C & D)
= (35 + 56 + 30) : (56 + 32 + 32) = 121 : 120

213) Ans. (B)


% of Mobiles sold by B with Mobiles & LED’s sold by E = 40 × 100/90 =
44.44%

214) Ans. (D)


Total LED’s & Washing Machine sold by B & C
= 2000/1.25 × (0.56 + 0.04) + 2000 × (0.32 + 0.12)
= 1600 × 0.6 + 2000 × 0.44 = 960 + 880 = 1840

215) Ans. (C)


Total Mobiles & LED’s sold by Shop A = 63 × 65/35 = 117
https : https : https : https :
//youtube.co //www.face //instagram.c
m/channel/U book.com/a om/aashishar //t.me/
CYa4_JrOrf8R orasocial?ut
364 5Kz2uOtccXQ
a s hisharora
m_medium = studifie
s ocial
copy_link d
(216 – 220)
Month  January February March April May Total
Male 600 600 420 639 900 3159
Female 840 450 1050 639 450 3429
Children 480 900 1050 852 720 4002
Total 1920 1950 2520 2130 2070 10590

216) Ans. (A)


In February, % of Males with Total number or Females & Children
= 4 × 100/9 = 44.44%

217) Ans. (B)


Revenue in February = 1950 × 100 = 195000 Rs.
Revenue in March = 2520 × 160 = 403200 Rs.
% increase in Revenue = (403200 – 195000) × 100/195000 = 106%

218) Ans. (D)


Males & Females went individually in January
= (600 – 480) + (840 – 480) = 120 + 360 = 480

219) Ans. (B)


Female in all the months are more than Males in all the months by
= 3429 – 3159 = 270

230) Ans. (C)


Average number of Children present in January, March & May
= (480 + 1050 + 720)/3 = 2250/3 = 750

(221 – 225)
Class  VIII IX X XI XII Total
Section A 1152 1440 918 1131 693 5334
Section B 1248 1260 2091 1755 1155 7509
Section C 2400 1800 2091 1014 1452 8757
Total 4800 4500 5100 3900 3300 21600
https : https : https : https :
//youtube.co //www.face //instagram.c
m/channel/U book.com/a om/aashishar //t.me/
CYa4_JrOrf8R orasocial?ut
365 5Kz2uOtccXQ
a s hisharora
m_medium = studifie
s ocial
copy_link d
221) Ans. (C)
In Section C, Class IX students are less than Class VIII students by
= (2400 – 1800) × 100/2400 = 25%

222) Ans. (A)


Section B of class (X, XI & XII) : Section A of class (VII, IX & X)
= (2091 + 1755 + 1155) : (1152 + 1440 + 918) = 5001 : 3510 = 1667 : 1170

223) Ans. (D)


Passed students of Section C of Class 10th
= 5100 × 1/3 – 918 × 1/6 – 2091 × 2/3 = 1700 – 153 – 1394 = 153

224) Ans. (D)


Average of All students of Section C = 8757/5 = 1751.4

225) Ans. (C)


Total Students in Section B – Total Students in Section A
= 7509 – 5334 = 2175

(226 – 230)
226) Ans. (C)
Average % of Apple mobiles sold in All year
= (42 + 28 + 36 + 44 + 54) /5 = 40.8
Average Apple mobiles sold in All years = 15200 × 0.408 = 6201.6

227) Ans. (A)


In 2012 & 2015, Samsung mobiles sold – Blackberry mobiles sold
= 19200 × (0.38 – 0.34) + 18500 × (0.28 – 0.18)
= 19200 × 0.04 + 18500 × 0.1 = 768 + 1850 = 2618

228) Ans. (D)


Samsung mobiles sold in 2011 : Apple mobiles sold in 2013
= (19 × 20) : (15 × 36) = 19 : 27

229) Ans. (B)


In 2014, Samsung mobiles sold are less than Apple & Blackberry mobiles
sold by = (70 – 30) × 100/70 = 57.14%
https : https : https : https :
//youtube.co //www.face //instagram.c
m/channel/U book.com/a om/aashishar //t.me/
CYa4_JrOrf8R orasocial?ut
366 5Kz2uOtccXQ
a s hisharora
m_medium = studifie
s ocial
copy_link d
230) Ans. (B)
Apple mobiles sold in 2012 = 4712 × 28/38 = 3472

(231 – 235)
Shop  A B C D E Total
Anars 3645 1440 1800 2160 1800 10845
Sparklers 2835 2880 2880 3060 1440 13095
Rockets 3240 3240 1440 1620 2520 12060
Total 9720 7560 6120 6840 5760 36000

231) Ans. (C)


Anars sold by Shop B, C & E : Sparklers sold by A, C & D
= (1440 + 1800 + 1800) : (2835 + 2880 + 3060) = 5040 : 8775 = 112 : 195

232) Ans. (B)


Sparklers sold by B are more than Rocket sold by C by
= (2880 – 1440) × 100/1440 = 100%

233) Ans. (D)


Total Revenue of Shop A & Shop B = 5 × (3645 + 1440) + 10 × (2835 + 2880)
+ 15 × (3240 + 3240) = 25425 + 57150 + 97200 = 179775 Rs.

234) Ans. (B)


New number of Crackers sold by Shop E
= 1800 + 1440 × 7/12 + 2520 × 22/15 = 1800 + 840 + 3696 = 6336

235) Ans. (B)


Average Number of Rocket sold by All shop together = 12060/5 = 2412

(236 – 240)
Showrooms  A B C D E Total
January 1258 572 816 615 1750 5011
February 962 988 1156 984 1375 5465
March 1166 714 1014 560 1300 4754
Total 3386 2274 2986 2159 4425 15230
https : https : https : https :
//youtube.co //www.face //instagram.c
m/channel/U book.com/a om/aashishar //t.me/
CYa4_JrOrf8R orasocial?ut
367 5Kz2uOtccXQ
a s hisharora
m_medium = studifie
s ocial
copy_link d
236) Ans. (C)
Chairs sold by C in January is less than Chairs sold by E in March by
= (1300 – 816) × 100/1300 = 37.23%

237) Ans. (D)


Chairs sold by D in January & March = 615 + 560 = 1175
Unbroken chairs = 615 × 7/15 + 560 × 5/7 = 287 + 400 = 687
% of Unbroken Chairs = 687 × 100/1175 = 58.4%

238) Ans. (A)


Chairs sold in January by A, B & C – Chairs sold in March by D & E
= (1258 + 572 + 816) – (560 + 1300) = 2646 – 1860 = 786

239) Ans. (C)


Cost Price of Chairs sold by C in February
= 1156 × (1.375 × 1856) = 2950112 Rs.

240) Ans. (D)


Average Chairs sold in February = 5465/5 = 1093

(241 – 245)
School = > A B C D E Total
Boys 360 400 300 350 360 1770
Girls 200 320 160 230 290 1200
Teachers 40 140 50 80 60 370
Total 600 860 510 660 710 3340

241) Ans. (B)


% of Teachers in E with Boys in C = 60 × 100/300 = 20%

242) Ans. (A)


Girls in School B & C = 320 + 160 = 480
Boys in School A & E = 360 + 360 = 720
Girls in B & C are less than Boys in A & E by
= (720 – 480) × 100/720 = 33.33%

https : https : https : https :


//youtube.co //www.face //instagram.c
m/channel/U book.com/a om/aashishar //t.me/
CYa4_JrOrf8R orasocial?ut
368 5Kz2uOtccXQ
a s hisharora
m_medium = studifie
s ocial
copy_link d
243) Ans. (C)
In School C, (Boys + Girls) : Teachers
= [300 × 22/15 + 160 × 1.25] : 50 = (440 + 200) : 50 = 64 : 5.

244) Ans. (A)


Average Teachers in all schools = 370/5 = 74

245) Ans. (D)


Boys in C, D & E = 300 + 350 + 360 = 1010
Girls in A, B & C = 200 + 320 + 160 = 680
Difference = 1010 – 680 = 330

(246 – 250)
Shop  A B C D E Total
TV 400 525 650 550 850 2975
AC 350 275 400 400 600 2025
Total 750 800 1050 950 1450 5000

246) Ans. (A)


TV sold by B : TV sold by C = 525 : 650 = 21 : 26

247) Ans. (C)


TV sold by B are more than AC sold by D by
= (525 – 400) × 100/400 = 31.25%

248) Ans. (B)


By E, TV sold = 0.8 × 850 = 680, AC sold = 600 × 17/12 = 850
AC sold are more than TV sold by = (850 – 680) × 100/680 = 25%

249) Ans. (C)


TV sold by A, D & E = 400 + 550 + 850 = 1800
AC sold by B, C & E = 275 + 400 + 600 = 1275
Difference = 1800 – 1275 = 525

250) Ans. (A)


Average CTV sold by all shops = 2975/5 = 595
https : https : https : https :
//youtube.co //www.face //instagram.c
m/channel/U book.com/a om/aashishar //t.me/
CYa4_JrOrf8R orasocial?ut
369 5Kz2uOtccXQ
a s hisharora
m_medium = studifie
s ocial
copy_link d
(251 – 255)
PB – I PB – II
School
Total Passed Failed Total Passed Failed
P 800 336 464 700 329 371
Q 600 330 270 500 145 355
R 560 224 336 700 259 441
S 700 245 455 980 392 588
T 660 165 495 780 468 312
Total 3320 1300 2020 3660 1593 2067

251) Ans. (C)


Passed in School P in PB – I : Passed in School R in PB – II = 336 : 259

252) Ans. (A)


In School Q, Students passed in PB – I is more than Students passed in
PB – II by = (330 – 145) × 100/145 = 127.5%

253) Ans. (D)


In School T, Students passed in PB – I = 165 × 26/15 = 286
Students passed in PB – II = 468 × 0.75 = 351
Difference = 351 – 286 = 65

254) Ans. (B)


Average Students appeared in PB – II = 3660/5 = 732

255) Ans. (A) = 805


(Students passed in PB – II in School P, Q, R & S) – (Students passed in
PB – I in School P, R & S)
= (329 + 145 + 259 + 392) – (336 + 224 + 245) = 1125 – 805 = 320

(256 – 260)
Seller  P Q R S T Total
Bike 3250 2250 2625 4000 3250 15375
Cars 1750 1250 1875 2250 2500 9625
Total 5000 https : 3500 4500
https : 6250https : 5750 https25000
:
//youtube.co //www.face //instagram.c
m/channel/U book.com/a om/aashishar //t.me/
CYa4_JrOrf8R orasocial?ut
370 5Kz2uOtccXQ
a s hisharora
m_medium = studifie
s ocial
copy_link d
256) Ans. (C)
Bikes sold by S are more than Cars sold by Q
= (4000 – 1250) × 100/1250 = 220%

257) Ans. (B)


Cars sold by P & Q = 1750 + 1250 = 3000
Bikes sold by R & T = 2625 + 3250 = 5875
Difference = 5875 – 3000 = 2875

258) Ans. (B)


By U, Bikes sold = 1875 × 19/15 = 2375, Cars sold = 2250 × 11/9 = 2750
Vehicles sold by U = 2375 + 2750 = 5125

259) Ans. (C)


Average number of cars sold by all shops = 9625/5 = 1925

260) Ans. (C)


Non – electric vehicles sold by P
= 0.8 × 3250 + 0.7 × 1750 = 2600 + 1225 = 3825

(261 – 265)
Shopkeeper  A B C D E Total
Diyas 825 775 900 1500 1000 5000
Candles 875 1225 1400 0 1500 5000
Total 1700 2000 2300 1500 2500 10000

261) Ans. (C)


Diya sold by C are less than Candles sold by E
= (1500 – 900) × 100/1500 = 40%

262) Ans. (D)


Shopkeeper who do not sell any Candles = D

263) Ans. (A)


Unbroken Diyas & Candles with E
= 1000 × 0.625 + 1500 × 8/15 = 625 + 800 = 1425
% of Unbroken Diyas & https :
Candles https×: 100/2500https
= 1425 = 57%: https :
//youtube.co //www.face //instagram.c
m/channel/U book.com/a om/aashishar //t.me/
CYa4_JrOrf8R orasocial?ut
371 5Kz2uOtccXQ
a s hisharora
m_medium = studifie
s ocial
copy_link d
264) Ans. (C)
Revenue of Shopkeeper A
= 825 × 2.5 + 875 × 7.5 = 2062.5 + 6562.5 = 8625 Rs.

265) Ans. (A)


Average number of Candles sold by All shopkeepers = 5000/5 = 1000

(266 – 270)
Item  P Q R S T
CP 8 7 6 8 11
MP 15 12 11 9 16
SP 13 7 8 8 13
Discount 2 5 3 1 3
Profit 5 0 2 0 2

266) Ans. (B)


SP of R : SP of T = (2166 × 8/6) : (2354 × 13/11) = 2888 : 2782 = 1444 : 1391

267) Ans. (A)


Discount given by Q = 784 × 5/7 = 560 Rs.
Discount given by R = 792 × 3/6 = 396 Rs.
Difference = 560 – 396 = 164 Rs.

268) Ans. (D)


New SP for P = 15 × 8/15 = 8 Rs.
No Profit

269) Ans. (D)


SP of P = 512 × 13/8 = 832 Rs., SP of S = 528 Rs.
SP of P is more than by SP of S = (832 – 528) × 100/528 = 57.57%

270) Ans. (D)


Shopkeeper who sell on no profit no loss = Q & S

https : https : https : https :


//youtube.co //www.face //instagram.c
m/channel/U book.com/a om/aashishar //t.me/
CYa4_JrOrf8R orasocial?ut
372 5Kz2uOtccXQ
a s hisharora
m_medium = studifie
s ocial
copy_link d
(271 – 275)
Year  2015 2016 2017 2018 2019 Total
Selected 810 2400 3528 1440 2790 10968
Not – Selected 5670 3360 4032 3600 8370 25032
Total 6480 5760 7560 5040 11160 36000

271) Ans. (B)


% of Selected students in 2015 with not selected students in 2019
= 810 × 100/8370 = 9.67%

272) Ans. (D)


Students appeared in 2020 = 7560 × 23/15 = 11592
Selected students = x, Not selected Students = 11592 – x
(11592 – x) × 5/7 = x
8280 = x + 5x/7 = 12x/7
x = 4830

273) Ans. (A)


Not – Selected students in 2016 : Not – Selected students in 2017
= 3360 : 4032 = 5 : 6

274) Ans. (C)


Selected students in 2016, 2018 & 2019 – Selected students in 2015 & 2017
= (2400 + 1440 + 2790) – (810 + 3528) = 6630 – 4338 = 2292

275) Ans. (C)


Average of not – selected students in 5 years = 25032/5 = 5006.4

(276 – 280)
Colonies  A B C D E Total
Swimming 95 87 49 72 51 354
Football 74 92 86 104 52 408
Basketball 80 112 98 54 108 452
Cricket 133 116 63 54 85 451
Total 382 407 https : 296 284 296
https : https : https1665
:
//youtube.co //www.face //instagram.c
m/channel/U book.com/a om/aashishar //t.me/
CYa4_JrOrf8R orasocial?ut
373 5Kz2uOtccXQ
a s hisharora
m_medium = studifie
s ocial
copy_link d
276) Ans. (C)
People like swimming in colony A, D & E : People like football in colony B,
C & E = (95 + 72 + 51) : (92 + 86 + 52) = 218 : 230 = 109 : 115

277) Ans. (A)


People playing in Colony A are more than that of Colony E by
= (382 – 296) × 100/296 = 29.05%

278) Ans. (D)


Average of Basketball players in all colonies = 452/5 = 90.4

279) Ans. (A)


Participant in Cricket tournament
= 133 × 2/7 + 116 × 0.25 + 63 × 1/3 + 54 × 1/6 + 85 × 0.2
= 38 + 29 + 21 + 9 + 17 = 114

280) Ans. (B)


Average of cricket players in all colonies = 451/5 = 90.2

(281 – 285)
281) Ans. (B)
Crackers sold in, City B in 2021 : City C & D in 2020
= (1.16 × 1.25) : (1.24 + 1.56) = 1.45 : 2.8 = 29 ; 56

282) Ans. (C)


Total increase in number of crackers sold in City E from 2019 to 2021
= (1.38 × 1.5 – 1) × 100 = 107%

283) Ans. (A)


Average crackers sold in 2020
= 100 × (1.2 + 1.16 + 1.24 + 1.56 + 1.38) /5 = 20 × 6.54 = 130.8
Average crackers sold in 2021
= 100 × (1.2 × 1.15 + 1.16 × 1.25 + 1.24 × 1.75 + 1.56 × 1.25 + 1.38 × 1.5) /5
= 20 × (1.38 + 1.45 + 2.17 + 1.95 + 2.07) = 20 × 9.02 = 180.4
Difference = 180.4 – 130.8 = 49.6

284) Ans. (B)


Crackers sold in 2021https :
in City × 1.25
D = 1.56https : = 1.95https : https :
//youtube.co //www.face //instagram.c
m/channel/U book.com/a om/aashishar //t.me/
CYa4_JrOrf8R orasocial?ut
374 5Kz2uOtccXQ
a s hisharora
m_medium = studifie
s ocial
copy_link d
Crackers sold in 2020 in City B & C = 1.16 + 1.24 = 2.4
% of crackers sold in City D in 2021 with crackers sold in City B & C in
2020 = 1.95 × 100/2.4 = 81.25%

285) Ans. (D)


Crackers sold in 2020 in City A & C : Average crackers sold in 2021 in all
Citites = (1.2 + 1.24) : (9.02/5) = 2.44 : 1.804 = 610 : 451

(286 – 290)
Shopkeeper  A B C D E Total
Sweet X 349 578 690 388 427 2432
Sweet Y 115 114 390 100 281 1000
Total 464 692 1080 488 708 3432

286) Ans. (A)


% of Sweet Y sold by B with Total Sweets sold by E
= 114 × 100/708 = 16.10%

287) Ans. (B)


Profit of C – Profit of B = 330 × 690 – 240 × 114 = 227700 – 27360 = 200340

288) Ans. (A)


Average of Sweet Y sold by B, C & D = (114 + 390 + 100) /3 = 604/3 = 201.33

289) Ans. (C)


Sweet Y sold by A & E = 115 + 281 = 396
Total sweets sold by B is more than Sweet Y sold by A & E by
= (692 – 396) × 100/396 = 74.74%

290) Ans. (B)


Sweet X sold by A & B : Sweet Y sold by C & E
= (349 + 578) : (390 + 281) = 927 : 671

(291 – 295)
291) Ans. (B)
Average Students in Blue House = (140 + 260 + 180 + 240) /4 = 820/4 = 205
https : https : https : https :
//youtube.co //www.face //instagram.c
m/channel/U book.com/a om/aashishar //t.me/
CYa4_JrOrf8R orasocial?ut
375 5Kz2uOtccXQ
a s hisharora
m_medium = studifie
s ocial
copy_link d
292) Ans. (A)
Students in Pink House in 2019 : Students in Orange House in 2021
= 150 : 200 = 3 : 4

293) Ans. (C)


Total Students, in 2018 = 140 + 240 + 148 + 260 = 788
in 2021 = 240 + 180 + 200 + 200 = 820
Students in 2018 are less than in 2021 by = (820 – 788) × 100/820 = 3.9%

294) Ans. (C)


Increase in number of Students compared to previous year was highest in
= 2021

295) Ans. (D)


Total Students in Blue House & Orange House
= (140 + 260 + 180 + 240) + (260 + 148 + 242 + 200) = 820 + 850 = 1670

(296 – 300)
296) Ans. (D)
% of Students voted for Alia with Total Students voted for Katrina &
Deepika = 10 × 100/(38 + 29) = 14.92%

297) Ans. (C)


Student voted for Katrina & Priyanka : Student voted for Alia & Shraddha
= (38 + 13) : (10 + 10) = 51 : 20

298) Ans. (A)


Remaining votes for Katrina = 1500 × 0.38 × 0.8 = 570 × 0.8 = 456

299) Ans. (B)


Average Votes for Katrina & Alia – Average Votes for Priyanka & Shradha
= 15 × (38 + 10) /2 – 15 × (13 + 10) /2 = 15 × (24 – 11.5) = 15 × 12.5 = 187.5

300) Ans. (D)


Students voted for Katrina are more than Students voted for Shradha by
= (38 – 10) × 100/10 = 280%

https : https : https : https :


//youtube.co //www.face //instagram.c
m/channel/U book.com/a om/aashishar //t.me/
CYa4_JrOrf8R orasocial?ut
376 5Kz2uOtccXQ
a s hisharora
m_medium = studifie
s ocial
copy_link d
https://t.mhttps/t.me
e/studified/studified

https://instagram.com/aashisharora
social?utm_medium = copy_link

https://youtube.com/channel
/UCYa4_JrOrf8R5Kz2uOtccXQ

https://www.facebook.
com/aashisharorasocial

https://youtu https://instagra
https ://ww
be.com/chan w.fa cebook.
m.com/aashish https://
arorasocial(?)
nel/UCYa4_Jr com/a ashis utm_medium = t.me/st
377 Orf8R5Kz2uO
tccXQ
ha rorasocial copy_link
udified
1. Train A running at a speed of 36 km/hr crosses train B in 20
seconds. Find the speed of train B (in km/hr), if the sum of the
length of train A & B is 600 meters and both trains running in the
opposite direction.
ट्रे न A 36 ककभी/घॊटा की गनत से चरते हुए ट्रे न B को 20 से कॊड भें ऩाय कयती है, ट्रे न
B की गनत ऻात कीजजमे (ककभी/घॊटे) , मदद ट्रे न A औय B की रम्फाई का मोग 600
भी है औय दोनों ट्रे न ववऩयीत ददिा भें जाती हो?
(A) 72 km/hr (B) 54 km/hr (C) 48 km/hr
(D) 108 km/hr (E) 81 km/hr

2. A invests Rs. X in a business. After four months B joined him with


Rs. 2X and A double his investment. If at the end of the years total
profit is Rs. 13950, then find the profit share of A?
A, X रु. ककसी व्मवसाम भें ननवे ि कयता है। चाय भहीने फाद B, 2X रु. के साथ
इनभें िाशभर होता है औय A अऩना ननवे ि दग ु न
ु ा कय दे ता है, मदद वर्षष के अॊत भें
कु र राब 13950 रु. है तो A का राबाॊि ऻात कीजजमे
(A) 7250 Rs. (B) 7750 Rs. (C) 8750 Rs.
(D) 6750 Rs. (E) 7050 Rs.

3. A man invested an amount in two schemes in the ratio of 2 : 3 at


the rate of 20% p.a. and 10% p.a. on compound interest respectively.
If the man gets a total interest of Rs. 1208 after two years from both
the schemes, the find amount invested by man?
एक व्मजक्त एक धनयाशि को दो स्कीभ भें 2 : 3 के अनुऩात भें चक्रवरवि ब्माज ऩय
क्रभि् 20% प्रनतवावर्षषक औय 10% प्रनत वावर्षषक से ननवे ि कयता है, मदद व्मजक्त को
दोनों स्कीभ से दो वर्षों फाद 1208 रु. कु र ब्माज शभरता है तो व्मजक्त द्वाया ननवे शित
याशि ऻात कीजजमे
(A) 6000 Rs. (B) 4800 Rs. (C) 5000 Rs.
(D) 4500 Rs. (E) 4000 Rs.

4. The sum of the length of a rectangle and the side of a square is


72 meters. If the perimeter of the rectangle is 84 meters and the
breadth of the rectangle is 18 meters, then find the measurement of
the side of the square (in meters).
एक आमत की रम्फाई औय एक वगष की बुज ा का मोग 72 भी है, मदद आमत का
ऩरयभाऩ 84 भी औय आमत की चौडाई 18 भी हो तो वगष की बुज ा का भाऩ भी भें
ऻात कीजजमे
https : https : https : https :
//youtube.co //www.face //instagram.c
m/channel/U book.com/a om/aashishar //t.me/
CYa4_JrOrf8R orasocial?ut
378 5Kz2uOtccXQ
a s hisharora
m_medium = studifie
s ocial
copy_link d
(A) 48 (B) 36 (C) 54
(D) 60 (E) 30

5. The ratio of time taken by A, B & C to complete work alone is 4 :


6 : 3 respectively. If all three together complete the work in 8 days,
then find in how many days A alone complete three – fourth of the
same work?
A, B औय C द्वाया अके रे कामष ऩूय ा कयने भें शरए गमे सभम का अनुऩात क्रभि् 4 :
6 : 3 है, मदद सबी तीनों शभरकय कामष को 8 ददनों भें ऩूय ा कयते हैं ककतने ददनों भें A
अकेरे सभान कामष के तीन – चौथाई बाग को ऩयू ा कये गा?
(A) 12 (B) 30 (C) 15
(D) 24 (E) 18

6. In the following questions, there are two equations in x and y. You


have to solve both the equations and give answer
ननम्न प्रश्नों भें , दो सभीकयण x औय y हैं , आऩको दोनों सभीकयण को हर कयना है
औय उत्तय दे ना है
I. 2x² – 3x + 1 = 0, II. 2y² – 5y + 3 = 0
(A) if x > y (B) if x < y (C) if x ≥ y
(D) if x ≤ y (E) if x = y or there is no relation between x and y

7. In the following questions, there are two equations in x and y. You


have to solve both the equations and give answer
ननम्न प्रश्नों भें , दो सभीकयण x औय y हैं , आऩको दोनों सभीकयण को हर कयना है
औय उत्तय दे ना है
I. x² + 21x + 110 = 0, II. y² + 17y + 72 = 0
(A) if x > y (B) if x < y (C) if x ≥ y
(D) if x ≤ y (E) if x = y or there is no relation between x and y

8. In the following questions, there are two equations in x and y. You


have to solve both the equations and give answer
ननम्न प्रश्नों भें , दो सभीकयण x औय y हैं , आऩको दोनों सभीकयण को हर कयना है
औय उत्तय दे ना है
I. x² = 4, II. y² – 6y + 8 = 0
(A) if x > y (B) if x < y (C) if x ≥ y
(D) if x ≤ y (E) if x = y or there is no relation between x and y

9. In the following questions, there are two equations in x and y. You


have to solve both the equations and give answer
ननम्न प्रश्नों भें , दो सभीकयण x औय y हैं , आऩको दोनों सभीकयण को हर कयना है
औय उत्तय दे ना है
I. x² + 9x – 22 = 0, II. 2y² – 7y + 6https
= :0
https : https : https :
//youtube.co //www.face //instagram.c
m/channel/U book.com/a om/aashishar //t.me/
CYa4_JrOrf8R orasocial?ut
379 5Kz2uOtccXQ
a s hisharora
m_medium = studifie
s ocial
copy_link d
(A) if x > y (B) if x < y (C) if x ≥ y
(D) if x ≤ y (E) if x = y or there is no relation between x and y

10. In the following questions, there are two equations in x and y.


You have to solve both the equations and give answer
ननम्न प्रश्नों भें , दो सभीकयण x औय y हैं , आऩको दोनों सभीकयण को हर कयना है
औय उत्तय दे ना है
I. 6x² + 5x + 1 = 0, II. 15y² + 11y + 2 = 0
(A) if x > y (B) if x < y (C) if x ≥ y
(D) if x ≤ y (E) if x = y or there is no relation between x and y

11. The population of a village is decreased by 10% in the first year


and then increased by 20% in the second year. Find the population
of the village at the end of the second year if two years ago it was
15,000?
एक गाॉव की जनसॊख्मा ऩहरे वर्षष भें 10% कभ होती है औय कपय दस ू ये वर्षष भें 20%
फढ जाती है, दस
ू ये वर्षष के अॊ त भें गाॉ व की जनसॊ ख् मा ऻात कीजजमे मदद दो वर्षष ऩहरे
मह 15,000 थी
(A) 16180 (B) 16200 (C) 16320
(D) 16360 (E) 16480

12. A shopkeeper marked up a shirt 50% above the cost price and
allows successive discounts of 11 1/9 % and 10% on it. If the
difference between profit earned and discount given is Rs. 200then
find the marked price of the shirt?
एक दुक ानदाय एक िटष को क्रम भूल्म से 50% अचधक अॊककत कयता है औयइस ऩय 11
1/9 % औय 10% की क्रशभक छू ट दे ता है मदद अजजषत राब अॊतय है तो िटष का
अॊककत भूल्म ऻात कीजजमे
(A) Rs. 2100 (B) Rs. 3000 (C) Rs. 2400
(D) Rs. 3600 (E) Rs. 3300

13. The downstream speed of a boat is 5 km/hr more than its


upstream speed and the ratio of the speed of the boat in still water
to the speed of the stream is 19 : 5. Find the total time taken by
boat to travel 42 km downstream and 31.5 km upstream?
एक नाव की धाया के प्रनतकू र गनत धाया की अनुकू र गनत से 5 ककभी/घॊटे अचधक है
तथा िाॊत जर भें नाव की गनत का धाया की गनत से अनुऩात 19 : 5 है तो नाव
द्वाया धाया के अनुकू र 42 ककभी औय धाया के प्रनतकू र 31.5 ककभी तम कयने भें
शरमा गमा सभम ऻात कीजजमे
(A) 7(1/2) hr (B) 8 hr (C) 9 hr
(D) 9(1/2) hr (E) 10 hr
https : https : https : https :
//youtube.co //www.face //instagram.c
m/channel/U book.com/a om/aashishar //t.me/
CYa4_JrOrf8R orasocial?ut
380 5Kz2uOtccXQ
a s hisharora
m_medium = studifie
s ocial
copy_link d
14. A container contains a mixture of two liquids P and Q in the ratio
5 : 3. If 16 liter of the mixture is taken out and replaced with liquid
Q, then the new ratio of liquid P to liquid Q becomes 1 : 1. Find the
initial quantity of mixture in the container.
एक ऩात्र भें दो शरजक्वड P का Q का शभश्ण 5 : 3 के अनुऩात भें है, मदद 16 री
शभश्ण ननकारा जाता है औय शरजक्वड Q से फदरा जाता है तो शरजक्वड P से शरजक्वड
Q का नमा अनुऩात 1 : 1 हो जाता है तो ऩात्र भें शभश्ण की आयॊ शबक भात्रा ऻात
कीजजमे
(A) 80 L (B) 60 L (C) 70 L
(D) 48 L (E) 96 L

15. The ratio of the present age of A to B is 8 : 5 and the average


of the present age of B and C is 35 years. If five years ago, the sum
of ages of A and B is 55 years, then find the difference between the
present age of A and C.
A की वतषभान आमु का B की वतषभान आमु से अनुऩात 8 : 5 है तथा B औय C की
वतषभान आमु का औसत 35 वर्षष है, मदद ऩाॊच वर्षष ऩूव,ष A औय B की आमु का मोग
55 वर्षष है तो A औय C की वतषभान आमु के भध्म अॊतय ऻात कीजजमे
(A) 12 years (B) 5 years (C) 9 years
(D) 8 years (E) 4 years

16. The line graph given shows the total number of posts (Photos +
Videos) shared by six (A, B, C, D, E & F) people in December 2019.
Read the data carefully and answer the questions.
ननम्न राइन ग्राप 2019 भें छह व्मजक्तमों(A, B, C, D, E औय F) द्वाया िे मय
ऩोस्ट (पोटो + ववडडमो) की कु र सॊख्मा को दिाषता है, डे टा का ध्मानऩूवषक अध्ममन
कीजजमे औय प्रश्नों के उत्तय दीजजमे

https : https : https : https :


//youtube.co //www.face //instagram.c
m/channel/U book.com/a om/aashishar //t.me/
CYa4_JrOrf8R orasocial?ut
381 5Kz2uOtccXQ
a s hisharora
m_medium = studifie
s ocial
copy_link d
A. The total post shared by C is what percent less than the total
post shared by D?
C द्वाया िे मय की कु र ऩोस्ट D द्वाया िे मय की गमी कु र ऩोस्ट से ककतने प्रनतित
कभ है?
(A) 20 % (B) 25% (C) 10%
(D) 10% (E) 15%

B. In January 2020 total posts shared by B & F is 12 and 15 more


than previous month respectively, then find the total number of the
post shared by B & F in January 2020?
जनवयी 2020 भें B औय F द्वाया िे मय की गमी कु र ऩोस्ट वऩछरे भहीने की तुरना
भें क्रभि् 12 औय 15 अचधक है, तो जनवयी 2020 भें B औय F द्वाया िे मय की गमी
कु र ऩोस्ट की सॊख्मा ऻात कीजजमे
(A) 95 (B) 91 (C)93
(D) 97 (E)99

C. Find the average number of posts shared by A, C & F?


A, C औय F द्वाया िे मय की गमी ऩोस्ट की औसत सॊख्मा ऻात कीजजमे
(A) 42 (B) 48 (C)40
(D) 36 (E) 44

D. Total photos shared by E is four more than total videos shared by


him, then find total videos shared by E?
E द्वाया िे मय की गमी कु र पोटो उसके द्वाया िे मय की गमी कु र ववडडमो से चाय
अचधक है तो E द्वाया िे मय की गमी कु र ववडडमो ऻात कीजजमे
(A) 24 (B) 20 (C) 28
(D) 22 (E) 30

E. If the ratio of total photos to total videos shared by B is 5 : 9,


then find total photos shared by B?
मदद B द्वाया िे मय की गमी कु र पोटो का कु र ववडडमो से अनुऩात 5 : 9 है तो B
द्वाया िे मय की गमी कु र पोटो ऻात कीजजमे
(A) 10 (B) 18 (C)12
(D) 14 (E) 16

17. What will come in the place of question mark in following


questions.
ननम्न प्रश्नों भें प्रश्नचचह्न के स्थान ऩय क्मा आएगा?
(? ‚ 5 ‚ 7) × 14 + 112 = 420
(A) 710 (B) 720 (C) 660
(D) 770 (E) 750
https : https : https : https :
//youtube.co //www.face //instagram.c
m/channel/U book.com/a om/aashishar //t.me/
CYa4_JrOrf8R orasocial?ut
382 5Kz2uOtccXQ
a s hisharora
m_medium = studifie
s ocial
copy_link d
18. What will come in the place of question mark in following
questions.
ननम्न प्रश्नों भें प्रश्नचचह्न के स्थान ऩय क्मा आएगा?
(?)(4 X 16 ‚ 31 + 1) + 122 = 360
(A) 4 (B) 6 (C) 8
(D) 2 (E) 3

19. What will come in the place of question mark in following


questions.
ननम्न प्रश्नों भें प्रश्नचचह्न के स्थान ऩय क्मा आएगा?
50% of 128 + √16 x 4 = (?) – 12
(A) 64 (B) 68 (C) 60
(D) 56 (E) 82

20. What will come in the place of question mark in following


questions.
ननम्न प्रश्नों भें प्रश्नचचह्न के स्थान ऩय क्मा आएगा?
∛1331 + √81 + ? = 27
(A) 19 (B) 18 (C) 17
(D) 16 (E) 15

21. What will come in the place of question mark in following


questions.
ननम्न प्रश्नों भें प्रश्नचचह्न के स्थान ऩय क्मा आएगा?
(13) (2) + 179 + (5)2 = (?)2
(A) 10 (B) 20 (C) 30
(D) 40 (E) 22

22. What will come in the place of question mark in following


questions.
ननम्न प्रश्नों भें प्रश्नचचह्न के स्थान ऩय क्मा आएगा?
28.5 x 34 + 2320 ‚ 8 = (36)2 – ?
(A) 51 (B) 47 (C) 43
(D) 34 (E) 37

23. What will come in the place of question mark in following


questions.
ननम्न प्रश्नों भें प्रश्नचचह्न के स्थान ऩय क्मा आएगा?
47 × 27 + 15600 ‚ 8 + 181 = ?
(A) 3320 (B) 3420 (C) 3370
(D) 3400 (E) 3460
https : https : https : https :
//youtube.co //www.face //instagram.c
m/channel/U book.com/a om/aashishar //t.me/
CYa4_JrOrf8R orasocial?ut
383 5Kz2uOtccXQ
a s hisharora
m_medium = studifie
s ocial
copy_link d
24. What will come in the place of question mark in following
questions.
ननम्न प्रश्नों भें प्रश्नचचह्न के स्थान ऩय क्मा आएगा?
112.5 × 5 + 4560 ÷ 6 – 175 × 7 = ?
(A) 103.5 (B) 91.5 (C) 97.5
(D) 110.5 (E) 115.5

25. What will come in the place of question mark in following


questions.
ननम्न प्रश्नों भें प्रश्नचचह्न के स्थान ऩय क्मा आएगा?
80% of ? = 250 X 44  40%of 8500
(A) 80 (B) 120 (C) 150
(D) 180 (E) 240

26. What will come in the place of question mark in following


questions.
ननम्न प्रश्नों भें प्रश्नचचह्न के स्थान ऩय क्मा आएगा?
? × 40 ‚ 24 × 27 = 594/11 × 2300/264
(A) 1 (B) 2 (C) 3
(D) 4 (E) 5

27. What will come in the place of question mark in following


questions.
ननम्न प्रश्नों भें प्रश्नचचह्न के स्थान ऩय क्मा आएगा?
? + 13 × 50 = 420 + 45% of 800 + 220
(A) 300 (B) 350 (C) 400
(D) 450 (E) 250

28. What will come in the place of question mark in following


questions.
ननम्न प्रश्नों भें प्रश्नचचह्न के स्थान ऩय क्मा आएगा?
12 % of? + 12.5% of 960 = 16 × 12
(A) 840 (B) 960 (C) 800
(D) 600 (E) 400

29. What will come in the place of question mark in following


questions.
ननम्न प्रश्नों भें प्रश्नचचह्न के स्थान ऩय क्मा आएगा?
15 × ? + 20% of 450 = 360
(A) 12 (B) 20 (C) 24
(D) 16 (E) 18
https : https : https : https :
//youtube.co //www.face //instagram.c
m/channel/U book.com/a om/aashishar //t.me/
CYa4_JrOrf8R orasocial?ut
384 5Kz2uOtccXQ
a s hisharora
m_medium = studifie
s ocial
copy_link d
30. What will come in the place of question mark in following
questions.
ननम्न प्रश्नों भें प्रश्नचचह्न के स्थान ऩय क्मा आएगा?
75% of ¾ x 8/9 x 1800 = ? + 600
(A) 300 (B) 400 (C) 250
(D) 480 (E) 540

31. What will come in the place of question mark in following


questions.
ननम्न प्रश्नों भें प्रश्नचचह्न के स्थान ऩय क्मा आएगा?
? % of 900 + 500 = 4 × 197
(A) 28 (B) 25 (C) 36
(D) 40 (E) 32

https : https : https : https :


//youtube.co //www.face //instagram.c
m/channel/U book.com/a om/aashishar //t.me/
CYa4_JrOrf8R orasocial?ut
385 5Kz2uOtccXQ
a s hisharora
m_medium = studifie
s ocial
copy_link d
https://t.mhttps/t.me
e/studified/studified

https://instagram.com/aashisharora
social?utm_medium = copy_link

https://youtube.com/channel
/UCYa4_JrOrf8R5Kz2uOtccXQ

https://www.facebook.
com/aashisharorasocial

https://youtu https://instagra
https ://ww
be.com/chan w.fa cebook.
m.com/aashish https://
arorasocial(?)
nel/UCYa4_Jr com/a ashis utm_medium = t.me/st
386 Orf8R5Kz2uO
tccXQ
ha rorasocial copy_link
udified

You might also like